You are on page 1of 238

SISTEMI KARDIOVASKULAR

1. Ju jeni emëruar si këshilltar i epidemiologut kardiak në një komitet ndërkombëtar që vendos politikat shëndetësore
globale. Komiteti juaj është caktuar të parashikojë tendencat shëndetësore në një vend Pacifik të Azisë Lindore me një
popullsi dhe mjedis shumë te ngjashëm me Kinën. Në këtë pikë të historisë, kombi është duke lëvizur nga faza e parë
klasike në tranzicionin epidemiologjik ("murtaja dhe uria") në fazen e dytë klasike ("pandemitë që tërhiqen"). Në lidhje
me modelet e parashikuara të kardiovaskulare sëmundja (CVD) në këtë komb, cila nga të mëposhtmet është e vërtetë?
A. Ndërsa ky komb hyn në fazën e tërheqjes së pandemive, ju do të prisni qe ne shumicen e CVD sëmundshmëria dhe
vdekshmëria duhet të vijnë nga kardiomiopatitë dytësore ndaj agjentëve infektivë.
B. Çdo komb, vend gjeografike përparon përmes fazave tracizionit epidemi në mënyrë identike sipas rrezikut CVD.
C. Në këtë komb, ku vdekshmëria nga CVD arrin kulmin, ju do të prisnit që vdekshmëria nga goditja në tru të jetë
më e madhe se vdekshmëria nga sëmundjet koronare.
D. Dikush priste një model homogjen epidemiologjik të rrezikut të CVD në të gjithë këtë komb me kalimin e kohës.
E. Ju parashikoni që vdekshmëria nga CVD të rritet në mënyrë të pashmangshme ndërsa ky komb përparon përmes
fazave te tranzicionit epidemiologjik

2. Ju jeni duke punuar në një klinikë shëndetësore rurale në Indinë Veriore. Ju ekzaminoni një djalë 8-vjeçar që nuk
eshte parë me pare nga një mjek. Nëna e tij ju tregon se ai nuk është në gjendje të vazhdojë me moshatarët e tij për
sa i përket aktivitetit fizik. Në ekzaminimin fillestar të lëkurës së tij, vëreni gishtat e Hipokratit dhe cianozë në këmbët
e tij, por duart normale. Pa ndonjë ekzaminim të mëtejshëm, ju dyshoni se ai ka cilen nga anomalite kongenitale?
A. Defekti i septumit atrial
B. Dextro-transpozimi i arterieve të mëdha (TGA)
C. Ductus arteriosus i patent me hipertension pulmonar sekondar
D. Tetralogjia e Fallot
E. Defekti i septumit ventricular

3. Një grua afrikano-amerikane 55-vjeçare, pa histori të kaluar mjekësore, paraqitet në departamentin e urgjencës me
sinkop dhe elektrokardiograme (EKG) te shënuar në Figurën V-3A
më poshtë. Ju gjithashtu vëreni nyje të dhimbshme subkutane në këmbët e saj (të ngjashme me ato të paraqitura në
Figurën V-3B më poshtë). Ju dyshoni se anomalitë e zemrës dhe të lëkurës ka shumë të ngjarë për shkak të:
A. Sindroma A. Carney
B. Hipotiroidizmi
C. Sarkoidoza
D. Lupusi eritematoz sistemik
E. Sifilizi terciar

4. Ju po vlerësoni z. Estebez, një pronar 67-vjeçar i një zinxhiri shumë të suksesshëm të restoranteve sushi. Ai ankohet
për dispne ne sforcim, edemë të ekstremiteve të poshtme dhe zgjimi gjatë natës nga veshtiresia ne frymemarrje. Ju
dëshironi të vlerësoni statusin e vëllimit të tij dhe ta dini se vlerësim i pulsit venoz jugular (JVP) është ekz fizik i vetëm
më i rëndësishëm për t'ju ndihmuar . Cila nga pohimet e mëposhtme në lidhje me matjen e JVP është e vërtetë?
A. Nëse bëhet siç duhet, këndi i inklinimit ka pak rëndësi për matjen e JVP.
B. Në pacientët normalë, JVP rritet ne inspirim për shkak të vëllimit të shtuar të ngarkesës ne zemren e djathte.
C. Matja ne lartësinë e majës së JVP dhe pikës së përthithjes sternale (këndi i Louis) do të sigurojë një matje shumë
të saktë të presionit venoz qendror.
D. Jugularia externe preferohet mbi venën jugulare interne për shkak të lehtësisë së dukjes.
E. Pulsimet venoze mbi klavikulën në pozicionin ulur janë anormale.

5. Cila nga pohimet e mëposhtme në lidhje me matjet e presionit të gjakut është e vërtetë?
A. Presioni sistolik rritet dhe presioni diastolik zvogëlohet kur matet në arterie me distalisht
B. Presionet sistolike të gjakut në këmbë janë zakonisht 20 mmHg më të ulëta se presionet e gjakut në krah.
C. Koncepti i "hipertensionit të bluzes së bardhë" (presionet e gjakut të matura në ambiente klinike ose spitali,
dukshëm më e lartë se në mjediset joklinike) është treguar të jetë një mit.
D. Diferenca në presionin e gjakut e matur në të dy krahët duhet të jetë më pak se 20mmHg.
E. Përdorimi i manshete te vogel do të rezultojë në një nënvlerësim të theksuar të vleres reale te presionit te gjakut.

6. Një burrë 75 vjeç paraqite ne urgjencë mjaft i sëmurë. Familjathotë ai nuk ka pasur energjinë normale 6 muajt e
fundit dhe ishte i hutuar dhe letargjik gjate ditëve te fundit. Pulsin radial të pacientit ka ndryshueshmëri të rregullt të
amplitudës së pulsit, megjithëse ritmi i rregullt. Presioni gjakut, çdo fazë tjetër I (sistolike) tingulli Korotkoff dëgjohet
pasi presioni i manshetës ulet ngadalë dhe kjo e pavarur nga cikli i frymemarrjes. Ca dyshoni ?
A. Fibrilacioni atrial
B. Tamponada kardiake
C. Perikarditi konstriktiv
D. Embolia pulmonare
E. Insuficience e rëndë e ventrikulit të majtë

7. Një 78-vjeçar është pranuar në njësinë e kujdesit intensiv me insuficense kardiake te dekompensuar. Ai ka
kardiomiopati ishemike. EKG tregon fibrilacionin atrial dhe bllokun e degës majtë. Radiografi gjoksit tregon
kardiomegali, infiltrate bilaterale alveolare me linjat Kerley B. Cila ka më pak të ngjarë të jetë e pranishme në ekz fizik?
A. Tingulli i katërt i zemrës
B. Frekuence jo e rregullt kardiake
C. Shenja Kussmaul
D. Ndarja paradoksale e tingullit të dytë të zemrës
E. Tingulli i tretë i zemrës

8. Një 24-vjeçar i referohet kardiologut pas një episodi sinkopi ndërsa luante basketboll. Ai nuk ka asnjë kujtim të
ngjarjes, por ai u tha se ai u shemb ndërsa vraponte. Ai u zgjua i shtrirë në tokë dhe pësoi kontuzione të shumta si
rezultat i rënies. Ai gjithmonë ka qenë një individ aktiv por së fundmi ka zhvilluar disa dhimbje gjoksi me sforcim që i
ka shkaktuar atij kufizim te veprimtarisë e tij. Babai i tij vdiq në moshën 44 vjeç kur ngjitej në shkëmb. Ai beson se ishte
vdekje e papritur kardiake dhe kujton se babait të tij i ishte zmadhuar zemra. Në ekzaminim, pacienti ka një murmuritje
krescendo-dekrescendo mes-sistolike III / VI. EKG-ja e tij tregon hipertrofi VM. Ju dyshoni në kardiomiopatinë
hipertrofike si shkakun e sëmundja e zemrës. Cila nga manovrat pritet rritse zhurmën e murmuritjes?
A. Berja grusht e dores
B. Squatting
C. Në këmbë
D. Manovra e Valsalvës
E. A dhe B
F. C dhe D
9. Një grua 75-vjeçare me kancer të mushkërive është pranuar në njësine e kujdesit intensiv me presion sistolik të
gjakut 73/52 mmHg. Ajo paraqiti duke u ankuar edhe keqësimi te dispnesë gjatë 3-5 ditëve të fundit. Ekzaminimi i saj
fizik tregon vena te zmadhura te qafes. Radiografia e kraharorit tregon një hije zemre masive, në formë shishe uji dhe
asnjë infiltrat te ri ne pulmone. Cila nga gjetjet shtese ka shumë të ngjarë të jetë e pranishme në ekzaminimin fizik?
A. Bie presioni sistolik i gjakut> 10 mmHg ne inspirim
B. Mungesa e rënies së presionit venoz jugular ne inspirim
C. Zhurma e vonë diastolike me hapje të shpejtë
D. Pulsus parvus et tardus
E. Zbritja e shpejtë y e presionit venoz jugular

10. Cila nga pohimet e mëposhtme në lidhje me modelet normale të depolarizimit të zemrës është e vërtetë?
A. Çdo rrahje normale e sinusit fillohet nga depolarizimi spontan në nyjen atrioventrikular (AV)
B. Rendi normal i depolarizimit: nyja SA - miokardi atrial – AV nyja – Tufa e Hisit - Fijet Purkinje - miokardi ventrikul
C. Dega e tufës së djathtë dyzohet në një fashikul të përparmë dhe të pasmë.
D. Nyja SA është unike në aftësinë e saj për të depolarizuar në mënyrë spontane, një cilësi e njohur si automatizëm.
E. Brenda miokardit ventrikular, depolarizimi fshihet nga epikardi në endokard.

11. Të gjitha format e EKG-së përputhen saktësisht me ciklin kardiak që ato përfaqësojnë PËRVEÇ:
A. Vala P - repolarizimi atrial
B. Intervali i PR - repolarizimi atrial
C. Kompleksi QRS - depolarizimi ventrikular
D. Vala T - repolarizimi i ventrikulit
E. Vala U - repolarizimi i ventrikulit

12. Cili nga vektorët mesatar të QRS përputhet me planin EKG frontal të përshtatshme për përcaktimin e tij?
A. –20 gradë - boshti normal
B. –35 gradë - devijimi i boshtit të djathtë
C. –110 gradë - devijimi i boshtit të majtë
D. –80 gradë - devijimi ekstrem i boshtit
E. Të gjitha më lart janë të pasakta.

13. Cila nga të mëposhtmet përfaqësohet në EKG të paraqitur në Figurën V-13?

A. Blloku i zemrës i shkallës së parë


B. Blloku i degës së tufës së majtë
C. P-pulmonale
D. Blloku i degës së tufës së djathtë
E. S1Q3T3 (modeli McGinn-White) tregues i sforcimit të ventrikulit të djathtë
14. Cila nga opsionet e mëposhtme përshkruan gjetjen kryesore në EKG të treguar në Figurën V-14?

A. Hipertrofia e ventrikulit të majtë


B. EKG normale
C. Valët T të pikut, mbase hiperkalemia
D. Bradikardia e sinusit
E. ngritje ST ne lidhjet prekordiale; dyshoni për ishemi të përparme të miokardit

15. Paraqitet një punëtor ndërtimi 56 vjeçar me hipertension dhe një histori paraprake të abuzimit me duhanin në
departamentin e urgjencës me 30 minuta nauze me shfaqje akute, dispne dhe presion ne gjoks. EKG-ja e tij fillestare
paraqitet në Figurën V-15. Të gjitha sa më poshtë janë të pranishme në këtë EKG PERVEÇ:

A. IM Inferior
B. Valët P
C. IM Posterior
D. Takikardia sinusale
E. Takikardia ventrikulare

16. Në pacientin në Pyetjen V-15, cila nga arteriet koronare të mëposhtme është më e mundshme te jete e mbyllur?
A. A. Perforuese septale e pare
B. Arteria zbritëse e përparme e majtë proksimale
C. Arteria cirkumflexe e majtë
D. LAD proximale dhe circumflexia
E. Arteria koronare e djathtë

17. Një grua 48-vjeçare ju takon në klinikën e kujdesit parësor për vlerësim fillestar pasi ka ardhur nga nje vend tjeter.
Ju nuk keni të dhëna mjekësore të së kaluarës, megjithëse ajo këmbëngul se ato u dërguan me postë një javë më parë.
Ajo shprehet se ka pasur «disa telashe zemre», por nuk është e qartë për detajet. Gjithashtu, ajo merr tableta për
"kolesterolin dhe presionin e gjakut". EKG fillestare tregohet në Figurën V-17. Cili nga deklaratat e mëposhtme në
lidhje me këtë EKG është e vërtetë?
A. Ka të ngjarë që ajo të ketë pësuar një infarkt paraprak të miokardit.
B. Ajo është në ritëm normal të sinusit.
C. Prania e një blloku të degës së tufës së majtë në këtë EKG tregon dissinkronizim mekanik.
D. Prania e kontraksioneve të parakohshme ventrikulit dhe takikardisë shqetësuese për crregullime te elektroliteve
E. Prania e valeve T te permbysura ne lidhjet anteriore është shqetësuese në këtë EKG për miokardin akut ishemi.
18. Ju kërkohet të ekzaminoni një student mjekesie 27-vjec që raporton 1 javë kollë, coryza, dhe temperature te ulët.
Sot, ai ka zhvilluar ndjeshëm shqetësime në gjoks duke u përshkallëzuar ndërsa ishte ne spital. Ai vëren se dhimbja
bëhet më e fortë kur merr frymë thellë. Ju kryeni një EKG standarde me 12 lidhje (shih Figurën V-18). Në ekzaminim,
presioni i saj i gjakut është normal, ai eshte afebril, dhe pulsi i tij venoz jugular nuk është i ngritur. Megjithatë, ai duket
lehtesisht i shqetesuar nga dhimbjet e gjoksit. Hapi tjetër më i përshtatshëm do të ishte cili nga të mëposhtmit?

A. Administroni aspirinë, heparinë intravenoze, nitroglicerinë sublinguale dhe klopidogrel.


B. Kryeni ekokardiogramë transtorakike në mënyrë emergjente me perikardiocentezë të mundshme.
C. Kryen në mënyrë emergjente Koro për të vlerësuar IAM.
D. Pëshkruaj ibuprofen dhe kolkicinë.
E. Referojuni proves ushtrimore.
19. Një stuhi e madhe dëbore godet zonën tuaj të enjten dhe rrugët janë kryesisht të pakalueshme deri te henen e
ardhshme. Të hënën në mëngjes, një 48-vjeçar është sjellë nga urgjenca mjekësore në departamentin e urgjencës pasi
u gjet me statusin te alteruar mendor nga fqinji. Ai shqetësohet dhe nuk është në gjendje të pershkruaje ngjarjen. Ju
vini re një fistule te hemodializes ne arterien brakiale sinister. EKG-ja e tij fillestare tregohet në Figurën V-19. Cilen nga
anomalitë e elektroliteve do të prisnit të gjenit tek ky pacient?
A. Hiperkalcemia
B. Hiperkalemia
C. Hipokalemia
D. Hipomagnezemia
E. Hiponatremia
20. Një 66-vjeçar është shtruar në spital nga dispnea progresive gjatë aktivitetit dhe lodhje. Ai ka një histori të kaluar
të abuzimit me duhanin dhe ka udhetuar kohet e fundit ne Ameriken e Jugut. Në prezantim, FK janë 104 / min dhe i
parregullt. Presioni i gjakut është 96/76 mmHg. Ju vini re një puls të ngritur venoz jugular dhe edeme te theksuar te
ekstremiteteve te poshtme. Ekokardiograma zbulon EF rreth 55%, dhe akoma imazhet nga ekokardiograma e tij
tregohen në Figurën V-20. Për të sqaruar etiologjinë insuficiences kardiake, cili nga sa më poshtë është testi
diagnostikues më i përshtatshëm për të kryer më pas?

A. Biopsia e palcës kockore


B. Testimi gjenetik për mutacionet transthyretin
C. Testi proves ushtrimore nepermjet tomografisë me emision me pozitorron
D. Serologjitë për Trypanosoma cruzi
E. Elektroforeza e proteinave të serumit dhe urinës dhe analiza e zinxhirit të lehtë
21. Zonja Jackson është një grua afrikano-amerikane 45-vjeçare me një histori të abuzimit të duhanit, kancer gjiri
(statusi: post-mastektomia dhe rrezatimi), dhe alergjia ndaj butakëve. Ajo u paraqit ne kliniken e kardiologjisë 2 javë
para raportimit të dispnesë ndërsa ecte një kodër afër shtëpisë së saj, gjendje e tille prej 6 muajsh. Ajo kurrë më parë
nuk ka pasur vështirësi me këtë kodër. Së bashku me dispne, ajo përjeton disa të përziera të paqarta dhe diaforeze te
shtuar EKG-ja në pushim dhe aktivitet fizik nuk shfaqin ndryshime. Ju e referuat atë për një tomografi kompjuterike
me emision me një foton stërvitje (SPECT) skanimi i sestamibit të perfuzionit të miokardit technetium-99m (99mTc)
dhe mori rezultatet, në foto në Figura V-21. Ju shpejt vlerësoni se skanimi i saj tregon ishemi të kthyeshme. Cili territor
arterial ka shumë të ngjarë të përfshihet në skanimin në fotografi?
A. LAD
B. Arteria cirkumlexe e majtë
C. Arteria koronare kryesore e majtë
D. Arteria posterior zbritëse

22. Nyja SA shërben si pacemaker në ritmin normal të sinusit. Çfarë vetie qelizave veprojë si stimulues kardiak primar?
A. Vendndodhja afër zonës anësore superiore të atriumit të djathtë
B. Disqe më të shumta intercalate se ne cdo ind tjeter ne miokard
C. Faza më e shpejtë depolarizimi 0
D. Vetia e depolarizimit spontan
E. Depolarizimi spontan gjatë fazës 4, potencialit veprimit me shpejtësi më shpejtë se çdo qel tjeter e miokardit

23. Z. Hendricks është një somelier 21-vjeçar në një restorant të njohur në qytet. Hobi i tij eshte gara me bicikleta dhe
ai ka një garë 150 km fundjavën e ardhshme për të cilën ai ka qenë duke u trajnuar për 6 muajt e fundit. Koordinatorët
e garës kërkojnë që secili garues të plotësojë një vlerësim gjithëpërfshirës kardiovaskular para konkurrimit, dhe kështu
ai vjen për të vizituar klinikën tuaj. Në vlerësimin tuaj, ju vini re një frekuence kardiake pushimi ose 45 bpm me një
pauzë të rastit deri në 2 sekonda. Presioni i tij i gjakut është 108/72 mmHg. Ai ndihet mirë dhe nuk raporton asnjë
episode sinkopal ose presinkopal në pushim ose gjatë ushtrimeve. Përveç bradikardisë, nuk vëreni asnjë anomali tjetër.
EKG-ja e tij tregon ritem sinusal me një interval PR prej 128 msec, kohëzgjatja QRS 80 msec, dhe pauza të
herëpashershme për 2.2 sekonda. Cila nga të mëposhtmet do të ishte e përshtatshme këshilla për Z. Hendricks?
A. Kryeni vlerësimin elektrofiziologjik për shqyrtimin e një stimuluesi kardiak.
B. Ai nuk duhet të garojë në garën e ardhshme për shkak të bradikardise dhe do të duhet nje prove tilt-table
C. Nuk ka nevoje per ndjekje te metejshme! Pac fat me garen!
D. Holter 48 oresh.
E. Ai duhet t'i nënshtrohet një prove ushtrimore për të përcaktuar praninë e kompetencës kronotrope.

24. Një 60-vjeçar po i nënshtrohet një studimi elektrofiziologjik për vlerësimin e sinkopës. Pas kanjulimit venoz te
kujdesshëm dhe vendosjes se kateterëve dhe pas administrimit te 0,2 mg / kg propranolol dhe 0,04 mg / kg atropinë,
FK eshte 65 bpm. Pas ndalimit të barnave dhe lejimit të kohës së mjaftueshme për larje, atriumi i tij i djathtë superior
/ lateral ka frekuence 140 bpm. Me ndërprerjen e pace-imit, rrahja e tij e radhës sinusale ndodh 1800 msec më vonë.
Bazuar në këto vëzhgime, pacient mund të diagnostikohet me cilën nga sa më poshtë?
A. Kardiomiopatia amiloide
B. Sëmundja e nyjes AV
C. Fibrilimi atrial paroksizmal
D. Sëmundja e nyjes SA
E. Sindroma Tachy-brady
25. Të gjitha sa më poshtë janë shkaqe të kthyeshme të mosfunksionimit të nyjes SA pervec:
A. Hipotermia
B. Hipotiroidizmi
C. Rritja e presionit intrakranial
D. Toksiciteti i litiumit
E. Terapia me rrezatim

26. Cili nga sa më poshtë është një faktor rreziku për zhvillimin e tromboembolisë në pacientët me variantin e
takikardisë-bradikardisë së sindromës së sinusit të sëmurë?
A. Mosha> 50 vjeç
B. Dilatimi atrial
C. Diabetes mellitus
D. Mutacioni protrombina 20210
E. Asnjë nga më sipër; s`ka rrezik të rritur tromboembolie ne sem sinusit te semure me variantin takikardia-bradikardi

27. Qelizat normale brenda nyjës AV shfaqin një veti të njohur si përçueshmëri ngadalesuese. Nëse ti do ta
demonstrosh këtë veti gjatë një studimi të elektrofiziologjisë, çfarë manovre mund të kryeni?
A. Pejsoni AD=atriumin dexter, duke zvogeluar gjatesine e ciklit dhe matni kohen e percimit ne tufen e Hisit
B. Pejsoni ventrikulin dhe regjistroni potencialet e AD.
C. Administroni atropinë 0,04 mg / kg dhe regjistroni kohën e përçimit te nje implusi deri ne tufen e Hisit
D. Administroni metoprolol 10 mg IV dhe regjistroni kohën e përçimit te nje implusi deri ne tufen e Hisit
E. Administroni adenozinë 12 mg IV dhe regjistroni kohën e rigjenerimit te nyjes AV.

28. Një burrë 87-vjeçar me histori të hipertensionit dhe stenozës së aortës është bërë simptomatik nga stenoza e tij e
aortës gjatë 2 muajve të fundit. Dje, ai iu nënshtrua zëvendësimit te valvulës me një valvul bioprotetike 25 mm me
rezultate të shkëlqyera intraoperative. Ai ishte shkeputur me shpejtësi nga by-passi kardiopulmonar dhe ekstubohet
brenda 24 orësh. Për protokoll kirurgjik, ai kishte tela të përkohshëm të ritmit epikardial të vendosur në sipërfaqen e
ventrikulit dhe ka ecur me ritëm në 90 bpm. Në kontrollet e këtij mëngjesi, ju ndaloni pacimin ventrikular për të
kontrolluar ritmin e tij bazik . Ju vini re një Frekuence atriale rreth 80/min , por një frekuence ventrikulare prej 32/
min me një QRS të gjerë komplekse. Nuk ka asnjë lidhje midis valëve P dhe komplekseve QRS. Ventrikuli i këtij pacienti
bradikardia ka shumë të ngjarë për shkak të cilës prej më poshtë?
A. Zhvillimi i një sëmundjeje sistemike siç është sarkoidoza ose sëmundja Lyme që shkakton nyjen AV
A. mosfunksionim
B. Endokarditi i bioprotezës së re të aortës që shkakton bllokimin e nyjeve AV
C. Sëmundja e nyjës SA
D. Shërimi i ngadalësuar i nyjeve AV pas ritmit të shpejtuar
E. Demtimi kirurgjikal i nyjes AV

29. Zonja Hellwig është një grua 25-vjeçare me lupus eritematoz sistemik (SLE) e komplikuar nga nefropatia, anemi
hemolitike dhe pleuriti. Sëmundja e saj është e kontrolluar mirë me terapi. Ajo së fundmi, zbuloi se është shtatzënë
dhe paraqitet sot për këshillim para lindjes. Ajo në mënyrë specifike është e shqetësuar për efektin që sëmundja e saj
autoimune mund të ketë tek foshnja. Ju i thoni asaj se ndërlikimi më i zakonshëm kardiak në fëmijët e lindur nga nëna
me SLE është:
A. Blloku AV
B. Sëmundja e arterieve koronare
C. Kardiomiopatia e dilatuar
D. Hipertensioni pulmonar me insuficience të ventrikulit të djathtë
E. Endokarditi Libman-Sacks steril
30. Z. Hofman, një ish-82-vjeçar ish-interpretues i litarëve të ngushtë, ju paraqet me ankesen e sinkopes. Ai deklaron
se dy herë gjatë javës së kaluar, ka humbur ndergjegjen pa paralajmërim. Një herë, ai goditi fytyrën e tij dhe ju vini re
se ai ka ekimozë periorbitale. Ju nuk keni asgjë anormale tjeter gjatë ekzaminimit. Ju kërkoni një EKG dhe dilni nga
dhoma për të filluar dokumentacionin tuaj. Pas pak, asistenti juaj mjekësor kërkon praninë tuaj urgjente në dhomën e
Z. Hofman. Ai kishte humbur vetëdijen. Ndihmësi mjekësor kapi episodin në EKG, foto në Figurën V-30. Cfare tipi i
bllokut AV është i pranishëm dhe cili eshte trajtimi i duhur apo ekzaminimi i nevojshem?

A. Bllok i plotë i SA node - Implantim i përhershëm pacemaker kardiak


B. Bllok AV i shkallës së parë - Administroni atropinën
C. Blloku AV node i shkallës së dytë të Mobitz të tipit II – Prove ushtrimore
D. Bllok AV i Mobitz-it të shkallës së dytë - Nuk ka nevoje per nderhyrje
E. Blloku i AV node të shkallës së dytë të Mobitz - Implantimi i përhershëm i stimuluesit kardiak

31. Një vrapues i distancave të gjata 19-vjeçar, i cili doli ne 10 te paret vitin e kaluar, paraqitet për vlerësimin e zemrës
pasi mjeku i tij i kujdesit parësor sugjeroi një monitorim Holter për qëllime screening. Në raportin e tij Holter, disa
episode të shkallës së dytë, Mobitz I (Wenckebach) u vërejtën, të gjitha ndodhën gjatë gjumit. Pacienti nuk raporton
asnjë simptomë, por mendon se gjyshi i tij ka vendosur nje pacemaker ne moshe te avancuar. Cili eshte hapi i rradhes?
A. Prove ushtrimore
B. Studimi elektrofiziologji invaziv
C. Rivleresimi
D. Referim për implantimin e pacemaker
E. Testimi serologjik duke përfshirë nivelet e hormoneve stimuluese të tiroides

32. Jeni duke vlerësuar një grua 47-vjeçare, me një histori të abuzimit me duhani dhe kolit ulceroz, per palpitacione
me ndërprerje. Ajo raporton se për 6 muajt e fundit, çdo 2-4 ditë ajo vë në dukje një ndjesi te cuditshme flip-flopping
te zemres qe zgjat rreth 5 min. Ajo referon ndonjë faktor nxitës dhe nuk ka patur marramendje, te fiket dhe as dhimbje
gjoksi me këto episode. Ekzaminimi fizik është normal. Një EKG në pushim zbulon ritem sinusal dhe asnjë anomali.
Përveç vleresimit te elektroliteve ne serum, cili nga të mëposhtmit është testi më i përshtatshëm?
A. Tomografia e kompjuterizuar e barkut (CT) me kontrast oral dhe intravenoz (IV)
B. Monitoruesi i ngjarjes
C. Monitori Holter
D. Rivleresim, pa ndonjë provë të mëtejshme të nevojshme
E. Referimi për studimin e elektrofiziologjisë
33. Pas provave të mëtejshme, pacienti në pyetjen V-32 zbulohet se ka disa episode te ekstrasistolave atriale. Cila nga
pohimet e mëposhtme në lidhje me crregullimin e ritmit eshte e sakte?
A. ES atriale jane me pak te zakonshme se ato ventrikulare gjate monitorimit te zgjatur te EKG-së.
B. Eko kardiake per te percaktuar nese ka ndonje anomali strukturore
C. Metoprololi për kontrollin e simptomave.
D. Pacienti duhet të sigurohet se kjo nuk është një gjendje e rrezikshme dhe nuk kërkon vlerësim te mëtejshëm.
E. Pacienti duhet t’i nënshtrohet proves ushtrimore për të përcaktuar nëse është e pranishme ishemia.
34. Znj. Milsap është një 18 vjecare, yll volejbolli i shkollës së mesme me një bursë sportive ne universitetin vendas.
Si pjesë e procesit të saj të pranimit, asaj i kërkohet t'i nënshtrohet një vlerësimi të plotë mjekësor para se të marri
pjesë në sporte kolegjiale. Ekzaminimi fizik nuk zbulon anomali, megjithëse ajo raporton episodin e rralle të
palpitacioneve, marramendjes dhe te fiketit. EKG zbulon një interval PR prej 0,06 msec, kohëzgjatja e QRS prej 140
msec dhe një valë në pjesën fillestare të QRS. Ti diagnostikon saktë këtë si model Wolff -Parkinson-White. Cila nga
pohimet e siguron se znj. Milsap nuk do kete shfaqje klinike sinjifikante dhe nuk ka nevoje per ablacion me kateter
per shkak te kesaj anomalie?
A. Aftësia për të arritur FK 185 bpm në një provë ushtrimore
B. Studimi i elektrofiziologjisë që demonstron se rruga aksesore ka percim anterograde dhe retrograde
C. Studimi i elektrofiziologjisë që demonstron se rruga aksesore ndodhet në rajonin posteroseptal
D. Prova ushtrimore që demonstron zhdukjen e valës delta dhe QRS të gjerë kur arrihet FK 120 bpm
E. Monitorimi i Holterit duke demonstruar FA paroxistik

35. Një grua 85-vjeçare pa histori të mëparshme kardiake paraqitet në departamentin e urgjencës me 2 orë
palpitacione. Presioni i gjakut, oksigjenimi dhe rrahjet e zemrës janë normale, megjithëse ju shënoni një ritëm të
parregullt gjatë ekzaminimit. EKG tregon një QRS të parregullt, të ngushtë pa valë P të dallueshme në një frekuence
prej 75 bpm. Ekokardiograma nuk zbulon ndonjë sëmundje strukturore të zemrës. Pavarësisht rrahjeve normale të
zemrës, pacientja eshte simptomatike dhe deshiron kthimin ne ritem sinusal. Të gjitha ndërhyrjet e mëposhtme mund
të jenë përfituese PERVEÇ:
A. Adenozina IV
B. Amiodaroni IV
C. Kardioversioni direkt
D. Dofetilide oral
E. Flecainide oral

36. Një burrë 79-vjeçar me histori të sëmundjes së arterieve koronare, kardiomiopati ishemike me EF rreth 30%, dhe
hypertension paraqitet në kliniken tuaj pa ndonjë ankese te re. Presioni i gjakut është 108/56 mmHg, FK eshte 88 bpm,
dhe saturimi eshte 98%. EKG e tij tregohet në Figurën V-36. Bazuar në këtë EKG, pacienti tani ka një (klasa I) indikacion
definitiv për cilën nga terapitë e mëposhtme?

A. Amiodaroni 400 mg në ditë


B. Aspirina 325 mg në ditë
C. Flecainide 600 mg palpitacione PRN
D. Antikoagulimi sistemik me warfarin ose një antikoagulues oral te ri
E. Ekokardiografia transesofageale e ndjekur nga kardioversion direkt
37. Një grua 43-vjeçare shihet në departamentin e urgjencës pas fillimit të papritur të palpitacioneve 30 minuta para
vizitës së saj. Ajo ishte ulur në kompjuterin e saj të punës kur filluan simptomat. Përveç dhimbjes se mesit, ajo është e
shëndetshme. Në triazh, FK eshte 178 bpm dhe presioni i gjakut është 98/56 mmHg me oksigjenim normal. Në
ekzaminimin fizik, ajo ka një "shenjë bretkocë" në qafë dhe takikardi. EKG tregon një takikardi me komplekse te
ngushte pa vale P. Cili nga sa më poshtë është hapi i parë më i përshtatshëm për të menaxhuar atë takikardi?
A. 5 mg metoprolol IV
B. 6 mg adenozinë IV
C. 10 mg verapamil IV
D. Masazhi i sinusit karotid
E. Kardioversioni DC duke përdorur 100 J

38. Cila nga pohimet e mëposhtme në lidhje me rivendosjen e ritmit sinusal pas FA është e vertete?
A. Dofetilidi mund të fillojë në mënyrë të sigurt si ambulator.
B. Në pacientët që trajtohen me farmakoterapi dhe zbulohet se janë në ritem sinusal, behet monitorimi me Holter
per te percaktuar nese stopimi i terapise antikoaguluese eshte i sigurte
C. Pacientët që kanë ruajtur ritmin sinusal farmakologjikisht pas FA kanë mbijetesë me te larte në krahasim me
pacientët që trajtohen me kontroll të frekuences dhe antikoagulim.
D. Përsëritja e FA është e pazakontë kur farmakoterapia përdoret për të ruajtur ritmin sinusal

39. Një grua 76-vjeçare me histori hipertensioni, sëmundje pulmonare obstruktive kronike (COPD), diabeti mellitus
dhe osteoporoze paraqitet në departamentin e urgjencës pas një rënieje në shtëpi e ndjekur menjëherë nga dhimbje
të forta të art koxofemoral te majte. Ajo u gjet nga një fqinj pas disa orësh.Pacientja nuk mund të kujtojë nëse e humbi
vetëdijen. Ajo është jashtëzakonisht e ndjeshme ne palpim dhe këmba e saj është shkurtuar dhe rrotullohet nga jashtë.
Mukozat e saj janë të thata. Presioni i gjakut është 170/80 mmHg dhe rrahjet e zemrës janë 130 bpm. ECG është
treguar në Figurën V-39. Cili është hapi i parë më i përshtatshëm për takikardinë e saj?

A. Adenozina 6 mg IV
B. Kardioverzioni DC
C. Digoxin 250 μg IV
D. Metoprolol 5 mg IV
E. Kontrolli i dhimbjes dhe hidratimi IV

40. Një grua e re është sjellë në departamentin e urgjencës pasi dëshmitarët e vëzhguan atë papritmas te humbe
vetëdijen ndërsa vraponte në parkun aty pranë. Ajo ka një hundë të thyer dhe dhëmbë të thyer nga rënia e saj. Nuk
eshte e identifikuar. Ajo nuk reagon ndaj stimujve të dhimbshëm. Presioni i gjakut është 50 / prekje dhe FK rreth 280
bpm. Një EKG me 12 lidhje është treguar në Figurën V-40. Cila është hapi i rradhes me i pershtatshem?
A. Amiodaroni 150 mg bolus IV
B. Lidokainë 1 mg IV bolus
C. Metoprolol 10 mg IV
D. Debrilacioni DC
E. CT me kontrast për të vlerësuar për TEP
41. Cili eshte ritmi ne pacientin e pyetjes 40?
A. Fibrilacion atrial me konduksion antegrade permes nje trakti aksesor?
B. Flater atrial me konduksion 1:1
C. Takikardi AV me mekanizmin e rihyrjes
D. Takikardi nodale AV rihyrese
E. Takikardi ventrikulare
42. Nje burre 67 vjec me nje histori hipertensioni dhe hyperlipidemie vjen para 3 oresh ne urgjence me nje dhimbje
gjoksi substernale shtypese dhe dispne me fillim akut. Ka ngritje te ST ne EKG ne lidhjet anteriore dhe laterale. Iu
administruan trombolitike dhe u dergua ne terapi intensive kardiake. Iu therret infermierja sepse ka ndryshime ne
monitor. Ritmi ne EKG me poshte, lidhja V6?

A. Fibrilacion atrial
B. Bllok i plote me nje junctional escape
C. Ritem idioventrikular
D. Ritem sinusal me bllok te deges se majte
E. Takikardi ventrikulare
43. Je ne turn ne urgjence. 4 ditet e fundit ka pas bllokim nga debora. Nje ambulance sjell nje grua te moshes se mesme
ne gjendje te obtunduar. Komshinjte se kane pare 4 ditet e fundit dhe policia e gjeti te pavetedijshme ne toke. Ti ndjen
shume pak pulsin radial. Ti dallon nje graft fistule dialize ne krahun e majte. TA=60/palp dhe EKG eshte e tille. Ti vendos
per defibrilim dhe kerkon nje bilanc biokimik. Cfare anomalie elektrolitike pret te gjesh?
A. Hiperkalcemi
B. Hiperkalemi
C. Hipokalemi
D. Hipomagnezemi
E. Hipofosfatemi
44. Nje grua 71 vjec me kardiomiopati ishemike dhe fraksion ejeksioni te ventrikulit te majte 38% ka qene e
hospitalizuar javen e fundit me diagnozen e insuficiences kardiake akute te dekompensuar. Pas stabilizimit te diurezes
dhe medikamenteve, ajo po ndjehej shume me mire. Ajo eshte ne dozen max te tolerueshme te ACE-I, beta bllokuesve
dhe doze te pershtatshme diuretiku. Ju po planifikoni daljen sot, por infermierja evidenton qe pacientja ka disa rrahje
te shkurta (5-10) te nje takikardie ventrikulare te paqendrueshme dhe PVCs multiple (extrasistola ventrikulare) gjate
nates, pavaresisht se mbetet asimptomatike. Nje student mjekesie pyet nese takikardia ventrikulare e paqendrueshme
(NSVT) ka ndonje rendesi prognostike dhe nese ndonje nderhyrje duhet. Pergjigja me e pershtatshme?
A. NSVT eshte e shpeshte ne pacientet me kardiomiopati dhe nuk ka sinjifikance.
B. NSVT eshte shqetesuese ne kete pacient. DUhet te ndalojme PVCs dhe NSVT me amiodarone.
C. NSVT e lidhur me rritje te vdekshmerise ne pacientet me IK.Refero per defibrilator automatik te implantueshem.
D. NSVT eshte e lidhur me nje rritje te vdekshmerise ne pacientet me IK. Do e stabilizojme me flecainide.
E. NSVT eshte e lidhur me nje rritje te vdekshmerise ne pacientet me IK. Megjithate, supresioni i PVCs dhe NSVT
me barna antiaritmike nuk e ndryshon prognozen.
45. Ju po kudeseni ne ICU kardiake per nje pacient 62 vjec z. Wittstine me hipertension, hyperlipidemi dhe fumator, i
cili kaloi nje infarkt miokardi anterior te gjere para 4 ditesh. Ai erdhi vone ne urgjence dhe pavaresisht nderhyrjes se
shpejte angioplastike dhe stentim te LAD, ekokardiograma sot tregon nje FE 25%. Nuk ka patur aritmi ventrikulare, ka
nje dispne te lehte, rale bilaterale dhe puls venoz jugular te rritur. Ai merr aspirine, clopidogrel, atorvastatin. Te gjitha
terapite e do ulin mortalitetin ne 40 ditet e ardhshme pervec:
A. Defibrilator automatik i implantueshem
B. Eplerenone
C. Lizinopril
D. Metoprolol
E. Te gjitha me lart
46. Nje studente 21 vjec qe studion muzike, i bashkohet nje grupi kori. Kur kendon solo, ajo ndjen palpitacione dhe nje
ndjenje marrje mendsh dhe te fiket. Nuk ka humbur kurre ndergjegjen, por nganjehere duhet te ulet per shkak te
marramendjes. Vjen ne klinike dhe ekzaminimi fizik rezulton normal, edhe EKG ne qetesi po ashtu. Ju i kerkoni te
kendoje nje solo dhe regjistroni nje EKG. Simptomat perseriten dhe ju e dergoni per nje ekokardiografi dhe MRI
kardiake, qe rezultojne normale. Nga se dyshoni qe i vijne palpitacionet?
A. Displazi aritmogjene e ventrikulit te djathte
B. Sindromi Brugada
C. Kardiomiopati hypertrofike
D. Takikardi ventrikulare intrafashikulare sinister
E. Takikardi ventrikulare e traktit dales ventrikular dexter
47. Nje femer 47 vjec, nen trajtim me metadone per nje histori te meparshme abuzimi me substanca narkotike, kalon
nje infeksion te traktit respirator te siperm dhe nje shoqe i jep disa tableta Eritromicin te ngelura. Sot ndjen episode
multiple palpitacionesh dhe marramendje dhe gjendje te fiketi. Ne EKG verehet aritmi e paqendrueshme. Ti
administron 2 mg iv sulfat magnezi, por pa rezultat. Niveli i kaliumit normal Hapi i rradhes me i pershtatshem?

A. Metoprolol 5 mg IV duke e perseritur derisa frekuenca te jete 60/min


B. Amiodarone 150 mg IV
C. Referim urgjent per implantim defibrilatori
D. Infuzion izoproterenol IV te titrueshem deri ne nje fr. 100-120
E. Sedacion dhe defibrilim

48. EKG me takikardi me komplekse te gjera, cila e dhene mbeshtet me shume diagnozen e takikardise venttrikulare?
A. Disociacioni AV
B. Paterni klasik i bllokut te deges se djathte
C. Ritmi crregullisht i crregullt me komplekse QRS te ndryshueshme
D. Zgjatja e QRS>120 msec
E. Ulje e fr. kardiake me masazh te sinusit karotid
49. Djale 18 vjec, pa histori per probleme shendetsore te meparshme, paraqitet per kontroll te pergjithshem para
fillimit te universitetit. Pa te dhena ne ekzaminim objektiv. Ne EKG ritem i crregullt. Hapi tjeter me i pershtatshem?

A. Ekokardiografi
B. Studim elektrofiziologjik dhe ablacion te fokusit atrial ektopik
C. Prova ushtrimore nen EKG
D. Monitor Holter
E. Rivleresim
50. Znj. Hardy, nje triatlete, DM tip 1. Pra 3 ditesh kaloi nje krize ketoacidoze sepse ne farmaci iu dhane insulina te
skaduara. Me rehidrim masiv, zevendesim elektrolitesh dhe insuline, u stabilizua. Po planifikohet dalja neser. Ju shihni
je bradikardi ne monitor, po flen me zhurme. Vendi anatomik i bllokut kardiak?

A. Nyja AV
B. Tufa e Bachmann
C. Tufa e His
D. Dega e majte
E. Nyja SA
51. Nqs do administronit Atropine IV te pacientja e pyetjes 50, cfare do prisnit te ndodhte?
A. Rrit ritmin sinusal dhe mbaj bllokun AV 2:1
B. Rrit ritmin sinusal dhe permireso bllokun kardiak ne percueshmerine 1:1
C. Rrit ritmin sinusal dhe perkeqeso bllokun kardiak mbi 2:1
D. Pa ndryshime ne ritmin sinusal
E. Pa ndryshime
52. Nje pacient 87 vjec me semundjen e Parkinson merr levodopa. Pas mbarimit te ilaceve, nuk i mori prape. Ai vjen
ne urgjence me akutizim te semundjes. Ne vleresimin fillestar, ka kete pamje ne EKG. Hapi tjeter me i pershtatshem?

A. Antikoagulim me heparin IV
B. Kontrollo TSH dhe merr nje histori respiratore
C. Kardioversion direkt
D. Metoprolol 5 mg IV
E. Asnje nderhyrje apo testim tjeter
53. Z. Tillman, 68 vjec me nje histori kardiomiopatie joishemike, hipertension dhe nje atak ishemik tranzitor para 6
muajsh pa nje shkak te qarte. Ne ekzaminim, duket i lodhur. TA=110/78 mmHg dhe frekuenca kardiake 150/min. Venat
jugulare nuk jane te distenduara dhe pulmonet e pastra. EKG e tij ka pamjen e . Ju administroni 5 mg Metoprolol IV
dy here dhe frekuenca kardiake ulet ne 75/min. Ai ndjehet shume me mire. Ju konsultoheni me elektrofiziologun
kardiak per mundesine e ablacionit te situsit aritmik. Hapi tjeter me i pershtatshem?

A. Amiodarone
B. Aspirin
C. Digoxin
D. Furosemide
E. Antikoagulim sistemik
54. Znj. Schoop, nje pronare dyqani qeramike paraqitet per nje vleresim te palpitacioneve. Ka nje EKG si me poshte.
Ajo me shume mundesi ka nje trakt aksesor ne:
A. Septumi anterior
B. Pozicion anterolateral ne ventrikulin sinister
C. Pozicion inferior ne ventrikulin sinister
D. Septum inferior
E. Ventrikuli dexter
55. Nje burre 68 vjec me nje histori infarkt miokardi dhe IK eshte komfort ne qetesi dmth nuk ndjen gje. Por, kur ecen
per te makina ai perjeton dispne, lodhje dhe nganjehere palpitacione. I duhet te pushoje disa minuta qe te qetesohet.
Cili eshte klasifikimi i tij sipas NYHA?
A. Klasa I
B. Klasa II
C. Klasa III
D. Klasa IV
56. Nje femer, 47 vjec, postmenopauze ka nje dispne severe javet e fundit. Nuk referon per dhimbje gjoksi paraprirese,
kolle, sputum, temperature. Referon per edema ne kembe. TA=145/78 mmHg dhe fr.=123/min. Prezente ekzoftalmia,
krepitacione inspiratore bilaterale ne 1/3 inferior te toraksit dhe distendim i venave te qafes. Ka nje ton te 3 kardiak
pa zhurma. Edema bilaterale ne kembe, ekstremitete te ngrohta dhe tremor duarsh fin. Shpjegimi fizpatologjik me i
mundshem per IK ne kete rast?
A. Anemi me nje debit kardiak te rritur
B. HTA kronik sistemik me hipertrofi te ventrikulit te majte dhe disfunksion sistolik
C. Hemokromatoze me kardiomiopati restriktive konsekuente
D. Infarkt miokardi me funksion te rene sistolik te ventrikulit te majte
E. Tirotoksikoze me debik kardiak te rritur
57. Duke iu referuar epidemiologjise dhe prognozes se IK, cila nga theniet e eshte e vertete?
A. Mes pacienteve me IK me FE te ulur, semundja koronare eshte shkaku kryesor.
B. Anemia eshte nje shkak i shpeshte i IK mes pacienteve me nje zemer te shendoshe dhe me strukture normale.
C. Per shkak te zhvillimeve ne teknikat terapeutike me medikamente dhe pajisje, prognoza e te gjithe pacienteve me
IK eshte e shkelqyer, me nje mbijetese>90% per mbi 1 vit pas diagnozes
D. Kardiomiopatia familjare, gjenetike eshte e rralle, megjithate duhet te behet screening per te patur nje diagnoze
dhe trajtim te hershem ne anetaret asimptomatike te prekur.
E. IK me FE te ruajtur perfaqeson nje pakice te popullates totale me IK.
58. Nga perspektive patofiziologjike, te gjitha jane te mbishprehura (upregulated) ne IK me FE te ulur pervec:
A. Angiotenzina II
B. Peptidi natriuretik tip B (BNP)
C. Futja e Ca ne REP
D. Norepinefrine
E. TNF
59. Te gjitha situatat klinike me poshte lidhen me ortopnene pervec:
A. Asciti
B. Obeziteti abdominal
C. Dobesia diafragmatike
D. IK
E. Sindromi hepatopulmonar
60. Nje burre 67 vjec paraqitet ne urgjence me dispne. Iu gjet i larte niveli i BNP. Pervec IK, te gjitha me poshte mund
te shkaktojne rritje te BNP pervec:
A. Mosha e rritur
B. Seksi femer
C. Obeziteti
D. Embolia pulmonare me prekje te zemres se djathte (lodhje)
E. Insuf. Renale
61. Ju po trajtoni nje pacient me cor-pulmonar ne ICU. Fatkeqesisht, ai ka kaluar nje arrest respirator ne shtepi dhe
kerkon nje intubim dhe ventilim mekanik. Aktualisht me nje volum Tidal 500 mL, FiO2=0.4, presion pozitiv ne fund te
ekspirimit=20 mmHg, fr.resp=20, pH=7.4, PCO2=40 mmHg, saturimi=86%. Ju jeni i shqetesuar per mbingarkesen e
ventrikulit te djathte. Te gjitha me poshte mund ta rrisin mbingarkesen ne ventrikulin e djathte pervec:
A. Rritja e FiO2 per te rritur saturimin e O2 arterial ne 95%
B. Rritja e PEEP ne 35 mmHg
C. Rritje e VT ne 750 mL
D. Ulja e fr. respiratore ne 10
62. Z. George është një 52-vjeçar me hipertension të gjatë dhe diabet të kontrolluar keq. Ai paraqet ankesa për disa
muaj me dispne eforti, episode akute të dispnese kur shtrihet dhe edemë te ekstremiteteve të poshtme. Në
ekzaminim, ai ka një puls venoz jugular i ngritur dhe një S4 në auskultim. Ekokardiografia tregon një ventrikul të majtë
me FE prej 55% me një atrium të majtë te dilatuar. Ju dyshoni se ky pacient ka IK me Fe te ruajtur. Medikamente qe
targetojne cilen nga t ka shfaqur ulje te mortalitetit ne pacientet me IK me FE te ruajtur?
A. ACE
B. Receptor i angiotenzines
C. Fosfodiesterza-5
D. Na/K ATP-aza
E. Asnjera
63. Nje grua 64 vjec, me nje histori kardiomiopatie ishemike. Vjen ne urgjence per dispne. Evidentoni qe ka shtuar 11
kg qe nga vizita e fundit ne kardiologji para 2 muajsh. Ekzaminimi objektiv evidenton te dhena qe lidhen me IK te
dekompensuar, duke perfshire ketu rale pulmonare, rritje te presionit venoz jugular, ascit abdominal, edeme ne
anesite inferiore dhe nje vale katrore te presionit te gjakut ne pergjigje te Valsalves. Anesi te ngrohta, TA=110/78,
fr.=75/min, Na=128 mEq/L dhe creatinina=2.5 mg/dL (me nje rritje nga hera e fundit kur ishte 1.2). Radiografia e
toraksit tregon per nje kongjestion alveolar difuz ne perputhje me edemen pulmonare. Hapi tjeter me i pershtatshem?
A. Administro digoxin 250 mikrogram IV
B. Administro furosemide 40 mg IV
C. Vendos nje rruge venoze me permasa te medha dhe pergatit per ultrafiltrim
D. Vendos nje kateter ne arterien pulmonare per monitorim hemodinamik
E. Fillo dobutaminen ne doze 5 mikrogram/kg/min dhe titro ne nje output urinar prej 1 mL/kg/ore
64. Cila e vertete ne lidhje me nesiritide ne pacientet me IK dekompensuar qe kerkon monitorim hemodinamik invaziv?
A. Nesiritide nuk ka efekt sinjifikant ne presionin e gjakut
B. Nesiritide permireson perfuzionin renal ne gjendjet e dekompensimit kardiak
C. Nesiritide eshte i lidhur me nje ulje te rihospitalizimit, por jo vdekjes nga IK e dekompensuar
D. Nesiritide ul presionin ne kapilaret pulmonare (wedge pressure) me shpejt se nitratet IV
E. Asnjera
65. Z. Jones eshte nje mashkull 21 vjec, i cili prezantohet ne urgjence me nje dispne prej disa ditesh dhe letargji nje
jave pas nje infeksioni viral te traktit respirator te siperm. Familjaret e sollen pasi vune re qe ai nuk ngrihej dot nga
kolltuku pa ndihme. TA=88/72 mmHg, fr.kardiake=115/min, saturimi=90%. Ne ekzaminimin objektiv: krepitacione
pulmonare, presion venoz jugular i rritur, ritem galopi S3, anesi te ftohta. Eshte letargjik, pergjigjet pak ndaj pyetjeve.
Creatinina=2.3 mg/dL, BNP e larte, laktatet e rritura. Ekokardiografia evidenton FE te ventrikulit te majte 15% me nje
hipokinezi globale. Ju i filloni dobutamine ne doze 5 mikrogram/kg/min dhe pergatiteni per te vendosur nje kateter
arterial per monitorim hemodinamik. Cili nga parametrat hemodinamike ka me shume mundesi te jete i rrritur?
A. Debitit kardiak
B. Indeksi i forces goditese te ventrikulit te majte
C. Saturimi mix i oksigjenit venoz
D. Hedhja kardiake
E. Rezistenca vaskulare sistemike
66. Cili nga medikamentet e ka treguar se ul mortalitetin ne IK me FE te ulur?
A. Bucindolol
B. Nebivolol
C. Suksinati i metoprololit
D. Tartrati i metoprololit
E. Te gjitha me lart
67. Ju po vleresoni nje pacient me kardiomiopati joishemike me FE 35% te ventrikulit te majte, te diagnostikuar kohet
e fundit. Eshte euvolemik dhe NYHA klasa e I. TA=120/78 mmHg, fr.=80/min. Do i nisesh nje beta bllokues dhe nje ACE-
I ne doza optimale. Cili rregull eshte me i pershtatshem ne nisjen dhe titrimin e dozes per keto medikamente?
A. Filloji te dyja ne doze te ulet. Titro secilin cdo 2-4 jave ne dozen me te larte te tolerueshme.
B. Fillo ACE-I dhe titro ne dozen me te larte te tolerueshme, pastaj nis beta bllokuesin dhe titro ne dozen me te larte
te tolerueshme.
C. Fillo ACE-I ne doze te ulet, pastaj beta bllokues ne doze te ulet. Alterno titrimin per secilin ne dozen me te larte te
tolerueshme
D. Fillo beta bllokuesin dhe titroje ne dozen me te larte te tolerueshme dhe pastaj bej njesoj me ACE-I.
E. Strategjia specifike nuk ka aq rëndësi sa qëllimi i fillimit të frenuesit ACE dhe β-bllokues në kohën e duhur dhe
duke i titruar të dyja në dozat e tyre optimale, më të tolerueshme.
68. Ti po vlereson nje femer, 72 vjec me nje histori te kardiomiopatise ishemike dhe FE te ventrikulit te majte te 35%.
Ajo eshte NYHA klasa II ne raport me simptomat. Po trajtohet me lizinopril 40 mg ne dite, karvedilol 25 mg 2 here ne
dite, spironolaktone 25 mg ne dite. TA=102/82 mHg, fr.kardiake=60/min. Cili nga medikamentet e meposhtem shtese
mund te ule mortalitetin te ky pacient?
A. Aliskiren
B. Digoxin
C. Ivabradine
D. Valsartan
E. Asnjera
69. Ti po ndjek nje pacient 57 vjec, i cili ka kaluar nje infarkt miokardi anterior te gjere para 8 muajsh. Pavaresisht
ardhjes vone ne spital (>5h nga fillimi i dhimbjes se gjoksit), ai iu nenshtrua nje nderhyrjeje primare koronare me nje
rezultat te shkelqyer angiografik. Pas qendrimit ne spital, eshte ndjekur rregullisht. Fr=65/min, TA=104/82. Duket
shume mire, nuk referon asnje simptome, as ne efort. Merr keto medikamente: Aspirin, Klopidogrel, Atorvastatin,
Karvedilol dhe Valsartan. EKG tregon ritem sinusal, me bllok te deges se djathte dhe kohezgjatje te QRS 135 msec.
Ekokardiografia tregon nje FE 25% me nje zone te gjere diskinezie anteriore, ne perputhje me nje aneurizem. Cila nga
terapite e eshte e indikuar per te ulur mortalitetin ne kete pacient?
A. Defibrilator kardioversioni i implantueshem
B. Defibrilator kardioversioni i implantueshem me risinkronizim kardiak (lidhja e sinusit koronar)
C. Ivabradine
D. Rimodelim ventrikular kirurgjikal
E. Asnjera
70. Ti po vlereson nje burre 25 vjec me nje histori kardiomiopatie joishemike, i cili iu nenshtrua nje transplanti kardiak
ortotopik para 18 muajsh. Vjen per nje vizite rutine. TA=128/82, fr=90. EKG tregon ritem sinusal me nje interval PR
normal dhe aks vale P. Ne te gjitha kontrollet e bera, ju vini re qe fr.kardiake ne qetesi ka qene rreth 90/min. Ai ndihet
mire. Cili eshte hapi me i pershtatshem per fr. Kardiake te rritur ne qetesi?
A. EKG ne prove ushtrimore per vleresuar kompetencen kronotropike (dmth sa e afte te rrise fr. ne prgj te ushtrimit)
B. Fillo Diltiazem 120 mg 2 here ne dite
C. Fillo tartrat metoprololi 25 mg 2 here ne dite
D. Rivleresim
E. Referoje per studime elektrofiziologjike per vleresim te takikardise atriale ektopike
71. Ju jeni emëruar në një pozicion këshilluese me komitetin e qeverisë për organet transplantimi. Kryetari i komitetit
është kurioz për miratimin e nje strategjie kombetare per transplantin kardiak, ku vetem pozicioni dhe koha e pritjes
ne liste do te percaktonte rradhen. Ju keshilloni qe kjo strategji nuk eshte e frytshme sepse:
A. Grupet e gjakut janë grupuar në mënyrë jorandom në shkallë vendi, duke përjashtuar ndarjen vërtetë gjithë vendin
B. Rajonet e ndryshme kanë kritere shumë të ndryshueshme per percaktimin e ashpersisie se semundjes
C. Struktura financiare e rimbursimit / pagesës në të gjithë vendin nuk do ta lejonte atë
D. Ekziston një kufi fiziologjik për kohën "e ftohtë ishemike" (koha jashtë trupit) për zemrat e mbledhura që do të
përjashtonte ndarjen në të gjithë vendin
E. Asnjë nga sa më sipër nuk është e vërtetë
72. Nje mashkull 28 vjec vjen ne urgjence me dispne ne efort. Ka bere nje transplant ortotopik per kardiomiopati
joishemike 5 vite me pare. Pergjithesisht ka shkuar mire, por ka patur nje riaktivizim te citomegalovirusit brenda vitit
te pare. Ai ka vene re se me uljen e aktivitetit eshte reduktuar edhe dispnea. Ai eshte i prere kur thote se nuk ka
perjetuar dhimbje gjoksi ose presion gjate ketyre episodeve. Eshte kompliant ne trajtimin me takrolimus,
mykofenolate mofetil dhe doza te uleta te prednizonit. Ekokardiografia tregon nje funksion normal te VM dhe trashesi
normale te VM. EKG ne qetesi tregon nje ritem sinusal me nje fr=80/min. Cila nga t eshte shkaku me i mundshem i
simptomave te tij?
A. Refuzimi i mediuar nga antitrupat
B. Refuzimi qelizor
C. Semundja koronare
D. Endokardit
E. Efekt anesor i medikamenteve
73. Ju po vizitoni Znj. Block ne kliniken e insuficiences kardiake. Ajo eshte nje grua 68 vjec me nje histori limfome me
qeliza B, difuze, te medha, e trajtuar me sukses me kemoterapi dhe transplant i PK para 15 vitesh. Ajo kaloi nje
kardiomiopati te lidhur me kemoterapine dhe tani ka nje FE=15%. Eshte hospitalizuar 4 here ne 6 muajt e fundit me IK
te dekompensuar. Eshte NYHA klasa III-IV sipas simptomave. Se fundmi, iu nenshtrua nje prove ushtrimore
kardiopulmonare duke treguar nje konsum max te oksigjenit=9 mL/kg/min. Eshte ne terapi mjekesore optimale. U
vleresua se nuk eshte nje kandidate e pershtatshme per transplant ortotopik te zemres per shkak te moshes dhe
malinjitetit te meparshem. Ajo pyet cfare opsionesh te tjera jane te gatshme per te, pasi prioriteti nr.1 i saj eshte te
shohe nipin te diplomohet. Cila nga t eshte nje pergjigje e pershtatshme?
A. Infuzion i vazhdueshem i Milrinone do permiresoje funksionin kardiak dhe mbijetesen.
B. Nje pajisje asistuese ne VM permiresoje cilesine e jetes, por s`ka avantazh mbijetese ne raport me medikament
C. Nje pajisje asistuese ne VM, mund te rrise mbijetesen mbi 2-5 vite me teper sesa terapia medikamentoze.
D. Implantimi i qelizave stem cell eshte treguar te zgjase jeten ne paciente me kardiomiopati stadi i fundit.
E. Nuk ka opsione te tjera per te zgjatur jeten dhe ne duhet te konsiderojme masat paliative ne te ardhmen.
74. Cili nga pohimet ne lidhje me epidemiologjine e semundjeve kongenitale kardiake (CHD) ne SHBA eshte i vertete?
A. CHD mbeten te rralla, ne 0.1% te gjitha lindjeve te gjalla.
B. Duke i dhene perparesi kirurgjikas e kujdesit pre dhe postnatal, mbijetesa te neonatet me CHD ka arritur 90%.
C. Duke pasur parasysh ritmet në rënie të shtatzënisë në gratë me CHD, incidenca e CHD në të sapolindur në rënie
D. Popullata e adulteve me CHD ne renie, duke patur parasysh perpjekjet per te permiresuar screening prenatal.
E. Grate me CHD nuk jane ne risk te rritur per komplikacione gjate barres ne raport me popullaten normale.
75. Nje burre 62 vjec po vleresohet per kirurgji te zevendesimit te valvules mitrale per arsye te regurgitimit mitral. I
nenshtrohet nje ekokardiograme transezofageale e cila demonstron nje defekt te vogel te rrjedhjes djathtas-majtas
te fluksit ne Doppler gjate sistoles, permes septumit atrial. Defekti eshte i lokalizuar ne mes te septumit dhe dhe
ndodh kur ka nje hapje te vogel te nje pjese indi <1 mm. Nuk ka fluks diastolik, as nje hapje te dukshme ne ndonje
pjese te septumit gjate diastoles. Cila nga patologjite shpjegon me mire gjetjet ne ekokardiografi?
A. Defekt atrial tip ostium primum
B. Defekt atrial septal tip ostium secundum
C. Kthim venoz pulmonar parcial anormal
D. Foramen ovale persistent
E. Defekt atrial septal tip sinus venosum
76. Ju po vleresoni nje mashkull 21 vjec ne klinike. Ai here pas here ndjehet i lodhur, por ne pergjithesi asimptomatik.
Ju e pate per here te pare 6 jave me pare dhe evidentuat nje zhurme holosistolike e ashper ne kufirin sternal te
poshtem te majte qe shtohet me kapje dore. Duke dyshuar nje DIV, ju e referoni per ekokardiografi, e cila konfirmon
dyshimin tuaj, nje defekt i septumit interventricular 5 mm, tipi muskulor. Me tej, kateterizimi i zemres se djathte zbuloi
nje presion mesatar ne arterien pulmonare rreth 20 mmHg, rezistenca venoze pulmonare 2 Wood units dhe rezistenca
vaskulare sistemike 6 Wood units. Nga matjet seriale tsatO2 venoz ne venat centrale dhe zemren e djathte, debit
kardiak zemres se djathte=7.5 L/min dhe debit kardiak sistemik=6 L/min. Cilen rruge terapeutike rekomandoni?
A. Prova ushtrimore kardiopulmonare per te percaktuar konsumin max te oksigjenit
B. Mbyllje me nje pajisje ne rruge perkutane
C. Konsidero transplant zemre dhe mushkerie
D. Jo nderhyrje tani; kontrolle vjetore kardiologjike nese shfaqen simptoma te reja
E. Korrigjim kirurgjikal permes kirurgjise me zemer te hapur dhe riparim i patch
77. Ju po vleresoni nje pacient 52 vjec, i cili vjen ne urgjence me dhimbje gjoksi dhe enzima kardiake te rritura. Ne
auskultacion, ju mund te degjoni nje zhurme te vazhdueshme, siperfaqesore ne nivel te mesit te sternumit, e
padokumentuar me pare. Ekokardiografia eshte e limituar ne ate cka ofron, por arrin te vizualizoje te gjitha valvulat
kardiake, qe duken totalisht normale. Pacienti i nenshtrohet nje kateterizimi te zemres se djathte. Saturimi i oksigjenit
tregohet me poshte. Cila eshte anomalia pergjegjese per simptomat ne kete rast?

A. Anomali ne raportet mes arteries koronare sinister dhe arteries pulmonare


B. Defekt atrial septal
C. Fistula arteriovenoze koronare ne sinusin koronar
D. Duktus arteriousus persisitent
E. Defekt i septumit ventrikular
78. Z. Jenson, nje burre 40 vjec me bikuspidi te valvules se aortes, te cilin ju po e ndiqni prej me shume se nje dekade.
Ju e keni ndjekur me ekokardiograme cdo vit per te pare progresin e semundjes se tij, duke ditur qe valvulat bikuspide
shpesh zhvillojne stenoza ose regurgitime, qe kerkojne zevendesim ne moshe te mesme. Duke patur parasysh
anomaline kongenitale specifike, cilat struktura anatomike ndjekur ne ekokardiograme cdo dy vite?
A. Madhesia e rrenjes se aortes
B. Madhesia e atriumit te majte
C. Presionet ne arterie pulmonare
D. Funksioni i valvules pulmonare
E. Regurgitim trikuspidal
79. Te gjitha t jane karakteristika klasike te tetrades Fallot pervec:
A. Obstruksion i fluksit nga ventrikuli i djathte
B. Shalimi i aortes
C. Hipertrofi e ventrikulit te djathte
D. Atrezi trikuspidale
E. Defekt septal ventrikular
80. Nje burre 78 vjec po vleresohet per nje dispne qe fillon ne efort. Ka nje histori te gjate te abuzimit me duhanin,
obezitet, diabet mellitus. Medikamentet qe merr aktualisht jane metformin, aspirin, nganjehere ibuprofen. Ne
ekzaminim fizik, pulset periferike tregojne nje vonese ne pikun e tyre, ka dhe kontraksione te fuqishme, nje force
goditese e madhe proeminente e ventrikulit te majte. Ka nje ritem te rregullt, nje zhurme mid-sistolike IV/VI, e larte
ne bazen e zemres, perhapet ne arteriet karotide. Toni i katert i zemres prezent. Ekokardiografia konfirmon nje stenoze
aorte severe pa lezione te tjera valvulare. Cila nga t ka me shume mundesi te kete kontribuar ne zhvillimin e ketij
lezioni kardiak?
A. Bikuspidi kongenitale e valvules se aortes
B. Diabeti mellitus
C. Semundja reumatike e zemres okulte
D. Semundje e indit lidhor
E. Asnjera
81. Nje burre 63 vjec, prezantohet me sinkop ne efort, ka filluar kohet e fundit. Eshte diagnostikuar me stenoze aorte
severe. Ju i tregoni se rekomandimet terapeutike tuaja jane te bazuara ne nje observim ku pacientet e patrajtuar me
kete prezantim klinik kane nje jetegjatesi mesatare te parashikuar rreth:
A. 5 vite
B. 4 vite
C. 3 vite
D. 2 vite
E. 1 vit
82. Z. Belliard eshte nje burre 82 vjec, qe e keni ndjekur ne klinike. Eshte vizituar para 3 vitesh, ku beri nje gjendje
sinkopale te rastesishme. Me tej u konstatua nje stenoze aorte e kalcifikuar me nje siperfaqe valvulare 0.7 cm 2 dhe
gradient mesatar 45 mmHg. Ju rekomandoni zevendesim kirurgjikal te valvules se aortes. Diten e operimit, pacienti
ndryshon mendje dhe thote se do rikthehet kur te jete gati. Ai nuk ka bere me vizita mjekesore. Sot, ai paraqitet ne
urgjence pas disa javesh me letargji dhe dispne. TA=82/68 mmHg, fr=110/min, ne EKG ritem sinusal. Pulsi karotid i
dobet dhe i vonuar, anesi te ftohta ne prekje. Ne auskultacion ju evidentoni nje ton S4 dhe zhurme sistolike III/IV late
peaking (mbase telesistolike). Funksioni renal dhe hepatik te demtuar, diureza e ulur. Kardiokirurgu on-call e shtyn
proceduren kirurgjikale te zvnd valvular per shkak te prekjes akute renale dhe demtimit hepatik. Cila nga t do ishte
nje opsion terapeutik i pershtatashem per te permiresuar perfuzion ne short-term per pacientin, ne menyre qe te
behet i mundur zvnd valvular.
A. Atorvastatin 80 mg
B. Digoxin 250 mikrogram IV nje here dhe 125 mikrogram orale cdo dite me pas.
C. Metoprolol 5 mg IV ne doza te vazhdueshme per te arritur nje target te fr.kardiake 60-70/min
D. Valvuloplastike perkutane me ballon aortik
E. Infuzion i vazhdueshem IV i Fenilefrines te titruar deri ne nje presion arterial mesatar >65
83. Nje avokat ne pension, 85 vjec paraqitet me nje dispne eforti progresive dhe edema te anesive inferiore. Ne
ekzaminim, ju konstatoni nje zhvendosje te pikes max te impulsit dhe ritem galopi S4. Ka nje zhurme sistolike III/IV ne
baze, e cila perhapet ne karotide. Ekokardiografia transtorakale evidenton nje FE te VM 25% me hipokinezi globale.
Siperfaqja valvulare e llogaritur eshte 0.8 cm2 dhe gradienti mesatar 25 mmHg. Cili eshte hapi tjeter me i pershtatshem
per te percaktuar nese ky pacient do te mund te perfitoje nga zevendesimi i valvules se aortes?
A. MRI per te vleresuar fibrozen ventrikulare dhe morfologjine e valvules se aortes
B. PET per te percaktuar qendrueshmerine ventrikulare
C. Koronarografi per te percaktuar prezencen e semundjes obstruktive koronare
D. Ekokardiografi e stresit me Dobutamine
E. Kateterizim i zemres se djathte per te percaktuar debitin kardiak dhe presionet e mbushjes
84. Cili nga parametrat e meposhtem eshte zakonisht i reduktuar ne regurgitimin aotral kronik te rende?
A. Presioni diastolik
B. Pasngarkesa e ventrikulit te majte
C. Diametri i ventrikulit te majte
D. Parangarkesa e ventrikulit te majte
E. Volumi hedhes total i ventrikulit te majte
85. Ju po kujdeseni per nje hyrje te re ne njesine e kujdesit intensiv kardiak, nje grua 21 vjece me semundje te indit
lidhor. Ajo u paraqit me marrje fryme akute dhe nje radiografi te gjoksit qe tregon edeme pulmonare difuze.
Ekzaminimi fizik tregoi nje zhurme diastolike te hershme, te bute, te shkurter ne kufirin sternal te siperm te djathte,
dhe ekokardiografia tregoi regurgitim aortik te rende me kuspis koronar te djathte te rupturuar. CT nuk tregoi
disekacion te aortes. Ajo eshte e intubuar dhe e seduar. Presioni i gjakut eshte 110/50 dhe FK 115/min. Outputi urinar
eshte i ulet dhe ekstremitetet te ftohta. Ekipi i kirurgjise kardiotorakale eshte i zene me nje transplant zemre dhe nuk
do te jete e mundur ta marre kete pacient per te pakten 4 ore. Cila nderhyrje ka me shume gjasa te ndihmoje mbajtjen
e perfuzionit te organeve te saj deri ne nderhyrjen kirurgjikale?
A. Infuzioni i vazhdueshem me Esmolol iv i titruar per nje frekuence 60-70/min.
B. Pompe me balon intra aortale
C. Nitroprusid IV
D. Norepinefrine IV
E. Vazopresine IV
86. Cili eshte shkaku me i shpeshte i obstruksionit te fluksit qe vjen ne ventrikulin e majte?
A. Semundja kongenitale e valvules mitrale
B. Cor triatriatum
C. Endokarditi infektiv me vegjetacione te medha ne valvulen mitrale
D. Kalcifikime anulare mitrale
E. Semundja reumatizmale mitrale
87. Ne rastet e stenozes mitrale te rende, cili nga parametrat e meposhtem, eshte zakonisht i rritur?
A. Hedhja kardiake
B. Presioni ne atriumin e majte
C. Diametri i venrtikulit te majte
D. Presioni fund-diastolik ne ventrikulin e majte
E. Komplianca vaskulare pulmonare

88. Po kujdesesh per nje grua 42 vjec me histori te meparshme per ethe reumatizmale qe kane shkaktuar stenoze
mitrale. Semundja valvulare momentalisht eshte e moderuar. Ju e dini qe stenoza mitrale shkakton rritje te presionit
ne atriumin e majte, cka me kalimin e kohes mund te shkaktoje edeme pulmonare kardiogjene dhe hypertension
pulmonary. Te gjitha t mund te shkaktojne rritje te presionit ne atriumin e majte dhe perkeqesim te mundshem te
funksionit pulmonary, pervec:
A. Anemia
B. Isoproterenol
C. Metoprolol
D. Shtatzania
E. Vrapimi ne piste
89. Zonja Beam paraqitet ne departamentin e urgjences me perkeqesim akut te veshtiresise ne frymemarrje. Ajo eshte
nje grua 84 vjec me stenoze mitrale te rende qe eshte planifikuar per valvotomi mitrale perkutane me balon pas 3
ditesh. Megjithate, sot ndersa po gatuante sallate me pule, ajo verejti nje dobesi derrmuese dhe dispne. Ne vleresim,
ajo duket dispneike dhe me nje shqetesim te lehte. Saturimi i oksigjenit ne ajrin e dhomes eshte 91%, frekuenca
kardiake 55/minute dhe PA 110/80 mmHg. Ajo ka rale bilaterale ne fushat e mesme pulmonare. Ju vini re se ritmi i saj
eshte i crregullt dhe EKG konfirmon vendosjen e nje fibrilacioni atrial te ri. Ju dyshoni se ajo ka nje presion shume te
larte te artiumit te majte qe i ka shkaktuar edeme pulmonare. Te gjitha nderhyrjet e terapeutike do te ndihmojne ne
uljen e presionit ne atriumin e majte, pervec:
A. Furosemide IV
B. Valvotomia mitrale perkutane me balon
C. Kardioversion i sinkronizuar
D. Vendosje transvenoze e pacemakerit dhe pejsimi ne nje frekuence 90/min
E. Te gjitha te mesipermet do te ndihmojne uljen e presionit ne atriumin e majte

90. Nje burre 34 vjec me stenoze mitrale reumatizmale eshte sjelle tek ju per vleresim. Atij i pelqen te luajte futboll ne
kohen e lire dhe nuk ka kufizime ose simptoma. Frekuenca kardiake eshte 65/min ne qetesi. Ekokardiografia
transtorakale tregon nje madhesi dhe funksion normal te ventrikulit te majte, nje atrium te majte mesatarisht te
zgjeruar, nje siperfaqe te valvules mitrale 1.7 cm² me flete relativisht te holla te pakalcifikuara. EKG tregon zmadhim
te atriumit te majte dhe ritem sinusal. Gjate testimit ne effort, prsioni sistolik i llogaritur i arteries pulmonare ne
ushtrimin maksimal eshte 40 mmHg. Cilin nga planet e te trajtimit rekomandoni?
A. Metoprolol 25 mg nga goja 2 here ne dite
B. Valvotomi mitrale perkutane me balon
C. Vleresime periodike kardiologjike dhe monitorim me ekokardiografi
D. Sildenafil 20 mg 2 here ne dite
E. Zevendesim kirurgjikal i valvules mitrale

91. Ju po vleresoni nje burre 65 vjec qe ka nje histori 15 vjecare te kardiomiopatise joishemike me ventrikulin e majte
te dilatuar dhe fraksion ejeksioni 15%. Ekokardiogramat vjetore gjate 5 viteve te fundit kane treguar regurgitim mitral
te rende. Me terapi mjekesore optimale, pacienti ka simptomat NYHA, klasa II. Sot ai pyet nese valvula e tij duhet te
korrigjohet per te zgjatur mbijetesen. Cfare duhet ti thoni ju?
A. “Nqs do te kishit presion te larte te arteries pulmonare ose do te zhvillonit fibrilacion atrial te ri, do te benim
riparimin e valvules.”
B. “Ne paciente si ju, riparimi i valvules nuk ka treguar kurre zgjatje te mbijeteses.”
C. “Duhet ta konsiderojme kirurgjine valvulare vetem nqs riparimi i saj eshte i mundur. Zevendesimi nuk do ta
permiresonte mbijetesen tuaj.”
D. “Nderkohe qe metodat kirurgjikale nuk kane treguar perfitim ne mbijetese, riparimi perkutan i valvules mitrale ka
treguar nje reduktim te mortalitetit ne paciente si ju.”
E. “Po, valvula juaj duhet te ishte korrigjuar vite me pare.”

92. Ju po performoni nje kateterizim te zemres se djathte dhe te majte ne nje pacient me stenoze te dy valvulave. Cilat
valvula jane te perfshira bazuar ne te dhenat e presioneve?
Atriumi i djathte 15 mmHg
Ventrikuli i djathte 25/6 mmHg
Arteria pulmonare 25/12 mmHg
Pulmonary arterial wedge pressure 12 mmHg
Ventrikuli i majte 105/6 mmHg
Aorta 105/75 mmHg

A. Aortale dhe mitrale


B. Aortale dhe trikuspidale
C. Mitrale dhe pulmonare
D. Mitrale dhe trikuspidale
E. Pulmonare dhe trikuspidale

93. Ju po vleresoni nje grua 50 vjec me hypertension arterial pulmonar idiopatik. Ekokardiograma transtorakale e
fundit ka treguar regurgitim trikuspidal te rende, ventrikul te djathte hipokinetik te dilatuar dhe presione sistolike te
arteries pulmonare qe i kalojne 70 mmHg. Ne ekzaminim, ka edema te ekstremiteteve te posthme, hepatomegaly me
hepar pulsues, rritje te pulsit jugular me vale c-v te dukshme, nje vale y zbritese prominente dhe nje kercim te
ventrikulit te djathte (right ventricular heave). Ajo raporton per marrje frymene sforcim mesatar. Cili eshte trajtimi me
i mire per regurgitimin trikuspidal te rende te saj?
A. Diuretike dhe kufizim i kripes, shoqeruar me terapi medikamentoze qe targeton presionin e larte te arteries
pulmonare
B. Valvotomi perkutane me balon
C. Riparim perkutan i valvules trikuspidale
D. Zevendesim kirurgjikal i valvules mitrale
E. Riparim kirurgjikal i valvules trikuspidale
94. Ju po shikoni nje grua 21 vjec per here te pare ne kliniken e kujdesit primar. Ajo nuk ka qene kurre me pare te
mjeku sepse prinderit e saj nuk besojne ne mjekesine perendimore. Ne anamneze, ajo thote se ndihet here pas here
e lodhur dhe nuk ndihet njesoj me bashkemoshataret e saj ne klasat e edukimit fizik. Ne ekzaminim, ju vini re nje
zhurme sistolike ne hapesiren e dyte interkostale te majte, te paraprire nga nje klik presistolik. Ekokardiograma
transtorakale konfirmon pranine e stenozes pulmonare me nje gradient maksimal 60 mmHg dhe doming (ngritje si
kube?) te valvules pulmonare pa regurgitim pulmonar. Cili eshte opsioni me i mire per trajtimin e saj?
A. Diuretike dhe kufizim i kripes
B. Nuk ka nevoje per terapi
C. Valvotomi perkutane me balon
D. Zevendesim perkutan i valvules pulmonare
E. Zevendesim kirurgjikal i valvules pulmonare.

95. Ju po kujdeseni per nje pacient 77 vjec me stenoze aortale te rende ne njesine e kujdesit intensiv. Zevendesimi
kirurgjikal i valvules aortale eshte planifikuar per neser. Megjithate, papritur ai ka veshtiresi te rende ne frymemarrje
dhe manifeston shenja te edemes pulmonare akute. Ne auskultim ju vleresoni tashme nje zhurme te bute te shkurter
sistolike apikale (pervec zhurmes se meparshme te stenozes se aortes), qe nuk ka qene e pranishme me pare. Ju
dyshoni se ai ka vuajtur nje rupture ne kordave te valvules mitrale dhe tani ka regurgitim mitral akut te rende. Cili nga
parametrat do te rritet per shkak te regutgitimit mitral te ri te rende?
A. Gradienti i valvules aortale
B. Siperfaqja e llogaritur e valvules aortale
C. Volumi efektiv i hedhjes
D. Fraksioni i ejeksionit
E. Pasngarkesa e ventrikulit te majte

96. Z. Milsap eshte nje nga pacientet tuaj per nje kohe te gjate ne klinike, qe ka nje histori per semundje reumatizmale
te zemres. Ekokardiograma e fundit tregoi nje gradient te valvules mitrale 11 mmHg, me siperfaqe te llogaritur te
valvules 1.3 cm2, me frekuence kardiake 60/min. Ai paraqitet sot duke u ankuar per veshtiresi ne frymemarrje te
perkeqesuar dhe EKG tregon fibrilacion atrial me frekuence 60/min. Nuk ka pasur kurre episode hemorragjie dhe kishte
vlera normale ne analizat e gjakut para 2 javesh. Cili nga opsionet e per profilaksine e tromboembolise eshte i
pershtatshem? Mr. Milsap is one of your longstanding clinic patients who has a history of rheumatic heart
A. Apixaban
B. Dabigatran
C. Rivaroxaban
D. Warfarin
E. Nevojitet me shume informacion para se te filloj profilaksine e tromboembolise.

97. Te gjithe te jane faktore risku per zhvillimin e kardiomiopatise peripartum, pervec:
A. Mosha maternale e avancuar
B. Kequshqyerja
C. Primipariteti
D. Shtatzania me binjake
E. Perdorimi i tokolitikeve

98. Ju po vleresoni nje pacient te ri ne klinike. Pacienti 25 vjecar eshte diagnostikuar me insuficience kardiake ne nje
shtet tjeter dhe me pas eshte zhvendosur. Ai ka simptoma NYHA klasa II dhe thote se nuk ka pasur angine. Paraqitet
per vleresim dhe menaxhim. Ne anamneze, pacienti ka qene ne karrige me rrota per shume vite dhe ka skolioze te
rende. Nuk ka histori familjare per hiperlipidemi. Ne ekzaminimin fizik, zbulohen krepitanca bilaterale ne mushkeri,
nje S2 dhe nuk ka cianoze. Behet nje EKG ne klinike dhe tregon vale R te larta ne V1 dhe V2 dhe Q te thella ne V5 dhe
V6. Nje ekokardiograme tregon disfunksion te rende global te ventrikulit te majte me fraksion ejeksioni te reduktuar.
Cila eshte diagnoza me e mundshme?
A. Skleroza amiotrofike laterale
B. Defect i septumit atrial
C. Semundje tromboembolike kronike
D. Distrofia muskulare e Duchene
E. Kardiomiopatia ishemike

99. Cili nga te eshte modeli me i shpeshte i trashegimise per kardiomiopatite familjare?
A. Autozomal dominant i shkaktuar nga dyfishimi i ekzoneve
B. Autozomal dominant nga mutacionet missense
C. Autozomal recesiv nga delecioni i ekzoneve
D. Autozomal recesiv nga dyfishimi i ekzoneve
E. I lidhur me X nga delecioni i ekzoneve

100. Nje burre 45 vjec me histori obeziteti vjen ne departamentin e urgjences me dispne, lodhje dhe nje kolle nocturne
qe eshte perkeqesuar muajt e fundit. Ai mohon te kete pasur dhimbje ose presion ne gjoks ne qetesi apo effort. Ne
vleresim, ai ka shenja te kardiomegalise me PMI (point of maximal impulse) te zhvendosur dhe presione te rritura te
mbushjes me rale pulmonare bilaterale dhe puls venoz jugular te rritur. Ekokardiografia tregon nje fraksion ejeksioni
te ventrikulit te majte te ulur, 24% me ventrikul te majte te dilatuar. Cili nga testet e eshte nje rekomandim i nivelit
te pare per vleresimin e metejshem?
A. RM zemre
B. Angiografi koronare
C. ERS (Erythrocyte sedimentation rate)
D. Hekuri ne serum dhe saturimi i transferines
E. Niveli i TSH

101. Nje student 22 vjec pa histori te meparshme mjekesore, eshte paraqitur para 3 ditesh ne urgjence per coryza,
mialgji dhe ethe, cka ishte e zakonshme per semundjen virale respiratore te siperme qe ishte perhapur ne kampus. I
eshte dhene mjekim per qetesimin e kolles dhe antipiretike dhe u keshillua te qendroje i hidratuar. Sot vjen ne urgjence
me letargji dhe lodhje. Ka frekuence kardiake 120/min dhe PA 78/62 mmHg. Ekstremitetet jane te ftohta dhe pulsi
venoz jugular eshte i rritur prane mandibules. Auskultimi precordial shfaq tinguj kardiake shume te qete, nje gallop S3
dhe nje zhurme te bute te regurgitimit mitral. Ekokargiograma transtorakale urgjente nuk tregon efuzion pericardial,
tregon nje ventrikul te majte te padilatuar me fraksion ejeksioni 30% dhe regurgitim mitral te lehte. Biopsia
endomiokardiale tregon miokardit limfocitar. Cili nga pohimet i vertet per prognozen dhe terapine e ketij pacienti?
A. Shansi i tij per mbijetese eshte 10% pa transplant kardiak. Eshte i justifikuar listimi urgjent per transplant.
B. Shansi i tij per mbijetese eshte >50%, me shume paciente te ngjashem qe kane pasurrikuperim te plote ne funks
ventrikular gjate javeve/ muajve vijues. Suporti hemodinamik agresiv farmakologjik e mekanik i justifikuar.
C. Imunosupresioni me steroide sistemike me doze te larte do te rrise shanset per mbijetese.
D. Prezenca dhe titri i antitrupave kunder zemres (anti-heart antibodies) mund te ofroje informacion prognostik

102. Ju po vleresoni nje grua 42 vjec me histori te tiroiditit Hashimoro para shume vitesh qe u trajtua me sukses me
jod radioaktiv. Ajo vjen ne urgjence pasi ka humbur ndjenjat ne shtepi dhe raporton se per disa dite ka pasur nje letargji
qe perkeqesohej dhe discomfort ne gjoks. Ajo ka veshtiresi ne frymemarrje, me frekuence kardiake ne qetesi 110/min
dhe PA 77/62 mmHg. Ekstremitetet jane te ftohta dhe duket e pergjumur. Laktati total ne gjak eshte i rritur dhe outputi
urinar per 2 oret e para te qendrimit eshte minimal. Ne monitorin kardiak ajo ne menyre te perseritur ka nje bresheri
(salvos) te takikardise ventrikulare te paqendrueshme. Ekokardiograma tregon nje fraksion ejeksioni te ventrikulit te
majte 15% dhe biopsia urgjente endomiokardiale tregon lezione difuze granulomatoze te rrethuara nga infiltrat
inflamator i gjere. Cili nga pohimet e eshte i vertete ne lidhje me diagnozen e saj?
A. Shumica e pacienteve me kete etiologji te kardiomiopatise sherohen me trajtim mbeshtetes
B. Steroidet jane shume efektive ne trajtimin e kesaj forme te kardiomiopatise
C. Ecuria e kardiomiopatie e tmerrshme (dire), shpesh me perkeqesim te shpejte qe kerkon transplant urgjent.
D. Takiaritmite ventrikulare jane te rralla ne kete semundje.
E. Asnje nga te mesipermet nuk eshte e vertete.

103. Te gjitha lidhen me nje risk te shtuar per zhvillimin e kardiomioparise gjate kimioterapise me antracikline pervec:
A. Rrezatimi shoqerues i gjoksit
B. Doza e larte totale e antraciklines
C. Semundje kardiake preekzistuese
D. Administrimi i Trastuzumabit
E. Te gjitha te mesipermet lidhen me risk te shtuar te kardiomiopatise nga antraciklinat

104. Z. Kia eshte nje kitarist 32 vjecar, pa histori te meparshme mjekesore. Ai paraqitet me dhimbje gjoksi pasi ka
vuajtur nga coryza, kolla, ethet dhe dhimbje muskulore gjate javes se kaluar. Ai e pershkruan dhimbjen si konstante
dhe qe radiohet ne shpatullen e majte. Perkeqesohet nga qendrimi shtrire dhe frymemarrjet e thella. Ekzaminimi
zbulon nje zhurme ekstrakardiake te mprehte prezente ne 3 komponente per cdo rrahje zemre. EKG tregohet ne figure.
Nivelet e troponines I jane te pazbulueshme ne prezantim dhe pas 6 oresh. Presioni arterial, frekuenca kardiake dhe
oksigjenimi jane normal. Cili eshte hapi i rradhes me i pershtatshem?

A. Aspirin 81 mg ne dite, metoprolol 25 mg 2 here ne dite dhe atorvastatin 80 mg ne dite


B. Aspirin 1 g cdo 8 ore me omeprazol 20 mg ne dite
C. Heparin, aspirin, clopidogrel, dhe angiografi koronare e menjehershme
D. Prednisone 40 mg ne dite per 2 jave e ndjekur nga nje reduktim gradual i dozes gjate 2 muajve pasues
E. Ekokardiografi transtorakale
105. Po kujdeseni per nje burre 45 vjec ne njesine e kujdesit intensiv. Ai u paraqit me dhimbje gjoksi dhe fillimisht u
mendua per sindrom akut koronar, qe nxiti fillimin e agjenteve antitrombocitare dhe heparine iv. Pas vleresimit
komplet dhe ardhjes se enzimave kardiake negative u be e qarte se ne fakt kishte perikardit akut. Shpejt pas hyrjes ne
njesine e kujdesit intensiv, ai u be hipotensiv me vena te qafes te ngritura. Pulmonet jane te pastra ne auskultim.
Ekstremitetet ftohta dhe pulsi brakial eshte i palpueshem vetem gjate ekspirimit. Cila diagnoz e mundshme?
A. Disekacion i aortes
B. Tamponade kardiake
C. Okluzion i arteries koronare kryesore te majte
D. Ruptura e chordae tendinae
E. Defekt i septumit ventrikular
106. Ju kerkoni nje ekokardiografi urgjente per pacientin ne pyetjen 105. Ne kete pacient, cili ng ate mund te rritet
gjate inspirimit krahasuar me ekspirimin?
A. Diametri fund-diastolik i ventrikulit te majte
B. Volumi hedhes i ventrikulit te majte
C. Shpejtesia e prurjeve (inflow) ne valvulen mitrale
D. Presioni sistolik
E. Shpejtesia e prurjeve ne valvulen trikuspidale

107. Te gjitha karakteristikat EKG ndihmojne ne diferencimin e perikarditit akut nga infarkti akut i miokardit, pervec:
A. Mungesa e zhvillimit te valeve QAbsence of the development of Q waves
B. Forma konkave e ngritjes se ST
C. Depresioni i PR
D. Ngritja e ST ne V2
E. Invertimet e vales T pas kthimit te segmentit ST ne baseline

108. Nje grua 35 vjec paraqitet ne spital me gjendje te pergjithshme jo te mire, shtim ne peshe, rritje te perimetrit
abdominal dhe edeme. Simptomat kane filluar 3 muaj me pare dhe kane progreduar gradualisht. Pacienti raporton nje
rritje ne perimetrin e belit prej 15 cm. Edema ne kembe ka ardhur duke u perkeqesuar aq sa tani ndien dhe kofshet te
enjtura. Ajo ka dispne ne effort dhe ortopne, fle me 2 jasteke. Ka histori te meparshme per semundje Hodgkin te
diagnostikuar ne moshen 18 vjec. Eshte ttrajtuar ne ate kohe me kimioterapi dhe rrezatim mediastinal. Ne ekzaminim
fizik, ka temporal wasting (eshte nje humbje ne mase muskulare ne zonen temporalt) dhe duket e semure kronikisht.
Pesha aktuale eshte 96 kg, nje rritje prej 11 kg ne 3 muajt e fundit. Shenjat vitale jane normale. Presioni venoz jugular
eshte 16 cm dhe venat e qafes nuk kolabohen gjate inspirimit. Tingujt kardiake jane te larget. Ka nje tingull te trete
menjehere pas mbylljes se valvules aortale. Tingulli eshte i shkurter dhe degjohet me mire ne apex. Hepari eshte i
zmadhuar dhe pulsues. Asciti eshte prezent. Ka edeme, ku le shenje gishti, qe shtrihet gjate gjithe ekstremiteteve te
poshtme dhe ne murin abdominal. Ekokardiograma trefon trashje te perikardit, dilatim te venes kava inferior dhe
venave hepatike, dhe nderprerje te menjehershme te mbushjes ventrikulare ne diastole e hershme. Fraksioni i
ejeksionit eshte 65%. Cili eshte qendrimi me i mire per trajtimin e kesaj pacienteje?
A. Vetem diureze agresive
B. Transplant i zemres
C. Zevendesim i valvules mitrale
D. Rezeksion i perikardit
E. Perikardiocenteze

109. Nje llogaritare 45 vjec, pa histori te meparshme mjekesore, ka perjetuar ethe, artralgji simetrike dhe lodhje here
pas here ne 3 muajt e fundit. 2 jave me pare, ajo perjetoi vendosjen e papritur te dobesise se dores se majte, qe kaloi
pas 1 ore. Disa kultura gjaku kane dale negative. Vleresimi urgjent nedepartamentin e urgjences lokale, perfshiu nje
CT, qe nuk ofroi ndonje informacion me rendesi. Javen e fundit ka bere nje ekokardiograme transtorakale qe zbuloi
nje mase solitare 2 cm ne atriumin e majte, qe e merr fillimin nga septum interatrial ne afersi te fossa ovalis, qe duket
e pedunkuluar ne nje kercell fibrovaskular. Ju po e shihni serish sot ne klinike. Cili eshte hapi i metejshem ?
A. Heqje e mases sepermjet kirurgjise kardiotorakale
B. Biopsi me kateter e mases
C. Perserit kulturat e gjakut dhe informo laboratorin e mikrobiologjise ti vendose kulturat ne metoda te vecanta per
te vleresuar organizmat HACEK
D. Serologji per antitrupat antinukleare (ANA), anti-DNA dhe antitrupar antikardiolipin
E. PET scan i gjithe trupit per te kerkuar per malinjitete

110. Cili nga malinjitetet e ka riskun relativ me te larte per metastaza ne zemer?
A. Glioblastoma
B. Karcinoma hepatocelulare
C. Melanoma malinje
D. Adenokarcinoma pankreatike
E. Kanceri pulmonar me qeliza te vogla

111. Ju po ndiqni loje baseballi te djalit tuaj 12 vjecar. Sulmuesi e hedh topin gabimisht ne kraharorin e pritesit te topit.
Pritesi menjehere humbet ndjenjat dhe rrezohet. Ju e dini se mbijetesa varet me shume nga cila nga t:
A. Transferimi emergjent ne nje laborator te kateterizimit kardiak per nderhyrje perkutane koronare.
B. Transferimi emergjent ne nje qender traume per riparimin e nje disekacioni akut te aortes
C. Asnje nderyhyrje nuk mund ta ndikoje mbijetesen e ketij pacienti
D. Kirurgji kardiake e shpejte per te riparuar nje valvule aortale te rupturuar
E. Defibrilimi i shpejte

112. Nje grua 47 vjec me BMI 37 kg/m² eshte diagnostikuar kohet e fundit me diabet melit tipi 2. Si pjese e edukimit
te saj, ju e informoni se cili nga te eshte shkaku me i shpeshte i vdekjes ne adultet me diabet melit tipi 2?
A. Semundja e arterieve koronare
B. Infeksioni
C. Neuropatia
D. Insuficienca renale
E. Stroke
113 Z. Daniels eshte nje burre 49 vjec qe njihet mire ne departamentin e urgjences per vizita te shpeshta per
intoksikacion nga alkooli. Ju telefonoheni per te vleresar ate sot. Ne ardhje, ai eshte pak i intoksikuar dhe ne gjendje
te siguroje nje histori te kaluar. Ai thote se per 3 muajt e kaluar, ka pasur veshtiresi ne frymemarrje ne rritje edhe ne
effort minimal dhe perjeton lodhje derrmuese. Gjithashtu eshte zgjuar naten me veshtiresi ekstreme ne frymemarrje,
qe qetesohet vetem duke u ulur ne ane te krevatit. Kycet e kembeve kane qene te enjtura. Ne ekzaminim, presioni
arterial eshte 140/45 mmHg dhe frekuenca kardiake eshte 122/min. ai ka nje puls karotid kercyes dhe puls venoz
jugular te ngritur. PMI e tij eshte e zhvendosur dhe ju qartasi degjoni nje tingull kardiak te ulet qe ndjek menjeher S2.
Ju vini re glossitis dhe kuptoni se atij i mungon ndjesia per prekjen e lehte poshte mesit te pulpes ne te dyja kembet.
Testet laboratorike baze tregojne nje albumine 3.2 g/dl, kreatinine 1.4 mg/dl dhe Na 134 mEq/l. Ekokardiograma
transtorakale tregon nje diameter fund-diastolik te ventrikulit te majte prej 6.8 cm me fraksion ejeksioni 70%. Ju vini
diagnozen dhe i jepni nje barne per te trajtuar shkakun e gjendjes se tij. Gjate rivleresimit pas 24 oresh ju e gjeni te
permiresuar klinikisht, me nje presion 142/76 mmHg dhe frekuence kardiake 85/min. Perserisni ekokardiogramen ku
shihet diametric fund diastolic i VM prej 6.2 cm. Cila ka me shume mundesi ta kete permiresuar kaq shpejt gjendjen ?
A. Folate
B. Penicillamine
C. Thiamine
D. Thyroxine
E. Vitamin B12
114. Nje ish punetore ne fabrike, 65 vjec, me histori te meparshme per artrit rheumatoid, paraqitet ne kliniken e
kujdesit primar duke u ankuar per lodhje. Ajo gjithashtu thote se ka simptomat e : konstipacion, ndien vazhdimisht
ftohte edhe kur temperature eshte 26°C, floke te brishte dhe enjtje te ekstremiteteve te poshtme. Ju, me shpejtesi,
masni TSH, i cili eshte shume i rritur 79.4 mUI/L.Duke pasur parasysh gjendjen aktuale te sistemit te saj kardiovaskular,
ju prisni nje renie ne te gjitha matjet e , pervec:
A. Hedhja kardiake
B. Frekuenca kardiake
C. Presioni i pulsit
D. Intervali QT
E. Presioni arterial sistolik

115. Ne modelin actual qe shpjegon hapat e fillimit dhe evolucionit te aterosklerozes, cili = hapin e pare?
A. Qelizat endoteliale arteriale mbishprehin receptoret e adezionit per leukocitet
B. Pjesezat e lipoproteinave i nenshtrohen modifikimeve oksidative
C. Lipoproteinat me densitet te ulet akumulohen brenda intimes arteriale
D. Makrofaget dhe leukocitet e tjera vijne ne intimen arteriale
E. Makrofaget behen qeliza skuamoze te mbushura me lipide permes endocitozes se pjesezave lipoproteinike

116. Cili nga pohimet e ne lidhje me pasojat patfiziologjike te nje aterome eshte i vertete?
A. Nje aterome qe rezulton ne okluzion vaskular total ne menyre te pandryshueshme shkakton infarkt
B. Nje aterome ne zmadhim nuk kalon ne lumenin arterial derisa pllaka te tejkaloje 40% te lamina elastica interna
C. Shumica e ateromave perfundimisht do te prodhojne simptoma te pacienti
D. Pas formimit fillestar, pllaka ateromatoze zakonisht rritet perbrenda lumenit te enes.
E. Vazat e prekura nga aterogeneza tentojne te kontraktohen dhe te zvogelohen ne diameter.

117. Bazuar ne guidelinet e 2013 per kolesterolin nga American College of Cardiology (ACC) dhe American Heart
Association (AHA), te gjitha grupet e jane te percaktuar si perfitues nga statinat, pervec:
A. Te gjithe individet qe kane semundje kardiovaskulare aterosklerotike klinike (ASCVD)
B. Individet me PCR me sensitivitet larte te rritur pavaresisht nivelit LDL kolesterolit apo riskut per ASCVD.
C. Individet me LDL kolesterol ≥190 mg/dL pa nje shkak sekondar si marrja e tepert e yndyrnave te ngopura ose trans
ose barnave te ndryshme
D. Individet pa ASCVD ose diabet te vendosur, qe jane 40-75 vjec dhe kane LDL kolesterol 70–189 mg/dL dhe nje risk
per ASCVD te llogaritur ≥7.5%
E. Pacientet me diabet, pa semundje kardiovaskulare te vendosur, 40-75 vjec dhe kane LDL kolesterol 70–189 mg/dL

118. Nje burre 50 vjec vjen ne kliniken tuaj per check-up vjetor. Sot ai eshte i shqetesuar per riskun e tij ne te ardhmen
per atak kardiak dhe stroke pasi ka lexuar disa lajme shqetesuese. Ka histori per semundje vaskulare. Ka shtuar ne
peshe gjate dekades se fundit dhe BMI sot eshte 34 kg/m2. Paneli tij lipidik eshte si me poshte.
Kolesteroli total 220 mg/dl
Trigliceridet 283 mg/dl
HDL 29 mg/dl
LDL 132 mg/dl
Ju llogarisni riskun e tij per ASCVD 6% me menyren e llogaritjes qe ofrohet nga ACC/AHA. Pacienti ka lexuar se niveli i
ulet i HDL kolesterolit lidhet me riskun per semundje kardiovaskulare ne te ardhmen. Ai do te dije cfare barnash mund
ti jepni ne lidhje me nivelin e HDL qe te ule riskun kardiovaskular ne graden me te larte. Cila eshte pergjigja e sakte?
A. Nje agonist te peroxisome proliferator-activated receptor α
B. Nje inhibitor te cholesteryl ester transfer protein
C. Fenofibrate
D. Acid Nicotinik
E. Humbje ne peshe dhe aktivitet fizik
119. Te gjitha t jane percaktues madhore te kerkeses se miokardit per oksigjen pervec:
A. Frekuenca kardiake
B. Ritmi kardiak
C. Kontraktiliteti i miokardit
D. Stresi mural ventrikular
E. Te gjithe te mesipermit jane percaktues madhore

120. Z. Jackson po i nenshtrohet nje stress testi ushtrimor. Ju e dini se gjate ketij testi, miokardi i tij do te kerkoje me
shume oksigjen. Si pergjigjet zemra normale (e pasemure) per t’iu pershtatur kerkesave me te larta per oksigjen?
A. Ekstraktim i rritur i oksigjenit ne miokard, me fluks gjaku koronar pothuajse constant
B. Ekstraktim i rritur i oksigjenit nga dhomat ventrikulare
C. Zgjatje relative e kohes se diastoles qe lejon rritjen e fluksit koronar
D. Vazodilatacion i arterieve koronare epikardiale qe con ne ulje te rezistences dhe rritje te fluksit koronar te gjakut
E. Vazodilatacion i vazave koronare intramiokardiale qe con ne rritje te fluksit koronar te gjakut

121. Gjate kohes qe Z. Jackson (nga pyetja 120) vazhdon stress testin, ai fillon te ankohet per dhimbje gjoksi ne forme
shtrengimi. Ju dyshoni se ai po perjeton angine per shkak te nje arterie koronare pjeserisht te okluduar. Nqs do te
analizonit menjehere ndryshimet biokimike qe ndodhin ne miokardin ishemik, do te vinit re te gjitha pervec:
A. Nje rritje ne kalciumin citozolik
B. Nje rritje te pH intraqelizor
C. Nje ndryshim nga metabolizmi aerob ne ate anaerob
D. Zbrazje te depove miokardiale me fosfate me energji te larte
E. Dalje te kaliumit nga miocitet

122. Cili nga lokalizimet e eshte i pazakonte per radiimin e dhimbjes anginoize?
A. Shpina
B. Regjioni interskapular
C. Baza e qafes
D. Dhembet
E. Muskuli trapezius

123. Znj. Wilson eshte punonjese e postes amerikane, 66 vjec, postmenopauze. Ndersa ecte ne rrugen e saj per te cuar
posten gjate 9 muajve te fundit, ajo ne menyre rutine ka perjetuar nje shtrengim ne gjoks dhe dispne kur ngjitej ne
ndonje koder te pjerret. Shtrengimi qetesohej kur ajo pushonte per rreth 3 minuta. Ajo nuk ka humbur ndonje pune
per shkak te ketyre simptomave. Ju dyshoni se ajo po perjeton angine. Cili term dhe klasifikim i Canadian
Cardiovascular Society (CCS) Functional Class of angina eshte i pershtatshem per te pershkruar simptomat e saj?
A. Angine e qendrueshme – CCS class I
B. Angine e qendrueshme – CCS class II
C. Angine e qendrueshme – CCS class III
D. Angine e qendrueshme – CCS class IV
E. Angine e paqendrueshme

124. Nje llogaritar 50 vjec, me histori duhanpirje, perfundoi nje stress test ushtrimor ne piste. Gjate testit, ai zhvilloi
depresion te ST upsloping (inclined upward from a horizontal line, nuk di ta perkthej shqip) ne lidhjet II, III dhe aVF.
Frekuenca kardiake maksimale ishte 130/min. Presioni arterial u rrit nga 120/80 ne 155/95 mmHg. Cili eshte
interpretimi i pershtatshem per kete stress test?
A. Stress test negativ; pergjigje e papershtatshme hypertensive ndaj ushtrimit
B. Stress test negativ; ndryshime jospecifike te ST
C. Stress test jodiagnostik; pergjigje normale e presionit te gjakut
D. Stress test pozitiv; pergjigje e papershtatshme hypertensive ndaj ushtrimit
E. Stress test pozitiv; pergjigje normale e presionit te gjakut

125. Nje grua 70 vjec vjen ne urgjence me veshtiresi akute ne frymemarrje prej 3 oresh dhe shtrengim derrmues
substernale ne kraharor. EKG tregon depression te ST 1mm ne lidhjet II, III dhe aVF pa ndonje ngritje te ST. Niveli
fillestar i troponines eshte lehtesisht i rritur. Ju dyshoni qe ajo ja nje infarkt miokardi pa ngritje te segmentit ST
(NSTEMI). Te gjithe te jane shkaqe te NSTEMI, pervec:
A. Vazospazma koronare
B. Rritja e kerkeses se miokardit per oksigjen per shkak te semundjeve sistemike
C. Disfunksion endothelial mikrovaskular
D. Formimi i trombeve pjeserisht okluduese ne pllaket aterotrombotike te rupturuar
E. Ateroskleroza koronare progresive qe con ne obstruksion mekanik te rende

126. Z. Brian eshte nje trajner i skuadres se hocket, 57 vjec, me histori te meparshme per hypertension, hiperlipidemi
dhe abuzim me duhanin. Ai paraqitet ne urgjence sonte pasi ka perjetuar discomfort ne gjoks gjate nje praktike. Ai
thote se pervoja e pare me discomfort torakal ka nisur para 3 javesh gjate nje sforcimi te fuqishem ne praktike.
Megjithate, gjate javes se fundit eshte dashur me pak sforcim per te filluar diskomforti. Sot ndodhi kur ai thjesht po
qendronte i ulur ne stol. Ai e pershkruan diskomfortin si shtrengim, qe radiohet ne nofulle. Zakonisht zgjat rreth 10
minuta, EKG e tij tregon invertime te reja te vales T ne lidhjet I, II, aVL, V5, dhe V6 qe nga EKG e fundit e tij gjate nje
ekzaminimi fizik rutine. Cili nga entitetet klinike eshte nje diagnoze e pershtatshme per Z. Brian?
A. Infarkt miokardi pa ngritje te ST
B. Infarkt miokardi me ngritje te ST
C. Angine e qendrueshme
D. Angine e paqendrueshme
E. Nevojitet me shume informacion para se te vihet nje diagnoze

127. Z. Riviera eshte nje burre 42 vjen me semundje renale anurike te stadit te fundit per shkak te diabetit melit. Per
shkak te disa problemeve me makinen, ai i ka humbur takimet e fundit per dialize dhe paraqitet ne urgjence me
veshtiresi ne frymemarrje. Frekuenca kardiake eshte 105/min dhe presioni arterial 185/100 mmHg. Ai ka rale fine ne
fushat e poshtme pulmonare ne te dyja krahet dhe pulsi venoz jugular eshte i rritur. EKG tregon takikardi sinusale pa
devijacion te ST. Analiza fillestare e troponines I eshte 0.14 ng/dl (norma <0.06 ng/dL). Cilat jane hapat e metejshem
me te pershtatshem terapeutike dhe diagnostike?
A. Administro 80 mg furosemide IV. Ndiq vlerat e troponines I dhe EKG ne seri.
B. Administro 80 mg aspirine. Refero per stress test farmakologjik nuklear.
C. Fillo terapi antitrombocitar dyfishte (aspirine + clopidogrel) + heparine. Ndiq vlerat e troponines I dhe EKG ne seri.
D. Fillo terapi antitrombocitar dyfishte (aspirine + clopidogrel) + heparine. Refero per angiografi koronare urgjente.
E. Fillo hemodialize urgjente me ultrafiltrim per eleminimin e lengjeve. Ndiq vlerat e troponines I dhe EKG ne seri.

128. Nje shef 67 vjec paraqitet ne urgjence me vjendosje te papritur te nje dhimbje dermuese gjoksi substernale. Ai ka
histori per hypertension, hiperlipidemi dhe disfunksion erektil per te cilat merr perdite Amlodipine, Simvastatin dhe
Tadalafil. Ne vleresimin tuaj, presioni arterial eshte 145/85 mmHg dhe frekuenca kardiake eshte 90/min. Pulmonet
jane te paster dhe pulsi venoz jugular nuk eshte i ngritur. Nuk ka zhurma ose gallop ne auskultimin kardiak. Ai vazhdon
te ankohet per discomfort ne gjoks, qe e vlereson 8/10. Niveli fillestar i troponines I eshte i rritur 0.52 ng/dl (normal
<0.06 ng/dL), dhe EKG zbulon nje depression te ST 1 mm ne lidhjet II, III, aVF, V5, dhe V6. Te gjitha terapite
farmakologjike jane te arsyeshme pervec:
A. Aspirin nga goja
B. Clopidogrel nga goja
C. Heparin iv
D. Metoprolol iv
E. Nitroglycerin SL
129. Z. Gilotra eshte nje punetor nafte 57 vjec me histori hipertensioni, abuzim me duhanin dhe diabet melit. Ai
paraqitet ne urgjence me nje dhimbje gjoksi substernale derrmuese prej 30 minutash qe radiohet ne nofulle dhe
shoqerohet me djersitje profuze dhe veshtiresi ne frymemarrje. Presioni arterial eshte 115/90 mmHg dhe frekuenca
kardiake eshte 95/min. Cili test diagnostic do te ofronte metoden me te shpejte per te ndryshuar menaxhimin
terapeutik te Z. Gilotra?
A. EKG ne 12 lidhje
B. Angio CT koronare
C. Ekokardiograma
D. Nivelet e CK-MB ne serum
E. Niveli i troponines I ne serum

130. Ju po vleresoni nje pacient me dhimbje gjoksi ne urgjence. EKG nuk tregon vale Q ose ndryshime te ST. Do te donit
te dinit nqs pacienti po kalon nje infarkt miokardi. Cili eshte markuesi biokimik i preferuar per infarktin e miokardit?
A. Creatine kinase
B. CK-MB band
C. Erythrocyte sedimentation rate (ERS)
D. Lactate dehydrogenase
E. Troponin I
131. Ju po kryeni nje angiografi koronare diagnostikuese tek Z. Hayes. Ne pamjet fillestare angiografike, arteriet e tij
jane te pastra nga ndonje semundje obstruktive. Megjithate gjate nje angiograme selektive te arteries anteriore
descendente te majte, ju vini re se nje tromb eshte formuar ne maje te kateterit. Gjate injektimit te kontrastit, trombi
embolizon neper arterien koronare descendente anteriore te majte duke shkaktuar obstruksion te plote ndaj fluksit
te gjakut. Fatmiresisht ju jeni ne gjendje ta hiqni trombin me trombektomi thithese, pothuajse menjehere. Nqs nuk do
tmundur ta eleminonit thrombin, kur do te prisnit te kishit CK-MB te detektueshme ne nje analize standard te serumit?
A. 5–10 minuta
B. 1–2 ore
C. 4–8 ore
D. 12–24 ore
E. 24–48 ore
132. Vonesa me e madhe midis vendosjes se simptomave dhe terapise definitive per pacientet me STEMI, zakonisht
ndodh ne cilin interval kohe?
A. Mes vendosjes se dhimbjes dhe vendimit te pacientit per te kerkuar ndihme
B. Mes telefonates se pacientit dhe mberritjes se skuadres mjekesore te urgjences
C. Mes mberritjes se urgjences dhe mberritjes ne spital
D. Mes mberritjes ne spital dhe vendimit per te filluar terapine e reperfuzionit
E. Mes vendimit per te nisur terapine e riperfuzionit dhe nisjes reale te kesaj terapie

133. Te gjithe medikamentet e te perdorur ne menaxhimin akut te STEMI jane ciftuar ne menyre te pershtatshme
me mekanizmin e veprimit pervec:
A. Aspirin – reduktim i thromboxane A2
B. Abciximab – inhibim i receptorit te glikoproteines IIb/IIIa
C. Antagonistet β-adrenergjik – reduktojne konsumin e oksigjenit nga miokardi
D. Clopidogrel – inhibon receptorin e ADP
E. Nitroglycerin – redukton pasngarkesen kardiake

134. Nje burre 61 vjec paraqitet ne urgjence 2 ore me pare me dhimbje gjoksi anginoze dhe ngritje te ST ne I, aVL, dhe
V1–V3. Ne ate moment, kardiologu intevencionues ishte i perfshire ne nje rast kompleks dhe ishte e pamundur ta
merrte pacientin per intervent koronar perkutan te shpejte, prandaj u administrua aktivizues i plazminogenit indoor
(tPA). Megjithate, 120 minuta pas administrimit te tPA, pacienti vazhdon te ankohet per dhimbje gjoksi, dhe EG e tij
nuk tregon asnje korrigjim te ngritjeve te ST. Cili eshte hapi i metejshem me i pershtatshem?
A. Administro nje doze te dyte te tPA
B. Administro streptokinase 1.5 milione unite ne 1 ore
C. Procedo me bypass urgjent te arteries koronare
D. Procedo me intervent koronar perkutan urgjent
E. Prit per 60 minuta te tjera para se te vazhdosh me opsione te tjera terapeutike

135. Z. Cooper ju nenshtrua administrimit te tPA per STEMI akut para 1 ore. Ai pati nje pergjigje ekselente me korrigjim
te ngritjes se ST dhe dhimbjes se gjoksit. Megjithate infermierja ju therret per te vleresuar nje ndryshim ne ritmin
kardiak. Ju vini re nje ritem ventricular me komplekse te gjera me frekuence 75/min. Z. Cooper vazhdon te ndihet mire,
pa simptoma te reja dhe PA eshte 120/80 mmHg. Manovra terapeutike me e pershtatshme ?
A. Administro amiodarone 150 mg IV ne 10 minuta
B. Administro flecainide 400 mg nga goja
C. Administto metoprolol 5 mg IV cdo 5 minuta × 3
D. Masazh i sinusit karotid
E. Vazhdo observimin
136. Nje grua 84 vjec me histori diabeti dhe hiperlipidemie paraqitet ne urgjence 3 jave me pare me STEMI. Ajo ju
nenshtrua interventit koronar primar perkutan urgjent per nje arterie anteriore descendente te bllokuar ne menyre
akute. Nje stent 3.5 x 24 mm everolimus-eluting (nuk di ta perkthej) u vendos me rezultat angiografik ekselent dhe
qetesim komplet te simptomave. Qe atehere eshte ndier shkelqyer. Mjekimet pas daljes nga spitali ishin metoprolol,
aspirin, clopidogrel, rosuvastatin, lisinopril, dhe insulin me shkallezim dinamik. Kohet e fundit shkoi te okulisti, i cili vuri
re nje katarrakt ne syrin e djathte. Ai kontaktoi ju duke ju kerkuar mendim per te nderprere aspirinen dhe/ose
clopidogrelin per te reduktuar riskun e hemorragjise gjate kirurgjise per katarraktin, qe eshte planifikuar per javen
tjeter. Ai ndiehn se risku per hemorragji duke perdorur te dy medikamentet eshte shume i larte per te kryer kirurgjine.
Cilen nga t do te rekomandoni?
A. “Nderpre aspirinen, vazhdo clopidogrelin dhe procedo me kirurgjine per katarraktin.”
B. “Nderpre clopidogrelin, vazhdo aspirinen dhe procedo me kirurgjine per katarraktin.”
C. “Nderpreji te dyja dhe procedo me kirurgjine per katarraktin.”
D. “Shty kirurgjine per te pakten 6 muaj, me mire per 12 muaj, kohe ne te cilen clopidogreli mund te nderpritet.”
E. “Une keshilloj te kryeni kirurgjine nderkohe qe ajo merr aspirine dhe clopidogrel sepse asnjehere nuk do mundet
te nderprese asnjerin prej tyre.”

137. Znj. Constance eshte nje punonjese biblioteke qe kohet e fundit ka dale ne pension, 65 vjec, me histori per
hypertension. Pas daljes ne pension, ajo ka filluar nje program ushtrimor per ecje, por ka vene re qe kishte nje veshtiresi
te dukshme ne frymemarrje dhe nje shtrengim mesatar ne gjoks pas vetem 2 ose 3 blloqe pallatesh. Ajo paraqitet ne
zyren tuaj 2 jave me pare. Duke qene se u shqetesuat se simptomat e saj mund te vijne per shkak te semundjes se
arterieve koronare, ju e referuat tek nje kardiolog per angiografi koronare diagnostikuese. Kjo u krye 1 jave me pare
dhe zbuloi stenoza fokale te grades se larte ne arterien anteriore descentente te majte proksimale, arterien
circumflexa te majte dhe arterien koronare te djathte. Pervec ketyre lezioneve, ajo nuk kishte semundje qe kufizonin
fluksin. Kardiologu nuk kryejti intervent koronar perkutan (PCI) dhe rekomandoi nje bisede me kirurgun kardiotorakal
per te konsuderuar coronary artery bypass grafting (CABG). Z. Constance tashme kthehet ne kliniken tuaj per keshillim.
Gjate keshillimit te Z. Constance per zgjedhjen e strategjise se rivaskularizimit, cili nga pohimet i vertete?
A. CABG ka nje shkalle mortaliteti me te ulet ne nje vit se PCI per paciente si Z. Constance
B. Per 1 vit, paciente si Z. Constance qe i nenshtrohen CABG kane risk me te larte per te nevojitur procedura te tjera
rivaskularizimi se pacientet qe i nenshtrohen PCI.
C. PCI ka nje risk me te larte per stroke se CABG.
D. Karakteristikat e anatomise se koronareve dhe parashikimi i kardiologut per suksesin e PCI duhet te ndikojne
ne vendim.
E. PCI ka shkalle me te ulet te infarktit miokardial ne 1 vit sesa CABG per paciente si Z. Constance.
138. Z. Gruentzig paraqitet tek ju per veshtiresi ne frymemarrje dhe shtrengim te lehte ne gjoks gjate efortit. Pas nje
anamneze te kujdesshme dhe ekzaminimit fizik dhe nje diskutimi me pacientin per risqet dhe benefitet ju e referoni
per nje skaner me technetium-99m scan, i cili zbulon ishemi infero-apiko-posteriore. Ju menjehere e referoni te kolegu
juaj per angiografi koronare. Ne angiografi, gjendet qe ai ka nje system dominant te majte me arterien circumflexa te
majte teresisht te okluduar. Arteria circumflexa e majte distale merr gjak nga kolateralet qe vijne nga arteria anterior
descendens e majte. Kardiologia intervencionuese eshte shume e afte, por e kishte te pamundur te kalonte okluzionin
total ne circumflexan e majte duke perdorur nje tel hidrofil dhe me rrugen anteriore. Cili nga pohimet e lidhur me
nderhyrjen ne kete okluzion kronik total (CTO) eshte i vertete?
A. Nqs CTO nuk mund te kalohet me ndonje tel ne rrugen anteriore, mjeku duhet gjithmone ta braktise riparimin
perkutan dhe te konsideroje kirurgjine e bypassit koronar
B. Tento akses CTO permes rruges retrograde permes vaze kolaterale-opsion kur tentojme rivaskularizim perkutan.
C. Megjithese prezenca e CTO shoqerohet me nje shkalle me te larte mortaliteti, PCI e suksesshme e nje CTO kurre
nuk ka treguar ta ule shkallen e mortalitetit.
D. CABG e CTO rralle e suksesshme per shkak te natyres atretike te vazave distalisht segmentit kronikisht te okluduar.
E. Ne studime, rivaskularizimi i suksesshem i okluzioneve kronike totale koronare shoqerohet me qetesim te
simptomave por pa permiresim te funksionit te ventrikulit te majte.

139. Nje ish shofer kamioni 67 vjec ju nenshtrua CABG 4 vazale 12 vjet me pare me nje graft nga arteria mamare interne
e majte (LIMA) ne arterien anterior dencendens te majte, dhe 3 grafte nga venat saphena (SVGs) ne arteriet first
diagonal, first obtuse marginal, dhe posterior descendens te djathte. Ne viziten e sotme, ai raporton prej pothuajse 1
muaji, nje pershpejtim te dhimbjes se gjoksit dhe veshtiresise ne frymemarrje me sasi ne renie te efortit. Ju e referoni
per angiografi koronare, e cila tregon stenoze sinjifikante ne trupin e SVG ne degen first obtuse marginal. Cili nga
pohimet e eshte i vertete?
A. Deshtimi i graftit nga nje SVG eshte pothuajse gjithmone nga nje tromb i embolizuar nga siti i anastomozes aortale.
B. PCI ne kete graft te venes safena do te kerkoje vendosjen e nje pajisje protektive distale
C. PCI ne nje graft te venes safena eshte teper i rrrezikshem. Duhet te ndiqet nje menaxhim medikal agresiv me terapi
antianginoze.
D. Risku i PCI ne nje SVG eshte identik me ate te PCI ne nje vaze native
E. Graftet venoze, arteriale (psh SVG dhe LIMA) kane shkalle kalueshmerie identike gjate 5 viteve te para pas bypassit.

140. Znj. Edwards eshte nje grua 87 vjec me nje histori te gjate abuzimi me duhanin dhe COPD si pasoje, me nje volum
ekspirator te sforcuar te sekondes se pare (FEV1) 0.76 L, diabet te kontrolluar keq dhe semundje renale kronike stadi
3. Ajo ka qene me pare pacientja juaj, por nuk eshte ndjekur me ju gjate 8 viteve te kaluara. Megjithate, ajo cjen sot
ne kliniken tuaj duke u ankuar per shtrengim ne gjoks gjate efortit, enjtje te kembeve, veshtiresi ne frymemarrje dhe
lodhje te shpejte edhe me effort minimal. Ne ekzaminim, ju vini re nje zhurme sistolike te ashper, me nje pik te vonuar,
ne kufirin e siperm te djathte te sternumit, qe radiohet edhe ne arteriet karotide dhe me nje muzikalitet ne apeks. Ju
nuk mund te vleresoni nje tingull te dyte te zemres, dhe impulse karotide jane te heshtura dhe te vonuara krahasuar
me PMI ne palpacion. Ju dyshoni stenoze aortale te rende dhe vertete, nje ekokardiograme qe behet me vone
konfirmon stenozen aortale me nje siperfaqe te valvules 0.59 cm². Ju e referoni Znj Edwards ne nje kirurg kardiotorakal
qe ju informon se zonja eshte teper e riskuar per nje zevendesim kirurgjikal te valvules aortale, perkatesisht me nje
risk mbi 15% te mortalitetit ne spital. Ju do ta takoni Znj Edwards neser. Cili nga pohimet me i arsyeshem?
A. “Per kete situate, medikamentet kane treguar nje reduktim te mortalitetit.”
B. “Eshte okay qe nuk jeni kandidate per kirurgji. Gjendja juaj ka nje risk te ulet te mortalitetit 1 dhe 5 vjecar dhe ne
mund ti kontrollojme simptomat tuaja me medikamente.”
C. “Valvotomia aortike perkutane me balon eshte nje opsion i mire trajtimi jokirurgjikal dhe eshte pare se ka rezultate
te shkelqyera ne terma afatshkurter dhe afatgjate.”
D. “Meqenese ju nuk jeni nje kandidate per kirurgji, nuk ekzistojne procedura te tjera per zevendesimin e valvules
aortale per ju. Duhet te konsiderojme opsionin e kujdesit terminal.”
E. “Ne duhet te konsiderojme referimin tuaj per testime te metejshme per te pare nqs jeni kandidate per
zevendesim te valvules aortale permes kateterit.”
141. Te gjitha pohimet e ne lidhje me epidemiologjine e hipertensionit jane te verteta pervec:
A. Ne individet e moshes 60 vjec ose me shume, presioni sistolik mesatar eshte me i larte te grate.
B. Presioni diastolic rritet ne menyre te qendrueshme me moshen gjate jetes.
C. Prevalenca e hipertensionit eshte me e ulet ne meksikanet sesa ne te bardhet jo Hispanike.
D. Ne vendet e industrializuara, presioni i gjakut rritet ne menyre te qendrueshme gjate 2 dekadave te para te jetes.
E. Ne shtetet e bashkuara, afersisht 30% e adulteve kane hypertension.

142. Semundja e cilit organ eshte shkaku me i shpeshte i mortalitetit ne pacientet hypertensive?
A. Truri
B. Zemra
C. Veshka
D. Hepari
E. Mushkerite

143. Nje grua 28 vjec ka hypertension qe eshte i veshtire per tu kontrolluar. Ajo eshte diagnostikuar ne moshen 26
vjecare. Qe nga ajo kohe, ajo ka marre sasi ne rritje te medikamenteve. Regjimi aktual konsiston ne labetalol 1000 mg
dy here ne dite, Lisinopril 40 mg 1 here ne dite, clonidine 0.1 mg 2 here ne dite, dhe amlodipine 5 mg 1 here ne dite.
Ne ekzaminimin fizik ajo duket pa shqetesime. Presioni i gjakut eshte 168/100 mmHg dhe frekuenca kardiake 84/min.
Ekzaminimi kardiak eshte pa shenja te vecanta, pa ferkime, galope ose zhurma. Ajo ka pulse periferike te mira dhe nuk
ka edema. Pamja fizike nuk ka hirsutizem, keqshperndarje te dhjamit apo anomaly te organeve gjenitale. Studimet
laboratorike tregojne nje Kalium 2.8 mEq/dL dhe nje bikarbonat ne serum 32 mEq/dL. Glukoza ne gjak esell eshte 114
mg/dL. Cila eshte diagnoza me e mundshme?
A. Hiperplazia adrenal kongenitale Congenital adrenal hyperplasia
B. Displazia fibromuskulare
C. Sindromi Cushing
D. Sindromi Conn
E. Feokromocitoma
144. Cila eshte menyra me e mire per ta diagnostikuar semundjen te pacienti i pershkruar ne pyetjen 143?
A. Nivelet e renines ne venat renale
B. Mbledhja e urines se 24 oreve per metanefrinat
C. RM e arterieve renale
D. Mbledhja e urines se 24 oreve per kortizolin
E. Raporti aldosterone/renine ne plazme

145. Z. Wilkins eshte nje inxhinier i aeronautikes 65 vjec qe nuk ka qene me pare te mjeku dhe nuk merr asnje
medikament. Ai sillet ne urgjence nga gruaja e tij per dhimbje koke ne perkeqesim gjate dites se fundit dhe konfuzion
qe ka filluar rreth nje ore me pare. Presioni i gjakut te prezantim eshte 230/140 mmHg dhe frekuenca kardiake 90/min.
Saturimi i oksigjenit arterial eshte 95%. Ne ekzaminim, ai i leviz te gjitha ekstremitetet ne menyre te barabarte dhe
nuk ka deficite te dukshme ne funksionin e nervave kraniale por eshte qartesisht ne delir. Ekzaminimi kardiak tregon
nje PMI te zmadhuar dhe te forcuar dhe nje gallop S4; pulmonet jane te pastra ne auskultim. Studimet laboratorike
tregojne nje kreatinine 2.4 mg/dl, proteina ne urine 2+ me hematuri dhe nje hematokrit 32 % me numer trombocitesh
normal. Ju ekzaminoni strishon e gjakut periferik dhe dalloni shistocite. RM ugjente e trurit tregon ndryshime
mikrovaskulare te vjetra por pa infarkt akut ose hemorragji. Lidhur me menaxhimin e presionit te gjakut, cila ng a t do
te ishte rruga me e arsyeshme terapeutike?
A. 0.1 mg clonidine nga goja
B. 20 mg Lisinopril nga goja
C. 20 mg labetalol iv dhe fillim i nje injeksioni parenteral te vazhdueshem per te arritur presionin arterial mesatar
te deshirueshem 125 mmHg ne oren e pare
D. 90 mg nifedipine nga goja me clirim te menjehershem
E. Plazmafereze urgjente
146. Cili nga pohimet e lidhur me fiziologjine e perfuzionit renal eshte i vertete?
A. Per shkak te rregullimit te forte prekapilar, endoteli kapilar glomerular relativisht i mbrojtur nga demtimi presioni.
B. Fluksi renal kortikal i gjakur eshte me pak se fluksi medular, dhe shoqeruar me nevojat e larta metabolike te
korteksit renal, kjo e le kete regjion ne kufi te hipoksemise.
C. Perfuzioni renal rregullohet fort ne baze te nevojave metabolike te veshkes.
D. Ekskretimi urinar i albumines eshte parashikues i ngjarjeve aterosklerotike sistemike dhe mund te paraqitet vite
me pare semundjes klinike aterosklerotike.
E. Gjaku venoz qe kthehet nga korteksi renal ka perbajtje ulet O2 se gjaku venoz qe kthehet nga medulla renale.

147. Cili nga pohimet e lidhur me stenozen e arteries renale eshte i vertete>
A. Nje shpejtesi fluksi ne arterien renale anomalisht e reduktuar ne EKO Doppler eshte parashikuese per nje stenoze
sinjifikante hemodinamikisht.
B. Heret ne ecurine e stenozes se arteries renale, jane te zakonshme nivelet e rritura te renines sistemike.
C. Ne popullaten e pergjithshme, prezenca e displazise fibromuskulare eshte e rralle (prevalence <1%).
D. Nivelet e aktivitetit te renines te observuara ne stenozen e arteries renale jane parashikuese te pergjigjes ndaj
terapise medikale.
E. Zakonisht prezantimi i pare klinik i stenozes renale per shkak te displazise fibromuskulare - reduktimi funks renal.

148. Nje burre 75 vjec ju nenshtrua nje angiografie koronare diagnostikuese pas nje stress testi anormal. Aksesi arterial
u arrit lehte ne arterien femorale te djathte dhe angiografia u kompletua me 35 ml te kontrastit jodik. Fatmiresisht
nuk u gjeten stenoza koronare sinjifikante. 7 dite me vone, burri paraqitet ne urgjence me dhimbje abdominale dhe
nauze. Ai raporton se output urinar ka qene i ulet kohet e fundit. Ekzaminimi zbulon nje temperature te lehte (38.3°C)
dhe livedo reticularis ne ekstremitetet e posthme. Studimet laboratorike tregojne nje kreatinine 2.7 mg/dl (me pare
1.1 mg/dL), nr i leukociteve 10,500/mL me 21% eozinofile, dhe nje shkalle sediment te eritrociteve (ERS) 92 mm/ore.
Cila eshte diagnoza me e mundshme?
A. Nefriti intersticial akut
B. Semundja renale ateroembolike
C. Sindroma Churg-Strauss
D. Nefropatia e induktuar nga kontrasti
E. Sindroma Hipereozinofilike

149. Ne cilin nga pacientet e meposhtem qe paraqiten ne urgjence duke raportuar dispne akute, rezultati pozitiv i D-
Dimerit do te nxiste testim te metejshem per emboli pulmonare?
A. Nje grua 24 vjec qe eshte 32 jave shtatzane
B. Nje burre 48 vjec pa histori te meparshme mjekesore qe paraqitet me dhimbje te pulpes pas nje udhetimi ajror
te zgjatur; gradienti i oksigjenti alveolaro-arterial eshte normal.
C. Nje grua 56 vjec qe i nenshtrohet kimioterapise per kancer gjiri
D. Nje burre 62 vjec qe ju nenshtrua kirurgjise per proteze femorale 4 jave me pare
E. Nje burre 72 vjec qe ka pasur infarkt miokardi 2 jave me pare

150. Nje grua 42 vjec paraqitet ne urgjence me vendosje akute te veshtiresise ne frymemarrje. Se fundmi ajo ka shkuar
per vizite te prinderit e saj jashte shteti dhe ka ngare makinen per rreth 9 ore ne vajtje dhe ne kthim. 2 dite me pare
ajo ndjeu dhimbje te lehte te pulpes dhe enjtje, por mendoi se kjo nuk eshte e pazakonte pas qendrimit ulur me kembet
e varura gjate udhetimit te fundit. Kur vjen ne urjgence, eshte takipneike. Shenjat vitale jane: PA 98/60 mmHg,
frekuenca kardiake 114/min, frekuenca respiratore 28/min, saturimi i oksigjenit ne ajrin e dhomes 92% dhe pesha 89
kg. Pulmonet jane te pastra bilateralisht. Ka dhimbje ne pulpen e djathte gjate dorsifleksionit te kembes, dhe kemba e
djathte eshte me e enjtur krahasuar me te majten. Nje matje e gazeve ne gjakun arterial tregon pH 7.52, pCO2 25
mmHg, dhe pO2 68 mmHg. Funksioni hepatik dhe renal jane normal. Nje CT konfirmon emboli pulmonare. Te gjithe
agjentet e mund te perdoren te vetme si terapi fillestare ne kete pacient, pervec:
A. Enoxaparin 1 mg/kg SC 2 here ne dite
B. Fondaparinux 7.5 mg SC 1 here ne dite
C. Tinzaparin 175 unite/kg SC 1 here ne dite
D. Heparine e pafraksionuar iv e rregulluar per te mbajtur kohen e tromboplastines parciale te aktivizuar (aPTT) 2-3
here me shume se limiti i siperm i normes.
E. Warfarin 7.5 mg nga goja 1 here ne dite per te mbajtur raportin internacional te normalizuar 2-3

151. Cili nga pohimet e lidhur me emboline pulmonare eshte i vertete?


A. Rezistenca e rrugeve ajrore zakonisht ulet ne kuader te nje embolie pulmonare akute.
B. Pothuajse te gjithe pacientet me emboli pulmonare kane evidence te trombozes se venave te thella.
C. Hiperventilimi alveolar eshte nje anomali fiziologjike tipike ne prani te embolise pulmonare.
D. Hipotensioni ne kuader te embolise pulmonare akute shpesh per shkak disfunksionit sistolik te ventrikulit te majte.
E. Madhesia e gradientit aterial-alveolar lidhet ne menyre jo te ndryshueshme me madhesine e embolise pulmonare.

152. Nje burre 57 vjec me histori te hipertensionit, vjen ne spital me emboli pulmonare, pasi ka perjetuar nje vendosje
te menjehershme te diskomfortit ne gjoks. Presioni arterial eshte 132/62 mmHg, frekuenca kardiake eshte 85/min dhe
saturimi i oksigjenit 95% ne ajrin e dhomes. Ekokardiografia tregon madhesi dhe funksion normal te zemres dhe
troponina I kardiake eshte e pazbulueshme. EKO Doppler e ekstremiteteve te poshtme tregon nje tromb ekstensiv te
venave te thella nga vena popliteale femorale e djathte. Pacientit i fillohet heparina me peshe te vogel molekulare dhe
varfarine shoqeruese. Diten tjeter ai pyet nqs vendosja e nje filtri ne venen cava inferior do te ishte e pershtatshme.
Cila eshte pergjigja me e pershtatshme?
A. “Jo, ju nuk do te perfitonit nga nje filter i VCI ne kete pike.”
B. “Jo, aktualisht nuk ka indikacione per vendosje te filtrit ne VCI ne kuader te trombozes se venave te thella ose
embolise pulmonare.”
C. “Ne duhet te rishikojme imazhet e ekzaminimeve tuaja dhe te shikojme sa e madhe eshte ngarkesa trombike e
embolise pulmonare para se te vendosim nese nje filter ne VCI eshte apo jo i pershtatshem per ju.”
D. “Po, per shkak te prezences se trombozes reziduale ne ekstremitetet e poshtme, ne do ju planifikojme per vendosje
te filtrit ne VCI.”
E. “Po, duke pasur parasysh moshen dhe gjinine tuaj, ne do ju planifikojme per vendosje te filtrit ne VCI.”

153. Znj. Tupulo eshte nje grua 45 vjec me histori te obezitetit morbid (pesha 140 kg), hypertension dhe diabet. Ajo ka
qene ne spital per 4 dite pasi ka bere nje operacion te hapur per bypass gastrik. Ne diten e 4 pas operacionit, ajo
perjeton mungese frymemarrje te papritur, takikardi, hipoksi dhe nje ndjesi derrmuese te vdekjes se afert. Angio CT
urgjente tregon nje emboli pulmonare ne forme shale (ne bifurkacionin e trungut te arteries pulmonare). Presioni
arterial eshte 70/45 mmHg dhe output urinar eshte i dobet.ju injektuan 500 ml sol fiziologjik iv dhe ju fillua dobutamine
5 μg/kg/min me nje presion arterial pasues 82/55 mmHg. Cili eshte hapi i rradhes me i pershtatshem?
A. Enoxaparin
B. Filter ne VCI
C. Embolektomi pulmonare kirurgjikale
D. tPA
E. Urokinase

154. Nje burre 67 vjecar, ish duhanpires eshte ne diten e 4 postoperative nga nje kirurgji e hapur e bupassit
aortobifemoral per klaudikacion te rende. Ne CT rutine postoperator para daljes nga spitali, vihet re pseudoaneurizem
proksimalisht origjines aortale graftit te bypassit. Cila nga t perkufizion me mire nje pseudoaneurizem?
A. Nje dilatacion fokal ne nje vaze, ne te cilin shtresa e intimes dhe media jane prishur dhe segmenti i dilatuar
rrethohet vetem nga adventicia.
B. Nje dilatacion fokal i nje vaze qe perfshin vetem nje pjese te perimetrit te saj
C. Dilatim i dukshem vaze per shkak tengushtimi intrinsek proksimalisht e distalisht pikes se ngushtimit te dukshem.
D. Dilatacion i nje vaze, megjithese nuk ka madhesine e nevojshme qe te diagnostikohet si aneurizem i vertete
E. Pamja e nje dilatimi aneurizmal te nje vaze ne imazheri per shkak te kendit te arteries dhe teknikes imazherike.
155. Te gjithe faktoret e meposhtem shoqerohen me risk te shtuar per zhvillimin e nje aneurizme aortike, pervec:
A. Mosha
B. Duhanpirja
C. Seksi femer
D. Hiperkolesterolemia
E. Hipertensioni

156. Ju po ndiqni Z. Walker. Ai eshte nje djale 22 vjec me histori familjare per vdekje te papritur dhe aneurizma
arteriale. Ai eshte goxha i gjate me sy me distance te gjere dhe me uvule bifide. Ne viziten e pare ju kerkuat nje
ekokardiograme transtorakale per te investiguar nje zhurme diastolike dhe vini re nje aneurizme te rrenjes se aortes
4.9 cm. Ai ka me shume mundesi te kete nje mutacion ne cilin nga gjenet e ?
A. Fibrillin-1
B. SMAD3
C. Smooth muscle–specific α-actinin
D. TGF-β receptor
E. Type III procollagen

157. Cili nga pacientet ne disekacion te aortes ose hematoma menaxhohet me mire pa terapi kirurgjikale?
A. Femer 45 vjec me disekaciont te aortes distalisht origjines se vazave te medha por proksimalisht arterieve renale
B. Nje mashkull 74 vjec me nje disekacion qe perfshin rrenjen e aortes
C. Nje mashkull 58 vjec me nje disekacion te aortes qe perfshin aorten distale dhe arteriet renale bilaterale
D. Nje mashkull 69 vjec me nje hematoma intramurale brenda rrenjes se aortes
E. Te gjithe pacientet e mesiperm kerkojne menaxhim kirurgjikal per semundjen aortale

158. Nje grua 32 vjece vjen ne urgjence per veshtiresi akute ne frymemarrje. Nje CT nuk tregon evidence per emboli
pulmonare, por vihet re reastesisht nje dilatim i aortes ascendente ne 4.3 cm. te gjitha t mund te shoqerohen me kete
gjetje, pervec:
A. Arteriti me qeliza gjigande
B. Artriti Rheumatoid
C. Sifilizi
D. Lupusi eritematoz sistemik
E. Arteriti Takayasu

159. Z. Tomazelli eshte nje burre 75 vjec me diabet melit dhe hypertension. Pavaresisht keshillimit te vazhdueshem, ai
e ka pasur te pamundur te ndaloje duhanpirjen e cigareve pa filter. Ne viziten e sotme ai ankohet per dhimbje djegese
ne te dyja pulpat. Zakonisht ndodh pas rreth 2 blloqeve te pallateve gjate te ecurit ne rrge te drejte dhe qetesohet me
pushimin. Gjithashtu ka vene re dhimbje te pulpes gjate nates qe qetesohet kur ai ulet ne ane te krevatit. Cila nga t
eshte me me shume mundesi e vertete per Z. Tomazelli?
A. Ai ka nje stenoze arteriale kritike aorto-iliake.
B. Amplituda e kurbes se volumit te pulsit do te jete e mprehte dhe me maje.
C. Etiologjia me e mundshme e simptomave te tij eshte displazia fibromuskulare e arteries femorale.
D. Testi i metejshem diagnostik me i arsyeshem eshte angio RM.
E. Raporti i presionit sistolik ne kycin e kembes me presionin ne arterien brakiale eshte me pak se 0.9.

160. Pacienti i pershkruar ne pyetjen 159 u diagnostikua me semundje arteriale periferike simptomatike (PAD) me
indekset bilaterale kembe/krah 0.82. ai pyet per prognozen dhe hapin tjeter ne trajtimin e tij. Cili nga pohimet e lidhur
me prognozen dhe terapine eshte i vertete?
A. Antikoagulimi me varfarine eshte superior ndaj agjenteve antitrombocitare ne parandalimin e ngjarjeve
kardiovaskulare ne pacientet me PAD.
B. Beta bllokuesit perkeqesojne ishemine e gjymtyreve dhe nuk duhen perdorur ne PAD.
C. Ai duhet te ushtrohet perdite, duke ecur deri ne piken e klaudikacionit maksimal para se te ndaloje per te pushuar
dhe te lejoje qetesimin e simptomave te tij.
D. Risku i larte per morbiditet e mortalitet gjate 5 viteve te ardhshme - progresioni ne ishemi kritike te gjymtyreve.
E. Vazodilatatoret jane terapi e linjes se pare per PAD simptomatike.

161. Nje grua 37 vjec, pa histori sinjifikante mjekesore te meparshme, pervec nje zhurme ne femijeri, vleresohet per
dhimbje te rende me vendosje te menjehershme ne ekstremitetin e poshtem te djathte. Ekzaminimi tregon per nje
grua te re, jo komforte, me shenja vitale normale, pervec nje frekuence kardiake 110/min. Kemba e djathte ka zbehje
poshte gjurit dhe eshte e ftohte ne prekje, dhe pulsi ne arterien dorsalis pedis mungon. Kemba e majte eshte normale.
CIli nga ekzaminimet do te diagnostikonte me me shume mundesi shkakun themelor per paraqitjen e pacientes?
A. Angiografia e ekstremitetit te poshtem te djathte
B. Kulturat e gjakut
C. Echocardiogram with bubble study
D. Antitrupat citoplazmike antineutrofilike ne serum (c-ANCA)
E. EKO e venave te ekstremitetit te siperm te djathte

162. Nje pergjegjes ndertimi 32 vjec, paraqitet me dhimbje gjate sforcimit ne


te dy parakrahet dhe duart. Ai pi nje pakete cigare ne dite, por nuk ka histori
mjekesore te meparshme. Ai ka nje puls brakial lehtesisht te palpueshem por
pulset radiale dhe ulnare jane shume te dobet. Angiografia e ekstremitetit te
sipetm tregon lezione segmentale ne forme koni ne arteriet e vogla distale.
Cili nga trajtimet e ka me shume shans per sukses?
A. Cilostazol
B. Enoxaparin
C. Prednisone
D. Ndeprerje e duhanit
E. Warfarin

163. Cili nga pohimet lidhur me semundjen venoze kronike i vertete?


A. Me shume burra se gra vuajne nga venat varikoze.
B. Me shume burra se gra vuajne nga insuficienca venoze kronike me edema.
C. Mbi 50% e pacienteve mbi 70 vjec kane insuficience venoze kronike.
D. Shumica e pacienteve me insuficience venoze kronike do te zhvillojne ulcera venoze
E. Te gjitha te mesipermet jane te verteta

164. Cila nga t eshte vena me e gjate ne trup?


A. Vena Azygous
B. Vena Saphena magna
C. Vena cava inferior
D. Vena iliake komune e majte
E. Vena femorale komune e djathte
165. Ju po vleresoni nje grua 77 vjec me histori te insuficiences
kardiake me fraksion ejeksioni te ruajtur, diabet melit dhe tromboze
venoze te thelle te ekstremitetit te poshtem te majte, status pas
kursit te antikoagulimit. Per disa muaj te fundit, eshte ankuar nga
nje ndjesi djegje dhe krampesh ne kembet e saj. Sot vjen per
vleresim ten je ulcere ne lekure, si ne figure. Ne ekzaminim ju vini
re nje ulcerim ne lekure mbi maleolin medial dhe nje edeme si dru,
qe nuk le shenje gishti ne te dyja pulpat dhe kofshet. Lekura aty
eshte me e erret. Cili eshte shkaku me i mundshem?
A. Helmim nga merimanga
B. Infeksion nga Bacillus anthracis
C. Insuficience venoze kronike
D. Ulcere e kembes diabetike
E. Semundje arteriale periferike

166. Ju po vleresoni nje vajze 19 vjec ne kliniken e universitetit. Ajo ankohet per nje enjtje te padhimbshme te
ekstremitetit te poshtem. Ne ekzaminim kemba ka lekure te trashur dhe teksture si dru. Nje EKO prane shtratit
konfirmon vena te ekstremiteteve te poshtme te dukshme pa tromboze. Ju dyshoni limfedeme dhe mendoni se
kjo eshte me e mundur te jete pasoje e shkakut me te shpeshte te limfedemes sekondare. Cfare shkaku dyshoni?
A. Kancer qe perfshin limfonodujt inguinale
B. Lymphogranuloma venereum
C. Filariasis limfatike
D. Limfangit bakterial rekurrent
E. Tuberculoz

167. Ju po kujdeseni per nje paciente qe vuan nga limfedema kronike per shkak te limfangitit recurrent kur ishte
femije. Ajo e gjen enjtjen e kembes te pakendshme dhe pyet per opsione terapeutike. Te gjitha t jane opsione
terapeutike te arsyeshme per limfedemen kronike, pervec:
A. Fizioterapi dekongjestive
B. Terapi me diuretike
C. Ngritje e shpeshte e kembes
D. Pajisje per kompresion pneumatik intermitent
E. Liposuksion

Case files
3.1 Nje grua 36 vjec ka nje dhimbje te rende djegese ne gjoks, qe radiohet ne qafe. Dhimbja fillon vecanerisht pas
vakteve, sodomos kur shtrihet dhe nuk precipitohet nga lodhja. Vjen per observim. EKG ne seri dhe nivelet e troponines
I jane normale. Cili nga t eshte hapi me i mire i rradhes?
A. Stress test ne piste me talium
B. Fillimi i nje PPI
C. Angiografi koronare
D. Fillimi i nje antidepresanti si psh SSRI
E. Referim te nje psikiater

3.2 Nje burre 56 vjec vjen ne spital per dhimbje gjoksi me kohezgjatje 2 ore. Frekuenca kardiake eshte 42/min, me
bradikardi sinusale ne EKG, si dhe nje ngritje te ST ne lidhjet II, III dhe aVF. Cila eshte diagnoza e mundshme?
A. Ai eshte ne gjendje te mire fizike me rritje te tonusit vagal.
B. Ai ka pesuar nje infarkt miokardi te murit inferior.
C. Ka nje aneurizme te VM.
D. Frekuenca kardiake e ulet eshte reflektim i fraksionit te mire te ejeksionit.
3.3 Nje grua diaberike 59 vjec ka vuajtur nga nje IAM i murit anterior. 5 dite me vone, ai ka nje grindje me te shoqin
dhe ankohet per dhimbje gjoksi. EKG fillestare nuk tregon ndryshime ishemike, por niveli i troponines I kardiake ne
serum del pak i rritur, Cili eshte hapi i rradhes me i mire?
A. Perdor terapi trombolitike
B. Trajto me PCI
C. Performo bypass te arteries koronare
D. Performo EKG ne seri dhe merr nivelet e CK-MB.
E. Pregatit pacientin per dialize.

3.4 Nje mashkull 59 vjec, duhanpires, ankon per dhimbje gjoksi shtypese substernale te rende me kohezgjatje 30
minuta. Paramedics i kane dhene nitroglicerine sublinguale dhe oksigjen me kanjule nazale. Presioni i gjakut eshte
110/70 mmHg dhe frekuenca 90/min kur arrin ne urgjence. EKG eshte normale. Cili eshte hapi i rradhes me i mire?
A. Ekokardiografi
B. Stress test me Thallium
C. Aspirine
D. Angiografi koronare
E. Bypass i arteries koronare

4.1 Nje burre 55 vjec eshte diagnostikuar me insuficience kardiake kongjestive mesatarisht te rende me funksion
sistolik te demtuar. Cila nga barnat ka me shume shanse te ule riskun per mortalitet?
A. ACE-I
B. Diuretiket e anses
C. Digoxin
D. Aspirin

4.2 Ne USA, cili eshte shkaku me i mundshem i insuficiences kardiake ne pacientin e pershkruar ne pyetjen 4.1?
A. Diabeti
B. Ateroskleroza
C. Alkooli
D. Semundja reumatizmale e zemres

4.3 Nje burre 75 vjec ka dhimbje gjoksi gjate lodhjes dhe kohet e fundit ka humbur ndjenjat. Ne ekzaminim eshte vene
re nje zhurme sistolike e ashper. Cila do te ishte terapia me e mire per gjendjen e tij?
A. Bypass i arteries koronare
B. Angioplastike
C. Zevendesim i valvules
D. Endarterektomi karotide

4.4 Nje burre 55 vjec, me insuficience kardiake kongjestive thote se eshte mire ne qetesi por behet dispneik edhe kur
ecen deri ne tualet. Ne Ekokardiografi vihet re nje fraksion ejeksioni 47%. Cili eshte pershkrimi me i mire per gjendjen
e ketij pacienti?
A. Disfunksion diastolik
B. Disfunksion sistolik
C. Kardiomiopati e dilatuar
D. Semundje e perikardit

5.1 Nje burre 59 vjec ankohet per dhimbje gjoksi te rende qe radiohet ne shpine. Pulset brakiale duken te pabarabarte
mes krahut te djathte dhe te majte. Ai duket hemodinamikisht stable. Ne radiografine e toraksit, ai ka nje mediastin
te zgjeruar. Cili eshte hapi i rradhes me i mire?
A. Fillo terapine trombolitike
B. Bej CT toraksi me kontrast iv
C. Fillo aspirin dhe heparin.
D. Mat nivelet serike te enzimave kardiake

5.2 Nje grua 45 vjec me vendosje te re te regurgitimit aortal, i gjendet nje disekacion i aortes ascendente dhe harkut
te aortes ne ekokardiografi. Ajo eshte relativisht asimptomatike. Cili eshte trajtimi me i mire?
A. Terapi me atenolol nga goja dhe monitoro disekacionin
B. Angioplastike
C. Riparim kirurgjikal i disekacionit
D. Terapi me warfarine nga goja

5.3 Nje burre 75 vjec i shendetshem, duke ju nenshtruar nje EKO per dyshim te semundjes se kolecistes, rastesisht i
gjendet nje aneurizme 4.5 cm e aortes abdominale. CIli eshte trajtimi me i mire per kete pacient?
A. Riparim kirurgjikal i aneurizmes
B. Ekzaminime ekografike cdo 6 muaj
C. RM urgjente
D. Terapi me beta agoniste

5.4 Nje burre 45 vjec eshte i shqetesuar sepse baba ii tij ka vdekur nga nje rupture e aneurizmes se aortes abdominale.
Ne vleresim, gjendet se ai ka nje valvule aortale bikuspidale. Cili eshte pohimi me i pershtatshem lidhur me gjendjen ?
A. Ai eshte ne risk per nje aneurizem te aortes ascendente
B. Ai eshte ne risk per nje aneurizem te aortes abdominale
C. Ai nuk ka risk te shtuar per aneurizma te aortes
D. Ai duhet ti nenshtrohet korrigjimit kirurgjikal te valvules aortale

6.1 Nje grua 30 vjec vihet re se ka presione gjaku ne rangun 160/100 mmHg. Ajo gjithashtu ka obezitet te shtuar,
vecanerisht rreth barkut, ku gjithashtu ka disa strie. Ajo mavijoset shume lehte dhe ka rritje te shtuar te qimeve ne
fytyre dhe gjoks. Cila eshte diagnoza me e mundshme?
A. Hipertiroidizmi
B. Koarktacioni i aortes
C. Sindroma Cushing
D. Feokromocitoma

6.2 Nje burre 45 vjec eshte diagnostikuar me hypertension esencial bazuar ne dy matje te presionit 150/100 dhe
156/102 gjate dy vizitave te ndryshme. Cila nga t ka me shume mundesi te ofroje informacion prognostik ?
A. Biopsi vaskulare
B. End-organ effects nga hipertensioni
C. Regjistrimi i pacientit ne nje clinical trial
D. Matja e niveleve te homocisteines ne serum

6.3 Nje grua 34 vjec qe po mendon per nje shtatzani eshte diagnostikuar me hypertension te stadit te pare dhe pas nje
vleresimi eshte vene re se nuk ka kompliacione. Cila nga klasat antihipertensiveve i pershtatshme per kete individ?
A. Labetolol
B. ARB
C. Inhibitore direkt te renines
D. ARB
E. Hidroklortiazid
6.4 Nje burre 45 vjec me diabet tipi 2, ka pasur presion gjaku 145/90 dhe 150/96 ne dy raste te ndryshme. Cila eshte
terapia me e mire fillestare per kete pacient?
A. Hidroklortiazid
B. ACE-I
C. Beta-bllokues
D. Clonidine

7.1 Nje burre 50 vjec me hypertension kronik paraqitet ne klinike pasi i kane mbaruar medikamentet, lizinopril dhe
amlodipine, >1muaj. Ai eshte asimptomatik dhe ka presion arterial 200/104 mmHg. Cili eshte menaxhimi me i mire?
A. Hospitalizo dhe fillo nitroprusid iv
B. Pershkruaj ne recete Clonidine 0.1 mg 3 here ne dite dhe rikontrollo presionin e gjakut pas 24-48 oresh.
C. Rifillo ACE-I dhe kalcibllokuesin dhe rikontrollo presionin e gjakut pas 24-48 oresh.
D. Refero te nje punonjes social dhe mos i jep asnje antihipertensiv

7.2 Nje grua 80 vjec pa histori per hypertension i nenshtrohet kirurgjise per frakture te legenit. Presioni i gjakut ne
diten e pare pas operacionit eshte 178/110 mmHg. Asimptomatike pervec dhimbjes legenit. Hapi i rradhes me i mire?
A. Transfero pacientin ne njesine e kujdesit intensiv, mat nivelet e enzimave kardiake dhe ul presionin e gjakut ne
rangun 140/90 mmHg.
B. Kontrollo dhimbjen dhe monitoro presionin.
C. Fillo beta bllokues dhe monitoro presionin.
D. Ndalo vizitoret dhe ul zerin e televizorit, alarmeve dhe zhurmat e tjera.

7.3 Nje burre 61 vjec me semundje te arterieve koronare ankon per ortopne progresive dhe edeme te kembeve. Ai
eshte hospitalizuar me nje presion gjaku 190/105 mmHg. Nivelet e enzimave kardiake dhe EKG jane normale. I eshte
administruar furosemide iv. Cili eshte hapi i rradhes me i mire?
A. Pershkruaj nje beta bllokues per te ulur kerkesat e miokardit per oksigjen.
B. Fillo dopamine iv.
C. Observo statusin klinik.
D. Fillo nje ACE-I

7.4 Nje grua 58 vjec me afazi dhe dobesi te krahut te djathe me kohezgjatje 8 ore vjen ne urgjence. CT nuk tregon
hemorragji intrakraniale. Presioni i gjakut eshte 162/98 mmHg. Cili eshte hapi i rradhes me i mire?
A. Normalizo presionin e gjakut me beta-bllokues.
B. Transfero ne njesine e kujdesit intensive dhe trajto me nitroprusid natriumi.
C. Normalizo presionin e gjakut me nje ACE-I.
D. Observo presionin e gjakut.

8.1 Nje grua 28 vjec, i eshte thene se ka semundje reumatizmale te zemres, specifikisht stenoze mitrale. Cila nga
zhurmat ka me shume mundesi te jete prezente?
A. Rumble (gjemim) ne apeksin e zemres
B. Decrescendo (zhurme ne renie??) ne kufirin e siperm te djathte te sternumit)
C. Zhurme holosistolike ne apex
D. Zhurme sistolike me pik te vone ne kufirin e siperm te djathte te sternumit

8.2 Nje grua 48 vjec eshte diagnostikuar me fibrilacion atrial me nje frekuence venrikulare 140/min. Ajo ka marrje
medsh dhe dispne me nje presion sistolik 75/48 mmHg. Cili eshte hapi i rradhes me i pershtatshem?
A. Digoxin iv
B. DC kardioversion
C. Manovrat vagale
D. Diltiazem iv (Cardizem)
8.3 Nje student mjekesie ne vit te trete po lexonte mbi rrezikun e antikoagulimit te tepruar dhe potencialin hemorragjik
te tij. Ai po shikonte kartelat e disa prej pacienteve me fibrilacion atrial qe merrnin kumadine. Cili nga pacientet e
meposhtem eshte me i pershtatshem per nderprerjen e kumadines?
A. Nje burre 45 vjec me te dhena ekokardiografike normale dhe pa histori per semundje kardiake apo
hypertension, por me histori familjare per hiperlipidemi
B. Nje 62 vjecar me hypertension te rende kronik dhe atrium te majte te dilatuar por me FE normal
C. Nje 75 vjecar me gjendje te mire shendetesore por me nje insult te meparshem nga i cili nuk ka pasoja funksionale
D. Nje 52 vjecar me ortopne dhe dispne paroksizmale nocturne

8.4 Nje grua 59 vjec i eshte vendosur trajtimi me Warfarin (Coumadin) pasi u gjet se kishte fibrilacion atrial kronik. Ajo
ka nje INR 5.8, eshte asimptomatike dhe nuk ka hemorragji te dukshme. Cili eshte menaxhimi me i mire ?
A. Transfuzioni me eritrocite
B. Jepi vitamine K
C. Jepi plazme te fresket te ngrire
D. Mbaj trajtimin me Warfarine

8.5 Nje grua 45 vjec ka marrje mendsh, ndjenje goditje ne gjoks (pounding) dhe lodhje me kohezgjatje 3 ore. Ne
ekzaminim eshte vene re nje presion gjaku 110/70 dhe frekuenca kardiake 180/min. Ne EKG ka fibrilacion atrial dhe
nje EKG e meparshme ka treguar vale delta. Mjeku i urgjences e keshillon pacienten lidhur me kardioversionin, por ajo
refuzon. Cila eshte terapia me e mire per gjendjen e saj?
A. Digoxin
B. ACE-I
C. Kalci-bllokues
D. Procainamide

9.1 Nje grua 18 vjec sillet ne urgjence pasi humbi ndjenjat ne nje koncert. Ajo ne dukje e mori veten menjehere, nuk
pati ndonje aktivitet konvulsiv dhe nuk ka histori mjekesore. Frekuenca kardiake eshte 90/min dhe presioni i gjakut
110/70 mmHg. Ekzaminimi neurologjik eshte normal. Testi i shtatzanise eshte negative deh EKG tregon ritem sinusal
normal. Cili eshte menaxhimi me i pershtatshem?
A. Shtroje ne spital per vleresim kardiak.
B. Bej nje ekokardiograme ambulatore.
C. Perdor monitor Holter 24 oresh.
D. Qeteso pacienten dhe dergoje ne shtepi.

9.2 Nje grua 67 vjec ka diabet dhe hypertension te lehte. Eshte pare se ka retinopati diabetike dhe thote se nuk mund
ti ndieje kembet. Ajo ka episode rekurrente te light-headedness kur cohet ne mengjes. Ajo vjen sot pasi kete mengjes
ka humbur ndjenjat. Cili eshte shkaku me i mundshem i sinkopit te saj?
A. Hipersensitiviteti i sinusit carotid
B. Embolia pulmonare
C. Neuropatia autonome
D. Stenoza kritike e aortes

9.3 Nje burre 74 vjecar pa probleme te meparshme mjekesore i bie te fiket teksa ruhej. Pati nje permiresim te shpejte
dhe pa deficite neurologjike. Glicemia e tij eshte normal dhe EKG tregon ritem normal sinusal. Cili eshte testi diagnostic
me i pershtatshem per situaten e tij?
A. Masazhi i sinusit carotid
B. Ekokardiograma
C. CT koke
D. Enzimat kardiake
9.4 Nje burre 49 vjec futet ne njesine e kujdesit intensive me diagnozen e infarktit inferior te miokardit. Frekuenca
kardiake eshte 35/min dhe presioni i gjakut 90/50 mmHg. EKG tregon nje bllok Mobitz tipi I. Cili eshte hapi me i mire?
A. Atropine
B. Pacemaker transvenoz
C. Lidokaine
D. Observim
10.1 Nje burre 68 vjec me histori te semundjes renale te stadit te fundit vjen ne spital me dhimbje gjoksi. Ne ekzaminim
vihet re nje ferkim perikardial. EKG tregon ngritje difuze te ST. CIli eshte trajtimi perfundimtar me i mire?
A. AIJS
B. Dialize
C. Steroide
D. Kayexalate (sodium polystyrene sulfonate)
10.2 Pacienti i pershkruar ne pyetjen 10.1 hospitalizohet, pork a nje vonese ne fillimin e trajtimit. Ju thirreni prane
pacientit sepse eshte bere hipotensiv me PA 85/68 mmHg, FK 122/min dhe puls paradoks. Nje EKG e perseritur eshte
e pandryshuar nga ajo e pranimit ne spital. Cila eshte nderhyrja e menjehershme me e pershtatshme?
A. Bej nje hemokulture dhe fillo antibiotike te spektrit te gjere per sepsisin e dyshuar.
B. Furosemid iv per mbingarkesen ne likide.
C. Perikardiocenteze e drejtuar me ekokardiografi.
D. PCI per IAM.
10.3 Nje grua 25 vjec ankon per dhimbje artikulacionet interphalangeal proksimal e metakarpofalangeal dhe raporton
test laboratorik pozitiv per antitrupat antinukleare (ANA). Cila klinike nuk pershtatet me diagnozen e LES?
A. Efuzioni pleural
B. Rash malar
C. Sklerodaktili
D. Sediment urinar me kaste eritrocitare
11.1 Nje grua 35 vjec paraqet shenje Kussmaul positive. Cilen nga patologjite e ka te ngjare te kete?
A. Perikardit konstriktiv
B. Tamponade kardiake
C. Kardiomiopati e dilatuar
D. Ketoacidoze diabetike
11.2. Cila nga t eshte gjetja me sensitive ne pacientet me tamponade kardiake?
A. Zhdukja e pulsit radial gjate inspirimit
B. Renia e TA sistolik me shume se 10mmHg gjate inspirimit
C. Rritja ne frekuencen kardiake me shume se 20/min gjate inspirimit
D. Zhurmat kardiake te largeta
11.3 Gjate pritjes per perikardiocenteze, kujdesi mbeshtetes i menjehershem ne tamponade kardiake, perfshin?
A. Diureze me furosemide
B. Likide iv
C. Nitrate per te ulur kongjestionin venoz
D. Morfine per te qetesuar dispnene
11.4 Cila nga t eshte shkaku me i mundshem i kardiomiopatise restrictive?
A. Fibroza endomiokardiale
B. Miokarditi viral
C. Beriberi (deficit i tiamines)
D. Terapia me Doxorubicin
12.1 Nje burre 68 vjec eshte hospitalizuar me endokardit te valvules mitrale nga Streptococcus bovis qe sherohet
teresisht me terapine e pershtatshme. Cili eshte hapi i rradhes me i rendesishem?
A. Higjene dentare e mire dhe montim i duhur i protezave per te parandaluar riinfeksionin e valvulave te demtuara
te zemres nga flora orale.
B. Perserit Ekokardiografine pas 6 javesh per tu siguruar qe vegjetacionet jane larguar.
C. Kolonoskopi per te pare per lezione mukozale
D. Zevendesim i valvules mitrale per te parandaluar emboline sistemike si infarkti cerebral.

12.2 Nje perdorues i drogerave iv 24 vjec vjen ne spital me temperature prej 4 javesh. Ai ka pasur 3 hemokultura
positive me Candida spp dhe papritur nje gisht te kembes i behet blu dhe i ftohte. Cili eshte hapi me i pershtatshem?
A. Perserit ekokardiografine per te pare nqs vegjetacioni i madh aortic qe eshte pare me pare ka embolizuar.
B. Keshillim per kirurgji kardiovaskulare per zevendesim te valvules se aortes
C. Angiografi e aortes per te vleresuar per nje aneurizem mykotike qe mund te embolizoje
D. Kalo nga fluconazole ne amphotericin B.

12.3 Nje pacient me cilen nga patologjite e kerkon profilaksi antimikrobiale para kirurgjise dentare?
A. Defekt i septumit atrial
B. Prolaps i valvules mitrale pa regurgitim mitral
C. CABG e meparshme
D. Endokardit infektiv i meparshem
13.1 Nje burre 49 vjec, fumator, me hypertension, diabet dhe hiperkolesterolemi vjen per vizite me ankesat e dhimbjes
ne kembet e tij kur ai ecen 2-3 blloqe pallatesh. Terapia me perfitimin te madh ne reduktim simptom e ne mortalitet:
A. Aspirina
B. Revaskularizimi i kembeve
C. Cilostazoli
D. Nderprerja e duhanit
E. Statinat
13.2 Nje mashkull 31 vjec, fumator, me dhimbje qetesie ne kembet e tij dhe me nje ulcere ne kembe qe nuk sherohet.
Cili shkaku me i mundshem i insuficiences arteriale ne kete pacient?
A. Embol kolesteroli
B. Displazia fibromuskulare
C. Thromboangiitis obliterans (semundja Buerger)
D. Arteriti Takayasu
E. Dhimbja psikogjenike
13.3 Nje grua 21 vjec paraqitet me ethe, lodhje dhe pulse dhe presione te pabarabarta ne krahe. Cili shkaku me i
mundshem i insuficiences arteriale ne kete pacient?
A. Embol kolesteroli
B. Displazia fibromuskulare
C. Thromboangiitis obliterans (semundja Buerger)
D. Arteriti Takayasu
E. Dhimbja psikogjenike
13.4 Nje burre 62 vjec me livedo reticularis dhe 3 gishta te kembes blu, perfshire nje me gangrebe, pas kateterizimit
kardiak. Cili shkaku me i mundshem i gjetjeve ne kete pacient?
A. Embol kolesteroli
B. Displazia fibromuskulare
C. Thromboangiitis obliterans (semundja Buerger)
D. Arteriti Takayasu
E. Dhimbja psikogjenike
PS. Pyetja ne nje teze ishte e tille:
Gjej dhe perputh shkakun me te shpeshte te insuficiences arteriale ne rastet e :
A. Embol kolesteroli
B. Displazia fibromuskulare
C. Trombangiti obliterant
D. Takayasu
E. Dhimbje psikogjenike
1. Nje burre 31 vjecar fumator me dhimbje qetesie ne kembet e tij dhe ulcera
2. Nje 21 vjecare me ethe, dobesi dhe puls e presion arterial te pabarabarte ne krahet e saj
3. Nje 62 vjecar me livedo reticularis dhe gishtat e medhenj te kembes blu, perfshire dhe nje gisht me gangrene
pas kateterizimit kardiak.

13.5 Nje grua 67 vjec eshte diagnostikuar me semundje vaskulare periferike sinjifikante. Ajo vleresohet nga kirurgu
kardiovaskular por nuk ndihet te jete nje kandidate per kirurgji. Cila nga gjendjet e ka me shume mundesi te jete
prezente ne kete paciente?
A. Semundja aterosklerotike difuze
B. Dhimbje e kembes ne qetesi
C. Simptoma qe nuk permiresohen me menaxhimin farmakologjik
D. Ulcera te pulpes qe nuk sherohen

Harrison 18
1. Një grua 35-vjeçare është parë në nje vizite ne klinikë për vlerësimin e dispnese. Cila nga gjetjet fizike të mëposhtme
do të përshtatet me diagnozen e hipertensionit arterial pulmonar idiopatik?
A. Venat e ngritura të qafës, normale S1 dhe S2, II / VI zhurem mermeritje diastolike e dëgjuar në të djathtën, sipërme
kufirit stenal.
B. Venat e ngritura të qafës;nje ton i lartë i S2; II / VI zhurem sistolike murmuritese ne kufirin e poshtëm sternal
te majte.
C. Venat e ngritura të qafës; I lartë, I fiksuar, I ndarë S2; III / VI murmizëm sistolik ne kufirin e poshtëm sternal te
majte
D. Venat e ngritura të qafës, ndarje ne ekspirim e S2, II / VI murmuritje e ashpër sistolike ne kufirin e e sipërm te
majte sternal
E. Venat e ngritura të qafës, gjoksi si fuçi, faze ekspiratore e zgjatur

2. Një grua 75-vjeçare me kanceri te mushkrive jo me qeliza të vogla, gjeresisht metastatik ,pranohet në kujdesin
intensiv me presion sistolik të gjakut 73/25 mmHg. Ajo paraqitet duke u ankuar për lodhje dhe përkeqësim të dispne
gjatë 3-5 ditëve të fundit. Ekzaminimi i saj fizik tregon venat e ngritura të qafës. Radiografia e kraharorit tregon një
hije masive, në formë shishe uji te zemeres dhe pa infiltrate te reja pulmonare. Cila nga gjetjet shtesë vijuese ka të
ngjarë të jetë e pranishme në ekzaminimin fizik?
A. Rënia e presionit sistolik të gjakut më shume se 10 mmHg ne inspirim
B. Mungesa e rënies së presionit venoz jugular ne inspirim
C. zhurme diastolike e vonë with opening snap
D. Pulsus parvus et tardus
E. Zbritja e ngadaltë e presionit venoz jugular

3. Një burrë 78-vjeçar pranohet në njësinë e kujdesit intensiv me dështim të dekompensuar të zemrës. Ai ka kohe te
gjate me kardiomiopatia ishemike. Elektrokardiogram (EKG) tregon fibrilacioni atrial dhe blloku i degës se majtë.
Radiografia e kraharorit tregon kardiomegali dhe infiltrate alveolare bilaterale me linjat B të Kerley. Cila nga sa vijon
është më pak e mundshme te jesh prezent ne ekzaminimin fizik?
A. Tingulli i katërt i zemrës
B. Ritmi i parregullt i zemrës
C. Pulsus alternans
D. Ndarja e permbysur e tingullit te dyte të zemrës së.
E. Tingulli i trete i zemrës

4. Një burrë 45 vjeç është pranuar në kujdesin intensiv njësi me simptoma të dështimit kongjestiv të zemrës. Ai eshte
i varur nga heroina dhe kokaina dhe përdor të dyja ilaçetçdo ditë përmes injeksionit. Kulturat e tij të gjakut kanë dhënë
Staphylococcus aureus i ndjeshëm ndaj metililinës në katër nga katër shishe brenda 12 orësh. Shenjat e tij jetësore
tregojnë një presion të gjakut prej 110/40 mmHg dhe një rrahje zemre prej 132 rrahje / min.Ekziston një murmuritje
diastolike IV / VI e dëgjuar përgjatë kufirit stenal te majte .Një paraqitje skematike e pulsimit karotid është treguar në
figurën V-4A. Cila është më së shumti shkak i mundshëm I zhurmes murmuritese te pacientit?

A. Rregullimi i aortës
B. Stenoza aortale
C. Stenoza mitrale
D. Rregullimi i mitralit
E. Rregullimi i trikuspidit

5. Një burrë 72-vjeçar kërkon vlerësim për dhimbjen e këmbëve me mundesi per te ecur pa ndihme.Ai e përshkruan
dhimbjen si një dhimbje me krampe ne muskujt e kofshëve të tij. Dhimbja bie brenda disa minutave pushimi. Në raste
të rralla, ai ka vërejtur mpirje të këmbës së djathtë në pushim, dhe dhimbja në këmbën e djathtë e ka zgjuar natën. Ai
ka një histori të hipertensionit dhe sëmundje cerebrovaskulare. Katër vjet më parë kishtenjë sulm kalimtar ishemik
dhe iu nënshtrua endarterektomise se karotidit të djathtë. Ai aktualisht merr aspirinë, irbesartan,hidroklorotiazid, dhe
atenololi në doza ditore.Ne ekzaminim, vihet re se ka zvogëlim te pulseve ne dorsalis pedis dhe tibial posterior në
mënyrë bilaterale. Pulsi ne dorsalis pedis e djathte eshte I zbehur. Ka humbje tëqimeve në ekstremitetet distale.
Rimbushja e kapilarëve është afërsisht 5 sekondakëmba e djathtë dhe 3 sekonda në këmbën e majtë. Cila nga gjetjet
vijuese do të ishin sugjeruese të ishemisë kritikee kembes se djathte?
A. Indeksi ankle _brankial më pak se 0.3
B. Indeksi ankle -brachial më pak se 0.
C. Indeksi ankle_brachial më i madh se 1.2
D. Mungesa e pulsit të palpueshëm të dorsalis pedis
E. Prania e edemës pitting të ekstremiteteve

6. Një djale 24-vjeçar është referuar në kardiologji pas një episodi sinkopi ndërsa luante basketboll. Ai nuk kakujtimin
e ngjarjes, por atij iu tha që u rrëzua ndërsa vraponte. Ai u zgjua i shtrirë në tokëdhe pësoi kontuzione të shumta si
rezultat i rënies.Ai gjithmonë ka qenë një individ aktiv, por së fundmi ka zhvilluar disa dhimbje gjoksi me ushtrimet që
ka shkaktuar qe ai i të kufizojë veprimtarinë e tij. Babai i tij vdiq në moshën 44 vjeç, ndërsangjitje në shkëmbinj. Ai
beson se shkaku i vdekjes së babait të tij ishtevdekje e papritur kardiake dhe kujton se i ishte thënë se baba i tij kishte
një zemër të zmadhuar. Në ekzaminim, pacienti ka një murmuritje krescendo-decrescendo midsistolike III /
VI.Elektrokardiograma e tij tregon prova të hipertrofise se ventrikulit të majtë. Ju dyshoni në kardiomiopatinë
hipertrofikesi shkaku i sëmundjes së zemrës së pacientit. Cila nga t pas manovrave do të pritet të shkaktojë njërritje
në zhurmën e murmuritjes?
A. handgrip exercise
B. Squatting
C. Standing
D. Manorva e Valsalves
E. A dhe B
F. C dhe D

7. Blloku i degës së majtë është tregues i cili nga grupe vijuese të kushteve?
A. Defekti septal atrial, sëmundja koronare e zemrës, sëmundja e valvules se aortes
B. Sëmundja koronare e zemrës, sëmundja e valvulave aortale, sëmundja hipertensive e zemrës
C. Sëmundja koronare e zemrës, sëmundja e valvulave aortale, hipertensioni pulmonar
D. Embolia pulmonare, kardiomiopatia, semundja hipertensive e zemeres.
E. hipertensioni pulmonar, emboli pulmonare,stenoza mitrale.
8. Një burrë 57 vjeç me kardiomiopati ishemike ne kohe te gjate është në klinikë për një vizitë rutinë. Ai raporton
pajtueshmëri të mirë me regjimin e tij me diuretik,por ka parë që pesha e tij ka rene rreth 2 kg që ngahera e fundit e
vizites. Ekzaminimet biokimike rutine tregojnë një vlerë kaliumi prej 2,0 meq / L. Pacienti referohet në departamentin
e urgjencës për përsëritjen e kaliumit. Cfare mund te gjejme ne në EKG më parë se te behet administrimi i kaliumit?
A. Zvogëlimi i amplituda e valës P
B. Valët e Osborne
C. Zgjatja e intervalit Q
D. Valët e shquara U
E. lugezimi I segmentit ST

9. Një grua 55-vjeçare nga El Salvador është parë në departamenti i urgjencës për shkak të fillimit gradual të dispne ne
sforcim. Ajo mohon dhimbjen në gjoks, kollën, fishkellimat , sputumin ose ethe. Radiografia e saj e toraksit vihet re per
arteriet pulmonare te medha dhe zmadhimi atrial i majtë, por jo infiltrate parenkimale. EKG tregon një R të larte ne
V1 dhedevijimi i boshtitne te majte. Cila nga sa vijon ka më shumë të ngjarëtë gjenden në ekokardiografinë e saj?
A. Rregullimi i aortës
B. Stenoza aortale
C. Fraksioni I ejeksionit I ventrikulit të majtë I ulet
D. Stenoza mitrale
E. Stenoza trikuspidale

10.Nje grua 29 vjecare eshte ne njesin e kujdesit intensiv me rabdiomniolize si pasoje e sindromes se Kompartmentit
ne ekstremitetet e poshtme te zhvilluara nga nje aksident me makine. Klinika e saj eshte komplikuar nga nje IRA dhe
dhimbje e forte. Ajo I eshte nenshtruar nje fasciotomie dhe eshte derguar ne njesine e kujdesit intensiv. Nje ECG eshte
bere (figura 5.10) Cili eshte opsioni me I mire I veprimit ne kete pike?

A. 18-lead ECG
B. Kateterizim koronar
C. Hemodializ
D. Fluide intravenoze dhe lak diuretik
E. Ventilim/ perfusion imaging

11. Hipercalemia akute me cilen nga ndryshimet ne ECG eshte e lidhur?


A. rritje e intervalit PR
B. zgjatje e segmentit ST
C. vale prominente U
D. QRS zgjerim
E. vala T e sheshte

12. ECG e treguar me poshte eshte me shume e pershtatshme per cilin nga pacientet e meposhtem?

A. Nje femer 33 vjecare me dhimbje akute te forte koke, disorientim, dhe gjak ne hapesiren intraventrikulare ne
skanerin e kokes.
B. Nje burre 42 vjecar me dhimbje te papritur gjoksi nderkoh qe ishte duke luajtur tenis.
C. nje femer 54 vjecare me nje histori te gjate duhanpirje dhe 2 dite me veshtiresi ne frymarrje( frymarrje te
shkurtra) dhe fishkellima.
D. Nje femer 64 vjecare me infucicenc Renale ne stad te fundit e cila nuk ka shkuar ne dialize 4 ditet e fundit.
E. Nje burre 78 vjecar me sinkop, delayed carotide upstroke, dhe nje zhurme sisitolike e forte ne hapesiren
intercostale te dyte te djathte.

13.Je duke vleresuar nje pacient ne klinik i cili vjen me ECG e . ECG eshte bere dy jave perpara. Cfare ankesa prisni qe
t iu shprehe pacienti?

A. Dhimbje Gjoksi(Angine)
B. Hemoptisi
C. disne paraoksizmale ne okturne
D. dhimbje gjoksi pleuritike
E. Takipalpitacione

14.Te gjitha gjetjet e ECG-ve I perkasin hipertrofis ventrikulare te majte, pervec?


A. (S in V1+R in V5or V6) me e madhe se 35
B. R in a VL me e madhe se 11mm
C. R ne a VF me e madhe se 20mm
D. R ne I+S ne III me e madhe se 25mm
E. R ne a VR me e madhe se 8mm
15. Basuar ne elektrokardiogramen e fig 5.15 trajtimi I ciles gjendje mund te permiresoje takikardine e pacientit?

A. Anemi
B. Semundje pulmonare obstruktive kronike
C. Ishemia e miokardit
D. Dhimbje

16.Elektrkardiografia Dopler eshte me e perdorshme per te diagnostikuar cilin nga lezionet e kardiake?
A. Te vleresoje nje mase kardiake ne nje pacient me nje plop te degjueshem ne ekzaminim.
B. te vleresoj hedhjen e ventrikulit te majte tek nje pacient me histori te infarktit te miokardit
C. te diagnostikoje nje ishemi miokardi te nje pacient me dhimbje atipike te gjoksit.
D. te diagnostikoje nje efusion pericardial.
E. vleresimi I mbushjes ne diastole te nje pacient I dyshuar per insuficence kardiake me fraksione ejeksioni te
ruajtur

17. Nje burre 75 vjecar eshte duke ju nenshtruar nje kateterizimi rutine per te vleresuar nje angine te qendrueshme
e cila nuk po I pergjigjet trajtimit mjeksor. Ai eshte infromuar mbi rriskun qe mbart procedura. Cila nga gjendjet e
eshte komplikacioni me I shpeshte ne kateterizimin kardiak dhe angiografin koronar?
A. IRA
B. Bradiaritmia
C. Infarkti Miokardit
D. Takiaritmia
E. Gjakderdhje ne vendin ku vendoset aksesi vaskular

18. Cili nga pacientët e mëposhtëm është kandidati I duhur për kateterizimin e zemrës?
A. Një grua 54-vjeçare me dispne me etiologji te paqarte; një tingulli i 2 i zemrës, I lartë, me ndarje fikse; radiografi
normale e gjoksit; dhe dëshmi per shunt bidereksional rreth septumit te saj interatrial.
B. Një burrë 54-vjeçar me një episod të qëndrueshëm takikardie ventrikulare monomorfike ndërsa në kazino
përfundoi me bystander defribilation. Pasi mbërriti në departamentin e urgjencës, pacienti është hemodinamikisht
i qëndrueshem.
C. Një grua 63-vjeçare me histori të abuzimit me duhanin, hiperkolesterolemi dhe diabeti i tipit 2 melitus me dhimbje
gjoksi në pushim, një EKG normale, dhe ngritje e bute në vlerën e troponinës.
D. Një burrë 66 vjeç me histori diabeti dhe hiperkolesterolemi u soll në departamentin e urgjencës me 1 orë dhimbje
substernale të gjoksit dhe frymarrje te shkurtera.Presioni i tij i gjakut është 95/60 mmHg me një rrahje zemre prej
115 rrahje / min. Një EKG tregon një bllok te degës së majtë te ri, që prej 1 muaji të EKG se tij te mëparshëmme
E. Një burrë 79-vjeçar që shihet në klinikën e kardiologjisë përvlerësimi i stenozës së rëndë të aortës që gjendet në
ekokardiografi ,e kryer për vlerësimin e dispnes.

19. Nje femer 55 vjecare eshte duke u vleresuar per disnen ne sforcim. Ajo ka nje histori hipertensioni qe ne moshen
32 vjecare dhe eshte gjithashtu obese.me BMI 44kg/m2. Testet e funksioneve te saj pulmonare tregojne semundje
pulmonare te mesme restriktive. Nje ekokardiogram tregon nje mur te ventrikulit te majt te holluar. Fraksioni
ejeksionit ne ventrikulin e majte 70%, dhe gjetjet sygjerojne per nje hipertension pulmonar. Me nje presion sistolik te
ventrikulit te djathte te vleresuar 55mmHg, por ekokardiograma eshte teknikisht e veshtire per shkak te kualitetit te
ulet. Ajo vleresohet me nje kateterizim te zemres se djathte qe tregon keto rezultate:

Cili eshte shkaku me I mundshem I dispnese se pacientit.


A. Semundje tromboembolike kronike
B. Insuficence kardiake diastolike
C. Apne opstruktive e gjumit
D. Hipertension pulmonar
E. Insuficence kardiake sistolike

20.Cili nga faktoret e meposhtem eshte nje faktor risku per te zhvilluar tromboemboline tek pacientet me takikardi -
bradikardi variant I sindromes se sinusit te semure?
A. Mosha me e madhe se 50
B. Zmadhimi I atriumit
C. Diabeti melitus
D. Mutacioni 20210 protrombin
E. Asnje me siper, nuk ka rritje risku per tromboembolizem me variantin takikardi-bradikardi te sindromit te sinusit
te semure.

21. Një burrë 38-vjeçar vlerësohet për fillimin se fundmi te nje ndjenje lodhjeje (raskapitje). Ai është një triatlete
aktive. Ai garoi ne një sfide 1 javë më parë pa vështirësi por ndjehet i lodhur në raste të tjera. Ekzaminimet
laboratorike, duke përfshirë hematokritin dhe TSH, janë unremarkable. Mqs gruaja e tij raporton gërhitje me raste,
rekomandohet një studim i gjumit. Nuk ka apne të dukshme, por monitorimi i EKG-së gjatë natës tregon bradikardi
sinusale. Ritmi i zemrës së tij ndryshon midis 42 dhe 56 ndërsa flinte. Rrahjet e zemrës së tij zgjuar ne qetesi ishin 65–
72 rrahje / min. Cila nga sa vijon është menaxhimi më i përshtatshëm për bradikardine e tij?
A. Masazh i sinusit karotid
B. Zgjime intermitente nokturne
C. Matja e free T4
D. Nuk ka terapi specifike
E. Referimi për vendosjen e pacemakerit.

22. Te gjitha t jane shkaqe reversible te disfunxionit sinoatrial pervec:


A. Hipotermia
B. Hipotiroidizmi
C. Presioni I rritur intracranial
D. Toxiciteti nga litiumi
E. Radioterapia

23. Burre, 58 vjec pranohet ne spital pasi ka perjetuar 2 dite me dispne severe. Para 3 javesh ka kaluar nje IAM me
ngritje te segmentit ST, qe eshte trajtuar me trombolitike. E mban fort mire regjimin mjekesor dhe medikamentet e tij
jane atorvastatin, lisinopril, metoprolol dhe aspirin. Ne ekzaminim HR=44bpm TA= 100/45mmHG, pulmonet kane
zhurma bilaterale, bradikardi, venat e qafes te dallueshme, edeme bilateral ne kembe 2+. Ska gallop apo murmur te
ri, Ekg tregon bradikardi sinusale dhe shenjat e IAM se kaluar por jo ndryshime acute. Cila nga me poshte eshte shkalla
me e pershtatshme e menaxhimit:
A. Fillo dopamine
B. Ndalo metoprolol
C. Mat TSH
D. Refero per vendosje pacemaker
E. Refero per angiografi koronare urgjente

24. Student 23 vjecar kthehet ne shtepi ne vere. Ekzaminohet ne urgjence per marrje mendsh qe I ka filluar para 3
ditesh. Raporton nje rash ne kemben e djathte qe duket si nje target para disa ditesh,por nuk ka shqetesime te tjera.
Ekzaminimi fizik tregon bradikardi HR=40 rrahje/min, TA=88/42mmHg dhe saturimi O2 normal. Dallohet nje rash si sy
I demit ne pjesen e siperme te kofshes se djathte dhe EKG tregon per Bllok AV te grades III. Cili nga studimet e
laboratorike ka me shume gjasa te zbuloje etiologjine e shenjave dhe simptomave te tij:
A. ANA
B. HLA B27 testing
C. Borrelia burgdorferi Elisa
D. RPR
E. SCL-70

25. Ne figuren me poshte cili abnormalitet I konduksionit eshte prezent dhe ku gjendet zakonisht ne rrugen e
konduksinit ky bllok:

A. Bllok AV I,intranodal
B. Bllok AV II tip 1,intranodal
C. Bllok AV II tip 2,infranodal
D. Bllok AV II tip 2,intranodal

26. Grua 47 vjece, me histori per dunhanpirje dhe kolit ulcerative u vleresua per palpitacione intermitente. Raporton
se ne 6 muajt e fundit cdo 2-4 dite ajo ve re nje sensacion te zemres se saj qe duket sikur ben “flip-flopping” per rreth
5 minuta ne kraharor. Nuk ka vene re ndonje faktor precipitues dhe nuk ka ndjere marrje mendsh apo dhimbje gjoksi
me keto episode. Ne ekzaminim fizik eshte normal. EKG e qetesise tregon ritem sinusal dhe pa abnormalitete. Perpos
kontrollit te elektroliteve ne serum, cili nga te eshte testi me I pershtatshem?
A. CT abdominal me contrast oral dhe IV
B. Monitorim I eventit
C. Monitorim Holter
D. Rikonfirmin pa teste te metejshme
E. Referim per studim EP

27. Pas testeve te metejshme pacientja ne V-26 u gjet se ka disa episode te kontraksioneve premature atriale. CIli nga
pohimet ne lidhje me disritmine e pacientes eshte e vertete?
A. Kontraksionet atriale premature me pak te zakonta se ato ventrikulare premature ne monitorimin e zgjatur te EKG
B. Ekokardiografia indikohet per te percaktuar nese ka semundje strukturale kardiake
C. Metoprolol niset per te kontrolluar simptomat
D. Pacienti duhet te sigurohet qe nuk eshte nje kondicion I veshtire dhe nuk ka nevoje per vleresim te metejshem
E. Pacienti duhet te beje nje test stresi per te percaktuar nese eshte prezente ischemia
28. Burre 55 vjec me COPD ne fazen e fundit do te pranohet ne ICU me nje perkeqesim te semundjes se tij pulmonare
obstructive. Per shkak te insuficiences respiratore hiperkarbike,ai intubohet dhe vendoset ne ventilim te asistuar
mekanik. Pavaresisht qetesuesve,disa here ventilator I tij tregon alarm per arritjen e presioneve te pikut inspirator
shume te larte. Mjeku kerkohet prane shtratit te pacientit per te vleresuar takikardine. Ne ekzaminim dallojme
TA=112/68 mmHg, HR=180bpm. Ne ekzaminim kardiak dallojme ritem te rregullt,dhe jo abnormalitete te tjera.
Zhurmat pulmonare jane te ulura ne anen e djathte. EKG tregon takikardi me komplekse te ngushta. Me ndihmen e
masazhit te sinusit carotid HR ulet deri ne 130bpm,por pastaj rritet perseri ne 180bpm. Cili nga te eshte hapi I
metejshme me I pershtatshem ne menaxhim?
A. Adenozine 25mg bolus IV
B. Amiodarone 200 mg bolus IV
C. CXR(chest X-Ray)
D. Metoprolol 5 mg IV bolus
E. Sedacion I ndjekur nga kardioversioni

29. Te gjithe jane faktore risku per stroke ne pacientin me fibrilacion atrial pervec:
A. Diabeti Mellitus
B. Histori me IKK(CHF ne anglisht)
C. History stroke
D. HTA
E. Madhesia e atrium sinister me e madhe se 4.0cm

30. Cili nga pohimet ne lidhje me rikthimin e ritmit sinusal pas fibrilacionit atrial eshte I vertete?
A. Dofetilide eshte I sigurte ne perdorim te pacienti ne dalje
B. Nese pt trajtohet me farmakoterapi dhe ka ritem sinusal, nje monitor Holter duhet te perdoret per te percaktuar
se kur do te ndalohet terapia me antikoagulante pa rrezik
C. Pacientet qe e kane mbajtur ritmin sinusal farmakologjikisht pas fibrilacionit atrial kane jetegjatesine me te mire
krahasuar me pacientet qe trajtohen me antikoagulante dhe kontrollues ritmi
D. Rekurrenca e fibrilacionit atrial eshte e pazakonte kur perdoret farmakoterapia per te mbajtur ritmin sinusal

31. Grua 57 vjecare me histori te korrigjimit kirurgjikal te defektit ne septumin atrial ne femijeri paraqitet ne urgjence
me palpitacione prej 3 ditesh. Ne ekzaminim ka HR=153rrahje/min, TA=128/75mmHg dhe ne EKG dallohet flater atrial.
Ne ekokardiograme dallojme dilatacion te moderuar atrial dexter dhe sinister, ndryshime postoperative nga interventi
dhe funksion ventricular dexter dhe sinister normal. Cila nga t eshte e vertete?
A. Duhet te niset antikoagulimi me dabigatran
B. Nese ekokardiograma transezofageale nuk tregon tromb ne atriumin sinister,behet kardioversion pa antikoagulim
C. Heparina intravenoze duhet ti niset menjehere
D. Duhet ti behet menjehere kardioversioni
E. Ekokardiograma transtorakale eshte e pershtatshme per te perjashtuar prezencen e trombit ne atriumin e majte

32. Nje pacient paraqitet me palpitacione dhe veshtiresi ne frymemarrje prej 6 oresh. Në dhomën e pritjes të
departamentit të urgjencë kryhet një EKG. Cila nga sa me poshte ka më shumë gjasa të gjendet në ekzaminimin fizik?
A. Abdomen I bute, mbrojtje muskulare
B. Wheezing difuz polifonik me levizje te dobet ajri dhe hiperinflacion
C. Impuls prekordial ne ventrikulin e majte dhe zhurme S3
D. Limfadenopatia supraklavikulare
E. Rash vezikular mbi dermatomen T5 te djathte
33. Një grua 43-vjeçare është parë në departamentin e urgjencës pas fillimit të papritur të palpitacioneve 30 minuta
para vizitës së saj. Ajo ishte ulur në kompjuterin e saj të punës kur filluan simptomat. Me perjashtim te dhimbjes së
shpinës, ajo është e shëndetshme. Në triazh, rrahjet e saj te zemrës jane 178 rrahje / min, dhe presion I gjakut është
98/56 mmHg me saturim te O2 normal. Nëekzaminimin fizik, pervec një "frog sign" në qafë dhe takikardi, shfaqet
normale. EKG tregon një takikardi me komplekse te ngushta pa vale p të identifikueshme. Cila nga sa me poshte është
hap I pare më I përshtatshëm për të menaxhuar takikardinë e saj?
A. 5 mg metoprolol IV
B. 6 mg adenosine IV
C. 10 mg verapamil IV
D. Masazh I sinusit karotid
E. Kardioversioni DC, 100 J

34. Një burrë 37-vjeçar, i cili është i shëndetshëm përveç një operacioni te meparshem të gjurit, ekzaminohet në
departamentin e urgjencës për palpitacionet e papritura duke ngrene darkë. Eshte matur një rrahje zemre prej 193
rrahje / min, presion i gjakut 92/52 mmHg dhe saturim te O2 normal. Ekzaminimi fizik i tij është normal përveç
takikardisë dhe djersitjes se lehte.Një EKG e marrë para operacionit të tij te gjurit tregon valët delta ne lidhjet
prekordiale. EKG tregon takikardi me komplekse te gjera. Cila terapit kunderindikuar si trajtim per takiaritmine e tij?
A. Adenosine
B. Masazh I sinusit karotid
C. DC cardioversion
D. Digoxine
E. Metoprolol

35. Në një EKG me takikardi me komplekse te gjera, cila nga të dhënat e mëposhtme mbështet më fuqimisht diagnozën
e takikardisë ventrikulare?
A. Disosacioni atrio-ventrikular
B. Patern klasik I RBBD
C. Ritem irregular me komplekse QRS te ndryshueshme
D. Zgjatja e QRS duration me shume se 120 millisekonda
E. Ulje e frekuences kardiake nga masazhi i sinusit karotid

36. Një mashkull 40-vjeçar me diabet dhe skizofreni ka nisur nje terapi me antibiotikë për osteomielitin kronik në spital.
Osteomieliti i tij është zhvilluar ne vendin e nje ulçere, ku ai injektonte heroinë. Ai është gjetur papritur i
pandergjegjshem nga stafi i infermierisë. Cila nga substancat e eshte shkaku më i mundshëm i këtij ritmi?
A. Furosemide
B. Metronidazole
C. Droperidol
D. Metformine
E. Heroine

37. Ritmi normal I sinusit rikthehet me kardioversion elektrik te pacientit në fjalë V-36. Një elektrokardiograme 12-
lead është e dukshme pë rnjë interval të zgjatur QT. Përveç ndalimit të ilaçit te perdorur, menaxhimi më i përshtatshëm
për këtë shqetësim të ritmit duhet të përfshijë administrimin intravenoz të cilës nga sa me poshte?
A. Amiodarone
B. Lidocaine
C. Magnesium
D. Metoprolol
E. Potassium

38. Po kujdesesh për një pacient me anginë lidhur me rrahjet e zemrës. Me rritje minimale të rrahjeve të zemrës,
pacienti ka simptoma të anginës që ndikojnë cilësinë e tij të jetesës. Në një mbikqyrje prej 24 orësh me monitorin
Hotler, duket se pacienti ka takikardi sinusale në kohën e simptomave. Cili është mekanizmi i aritmisë së pacientit?
A. Pasdepolarizimi i vonshëm
B. Pasdepolarizim i shpejtë
C. Automaticiteti i rritur
D. Rruga e rihyrjes

39. Ku janë nxitësit më të zakonshëm të fibrilacionit atrial të lokalizuar anatomikisht?


A. Aurikula e atriumit të majtë
B. Anulusi mitral
C. Orificiumi i venës pulmonare
D. Sinusi venoz
E. Nyja sinuzale

40. Simptomat e fibrilacionit atrial varion dramatikisht nga pacienti në pacient. Pacienti me cilin nga kushtet klinike ka
më shumë gjasa të jetë simptomatik (p.sh., vështirësi në frymëmarrje); nëse pacienti pëson fibrilacion atrial?
A. Intoksikim akut nga alkooli
B. Kardiomiopati hipertrofike
C. Hipertiroidizëm
D. Hipotermi
E. Torakotomia postoperative

41. Një grua 47 vjeçare postmenopauzaleështë ekzaminuar për nisje të një dispneje të rëndë për javët e fundit. Ajo
raporton se nuk ka pasur dhimbje në kraharor, kollë, sputum, apo temperaturë, por raporton edemë në kembë. Në
ekzaminimim fizik dallojmë TA 145/78 mmHg dhe HR 123 rrahje/min. Exoftalmus është i pranishëm, ashtu si dhe
krepitacionet bilaterale inspiratore, që zënë afërsisht 1/3 e pjesës së poshtmë të kraharorit; distencion të venave të
qafës; ritëm kardiak normal, edhe pse ka takikardi dhe S3 pa murmur. Dallohet gjithashtu edemë bilaterale e
ekstremitetit të poshtëm dhe tremor të lehtë të duarve. Cila nga alternativat e mëposhtme ka më shumë gjasa të jetë
shpjegimi patopsikologjik i insuficincëssë saj kardiake?
A. Anemi me CO të lartë
B. Hipertension kronik sistemik me rezultat hipertrofi të ventrikulit të majtë dhe insuficincë kardiake josistolike
C. Hemokromatozë e ndjekur nga kardiomiopati restrictive
D. IAM me ulje të funksionit të ventrikulit të majtë
E. Tirotoksikozë me CO të lartë

42. Cila nga alternativat e vërtetë për matjen e BNP-s së plazmës për të diagnostifikuar insuficiencën kardiake?
A. Një nivel i rritur i BNP-s së plazmës në një pacient dispneik konfirmon diagnozën e insuficiencës kardiake të majtë
B. Në prezencën e insuficiencës renale, niveli i BNP-s nuk rritet edhe kur kemi insuficiencë kardiake
C. Niveli i BNP-s së plazmës mund të paraqitet I ulët në pacientët me obezitet dhe insuficiencë kardiake
D. Matja periodike e BNP-s në terapi insuficiencës kardiake të dekompensuar përdorur për të udhëhequr terapinë
E. Të gjitha alternativat janë të vërteta

43. Një burrë 64-vjeçar me kardiomiopati ishemike, fraksion të ejeksionit 35% dhe insuficiencë kardiake të fazës C
vlerësohet në klinikën kardiologjike për vlerësimin e gjendjes së sëmundjes së tij. Pacienti raporton një regjim të
rregullt me ecje në pistën e vrapin disa herë në javë dhe përkeqësim të herëpashershëm të edemës së këmbës së tij
qëai e menaxhon me një dozë ektra frosemide. Ai nuk është shtruar asnjëherë në spital për insuficiencë kardiake.
Regjimi i tij itanishëm mjekësor përmban lisinopril, aspirin, furosemide, atorvastatin, digoxin, spironolactone dhe
metoprolol. Ai dëshiron të ndalojë medikimet për shkak të kostos së lartë. Cila e vërtetë për regjimin e tij mjekësor?
A. Terapia me ACEI nuk duket të ketë përmirësuar simptomat e insuficiencës kardiake
B. Terapia me beta bllokues në këtë pacient mund të jetë duke përkeqësuar nevojën e tij të herëpashershme për një
dozë ekstra furosemide; ndaj duhet të ndalohet
C. Duhet të kalojë nga spironolactone në eplerenone për efikasitet më të lartë, siç duket në pacientët me fraksion
ejeksioni më të ulët se 35%
D. Nëse digoxina ndalohet, ai me shumë gjasa do të ketë përkeqësim simptomash
E. Nëse është intolerant ndaj lisinopril-it për shkak të kollës, do të ishte e arsyeshme të nderrohej në ARI

44. Nje grua e dobet, 78 vjecare eshte pare ne pavionin e urgjences pas disa javesh me dispne eforti qe progredoi drejt
dispnese ne qetesi, qe pasoi nje barbeque, ku ajo konsumoi shume perime turshi. Ajo gjithashtu ankon per enjtje ne
kembe, ortopne dhe disa raste zgjim gjate nates me dyspnea. Historia e saj mjekesore e kaluar shquhet per
hypertension sistemik per nje kohe te gjate, prolapse uterin dhe crregullim ankthi. Ekzaminimi konfirmon prezencen e
insuficiences kardiake, me zonen e impulsit kardiak maximal (pmi=point of maximum impulse, zona lateralisht-poshte
sternumit ku ndjehen me shume rrahjet e zemres, ictus cordis?) te zhvendosur lateralisht dhe te qendrueshme dhe
ton te katert kardiak. Ajo u pranua ne spital, iu dha diuretike dhe iu be ekokardiograme. Ne ekokardiografi shihet
hipertrofi e rende e ventrikulit te majte me fraksion ejeksioni 70%, por nuk ka levizje anormale fokale te murit
ventricular, dhe funksioni valvular aortic dhe mitral eshte intakt. Presioni sistolik I ventrikulit te djathte eshte 45mmHg.
Pas qetesimit te simptomave te insuf.kardiake me diuretike, pacientja eshte gati per dalje. Cili medikamente ka treguar
permiresim mortaliteti ne insuf.kardiake me fraksion ejeksioni te ruajtur dhe duhet te perfshihet ?
A. Digoxin
B. Lisinopril
C. Metoprolol
D. Sildenafil
E. Asnje

45. Nje burre 68 vjecar me histori infarct miokardi dhe insuf.kardiake kongjestive eshte I qete ne qetesi. Por, kur ecen
per tek makina e tij ai zhvillon dispne, lodhje dhe ndonjehere palpitacione. Ai duhet te qetesohet per disa minuta qe
keto simptoma te largohen. Cila nga alternativat e eshte klasifikimi I tij sipas New York Heart Association?
A. klasa I
B. klasa II
C. klasa III
D. klasa IV
46. Bashkeshorti I nje gruaje 68 vjecare me insuf.kardiake kongjestive eshte I shqetesuar pasi bashkeshortja e tij ndalon
frymemarrjen per nje kohe te caktuar kur fle. Ai ka vene re se ajo ndalon frymemarrjen per rreth 10 sekonda dhe pas
kesaj pason nje episode hiperventilimi per nje kohe te ngjashme. Kjo nuk e zgjon nga gjumi. Ajo nuk gerhet, ndihet e
qetesuar mire ne mengjes, por eshte shume dispneike edhe me aktivitetin me te lehte. Cili eshte hapi yt I metejshem
ne menaxhimin e pacientes?
A. Elektroencefalografi
B. Maximalizo menaxhimin e insuf.kardiake
C. CPAP (presion pozitiv I vazhdueshem ne rr.frymemarrjes) nazal gjate gjumit
D. Studim I gjumit
E. Jep bronkodilatatore

47. Nje burre 53 vjecar I nenshtrohet transplantit kardiak per kardiomiopati ishemike end-stage, pasoje e nje
crregullimi familjar hiperkolesterolemik. Dhuruesi ishte nje viktime 23vjecare e nje aksidenti me motor. Pacienti shkon
mire per 3 vitet e para pas transplantin me vetem 1 episod te flakjes akute. Ai tregohet compliant gjate trajtimit me
imunosupresore, qe perfshijne prednisone dhe sirolimus. Eshte vleresuar gjate nje vizite follow-up dhe raporton se ka
zhvilluar dispne ne effort. Testet e funksionit pulmonary jane te pandryshuara dhe radiografia e toraksit eshte normale.
Ai I nenshtrohet kateterizimit kardiak me biopsy te zemres se transplantuar. Angiografia koronare tregon
sem.koronare te rende, difuze, koncentrike dhe longitudinale, dhe histologjia nuk ka te dhena per flakje akute. Cila
nga theniet e per aterosklerozen koronare te gjetur ne kete pacient eshte e vertete?
A. Asnje imunosupresore nuk ka treguar t incidence me te ulet per ateroskleroze koronare pas transplantit kardiak.
B. Ateroskleroza koronare me shume mundesi-demtim imunologjik I endotelit vascular ne organin e transplantuar.
C. Ateroskleroza koronare aktuale pas transplantit kardiak - pasoje e aterosklerozes ekzistuese para transplantit.
D. Crregullimi baze I kolesterolit pacientit nuk e predispozoi per ateroskleroze koronar rekurent pas transplantit
E. Terapia me statina nuk eshte e lidhur me incidence te ulur te ketij komplikacioni te transplantit.

48. Cila nga alternativat eshte komplikacion I njohur I vendosjes se pajisjes se asistences ventrikulare ne pacientet me
insuf.kardiake end-stage?
A. Aksident cerebrovascular
B. Infeksion I vendit te inserimit
C. Deshtim mekanik I pajisjes
D. Tromboemboli
E. Te gjitha
49. Te gjitha jane komplikacione te mundshme te nje defekti septal atrial ne adulte pervec:
A. Emboli ajrore nga kateteri venoz central
B. Desaturim I oksigjenit arterial gjate efortit
C. Aksident cerebrovascular embolik
D. Hipertension arteria pulmonary
E. Angine e paqendrueshme
50. Nje grua 32vjecare eshte vizituar nga mjeku I saj I kujdesit primar gjate nje vizite rutine follow-up per
hipotiroidizmin e saj. Ajo ka gjithashtu nje semundje kardiake kongenitale komplekse me nje DIV te korrigjuar
pjeserisht me shunt kryesisht djathtas-majtas pergjate patch-it. Ajo eshte mire dhe eshte e afte te punoje ne sherbimet
roje (janitorial) pa dispne severe. Ajo refuzon te kete simptoma te insuf.kardiake apo neurologjike, por ka saturim
oksigjeni periferik 78%. Gjaku komplet rutine tregoi hematokitin 65%. Cila nga alternativat eshte me e pershtatshme
per menaxhimin e hematokritit te larte?
A. Fillo oksigjenoterapi
B. Kontrollo co-oximetry ne mostren e gazeve te gjakut arterial
C. Kontrollo nivelin e eritropoetines serike
D. Prit (expectant waiting)
E. Referoje ne hematologji per flebotomi
51. Nje burri 43vjecar iu gjet kohet e fundit nje DIA asimptomatike, qe u mbyll duke perdorur nje patch perkutan, para
1 muaji, pa komplikacione. Ai do ti nenshtrohet nje procedure stomatologjike (root canal) javes tjeter dhe per kete
telefonoi mjekun e kujdesit primar per te vendosur nqs profilaksia me antibiotike eshte e indikuar. Cila nga alternativat
eshte e vertete per profilaksine me antibiotike ne kete pacient?
A. Duke qene se kishte vetem nje semundje kardiake kongenitale te thjeshte, profilaksia nuk eshte e indikuar.
B. Duke qene se lezioni u korrigjua, profilaksia nuk eshte e indikuar.
C. Ai duhet shmange procedura dentare potencialisht bakteremike, vetem nqs ska alternative tjeter te mundshme.
D. Profilaksia rutine me antibiotike eshte e indikuar per procedura dentare bakteremike, sidomos nqs patch-I eshte
me pak se 6 muajsh.
E. Profilaksia rutin antibiotike e indikuar per procedure dentare bakteremike kurdohere qe material I huaj present

52. Nje mashkull 20 vjecar I nenshtrohet nje ekzaminimi fizik me grafi toraksi per perfshirje ne ushtri. Ai ka patur nje
femijeri normale pa ndonje semundje madhore. Nuk ka histori per sinusit, pneumoni apo semundje respiratore
kronike. Radiografia e toraksit tregon per dextrokardi. Ne nje examinim me te hollesishem fizik, maja e shpretkes eshte
e palpueshme ne anen e djathte te abdomenit dhe melcia perkutohet ne anen e majte. Cila nga alternativat eshte e
vertete per gjendjen e tij?
A. Ai ka te ngjare te kete stenoze aorte.
B. Ai ka te ngjare te kete aspermi.
C. Ai ka te ngjare te kete DIA.
D. Ai ka te ngjare te kete DIV.
E. Ai ka te ngjare te jete normal.
53. Nje mashkull 24vjecar kerkon vleresim mjekesor per fillimin e dhimbjeve te kokes kohet e fundit. Dhimbjet e kokes
pershkruhen si ‘goditese’ (pounding) dhe shfaqen gjate dites dhe nates. Ai ndjen nje lehtesim minimal nga
acetaminofeni. Ekzaminimi fizik tregon presion gjaku 185/115mmHg ne krahun e djathte, rrahjet e zemres 70/min, AV
nicking ne examinimin fundoskopik, vena jugulare dhe art.karotide normale, pressure loaded PMI me T4 apikal, pa
irradijim abdominal, dhe puls te reduktuar ne te dy extremitetet e poshtme. Rishikimi I simptomave eshte pozitiv
vetem per lodhje te kembeve pas efortit. Matje te metejshme te presionit te gjakut tregojne per:

A. MRI koke
B. MRI veshke
C. MRI toraksi
D. 5- HIAA ne urinen 24h
E. Kortizoli I lire ne urinen 24h
54. Pacienti I pershkruar ne pyetjen 53 ka te ngjare te kete nje nga anomalite kardiake , cilen?
A. Valvul aorte bikuspidale
B. Stenoze mitrale
C. Sindrome preeksitimi
D. Bllok I deges se djathte
E. Atrezi trikuspidale
55. Stenoza mitrale komplikohet shpesh me hypertension pulmonary. Cila nga alternativat e eshte shkak per
hypertension pulmonary ne stenozen mitrale?
A. Edeme intersticiale ne muret e vazave te vogla pulmonare
B. Transmetim pasiv I rritjes se presionit te atriumit te majte
C. Ndryshime obliterative ne shtratin vascular pulmonary
D. Konstriksion arterial pulmonary
E. Te gjitha
56. Nje burre 58vjecar me histori te hipertensionit sistemik, hiperlipidemi, dhe abuzim me duhan pranohet ne njesine
e kujdesit intensive me dhimbje derrmuese gjoksi e lidhur me rritje te segmentit ST dhe vale Q te vogla prekordiale.
Duke qene se simptomat e tij kane qene prezente per 36h, ai nuk eshte kandidat per trombolize. Kur u pranua ne
kujdesin intensive, presioni gjakut sistemik ishte 123/67mmHg, rrahjet e zemres 67/min pas beta bllokades dhe
saturimi 93% ne nje kanjule nazale 2L. Pjesa tjeter e examinimit fizik eshte normale. Ai u trajtua me Lisinopril, aspirin,
heparin dhe metoprolol. Para transferimit ne nje qender terciare, pacienti ankon per dispne extreme. U gjet diaforetik
dhe me FC=80 rrahje/min, PA= 84/56mmHg dhe saturim me oksigjen 93% on 100% non-rebreather. Pulmonet e tij
kane rale bilaterale difuze dhe venat e qafes jane te zgjeruara ne menyre te moderuar. EKG eshte e pandryshuar.
Radiografia e toraksit tregon per infiltrate te reja alveolare ne mushkerine e djathte, me shume se ne ate te majten.
Cila nga alternativat eshte gjetje e mundshme gjate ekzaminimit fizik?
A. Nje ton I 4 kardiak, zhurme sistolike III/VI, qe degjohet mire ne apex me kualitet ‘cooing’ qe irradijohet ne axille
B. Right ventricular heave (upward push on your hand when you palpate the precordium suggesting the presence of
hypertrophy), T2 I forte, zhurme III/VI qe rritet gate inspirimit ne kufirin e poshtem te djathte sternal
C. Nje ton I 3 kardiak, zhurme III/VI kreshendo-dekreshendo, qe degjohet me mire ne kufirin e siperm dex sternal
D. Reaksion urticarial difuz, wheezing gjate examinimit pulmonary
E. Edeme mukozale, edeme te gishtave, stridor
57. Cila nga alternativat eshte me e pershtatshme per hapin e rradhes ne terapine e pacientit te pyetjes 56?
A. Albuterol aerosol
B. Fillo infusion norepinephrine
C. Infuzion IV nitroprusidi
D. Metilprednisolon IV
E. Vendosje e balonit intraaortik
58. Nje grua e shendetshme 26 vjecare u vizitua per nje strisho pap ne nje vizite rutine. Ajo ndihet mire dhe ska ankesa,
as histori te rendesishme mjekesore. Internisti I saj performon nje ekzaminim fizik te plote dhe degjon nje klik
mezosistolik. Asnje zhurme apo gallop nuk eshte present. Ajo shqetesohet per kete gjetje. Cila nga theniet e eshte e
vertete persa I perket gjetjes gjate examinimit te saj fizik?
A. Ne shumicen e pacienteve me kete crregullim gjendet nje shkak baze si crregullim I trasheguar I indit lidhor.
B. Indikohet profilaksi per endokarditin infektiv gjate procedurave dentare potencialisht bakteremike.
C. Shumica e pacienteve jane asimptomatike nga ky lesion dhe ngelen keshtu gjate gjithe jetes.
D. Ajo duhet te filloje terapi me aspirine 325mg PO cdo dite.
E. Ky crregullim nuk mund te vizualizohet ne ekokardiografi.
59. Nje burre 78 vjecar vleresohet per fillimin e dispnese gjate efortit. Ai ka nje histori te gjate te abuzimit me duhan,
obezitet dhe diabet mellitus. Medikamentet e tij aktuale perfshijne metformin, aspirin, dhe ndonjehere ibuprofen.
Gjate examinimit fizik, pulset e tij periferike tregojne nje pik te vonuar dhe ai ka proeminent left ventricular heave. Ai
ka rritem te rregullt me zhurme IV/VI mezosistolike qe eshte me e forte ne bazen e zemres dhe irradijohet ne karotide.
Nje ton I 4 kardiak eshte present. Ekokardiografia konfirmon stenozen aortale severe pa lezione te tjera valvulare. Cila
nga alternativat ka me shume gjasa te kete kontribuar ne zhvillimin e lezionit te tij kardiak?
A. Valvul aorte bikuspide kongenitale
B. Diabet mellitus
C. Semundje kardiake reumatizmale okulte
D. Semundje ekzistuese e indit lidhor
E. Asnje
60. Nje burre 63vjecar shfaqet me sinkop new-onset pas efortit dhe iu gjet stenoze aorte. Gjate konsultes me pacientin,
I thua se rekomandimet e tua terapeutike jane te bazuara ne observimin qe pacientet e patrajtuar, me shenjat dhe
simptomat e tij, kane nje jetegjatesi mesatare prej:
A. 5 vite
B. 4 vite
C. 3 vite
D. 2 vite
61. Cila nga gjetjet e te examinimit fizik sugjeron per regurgitim aortic te rende?
A. pulsi Corrigan
B. pulsi alterans
C. pulsi bigeminus
D. pulsi paradox
E. pulsi parvus et tardus
62. Nje grua Somaleze 41 vjecare eshte vizituar per fillim te hemoptizise ne muajin e 6 te shtatezanise. Kjo eshte
shtatezania e saj e katert dhe te tjerat kane qene pa komplikacione, ajo ka qene 35 vjec ne lindjen e femijes se saj te
fundit. Para kesaj, ajo ka qene e shendtshme. Ajo tregon per dispne te lehte, qe filloi ne muajin e katert te shtatezanise
dhe shpejt pas kesaj edeme/fryrje te kembeve, qe ia atribuoi shtatezanise. Dyspnea eshte bere severe dhe ajo tani
eshte e kufizuar te ece rreth shtepise se saj. Ajo filloi te kollitej me sasi te vogla sputum me gjak para 5 ditesh. Skishte
as temperature as sputum purulent dhe nuk I eshte pergjigjur antibiotikeve te pershkruara nga obstetric I saj. Gjate
examinimit fizik u konstatua temperature normale, FC=100rrahje/min, PA=108/60mmHg dhe saturimi me oksigjen
90% ne ajrin e dhomes. Nuk eshte pare burim hemorragjie ne nares apo orofarinx. Pulmonet e saj kane krepitacione
difuze dhe examinimi kardiak tregon per rritje te moderuar te venave te qafes, ritem I rregullt kardiak, T2 I larte, dhe
nje low-pitched diastolic rumble qe degjohet me mire ne apex. Abdomen ka nje uterus gravid dhe eshte prezente
edeme e ekstremiteteve te poshtme 1+. Cila ka me shume te ngjare te demonstroje shkakun e simptomave te saj?
A. Bronkoskopi
B. CT toraksi me konstrast
C. Ekokardiogram
D. Kateterizim I zemres se djathte
E. Inspeksion I rrugeve te siperme te frymemarrjes nga nje ORL
63. Ne pacientin e pershkruar ne pyetjen 62, cila nga alternativat duhet pershkruar gjate vizites ne menyre qe te
lehtesohen simptomat e saj?
A. Benazepril
B. Digoxin
C. Furosemide
D. Heparin
E. Levofloxacin
64. Cili nga pacientet e meposhtem me evidence ekokardiografike te regurgitimit mitral te rendesishem ka indikacionin
me te mire per kirurgji me gjasat me te favorshme per nje rezultat pozitiv?
A. Nje burre 52vjecar me fraksion ejeksioni 25%, spimtoma te NYHA III dhe dimension 60mm fundsistolik te
ventrikulit te majte
B. Nje burre 54vjecar me fraksion ejeksioni 30%, simptoma te NYHA II dhe hypertension pulmonary
C. Nje burre 63vjecar me ritem sinusal pa simptoma, fraksion ejeksioni 65% dhe kateterizim normal zemres se djathte
D. Nje burre 66vjecar pa simptoma, me fraksion ejeksioni 50% dhe dimension 45mm fundsistolik te VM
E. Grua 72vjec asimptomatike me fibrilacion atrial se fundmi, fraksion ejeksioni 60%, dimension 35mm fundsistolik
65. Gjithe alternativat e jane shkaqe te mundshme te regurgitimit trikuspidal pervec:
A. Semundje kardiake kongenitale
B. Endokardit infektiv
C. Infakt miokardi I murit inferior
D. Hipertension arterial pulmonary
E. Semundje kardiake reumatizmale
F. Te gjitha do te shkaktojne regurgitim trikuspidal.
66. Te gjitha te meposhtem jane te verteta per zevendesimin e valvulave kardiake pervec:
A. Ne paciente te rinj preferohet zevendesimi valvular me valvula bioprostetike kundrejt atyre mekanike per shak
te kohezgjatjes me te madhe
B. Valvulat bioprostetike kane incidence te ulet te komplikacioneve tromboembolike
C. Risku I trombozes ne zevendesimin me valvul mekanike me I larte ne pozicionin mitral se sa ne pozicionin aortal
D. Valvulat mekanike jane kundraindikacion relativ ne paciente qe deshirojne shtatzani
E. Valvulat mekanike prostetike me disk te 2fishte ofrojne karakteristika hemodinamike superiore ndaj valv1disk.
67. Cili nga agjentet infeksioze te meposhtem eshte i lidhur me zhvillimin e miokarditit inflamator
A. Coxsackie virus
B. Diphtheria
C. Q fever
D. Trypanosoma cruzi
E. Te gjitha te mesipermet
68. Te gjitha t jane faktor risku per zhvillimin e kardiomiopatise peripartum pervec:
A. Mosha e rritur e nenes
B. Malnutricioni
C. Primipariteti
D. Barra binjake
E. Perdorimi i tokolitikeve
69. Nje burre 67 vjec me histori te gjate abuzimi me alkoolin paraqitet me gjetje konsistente te insuficiences
ventrikulare te majte, perfshire ketu dhe edeme pulmonare dhe kongjestion. I nenshtrohet kateterizimit te anes se
djathte dhe te majte te zemres. Nuk u konstatua semundje koronare sinjifikante. Cilat nga vlerat e te kateterizimit te
anes se djathte te zemres mbeshtet diagnozen e semundjes beriberi

A. prgj e sakte
70. Nje basketbollist 20 vjec vjen per vleresim te gjendjes shendetesore para fillimit te sezonit te ri. Ne kufirin e
poshtem te majte sternal degjohet nje zhurme e ashper sistolike, cila nga manovrat e do ta shtoje zhurmen nqs
shkaku eshte kardiomiopatia e dilatuar
A. Shtrengimi i dores (hand grip)
B. Perkulja para kur eshte I ulur
C. Shtrire ne anen e majte poshte
D. Squatting
E. Manovra e Valsaves
71. Nje grua 62 vjece paraqitet ne zyren tuaj me dipsne prej 4 muajsh. Ka histori per MGUS (monoclonal gammopathy
of unclear significance) dhe nuk ka bere follow-up qe para 5 vitesh. Mund te beje vetem aktivitet minimal pa pushuar,
por nuk ka simptoma ne qetesi. Ka zhvilluar ortopne por mohon te kete dispne paroksizmale nocturne. Ankon per
lodhje, marrje mendsh, dhe enjtje te ekstremiteteve te poshtme. Ne ekzaminim PA 110/90 mmHg dhe FK 94
rrahje/min. Ka rritje te presionit jugular venoz dhe vala jugulare venoze nuk bie me inspirimin. Eshte prezente nje S3
dhe S4 si dhe nje zhurme regurgitimi mitral. Pika e impulsit maximal nuk eshte e zhvendosur. Ekzaminimi abdominal
tregon ascit dhe hepar te zmadhuar, te ndjeshme dhe pulsatile. Radiografia e kraharorit tregon edeme pulmonare
bilaterale. EKG tregon bllok dege te majte te vjeter. Cila nga shfaqjet klinike diferencon perikarditin konstriktiv nga
kardiomiopatia restriktive
A. Rritja e presionit jugular venoz
B. Shenja Kussmaul
C. Presioni i pulsit(diferenca midis PAS dhe PAD) i ulet
D. Hepar pulsatil
E. Asnje nga te mesipermet
72. Po vleresoni nje pacient te ri ne klinike. 25 vjecari eshte diagnostikuar ne nje shtet tjeter me insuficience kardiake
dhe eshte zhvendosur qe atehere. Ka simptoma te klases II te New York Heart Association dhe mohon te kete angina.
Ai paraqitet per vleresim dhe trajtim. Pacienti ka qene per shume vite ne karroce invaliditeti dhe ka skolioze te rende.
Nuk ka histori familjare per hiperlipidemi. Ne ekzaminim fizik verehen kercitje pulmonare bilaterale, S3, por jo cianoze.
Ne EKG dallohen vale R te larta ne V1 dhe V2 me Q te thella ne V5 dhe V6. Ne ekokardiogram verehet disfunksion
ventrikular i majte global me reduktim te fraksionit te ejeksionit. Cila ka me shume gjasa te jete diagnoza
A. Skleroze laterale amiotrofike
B. Defekte septal atrial
C. Semundja tromboembolike kronike
D. Distrofia muskulare Duchenne
E. Kardiomiopatia ishemike
73. Nje grua 35 vjece me histori abuzimi me tobacco paraqitet ne urgjence me dhimbje kraharori te rende me perhapje
ne te dy krahet. Dhimbja ka filluar para 8 oresh dhe perkeqesohet ne inspirim. Ska mundur te shtrihet pasi kjo e
perkeqeson dhimbjen, por pacientja ndihet me mire kur perkulet para. Ne ekzaminim kemi FK 96 rrahje/min, PA
145/78 mmHg dhe Sat.O2 98%. Pulmonet jane te pastra dhe degjohet zhurme ferkimi me 3 komponente, qe degjohet
me shume ne kudirin e poshtem te majte sternal. Cila ka me shume gjasa te verehet ne EKG
A. Vale T te invertuara ne menyre difuze ne lidhjet prekordiale
B. Rritje e PR ne lidhjet II, III dhe aVF
C. Takikardi sinusale
D. Rritje e segmentit ST ne I, aVL dhe V2-V6 me konkavitet lart dhe depresion reciprok ne aVR
E. Rritje e segmentit ST ne V1-V6 me kurvature konvekse dhe depresion reciprok ne aVR.
74. Cili nga pohimet e eshte e vertete per pulsin paradoks
A. Konsiston ne rritje te PAS me me shume se 15 mmHg gjate inspirimit
B. Mund te gjendet ne paciente me semundje pulmonare obstruktive te rende
C. Eshte inversioni/ndryshimi i kundert i nje fenomeni normal gjate inspirimit
D. Rezulton nga distendimi ventrikular i djathte gjate ekspirimit qe con ne komprimimin e volumit ventrikular te
majte dhe per pasoje ulje te presionit te pulsit sistolik.
E. Te gjitha te mesipermet
75. Cila nga t jane karakteristika te triades Beck ne tamponade kardiake
A. Hipotension, alternime elektrike, x-descendent prominent ne venat e qafes
B. Hipotension, zhurma kardiake mbytese(muffled), alternime elektrike
C. Hipotension, zhurma kardiake mbytese(muffled), distendim i venave jugulare
D. Shenja Kussmaul, Hipotension, zhurma kardiake mbytese(muffled)
E. zhurma kardiake mbytese(muffled), hypotension, zhurma ferkimi
76. Nje grua 35 vjec vjen ne spital me dobesi/pafuqi, shtim ne peshe, rritje te perimetrim abdominal dhe edeme.
Simptomat kane filluar 3 muaj me pare dhe kane progresuar gradualisht. Pacientja referon per rritje te madhesise se
belit me rreth 15 cm. Enjtjet e kembeve kane ardhur duke u perkeqesuar sa tani ka dhe enjtje ne pjesen e siperme te
gjymtyreve (ne kofshe). Ka dispne ne efort dhe ortopne kur shtrihet mbi dy jasteke (two-pillow orthopnea). Ka histori
per semundjen Hodgkin te diagnostikuar ne moshen 18 vjec, per te cilen eshte trajtuar me kemioterapi dhe rrezatim
mediastinal. Ne ekzaminim fizik verehet humbje e mases muskulore te muskulit temporal (temporal wasting) dhe
duket e semure kronike. peshon 96 kg cka reflekton nje rritje prej 11 kg ne 3 muajt e fundit. Shenjat vitale jane normale.
Presioni jugular venos eshte rreth 16cm dhe venat e qafes nuk kolapsohen gjate inspirimit. Zhurmat kardiake jane te
largeta. Dallohet nje zhurme e trete kardiake pak pas mbylljes se valvules se aortes, e cila eshte e shkurter,e papritur
dhe degjohet me mire ne apeks. Hepari i zmadhuar dhe pulsatil, asciti prezent. Ka pitting edema ne gjymtyret e
poshtme dhe ne murin abdominal. Ekokardiograma tregon trashje perikardiale, dilatim VCI dhe venave hepatike, dhe
nderprerje e papritur e mbushjes ventriukulare ne diastolen e hershme. Fraksioni i ejeksionit 65%. Perqasja me e mire
A. Vetem diureze agresive
B. Transplant kardiak
C. Zevendesim i valvules mitrale
D. Rezeksion perikardial
E. Perikardiocenteze
77. Nje 19 vjecar lojtar hokeji, po mbronte porten kur u godit ne anen e majte te kraharorit me top dhe kolapsohet
menjehere ne akull. Trajneri vrapon drejt tij dhe e gjen te pandergjegjshem dhe pa puls. Cila nga t ka me shume gjasa
te jete pergjegjes per kete sindrome
A. Rupture aorte
B. Tamponade kardiake
C. “Commotio cordis”
D. Kardiomiopati hipertrofike
E. Pneumotoraks tensiv
78. Nje burre 48 vjec paraqitet per ekzaminim rutine. Nuk ka ankesa. PA eshte 134/82 mmHg me FK normale. BMI 31
kg/m2. Pjesa e tjereve e ekzaminimeve jane normale. Cila e vertete persa i perket modifikimit te stilit te jeteses
A. Ecje e shpejte per 10 min, 4 dite ne jave do ti ule PA ne vlera normale
B. Kufizimi i marrjes se NaCl ne me pak se 6 g ne dite do ti ule PA
C. Modifikimi i stilit te jeteses nuk do kete efekt ne PA
D. Reduktimi i konsumit te alkoolit ne 3 ose me pak gota ne dite do te ule PA
E. Humbja e peshes me rreth 9 kg pritet ta ktheje TA ne vlera normale
79. Nje grua 46 vjece pararqitet ne zyren tuaj e shqetesuar per diagnostikimin e saj me hipertension para 1 muaji. Ajo
pyet per riskun e shfaqjes se komplikacioneve te HTA, perfshire insuficiencen renale dhe stroke. Pacientja referon se
nuk ka histori mjekesore te kaluara te tjera pervec HTA dhe ska simptoma qe sugjerojne shkaqe sekondare. Aktualisht
po merr hydrochlorothiazide 25 mg/d. konsumon gjysme pakete cigare ne dite dhe alkool jo me shume se 1x/jave. Ka
histori familjare per HTA (prinderit). Mamaja a saj ka nderruar jete nga nje aksident cerebrovaskular. Babai eshte gjalle
por vuan nga semundje e arterieve koronare dhe eshte ne hemodialize. Pacientja ka PA 138/90 mmHg dhe BMI 23.
Nuk ka eksudat retinal apo shenja te tjera te retinopatise hipertensive. Pika e impulsit kardiak maksimal nuk eshte e
zhvendosur, Ritmi dhe frekuenca normale dhe pa galop, pulset periferike te mire. EKG tregon aks -30 grade me kritere
ne borderline te voltazhit te hipertrofise ventrikulare te majte. Kreatinina eshte 1.0 mg/dL. Cilet nga elementet faktor
risku per prognoze te varfer ne nje pacient me HTA:
A. Histori familjare per insuficience renale dhe semundje cerebrovaskulare
B. Rritje persistente e PA pas fillimit te terapise
C. Perdorimi i vazhdueshem i tobacco
D. Perdorimi i vazhduaeshem i alkoolit
E. Prania e hipertrofise ventrikulare te majte ne EKG
80. Nje 28 vjecare ka HTA te veshtire per tu mbajtur nen kontroll te diagnostikuar qe ne moshen 26 vjecare. Qe nga
ajo kohe ka marre mjekim me doze te larte. Regjimi aktual konsiston ne labetalol 1000 mg bid, lisinopril 40 mg qd,
clonidine 0.1 mg bid, dhe amlodipine 5 mg qd. Ne ekzaminim fizik PA 168/100 mmHg, FK 84 rrahje/min. ekzaminimi
kardiak eshte normal, ka pulse te mira periferike dhe ska edema. Ne pamjen e jashtme nuk verehet hirsutizem apo
keqshperndarje e yndyrnave apo anormalitete te genitaleve. Ne laborator verehet kalium 2.8 meq/dL dhe
bikarbonatet serike3 2 meq/dL, Glicemi esell 114 mg/dL. Cila eshte diagnoza e mundshme?
A. Hiperplazi adrenale kongenitale
B. Displazi fibromuskulare
C. Sindroma Cushing
D. Sindroma Conn
E. Feokromacitoma

81. Cila eshte menyra me e mire per dx e semundjes se pyetjes 80?


A. Nivelet e renines ne venen renale
B. Mbledhja e urines 24oreshe per metanefrina
C. MRI e arterieve renale
D. Mbledhja e urines 24 oreshe per kortizolin
E. Raporti plazmatik aldosteron/renin
82 Cili nga pacientet e meposhtem me disekacion aorte ose hematoma menaxhohet me mire pa terapi kirurgjikale?
A. Nje burre 74 vjec me disekacion qe perfshin rrenjen e aortes
B. Nje grua 45 vjece me disekacion qe perfshin aorten distalsisht origjines se vazave te medha por ne pjesen
cefalike te arterieve renale
C. Nje burre 58vjec me disekacion qe perfshin aorten distale dhe arteriet renale bilaterale
D. Nje burre 69 vjec me hematome intramurale ne rrenjen aortale
E. Te gjithe pacientet e mesiperm kerkojne menaxhim kirurgjikal te semundjes aortale

83. Nje burre 68 vjec paraqitet per follow-up rutine. Ai thote se ndihet mire dhe nuk ka ankesa. Ka histori mjekesore
per hipertension dhe hiperkolesteolemi. Vazhdon te konsumoje nje pakete duhan ne dite. Pacienti aktualisht merr
chlorthalidone 25 mg cdo dite, atenolol 25 mg cdo dite, dhe pravastatin 40 mg cdo nate. PA eshte 133/85 mmHg dhe
FK 66 rrahje/min. Ne ekzaminimin kardiak dhe pulmunar nuk verehet gje. Ne te majte te umbilikusit ndjehet nje mase
pulsatile rreth 4cm. Npm CT konfirmohet dx aneurizme e aortes abdominale me lokalizim infrarenal dhe permasa 4.5
cm. Te gjitha t jane te verteta ne lidhje me dx pervec:
A. Risku i ruptures ne 5 vite i nje aneurizme te kesaj permase eshte 1-2%.
B. Nderhyrja kirurgjike ose endovaskulare indikohet per shkak te madhesise se aneurizmes
C. Vendosja e nje stenti endovaskular infrarenal eshte nje opsion nqs aneurizma vazhdon te rritet
D. Nderhyrja kirugjikale ose endovaskulare indikohet kur pacienti shfaq simptoma te dhimbjes abdominale rekurente
apo dhimbje shpine
E. Nderhyrja kirugjikale ose endovaskulare indikohet nqs aneurizma zgjerohet mbi 5.5 cm
84. Nje grua 32 vjece vjen ne urgjence per marrje fryme akute. CT helical nuk tregon evidence per embol pulmonary,
por eshte bere nje shenim I rastesishem per dilatim te aortes ashendente ne 4.3 cm. Te gjitha lidhen me gjetje pervec:
A. Sifiliz
B. Arteriti Takayasu
C. Arteriti me qeliza te medha
D. Artrit Reumatoid
E. Lupus eritematoz sistemik
85. Nje burre 68 vjec me histori semundje te arteries koronare vjen ne kliniken e kujdesit paresor me ankese kolle me
sputum. Mjeku dyshon per pneumoni dhe I ben nje radiografi kraharori. Ne Rx aorta paraqitet tortuoze me mediastin
te zgjeruar. CT me contrast konfirmon pranine e nje aneurizme te aortes deshendente torakale me madhesi 4 cm pa
evidence disekacioni. Cila eshte menyra me e mire e menaxhimit:
A. Konsiderimi I radiologjise intervencionuese per vendosjen e nje stenti endovascular
B. Konsiderimi i kirurgjise torakale per riparim
C. Ska nevoje per vleresime te metejshme
D. CT me contrast I kraharorit cdo vit dhe referimi per riparim kirurgjikal nqs madhesia e aneurizmes > 4.5 cm
E. Trajtim me betabllokues, CT me contrast cdo vit dhe referim per riparim kirurgjikal nqs aneurizma rritet me
shume se 1 cm ne vit

86. Nje grua 37 vjece, pa histori te rendesishme mjekesore te kaluar pervec nje zhurme ne femijeri, vleresohet per nje
dhimbje te forte me fillim te papritur ne ekstremitetin inferior te djathte. Shenjat vitale normale pervec FK 110
rrahje/min. kemba e djathte e zbehur distalisht gjurit dhe eshte e ftohte ne prekje dhe mungon pulsi ne dorsalis pedis.
Cili nga t ka me shume gjasa te gjeje shkakun:
A. Angiografi e ekstremitetit inferior te djathte
B. Culture gjaku
C. Ekokardiogram me studim “bubble”
D. c-ANCA serike
E. Ultrasound venoze e ekstremitetit superior te djathte
SISTEMI PULMONAR Harrison 19
1. Te gjitha patologjite me poshte karakterizohen si semundje obstructive pulmonare me perjashtim te:
A. Asbestoza
B. Astma
C. Bronkektazia
D. Bronkiti kronik
E. Emfizema
2. Nje M 25vj eshte sjelle ne departamentin e emergjencave nga ambulanca pasi familja e tij e gjeti te pavetedijshem
ne shtepi. Ai ka nje histori te abuzimit te perdorimit te drogerave IV dhe HIV dhe mosrespektimit te rregullave
mjekesore. Matja e CD4 per here te fundit ka qene < 200/uL. Ne vleresimin fillestar, PA 120/75 mmHg, FK 105/min, FR
8/min, saturimi i O2 83% dhe temp 36 grade celcius. Matja e gazeve ne gjak tregon pH 7.16, presioni parcial i CO2 --
70mmHg, presioni parcial I O2—55mmHg. Kush eshte diagnoza e mundshme?
A. Astma
B. Overdoza narkotike
C. Pneumonia pneumokoke
D. Pneumonia pneumocistike
E. Embolizmi pulmonar
3. Në cilin vëllim të pulmonit barazohet tërheqja e jashtme e murit të gjoksit me tërheqjen elastike të brendshme e
pulmoneve?
A. Volumi rezerve residual
B. Kapaciteti funksional residual
C. Volumi residual
D. Volumi tidal
E. Kapaciteti total i mushkerive
4. Nje M 65vj eshte vleresuar me nje dispne progresive
ne sforcim e cila ka persistuar pergjate tre muajsh. Ka
histori mjekesore per pankreatit nekrotizant e cila ka
rezultuar ne insuficience mutiorganore dhe sindromin e
distresit respirator akut. Ai eshte trajtuar me ventilim
mekanik per 6 jave derisa eshte permiresuar. Ka histori
te perdorimit per 30 vite te duahnit, pork a 15 vite qe e
ka nderprere. Nuk eshte diagnostikuar per semundje
kronike obstructive pulmonare. Ne ekzaminimin fikzik,
degjohet nje fishkellime e lehte inspiratore dhe
ekspiratore, ndersa zhurma me e forte degjohet ne
zonen e mesme pulmonare. Ne testin e funksionit
pulmonar, FEV1→2.5L , FVC→ 4.00, FEV1/FVC→ 62.5%.

Bazuar ne te dhenat e mesiperme, kush eshte shkaktari i mundshem I ketyre simptomave?


A. Aspirimi i nje trupi te huaj
B. Semundja kronike obstructive pulmonare
C. Fibroza pulmonare idiopatike
D. Stenoza subglotike
E. Paraliza unilaterle e kordave vokale
5. Nje F 22 vj vjen ne departamentin e emergjencave ne javen e 23 te shtatezanise duke u ankuar per dispne akute.
Ajo ka nje shtatezani e shendetshme dhe nuk ka probleme te tjera mjekesore. Nuk po merr mjekim pervecse vitaminat
prenatale. Ne ekzaminim, ajo paraqitet dispenike. Shenjat jetesore jane si me poshte: PA 128/78mmHg, FK 126/min
FR 28/min, Saturimi I O2 96% ne ajrin e dhomes. Eshte afebrile. Ekzaminimi pulmonar dhe kardiak normal. Ka shenja
bilaterale tee demes. Grafia e toraksit rezulton normale ndersa EKG tregon per nje takikardi sinusale. Shikohen gazet
e gjakut arterial. pH 7.52, presioni parcial I CO2 ne arterie eshte 26mmHg, presioni parcial i oksigjenit arterial eshte
85mmHg. Kush eshte hapi I rradhes ne diagnozen dhe menaxhimin e pacientit?
A. Terapi fillestare me Amoxicillin per bronkit akut
B. Angiograme pulmonare e vezhguar nga CT
C. Ekokardiograme
D. Sqarimi i pacientit qe dyspnea normale gjate shtatezanise dhe nuk ka anormalitete te verejtura apo te testuara.
E. Trajtim me clonazepam per atak paniku.
Lidh secilin nga testet funksionale pulmonare me crregullimin respirator te mundshem
6: Miastenia gravis
7: Fibroza pulmonare idiopatike
8: Hipertension pulmonar familjar
9: Semundje obstructive kronike pulmonare
A. Rritje TLC, ulje VC, ulje FEV1/FVC Semundje obstructive kronike pulmonare
B. Ulje TLC, ulje VC, ulje RV, rritje FEV1/FVC , presioni maksimal inspirator normal
Fibroza pulmonare idiopatike
C. Ulje TLC, rritje RV, FEV1/FVC normale, presioni maksimal inspirator i ulur Miastenia gravis
D. TLC normale, RV normale, FEV1/FVC normale, presioni maksimal inspirator normal.
Hipertension pulmonar familjar
TLC: kapaciteti total pulmonary, CV: kapaciteti vital.
10. Nje F 78vj u pranua ne ICU me pneumoni multilobare. Ne prezantimin fillestar ne departamentin e urgjences
saturimi i O2 pa suport ishte 60% dhe u rrit deri 82% me faske faciale. Ajo konsiderohej me distres respirator dhe u
intubua ne dhomen e urgjences. Pas pranimit ne ICU, aju u sedua. Ventilatori u vendos ne system te kontrolluar
respirator me FR 24/min, volume tidal 6mL/kg, FiO2 1.0, dhe presion pozitiv ne fund-ekspiracioni 12cmH2o. Nje matje
e gazve te gjakut arterial rezultoi: pH 7.20, PCO2 32mmHg, PO2 54mmHg. Kush eshte shkaku i hipoksemise?
A. Hipoventilimi
B. Hipoventilimi dhe mosperputhja ventilim-perfuzion
C. Shunti
D. Mosperputhja ventilim-perfuzion
11. Zonja Wittstine, 72vj ankohet per temperature te lehte dhe dispne pergjate 2 javeve. Ajo ka nje histori 10 vjecare
te sklerodermise me perfshirjen e ezofagut. Ajo ka nje histori 30 vjecare te perdorimit te duhanit por e ka lene para 8
vitesh. Ne grafine e toraksit, ajo ka nje infiltratne lobin e djathte te poshtem. PET/CT zbulon qe lezioni ne lobin e djathte
te poshtem eshte me diameter 3 cm me karakteristika te infiltratit nodular, dhe rritje te kapjes se FDG
(fluorodeoxyglucose). Cila nga pohimet per zonjen Wittstine eshte me e sakte?
A. Duhen studime te metejshme diagnostike
B. Gjetjet ne PET/CT sugjerojne per nje infeksion te mundshem
C. Gjetjet ne PET/CT nuk sugjerojne per nje infeksion te mundshem
D. Gjetjet ne PET/CT sugjerojne per nje situate malinje te mundshme
E. Gjetjet ne PET/CT nuk sugjerojne per nje situate malinje te mundshme

12. Nje M 65vj u vleresua me dispne progresive ne sforcim dhe kolle te thate te cilat jane perkeqesuar ne 6 mujorin e
fundit. Ai nuk ka part dispne gjate pushimit dhe nuk pranon te kete patur zhurma gjate frymemarrjes. Ai nuk ka patur
eksperianca me dhimbje te gjoksit. Ka nje histori te semundjeve te arterieve koronare, dhe fibrilacionit atrial dhe se ka
bere nje bypass te a. koronare 12 vite me pare. Mjekimet e tij perfshijne: Metoprolol, Aspirin, Varfarin, Enalapril. Ai ka
konsumuar nga nje pakete cigare cdo dite per 40 vite, dhe e ka lene para 5 vitesh. Shenjat vitale jane: PA 122/68, FK
68, FR 18, Saturimi I O2 92% (pa suport te O2) . Ekzaminimi i toraksit demonstron kercitje bibasilare prezente ne 1/3
e rruges se siperme bilaterale. Nuk degjohet “wheezing”. Ai ka nje ritem crregullisht te crregullt me nje II/IV zhurme
holosistolike ne apex. Presioni venoz jugular nuk eshte i rritur. Nuk ka edeme prezente, por verehet “clubbing”. Testi
pulmonar: FEV1 65%, FVC 67%, FEV1/FVC 74%, TLC 68%, kapaciteti I difuzionit per CO (DLCO) 62%. Cili ekzaminim
eshte me efikas per te zbuluar etiologjine e dispense?
A. Bronkoskopi me biopsi transbronkiale te mushkerive
B. Angiografi nen CT pulmonic
C. Ekokardiografi
D. CT toraksi me rezolucion te larte
E. Nuclear medicine stress test
13. Kush eshte faktori kryesor i riskut per astmen?
A. Ndotja e ajrit
B. Atopia
C. Dieta
D. Pirja e duhanit gjate shtatezanise
E. Infeksione virale te traktit te siperm respirator
14. Nje F 24 vj ankohet per veshtiresi ne frymemarrje dhe “wheezing”. Ajo ka vene re qe simptomat perkeqesohen kur
ben ushtrime fizike dhe kur eshte prane maceve. Ajo ka patur rinit alergjik ne pranvere dhe ne vere per shume vite
dhe ka vuajtur nga eczema ne moshe feminore. Ne ekzaminim fizik, verehet nje “wheezing” ekspirator. Testet
pulmonare: FEV1 2.67 (79%), FVC 3.81 (97%), FEV1/FVC 70%. Pas dhenies se albuterolit, FEV1 u rrit ne 3.0L (12.4%).
Cila nga pohimet e lidhur me semundjen e pacientit eshte e vertete?
A. Konfirmimi i diagozes kerkon testin e proves me metakoline
B. Mortaliteti I lidhur me semundjen ka filluar te rritet ne dekaden e fundit
C. Faktori I riskut me i shpeshte ne individet me kete crregullim eshte predispozicioni gjenetik
D. Prevalenca e crregullimit nuk ka ndryshuar pergjate dekadave te fundit
E. Ashpersia e semundjes nuk varion ne menyre sinjifikative ne nje pacient me kete semundje.
15. Nje F 38 vj eshte sjelle ne departamentin e urgjences per status astmatik. Ajo menjehere perkeqesohet dhe ben
exitus. Te gjitha gjetjet patologjike kane mundesi te jene pare tek kjo paciente, me perjashtim te:
A. Infiltrimi I mukozes se rrugeve te frymemarrjes me eozinofile dhe Limfocite T te aktivizuar.
B. Infiltrimi I hapesirave alveolare me eozinofile dhe neutrofile
C. Bllokimi I lumenit te rrugeve te frymemarrjes me mucus
D. Trashje dhe edeme te murit te rrugee te frymemarrjes
E. Trashje e membranes bazale e rrugeve te frymemarrjes me depozitim te kolagjenit subepitelial
16. Cili nga pacientet e meposhtem eshte i diagnostikuar me astme?
A. F 24vj e trajtuar me kortikosteroide inhalatore per kollen dhe “wheezing” te cilat kane persistuar per 6 jave duke
u pasuar nga nje infkesion I rrugeve te siperme respiratore.
B. M 26vj i cili kollitet dhe me raste ka “wheezing” gjate ushtrimeve ne mot te ftohte.
C. F 34 vj e vleresuar per kolle kronike me FEV1/FVC 68%, me FEV1 i cili rritet nga 1.68L ne 1.98L pas perdorimit te
albuterolit (ndryshim FEV1 18%)
D. M 44vj i cili punon sinje teknik duke u kujdesur per minjte ne nje laborator te kerkimit mjekesor. Ankohet per
“wheezing”, veshtiresi ne frymemarrje, dhe kolle te cilat jane me te renda ne fundjave.
E. M 60vj I cili ka konsumuar dy paketa cigare ne dite per 40 vite. Ka dispne, kolle dhe hiperaktivitet te rrugeve te
frymemarrjes ne pergjigje te metakolines.
17. Nje F 24 vj u diagnostikua me astme 4 muaj me pare dhe u trajtua me albuterol inhalator sipas nevojes. Qe nga
vizita e fundit, ajo ndihet pergjithesisht mire dhe zakonisht ka neoje per te perdorur inhalatorin afersisht 4-7 here ne
jave kue gjendet prane polenave, maceve ose kur ben ushtrime fizike ne ajer te ftohte. Aluteroli inhalator zakonisht e
ndihmon dhe pacientja ka nevoje per nje perseritje te inhalacionit afersisht 2 here ne jave. Ajo nuk merr mjekime te
tjera dhe nuk pi duhan. Nuk mban kafshe shtepiake ne shtepi, pervecse nje peshku te quajtur Puffer. Bazuar ne
informacionin e mesiperm, cfare keshille do ti jepje pacientes?
A. Beklometason inhalator
B. Salmeterol inhalator dy here ne dite
C. Tiotropium inhalator
D. Te vazhdoje terapine qe ka aktualisht
18. Nje F 28 vjece me nje astme te lehte persistente per shume kohe vjen tek mjeku sepse zbuloi qe sihte shtatezene.
Mjekimi I saj I vetem eshte beklometasoni inhalator 2 here ne dite dhe albuterol sipas nevojes. Ajo tipikisht e perdor
albuterolin me pak se dy here ne jave. Ajo do te dije se cfare e pret rreth ashpersimit te astmes dhe nese duhet te beje
ndonje ndryshim te mjekimit ne kete periudhe. Cila nga pohimet eshte i sakte?
A. Duhet te vazhdoje terapine aktuale dhe te kotrolloje simptomat
B. Duhet te zevendesoje albuterolin inhalator sipas nevojes me tiotropium inhalator sipas nevojes
C. Duhet te zevendesojebeklometasonin inhalator me salmeterol inhalator
D. Ka nje mundesi me shume se 70% qe simptomat e astmes do te jene me pak te renda gjate shtatezanise
E. Ka nje mundesi me shume se 70% qe simptomat e astmes do te jene me te renda gjate shtatezanise.
19. Nje F 38 vj u pranua ne ICU me insuficience respiratore akute hipoksemike. Ajo ishte e mire me shendet deri para
4 diteve kur ajo papritur filloi te ndihej e semure me temperature, te dridhura, dhimbje gjoksi pleuritike bilaterale dhe
perkeqesim te vehtiresise se frymemarrjes. Ajo nuk ka histori semundjesh ne te kaluaren por se fundmi ka vuajtur nga
largimi nga jeta I babait te saj nga nje aksident me makine. Pas humbjes se tij ajo filloi te pinte duhan perseri pas 15
vitesh qe e kishte lene. Filloi te pite deri ne dy paketa duhan cdo dite. Pasi filloi te ndihej e semure, ajo filloi te perdorte
Acetaminophen dhe pseudoephedrine, por nuk merr mjekim tjeter. Sapo mberriti ne departamentin e urgjences,
saturimi i O2 pa suport ishte 78%. Me ane te maskes faciale, saturimi shkoi ne 92%. Shenjat vitale jane: temp 38.7
grade, FK 122, FR 28, PA 132/82mmHg. Ajo paraqitet ne veshtiresi respiratore te moderuar. Ka kercitje bilaterale
difuze. Ekz. Kardiovaskular tregon takiaritmi te rregullt pa zhurma. Presioni jugular venoz nuk eshte i ngritur dhe nuk
ka edeme prezente. Abdomeni eshte i bute dhe jo i tendosur. Nuk ka hepatosplenomegali. Ekzaminimet neurologjike
dhe te ekstremiteteve jane normale. Radiografia e toraksit tregon infiltrate bilaterale difuze. Ekokardiograma e saj
tregon fuksion normal te sistoles dhe diastoles se majte. Pacientja trajtohet me ceftriaxone 1 g IV cdo dite dhe
azithromycin 500 mg IV cdo dite. Ne 24h e para, klinika e pacientit filloi te perkeqesohej. Ajo qendronte febrile, dhe
kishte te nevojshem intubimin dhe ventilimin mekanik. Ventilatori u vendos ne kontroll ne FR 28, volum tidal 330ml,
FiO2 0.8, PEEP 12cmH2O. Parametrat e gazeve ne gjak: pH 7.28, PaCO2 68mmHg, PaO2 62mmHg. U krye nje lavazh
bronkoalveolar. Analizat : 58% neutrofile, 12% limfocite, 30% eozinofile. Hapi me i mire ne trajtimin e pacientit?
A. Konsulte per biopsi pulmonare.
B. Vazhdo regjimin actual me antibiotike IV nderkohe presim per te dhenat e kultures.
C. Fillo metilprednizolon 60mg IV cdo 6 ore
D. Fillo oseltamivir 75mg dy here ne dite
E. Fillo terapi me trimethoprim/sulfamethoxazole IV me prednizon 40 mg dy here ne dite.

20. Nje M 34 vj u referua ne spital per vleresimin e astmes persistente severe. Ai u diagnostikua me astme ne vitet e
adoloshences. Ne ate kohe, ai fillimisht zbuloi simptoma kur po vraponte. Ai zakonisht ka qene nje person aktiv, por
ne 10 vitet e shkuara astma e tij u be e veshtire per tu kontolluar. Ai ka qene I hospitalizuar 3 here per acarimet e
astmes ne 3 vitet e fundit. Pacienti nuk vrapon me dhe ka fituar 30 kg pergjate kohes ku ka karre ne menyre kronike
prednizon oral. Regjimi mjekesor actual eshte fluticasone/salmeterol 500/50 μg dy here ne dite, tiotropium 18 μg ne
dite, montelukast 10 mg ne dite, esmoprazole 40 mg ne dite, fluticasoneme ane te sprayt nazal cdo dite dhe
prednozone 10 mg cdo dite. Ai perdor nebulizatorin me albuterol rreth 4 here ne dite dhe zgjohet naten per ta perdorur
rreth tre here ne jave. Ka nje kolle kronike dhe sekrecione mukusi ngjyre kafe. Ne ekzaminim ka turbinatet nazale te
zmadhuara por jo popile. Degjohet “wheezing” bilateral difuz. Nuk ka kercitje prezente. Radiografia e toraksit tregon
hiperinflamim me nje infiltrat ne zonen e siperme pulmonare ne lobin e siperm te djathte. Tre muaj me pare, ai kishte
nje zone atelektaze me zona mukusi ne lobin e poshtem te majte. Eozinofilet periferike 750/μL. Cila nga ekzaminimet
e eshte e indikuar ne vleresimin dhe trajtimin e pacientit?
A. Desensibilizimi i aspirins
B. Ct toraksi
C. Nasal nitric oxide testing
D. Ig E ne serum
E. Sweat chloride testing
21. Nje F 34vj ka ankesa per kolle dhe dispne ne sforcim tecilat jane perkeqesuar ne tre muajt e fundit. Pacienti nuk ka
histori te shqetesimeve pulmonare dhe nuk ka patur kurre astme. Ajo filloi te punonte ne nje dyqan kafshesh shtepiake
afersisht 6 muaj me pare. Detyrat e saj perfshinin pastrimin e zvarranikeedhe kafazeve te zogjve. Ajo raporton qe ka
patur me raste temperature pak te larte por nuk ka patur “wheezing”. Kolla eshte e thate dhe joproduktive. Para tre
muajsh, pacientja nuk ka patur limitime ne tolerance e ushtrimeve fizike, por ajo tani tregon qe ka dispne kur ngjit
shkallet. Ne ekz fizik, pacientja paraqitet mire. Saturimi i O2 eshte 95% ne ajrin e dhomes gjate pushimit por
desaturohet ne 89% ne levizje. Temp eshte 37.7 grade celcius. Ekzaminimi pulmonar nuk ka te dhena ne vecanti. Nuk
ka cianoze apo “clubbing”. PAcienti ka nje radiograme normale te toraksit. Nje CT toraksi me rezolucion te larte tregon
“diffuse ground-glass infiltrates” ne lobet e poshtme me prezencen e nodulave centrolobulare. Biopsia transbronkiale
tregon infiltrate alveolar intersticial te qelizave plazmatike, limfocitee, dhe me raste te eozinofileve. Ka edhe disa
granuloma. Te gjitha kulturat jane negative per patogene bakteriale, virale dhe fungale. Cila eshte diagnoza?
A. Aspergiloza
B. Pneumoniti hipersensitiv
C. Pneumoniti intersticial jospecifik i lidhur me semundjen vaskulare te kolagjenit
D. Psittacosis
E. Sarkoidosis
22. Cilin trajtim do te rekomandoje per pacientin ne pyetjen 21?
A. Amphotericin
B. Doxycycline
C. Glukokortikoide
D. Glukokortikoide + azathioprine
E. Glukokortikoide + removal of antigen
23. Nje M 75vj eshte vleresuar per efuzion pleural te majte dhe veshtiresi ne frymemarrje. Ai ka punuar si punetor
izolimesh ne kantier detar per me shume se 30 vite dhe nuk ka veshur pajisjen mbrojtese respiratore. Ka dale ne
pension ne moshen 60 vjecare. Ai ka 50 vite qe perdor duhanin dhe njihet per SPOK te moderuar (FEV1 55%) dhe ka
kaluar infarct miokardi para 10 vitesh. Mjekimet e tanishme perfshijne: aspirin, atenolol, benazepril, tiotropium dhe
albuterol. NE ekzaminim fizik ka efuzion te zones se majte me timpanizem ne perkusion dhe ulje te hurmave te
frymemarrjes perreth gjysmes se hemitoraksit. Ne grafi toraksi ka nje efuzion pleuralte majte me kalcifikime pleurale
bilaterale dhe trashje plaurale apikale te majte. Nuk shihen masa ne mushkeri. Nje CT toraksi konfirmon grafine dhe
nuk gjen mase ne pulmon. Ka nje atelektaze ne lobin e poshtem te majte. U krye ne torakocenteze duke gjetur efuzion
eksudati me limfocite 65%, qeliza mezoteliale 25%, neutrofile 10%. Citologjia nuk demonstron mailinjitet. Kush nga
pohimet e lidhur me shkakun me te mundshem te efuzionit te pacientit apo gjendjen shkaktare eshte e vertete?
A. Pirja e duhanit e rrit mundesine e zhvillimir te kesaj gjendjeje
B. Vdekja ne kete semundje eshte zakonisht e lidhur me metastazat difuze
C. Ekspozimi I agjentit shkaktar 1-2 vite dhe latenca per shfaqjen e semundjes mund te jete edhe 40 vite
D. Ctologjia e lengut pleural e perseritur me shume mundesi mund te jape nje diagnoze definitive
E. Terapia me kombinim rezeksion kirurgjikal + kemioterapi adjuvant permireson mbijetesen per nje kohe te gjate
24. Silikoza kronike eshte e lidhur me nje risk te rritur per cilen nga situatat e ?
A. Infektimi me Aspergillus
B. Infektimi me Mycobacterium Tuberculosis
C. Kanceri i mushkerive
D. Artriti rheumatoid
E. Te gjitha te mesipermet
25. Te gjitha semundjet e te mushkerive jane te lidhura sakte me ekspozimin pervec:
A. Berylliosis—High-technology electronics
B. Byssinosis—Cotton milling
C. Farmer’s lung—Moldy hay
D. Fibroze masive progresive—Punonjesit ne kantier detar
E. Metal fume fever— Saldim
26. Nje M 53vj vjen ne urgjence me temperature te papritur, te dridhura, pa fuqi, veshtiresi ne frymemarrje po jo
“wheezing”. Ai nuk ka probleme mjekesore ne te shkuaren. Eshte fermer. Shenim: Pacienti me heret gjate dit ka
mbledhur kashte. Grafia e toraksit posteroanteriorisht dhe lateralisht, tregon infiltrate alveolare bilaterale difuze. Cili
organizem eshte me shume I mundshem te jete pergjegjes per kete situate?
A. Nocardia asteroids
B. Histoplasma capsulatum
C. Cryptococcus neoformans
D. Actinomyces species
E. Aspergillus fumigatus
27. Nje F 36vj eshte sjelle ne ICU pasi ra zjarri ne shtepi. Ajo eshte e keputur por nuk ka patur vuajtje apo djegie te
medha. Pacientja u trajtua per inhalimin e tymit. Perqendrimi fillestar I karboksihemoglobines eshte 25%. Ajo u trajtua
me 100% O2 por qendron prap e keputur. Ne menyre jokoshiente zhvillon “generalized seizures”. PA bie me shpejtesi
ne 60/40 mmHg. FK 150/min. Saturimi i O2 eshte 98%. Pacientja ka nje fytyre te kuqe dhe eshte e pranishme aroma e
bajames se hidhur. Ka bloze perreth hundes dhe gojes se pacientit. Pupilat jane te dilatuara dhe lekura eshte
diaforetike. Cili nga ekzaminimet e do te ndihmoje ne diagnoze?
A. Perqendrimi i amoniakut
B. Niveli i laktateve ne gjak
C. Niveli i karboksihemoglobines
D. Red blood cell cyanide concentration
E. Niveli i methemoglobines
F. Glukokortikoide
28. Nje F 51vj ankohet per kolle te perditshme e cila eshte produktiveme sputum viskoz dhe ngjyre jeshile. Kolla eshte
me e rende kur zgjohet ne mengjes. Ne kete periudhe ka me raste shenja te gjakut ne sputum. Kolla I ka nisur perpara
7 viteve dhe eshte perkeqesuar ne menyre progresive me rritjen e sasise se sputumit. Ne kete periudhe ajo ka vene re
se ka nxjerre sputum cdo dite sa gjysma e provezes. Pacientja tregon se ka patur nevoje shpesh per perdorimin e
antibiotikeve per traktin e poshtem respirator dhe infeksionet sinusale. Ne zonat pulmonare te poshtme degjohen
“Bilateral coarse crackles”. Nuk ka “clubbing” present. Testet pulmonare: FEV1 1.68L (53.3%), FVC 3.00L (75%),
FEV1/FVC 56%. Kultura e sputumit tregon per rritjen e Pseudomonas Aeruginosa. Cili eshte ekzaminimi I rradhes
A. Ekzaminimi me barium
B. Bronkoskopi
C. Grafi toraxi
D. CT toraxi
E. Sweat chloride testing
29. Nje F 62vj u diagostikua me bronkektazi dhe u trajtua per Mycobacterium avium-intracellulare, gje e cila u
konfirmua ne dy kultura positive te sputumit. Ajo eshte trajtuar per 24 muaj me clarithromycin, ethambutol dhe
rifampicin. Kulturat e sputumit kane qene negative per mykobakteret ne 12 muajt e fundit. Megjithate, ajo vazhdon
te prodhoje sputum cdo dite, dhe CT tregon bronkektazi ne lobin e mesem te djathte. Trajtimi me i mire per pacientin
A. Konsulte kirurgjie torakale per lobektomi te mesme dexter
B. Vazhdo trajtimin me clarithromycin, ethambutol dhe rifampicin
C. Fillo pastrimin e rrugeve te frymemarrjes me “oscillating positive expiratory pressure flutter valve”
D. Nis trajtimin me dornase (DNase)
E. Nis trajtimin me prednisone 20mg cdo dite
30. Nje foshnje e sapolindur u diagnostikua me fibroze cistike kur testi i djersitjes rezultoi pozitiv: 110 mmol/L . Ky kest
u kryes pasi screening test i te porsalindurit doli jonormal. Medikamenti Ivacaftor, aktualisht i aprovur nga FDA, eshte
i indikuar ne pacientet me mutacionin genetic te meposhtem:
A. G542X
B. G551D
C. F508del
D. 621 + 1G > T
31. Ivacaftor i cili perdoret ne pacientet me fibroza cistike ka kete veprim ne proteinen CFTR:
A. Corrects protein slicing abnormalities
B. Permireson percueshmerine permes kanalit te joneve klor
C. Nxit sintezen e CFTR
D. Ndihmon ne vendosjen e CFTR ne membranen qelizore
E. Stabilizon CFTR me qellim degradimin e ngadalshem
32. Nje F 28vj u vleresua per infeksione sinusale dhe pulmonare te perseritura. Ajo mban mend qe episode me te
hershme te bronkitit filluan ne moshen e hershme te adoloshences. Ne 5 vitet e kaluara, ajo tregon se ka marre
antibiotike te pakten tre here ne vit per infeksion sinusal dhe pulmonar. Ajo tregon gjithashtu se ka veshtiresi qe te
fitoje peshe dhe gjithmone eshte ndjere e shkurter duke u krahasuar me bashkemoshataret e saj. Ne ekzaminim fizik,
IBM: 18.5 kg/m2. Saturimi I O2 ne ajrin e dhomes eshte 95%. Ka prezente polipe nazale. Ne zonat e siperme pulmonare
bilateralisht degjohen kercitje dhe “Coarse rhonchi”. Grafia e traxit tregon bronkektazi bilaterale e lobit te siperm me
zona mukusi. Mjeku mendon per mundesine e nje fibroze cistike te padignostikuar. Cili nga ekaminimet e do te jepte
ndihmen me te madhe ne diagnostikimin e fibrzes cistike ne kete paciente?
A. Analiza e AND e cila demonstron nje kopje te alelit F508del
B. Decreased baseline nasal potential difference
C. Prezenca e P aeruginosa ne kulturat e perseritura te sputumit
D. Vlera e testit te djerses > 40 mmol/L
E. Vlera e testit te djerses > 60 mmol/L
33. Nje M 22 vj me fibroze cistike erdhi tek mjeku per nje kontroll rutine. Ai aktualisht po trajtohet me DNase dhe
albuterol me nebuliator /2 here ne dite. Teknikat e tij per te pastruar sputumin jane ushtrimet aerobike 5 here ne jave
dhe drenimi autogjenik. Pacienti pergjithesisht ndihet mire dhe ekzaminimi eshte normal. Testet pulmonare FEV1
4.48L (97%), FVC 5.70L (103%), FEV1/FVC 79%. Kultura e sputumit e marre ne menyre rutine tregon per rritjen e P
aeruginosa. Organizmi I vetem I izoluar ne kulturat Staphylococcus aureus. Cfare rekomandoni per kete pacient?
A. Oshilacione me frekuence te larte te murit te toraksit
B. Saline hipertonike (7%) me nebulizator, 2 here ne dite
C. Tobramycin inhalatore 300mg, 2 here ne dite
D. Cefepime IV dhe tobramycin IV per 14 dite
E. Rikthim pas 3 muajsh per te perseritur kulturen e sputumit dhe mjekim vetem P aeruginosa persistente.
34. Një 69-vjeçar me COPD është pranuar në spital tre herë gjatë vitit të kaluar për përkeqësime/ exacerbime të COPD.
Ai ka kollë të përditshme me sputum dhe parashikohet një FEV1 prej 45%. Ai më parë pinte një paketë cigare çdo ditë
për 50 vjet, la duhanin 1 vit më parë. Saturimi në ajrin e dhomës është 91%. Cili nga trajtimet e mëposhtme ka më
shumë të ngjarë të zvogëlojë frekuencën e exacerbimeve të tij?
A. Azithromycin 250 3x/jave
B. Oksigjen i vazhdueshem at 2 L/min
C. Nocturnal bilevel positive airway pressure with an inspiratory pressure of 18 cm H2O and expiratory pressure
of 12 cm H2O
D. Roflumilast 500 μg / d
E. Theophylline 300 mg / d
35. Te gjithe me poshte jane faktore risku per COPD, PERVEC:
A. Hiper reaktivitetit te rrugeve ajrore
B. Ekspozim nga pluhuri i qymyrit
C. Ekspozim pasiv i tymit te duhanit
D. Infeksionet rekurrente respiratore
E. Perdorimi i karburanteve me lende biologjike biomass ne zona me ventilim te keq
36. Një grua 65-vjeçare vlerësohet për dispne gjatë sforcimit dhe kollë kronike. Ajo ka një histori të gjatë të përdorimit
të duhanit, duke pirë 1.5 pako cigare çdo ditë që nga mosha 20 vjeç. Ajo është një grua e dobët dhe pa ndonjë
shqetësim/distress të dukshëm. Saturimi në ajrin e dhomës është 93% me FR prej 22 frymëmarrjesh / min. Mushkëritë
janë shume te zgjeruara ne perkusion me
tinguj frymëmarrjeje të zvogëluar në
fushat e sipërme pulmonare. Ju dyshoni
për COPD. Cilat janë gjetjet e pritshme në
testet e funks pulmonar?
A. A
B. B
C. C
D. D

37. Një burrë 70-vjeçar me COPD të njohur vjen për ndjekje. Ai ka qenë klinikisht i qëndrueshëm pa ndonjë
përkeqësim/exacerbim për 6 muajt e fundit. Sidoqoftë, ai përgjithësisht ndihet në shëndet të dobët dhe është i
kufizuar në atë që mund të bëjë. Ai raporton dispne ne aktivitet të zakonshem. Ai aktualisht po menaxhohet me
albuterol inhalator MDI (metered dose inhaler) 2x/d dhe sipas nevojës. Ai ka një histori 50-vjeçare të pirjes së duhanit
dhe e ka lënë 5 vjet më parë. Problemet e tij të tjera mjekësore përfshijnë sëmundjen vaskulare periferike, hipertension
dhe hiperplazinë beninje të prostatës. Ai menaxhohet me aspirinë, lizinopril, hidroklorotiazid dhe tamsulozinë. Gjatë
ekzaminimit, pacienti ka një saturim në pushim prej 93% në ajrin e dhomës. Ne perkusion ka mushkeri te tejzgjeruara
(hiperinflated) me tinguj të zvogëluar të frymëmarrjes në maja pulmonare dhe wheezing të dobët ekspirator. Testet e
funksionit të tij pulmonar demonstrojnë një FEV 1 prej 55% të parashikuar, FVC prej 80% të parashikuar dhe FEV1 /
FVC raport prej 50%. Cili është hapi tjetër më i mirë në menaxhimin e këtij pacienti?
A. Filloni një provë të glukokortikoideve orale për një periudhë prej 4 javësh dhe filloni fluticasone inhaluar nëse ka
një përmirësim të dukshëm në funksionin pulmonar.
B. Filloni trajtimin me fluticasone inhalator 110 μg / puff dy herë në ditë.
C. Filloni fluticasone inhalator 250 μg / puff në kombinim me salmeterol inhalator 50 mg / puff dy herë në ditë.
D. Filloni trajtimin me tiotropium inhalator 18 μg / ditë.
E. Kryeni oksimetri gjate stërvitjes dhe nokturnale dhe filloni terapi me oksigjen nëse këto demonstrojnë hipoksemi
të konsiderueshme
38. Një grua 56-vjeçare pranohet në ICU me një histori 4-ditore të rritjes së dispnese dhe kollës me prodhimin e shumtë
të sputumit. Ajo ka COPD të rëndë me një parashikim FEV1 prej 42%. Në prezantim, ajo ben nje astrup me një pH prej
7.26, PaCO2 prej 78 mmHg dhe PaO2 prej 50 mmHg. Ajo është në distres të dukshëm respirator me përdorimin e muskujve
aksesorë dhe retraksioneve. Tingujt respiratorejanë të qetë me wheezing ekspirator difuz dhe bronke. Asnjë infiltrat nuk
është e pranishme në radiografinë e gjoksit. Cila nga terapitë e mëposhtme është demonstruar të ketë uljen më të madhe
të shkallës së vdekshmërisë për këtë paciente?
A. Administrimi i bronkodilatatorëve inhalatore
B. Administrimi i glukokortikoideve IV
C. Administrimi i hershëm i antibiotikëve me spektër të gjerë me mbulim të P aeruginosa
D. Intubimi i hershëm me ventilim mekanik
E. Përdorimi i ventilimit jo-invaziv me presion pozitiv
39. Një burrë 63-vjeçar me një histori të gjatë të pirjes së duhanit vjen për t'ju parë për një histori 4-mujore të
veshtiresise ne frymemarrje/dispnese progresive dhe dispnesë gjatë sforcimit. Simptomat kanë qenë te buta/dobeta,
pa ndonjë përkeqësim/exacerbim të kohëve të fundit. Ai mohon ethe, dhimbje gjoksi ose hemoptizi. Ai ka një kollë
ditore prej 3 deri në 6 lugë gëlbazë/flegme të verdhë. Pacienti thotë se nuk ka parë një mjek për më shumë se 10 vjet
Ekzaminimi fizik është i dukshëm për shenjat vitale normale, një fazë të zgjatur ekspiruese, rhonki të shpërndarë,
pulsim të ngritur të venës jugulare dhe edemë kembe. Hematokriti 49%. Cila terapi zgjat mbijetesën e tij?
A. Atenolol
B. Enalapril
C. Oxygen
D. Prednisone
E. Theophyllin
40. Një burrë 62 vjeç vlerësohet për dispne gjatë sforcimit që është përkeqësuar në mënyrë progresive gjatë një
periudhe prej 10 muajsh. Ai ka një histori 50-vjeçare të duhanit, duke lënë 10 vjet më parë. Në ekzaminimin fizik,
ka një saturim në pushim prej 94%. Pas ecjes prej 100 m, saturimi i tij bie në 84%. Ai kërkon oksigjen në 3 L / min
për të mbajtur satrurimin e tij në më shumë se 90% me ecje. Ekzaminimi i tij i mushkërive tregon krepitacione
(crackles) fund-inspiratore difuze në të dy mushkëritë. Kapaciteti i tij total i mushkërive është parashikuar 72%,
volumi rezidual është parashikuar 68% dhe kapaciteti përhapës/difuzues është parashikuar 60%. CT-ja e kraharorit
me rezolucion të lartë është treguar në Figurën VI-40. Kerkimi serologjik për sëmundjen autoimune është
jokonkludues dhe historia e plotë nuk jep ekspozime që do të çonin në këto gjetje. Dyshoni për fibrozë pulmonare
idiopatike. Cila është gjetja e pritur ne patologjinë kirurgjikale?

FIGURA VI-40
A. Pneumoni intersticiale deskuamative
B. Demtim alveolar difuz
C. Inflamacion granulomatoz i c organizuar dhe fibroze
D. Pneumoni intesticiale jospecifike
E. Pneumoni intersticiale tipike

41. Cili është trajtimi i rekomanduar për pacientin në Pyetjen VI-40?


A. Azathioprine 125 mg në ditë plus prednisone 60 mg në ditë
B. Ciklofosfamid 100 mg në ditë
C. Nintedanib 150 mg dy herë në ditë
D. Prednizoni 60 mg në ditë
E. Asnjë terapi nuk është efektive për trajtimin e fibrozës pulmonare idiopatike.
42. Cili do të ishte gjetja e pritshme për lavazhin bronkoalveolar në një pacient me hemorragji difuze alveolare?
A. Pneumocitet atipike hiperplastike të tipit II
B. Trupat ferruginozë
C. Makrofagët e ngarkuar me hemosiderinë
D. Limfocitoze me një raport të rritur CD4: CD8
E. Pamja e qumështit me makrofagët e shkumëzuar
43. Një burrë 42 vjeç paraqet dispne progresive në sforcim, ethe të grades së ulët dhe humbje peshe gjatë 6 muajve.
Ai gjithashtu ankohet për një kollë kryesisht të thatë, edhe pse herë pas here ai kollitet me një sputum të trashë
mukoid. Nuk ka histori të kaluar mjekësore. Ai nuk pi cigare. Në ekzaminimin fizik, pacienti shfaqet dispneik me
sforcim minimal. Temperatura e pacientit është 37.9°C (100.3°F). Saturimi është 91% në ajrin e dhomës në qetësi.
Dëgjohen krepitacione te zbehta ne bazat pulmonare. Në studimet laboratorike, pacienti ka hipergamaglobulinemi
poliklonale dhe një hematokrit 52%.
CT zbulon infiltrate bilaterale alveolare që janë kryesisht me natyrë perihilare me pattern mozaiku. Pacienti i
nënshtrohet bronkoskopisë me lavazh bronkoalveolar. Rezultati del si qumesht. Citopatologjia tregon mbetje
amorfe me makrofagët pozitivë ne PAS. Cila është diagnoza?
A. Bronkiolit obliterans me pneumoni
B. Pneumoni intersticiale deskumative
C. Nokardioza
D. Pneumoni nga pneumocystis carinii
E. Proteinoza alveolare pulmonare
44. Për pacientin në Pyetjen VI-43, cili trajtim është më i përshtatshmi në këtë kohë?
A. Doxycycline
B. Prednizoni
C. Prednizoni dhe ciklofosfamidi
D. Trimetoprim-sulfametoksazol
E. Lavazh i mushkërive
45. Një grua 56-vjeçare paraqitet për vlerësim dispneje dhe kollë për 2 muaj. Gjatë kësaj kohe, ajo gjithashtu ka
pasur ethe intermitente, dobesi dhe një humbje peshe 5.5 kg. Ajo mohon të ketë ndonjë kontakt me ndonje të
sëmurë dhe nuk ka udhëtuar së fundmi. Ajo punon si një infermiere, dhe një test vjetor derivateve të proteinave i
kryer 3 muaj më parë ishte negativ. Ajo mohon çdo ekspozim ndaj pluhurit organik dhe nuk ka zogj si kafshë
shtëpiake. Ajo nuk ka ekspozime të tjera dhe asnjë simptomë autoimune. Ajo nuk merr asnjë ilaç rregullisht. Gjatë
ekzaminimit fizik, dëgjohen krepitacione (crackles) difuze inspiratore. CT i gjoksit zbulon infiltrat alveolar me patche
dhe trashje të murit bronkial. Testet e funksionit pulmonar zbulojne restriksion te lehte. Ajo i nënshtrohet një
biopsie kirurgjikale të mushkërive. Patologjia tregon indin granulacioni që mbush rrugët e vogla të frymëmarrjes,
kanalet alveolare dhe alveolat. Intersticiumi alveolar ka inflamacion kronik dhe pneumoni organizative. Cila është
terapia më e përshtatshme për këtë pacient?
A. Azathioprine 100 mg në ditë
B. Nentedanib 150 mg dy herë në ditë
C. Pirfenidon 2403 mg në ditë
D. Prednizoni 1 mg / kg në ditë
E. Referenca për transplantimin e mushkërive
46. Një burrë 53 vjeç pranohet me ethe dhe dhimbje gjoksi pleuritike të djathtë për 5 ditë. Ai ka një histori të
varësisë nga alkooli. Në prezantim, temperatura e tij është 39.2 ° C, rrahjet e zemrës janë 112 bpm, presioni i gjakut
është 102/62 mmHg, FR 24 frymëmarrje / min, dhe SaO2 është 92% në ajrin e dhomës. Ai nuk ka tinguj
frymëmarrjeje në pjesën e poshtme të kraharorit me shurdhim të goditjeve dhe renie të fremitusit taktil.
Radiografia e gjoksit konfirmon një konsolidim të lobit të poshtëm të djathtë me efuzionin shoqërues. Efuzioni nuk
rrjedh i lirë. Thorakocenteza fillestare demonstron pus te trashe në hapësirën pleurale dhe Gram pozitive për koket
gram pozitive në çifte dhe zinxhirë. Vendoset një tub i gjoksit me vrima të mëdha per drenim. Cili nga trajtimet e
mëposhtëm duhet gjithashtu të rekomandohet në këtë pacient për të përmirësuar zgjidhjen e empiemës?
A. Referim i menjëhershëm për decortication- heqje e cipes pleurale te trashur
B. Instilimi intrapleural i alteplazës 10 mg dy herë në ditë për 3 ditë
C. Instilimi intrapleural i alteplazës 10 mg plus deoksiribonukleazës 5 mg dy herë në ditë për 3 ditë
D. Instilimi intrapleural i deoksiribonukleazës 5 mg dy herë në ditë për 3 ditë
E. Instilimi intrapleural i streptokinazës 250,000 IU
47. Një grua 44-vjeçare me SIDA ka insuficience akute hipoksemike të frymëmarrjes për shkak të Pneumocystis
jiroveci. Ajo intubohet dhe ventilohet mekanikisht me cilësimet e mëposhtme: ventilim i asistuar, volumi tidal
350 ml (6 ml / kg peshe ideale), FiO2 1.0, FR 28 frymëmarrje / min dhe PEEP 12 cmH2O. Astrupi: pH 7.28, PaO2 68
mmHg dhe PaCO2 64 mmHg. Presioni plateau inspirator është 26 cmH2O. Ju thirreni shpejt pranë shtratit kur
presioni i saj i gjakut papritmas bie në 70/40 mmHg. Në të njëjtën kohë, bie alarmi i ventilatorit qe tregon presion
maje në 55 cmH 2O. Tingujt e frymëmarrjes janë të padëgjueshëm në anën e djathtë dhe janë të qartë në majtë.
Cili është rruga më e mirë e veprimit në këtë kohë?
A. Administroni një bolus likidi për të përmirësuar kthimin venoz.
B. Shkëputeni pacientin nga ventilatori për të lejuar një ekspirim të plotë.
C. Vendosni një gjilpërë me vrima të mëdha në hapësirën e dytë të djathtë të interkostale për të lehtësuar një
pneumotoraks tensioni.
D. Sedoni pacientin për të arritur sinkroninë e ventilatorëve
E. Perdorni suction për të hequr tapat obstruktive te mukusit.
48. Një grua 62-vjeçare pranohet në spital me një pneumoni të fituar komunitare me një histori 4-ditore të etheve,
kollës dhe dhimbje gjoksi pleuritike të djathtë. Radiografia e gjoksit në pranim identifikon një infiltrim të lobit të
poshtëm dhe të mesëm te djathte me një efuzion shoqerues. Të gjitha karakteristikat e mëposhtme të efuzionit
pleurale indikojne një efuzion të komplikuar që mund të kërkojë torakostominë me tub PERVEÇ:
A. Likid i lokalizuar
B. pH i Likidit pleural pH <7,20
C. Glukoza e likidit pleural <60 mg / dL
D. Ngjyrosja + Gram ose kultura e likidit pleural
E. Rekurenca/rishfaqja e likidit pas toracentezës fillestare
49. Një 58-vjeçar vlerësohet për dispne dhe zbulohet se ka një efuzion pleural te moderuar të anes se djathtë. Ai i
nënshtrohet thorakocentezës me karakteristikat e mëposhtme:
Cila nga t NUK eshte e mundshme te jete shkaku i efuzuonit pleural?
A. Cirroza
B. Kanceri pulmonary
C. Mezotelioma
D. Emboli pulmonare
E. Tuberkulozi

50. Një 28-vjeçar paraqitet në departamentin e urgjencës me dispne me fillim akut dhe dhimbje gjoksi pleuritike
në të djathtë që filloi 2 orë më parë. Ai në përgjithësi është i shëndetshëm dhe nuk ka histori mjekësore. Ai ka pirë
një paketë cigare çdo ditë që nga mosha 18 vjeç. Në ekzaminimin fizik, ai është i gjatë dhe i dobët, me një indeks
të masës trupore 19.2 kg / m2. Ai ka FR prej 24 frymëmarrjesh / min me saturim prej 95% në ajrin e dhomës. Ai ka
tinguj frymëmarrjeje te pakesuar në majën e djathtë të mushkërive. Një radiografi në gjoks demonstron një
pneumotoraks 20% në të djathtë. Cila nga pohimet e mëposhtme është e vërtetë në lidhje me pneumotoraksin ?
A. Një CT ka të ngjarë të tregojë ndryshime emfizematoze.
B. Nëse pacienti do të zhvillonte pneumotorakse të përsëritura, torakoskopia me abrazion pleural ka një sukses
100% për parandalimin e përsëritjes.
C. Shumica e pacientëve me këtë paraqitje kërkojnë torakostominë me tub për të zgjidhur pneumotoraksin.
D. Gjasat e pneumotoraksit përsëritës janë rreth 25%
E. Faktori kryesor i riskut për zhvillimin e pneumotoraksit spontan është një habitus i gjatë dhe i holle
51. Një grua 52-vjeçare nga Indiana paraqitet me përkeqësim të dispnesë gjatë sforcimit dhe kollës për një vit. Ajo
mohon dispne në qetësi. Kollitja është kryesisht e thatë, por herë pas here prodhuese e një mukusi si rere. Historia
e saj e kaluar mjekësore është pozitive për hipertension dhe hipotiroidizëm. Ajo merr benazepril dhe levotiroksinë.
Ajo ka punuar kryesisht si kopshtare/ peizazhiste gjatë gjithë jetës së saj adulte. Në ekzaminimin fizik, ajo nuk
shfaqet në distress. Saturimi është 95% në ajrin e dhomës. Gjoksi është i qartë ne auskultim. Ekzaminimi
kardiovaskular është jo konkludues. Ajo nuk ka edema periferike. Radiografia e gjoksit tregon një granulomë të
vjetër në mushkërinë e djathtë dhe kalcifikimet mediastinale. Kryhet një CT, i cili konfirmon granulomën e shëruar.
Shihet kalcifikim i gjerë mediastinal. Kalcifikimet mbështjellin venën cava superiore dhe bronkun e djathtë të
sistemit kryesor. Një analizë e interferon-γ është negative. Cila nga pohimet e mëposhtme në lidhje me kushtet e
pacientit është e vërtetë?
A. Një test i antigjenit Histoplazma në urinë do të jetë pozitiv.
B. Shkaku më i zakonshëm i gjendjes është histoplazmoza ose tuberkulozi.
C. Pacienti duhet të referohet në një qendër kirurgjikale të specializuar në trajtimin e kësaj gjendjeje.
D. Trajtimi me kortikosteroide do të përmirësojë gjendjen e pacientit.
E. Trajtimi me itrakonazol ose vorikonazol do të përmirësojë gjendjen e pacientit.

52. Një grua 52 vjeç pranohet në spital me letargji dhe simptoma të theksuara të mbingarkesës volumore. Ajo ka
një histori të kaluar mjekësore të obezitetit morbid me një BMI 52 kg / m2, apnea të rëndë obstruktive te gjumit,
hipertension dhe DM tip 1. Ajo është përgjithësisht në gjendje të keqe shëndetësore dhe nuk ka qenë kompliante/e
rregullt me insulinën e saj, si dhe CPAP pasi raporton klaustrofobi. Ajo nuk mund të kujtojë kur ka përdorur për
herë të fundit terapinë CPAP. Në ekzaminimin fizik, pacienti është somnolent, por zgjohet. Shenjat e saj jetike janë
si më poshtë: presioni i gjakut 168/92 mmHg, rrahjet e zemrës 92 bpm, FR 14 frymëmarrje / min, afebrile dhe SaO2
82% në ajrin e dhomës. SaO2 e saj rritet në 92% me 6 L / min nga kanula nazale, por statusi i saj mendor bëhet më
letargjik. Ajo ka tinguj të largët të zemrës dhe mushkërive pa krepitacione/crackles. Ka edemë 4+ në mënyrë
bilaterale deri në kofshë dhe mbi murin e barkut. Radiografia në gjoks tregon vëllime të ulëta të mushkërive. Vlerat
fillestare të astrupit në 6 L / min oksigjen nazal janë pH 7.22, PaCO2 88 mmHg dhe PaO2 72 mmHg. Cila nga pohimet
e mëposhtme është e vërtetë në lidhje me gjendjen e pacientit?
A. Anomalitë e gjenit PHOX2b shoqërohen me këtë gjendje.
B. Terapia CPAP e vetme është e përshtatshme për trajtimin e këtij pacienti
C. Trajtimi fillestar i gjendjes duhet të përfshijë intubimin dhe ventilimin mekanik duke pasur parasysh
intolerancën e njohur të pacientit ndaj terapisë CPAP.
D. Apnea obstruktive e gjumit bashkëjeton me diagnozën në rreth 75% të rasteve.
E. Humbja e peshës do të çojë në përmirësime në PaCO2 me kalimin e kohës.
53. Një pacient me sklerozë amiotrofike laterale te bute ndiqet nga një pneumolog për mosfunksionim respirator
që shoqërohet me sëmundjen e tij neuromuskulare. Cila nga simptomat e mëposhtme përveç PaCO2 ≥45 mmHg
do të kërkonte terapi me ventilim jo-invaziv me presion pozitiv për hipoventilimin?
A. Ortopne
B. Gjumë me cilësi të dobët
C. Kollë e dëmtuar
D. dispnea në aktivitetet e jetës së përditshme
E. Të gjitha sa më sipër
54. Një 52-vjeçar vlerësohet për gërhitjen me zë të lartë dhe lodhjen e ditës. Gruaja e tij e nxiti atë të vinte për
vlerësim sepse gërhitja e tij është bërë gjithnjë e më shqetësuese gjatë 2 viteve të fundit pasi ai fitoi rreth 13 kg.
Ajo shpesh fle në një dhomë tjetër dhe raporton se e kishte parë atë të ndalonte frymëmarrjen gjatë gjumit. Gjatë
ditës, ai ka veshtiresi të qëndrojë zgjuar kur është në takime ose ulur në kompjuterin e tij, veçanërisht pas drekës.
Ai ben një udhëtim 40-minutësh dhe i është dashur të ndaloje ne rrugë për shkak të ndjenjës së përgjumjes. Ai ka
një histori mjekësore të hipertensionit për 5 vitet e fundit dhe merr losartan 25 mg në ditë. Ai nuk pi duhan dhe pi
një birrë ose gotë verë çdo ditë. Babai i tij, i cili është 75 vjeç, përdor një makinë CPAP për apnea obstruktive të
gjumit dhe gjithashtu ka pasur një infarkt miokardi. Në ekzaminimin fizik, pacienti shfaqet mirë. Ai ka një indeks të
masës trupore prej 37.1 kg / m2. Kur pacienti hap gojën, ju mund të shihni pjesën më të madhe të uvulës së tij, e
cila duket disi edematoze. Indi tonsilar është i dukshëm dhe shtrihet përtej urave tonsilare. Circumferenca e qafës
është 43 cm. Cili është hapi tjetër në vlerësimin dhe trajtimin e këtij pacienti?
A. Polisomnogramë në laborator
B. Polisomnograme në shtëpi
C. Oksimetria gjate natës
D. Referimi për tonsillectomy
E. Trajtimi me CPAP me autotitrim
55. Një 48-vjeçar kohët e fundit është diagnostikuar me apnea obstruktive të gjumit me një indeks apnea-hipopnea
prej 21.2 / orë. Ai paraqitet në klinikë për ndjekje pasi ai provoi CPAP në polisomnographin dhe ndihej i
parehatshem me të. Ai pyet cilat janë rreziqet e mundshme për shëndetin e tij nëse zgjedh të heqë dorë nga
trajtimi. Çfarë këshille i jepni?
A. Apnea obstruktive e gjumit e patrajtuar ka një rrezik në rritje të vdekshmërisë për shkak të ngjarjeve
kardiovaskulare, përfshirë infarktin e miokardit dhe goditjen në tru.
B. Apnea obstruktive e gjumit e patrajtuar ka risk te rritur per depresion.
C. Apnea obstruktive e gjumit e patrajtuar shoqërohet me një rrezik shtatë herë më të madh të
aksidenteve automobilistike.
D. Apnea obstruktive e gjumit e patrajtuar ngre presionin e gjakut noktturnal, dhe trajtimi me CPAP çon
në një rënie të presionit të gjakut 2- deri në 4 mmHg.
E. Të gjitha më lart janë këshilla të mira për t'i dhënë pacientit.
56. Cili pacient më poshtë ka më
shumë të ngjarë të ketë modelin e
frymëmarrjes të parë në Figurën në
polisomnografi?

FIGURE VI-56
A. Një djalë 6 vjeç me performancë të
dobët në shkollë, gërhitje, dhe tonsila dhe
adenoide të zgjeruara
B. Një grua 36-vjeçare me histori të
varësisë nga heroina e cila po trajtohet me
terapi mbajtese me metadon në një dozë prej
100 mg në ditë
C. Një burrë 48 vjeç i vlerësuar për gërhitje të madhe dhe përgjumje të tepruar me një indeks të
masës trupore 36.8 kg / m2
D. Një grua 52-vjeçare me indeks të masës trupore 22 kg / m2 duke u ankuar për zgjime të
shpeshta të natës në territor te menopauzës; ajo nuk gërhet
E. Një burrë 68 vjeç me fibrilacion atrial dhe kardiomiopati ishemike me një fraksion ejeksioni prej
15%; Gjatë ekzaminimit, ai ka ngritje të presionit venoz jugular, refluks hepatojugular dhe
edemë periferike 3+.
57. Një burrë 48 vjeç me BMI 28.9 kg / m2 diagnostikohet me apnea obstruktive të gjumit me indeks
apnea-hipopnea 42 / orë dhe një saturim të oksigjenit 78%. Terapia fillestare më e përshtatshme ?
A. CPAP
B. Terapia me aparate orale
C. Terapia me oksigjen
D. Uvulopalatopharyngoplasty
E. Humbja e peshës
58. Një grua 47-vjeçare me hipertension arterial pulmonar arterial idiopatik ka dështuar në terapinë
mjekësore duke përfshirë IV epoprostenol. Ajo ka insuficiencë të përparuar të zemrës së djathtë me
disfunksion të rëndë të ventrikulit të djathtë në ekokardiografi dhe një indeks kardiak prej 1.7 L / min
/ m2. Ajo referohet për transplantim të mushkërive. Cila nga pohimet e mëposhtme është e vërtetë?
A. Ajo do të kërkojë transplantim zemër-mushkëri për dështimin e përparuar të zemrës së djathtë.
B. Pacientët me hipertension arterial pulmonar idiopatik kanë mbijetesë më të keqe 5-vjeçare sesa
marrësit e tjerë të transplantimit.
C. Transplantimi me një mushkëri është procedura e preferuar kirurgjikale për hipertensionin arterial
pulmonar idiopatik.
D. Funksioni i saj i ventrikulit të djathtë do të rikuperohet pas transplantimit të mushkërive.
E. Ajo është në rrezik për HTP të përsëritur pas transplantimit të mushkërive.
59. Një grua 25-vjeçare me fibrozë cistike referohet për transplantim të mushkërive. Ajo është e
shqetësuar për rezultatin e saj afatgjatë. Cila nga të mëposhtmet është pengesa kryesore për
mbijetesën afatgjatë pas transplantimit të mushkërive?
A. Sindroma e bronkiolitit obliterans
B. Infeksioni CMV
C. Sëmundja kronike e veshkave
D. Mosfunksionimi primar i graftit
E. Çrregullimi limfoproliferativ pas transplantimit

79
60. Një burrë 30 vjeç me fibrozë cistike në fazën përfundimtare i nënshtrohet transplantit të
mushkërive. Tre vjet më vonë, ai ka një rënie 6-mujore progresive të funksionit të tij renal. Cili nga
ilaçet e mëposhtëm është etiologjia më e mundshme e kësaj?
A. Prednisone
B. Takrolimus
C. Albuterol
D. Mycophenolate mofetil
E. Asnjë nga sa më sipër
61. Një grua 48-vjeçare pranohet në ICU me ARDS dhe shok për shkak të kolangitit ascendens. Rezultati
i saj i Fiziologjisë Akute dhe Shëndetit Kronik (APACHE II) është 19 në 24 orë. Cila deklaratë përshkruan
më mirë performancën e rëndesës/gravitetit në parashikimin e rezultateve për një pacient ?
A. Piket / Score te APACHE II dhe mjetet e tjera te gravitetit të sëmundjes janë të dobishme për
kerkim shkencor në sëmundjen kritike, veçanërisht në lidhje me ashpërsinë e sëmundjes në
pacientët e regjistruar në provat klinike.
B. Piket / Score APACHE II është një parashikues i rëndësishëm i mortalitetit individuale kur zbatohet
drejtpërdrejt në kujdesin e pacientit.
C. Piket / Score APACHE II duhet të përdoret për të drejtuar terapinë në këtë pacient.
D. Rezultati i Thjeshtuar i Fiziologjisë Akute (SAPS II) do të dilte më mirë në parashikimin e
vdekshmërisë sesa rezultati APACHE II.
E. Të gjitha sa më sipër janë të vërteta.
62. Një burrë 42 vjeç pranohet në ICU pas një aksidenti automobilistik. Ai pësoi një thyerje të përbërë
të femurit dhe gjithashtu kishtehemorragji të brendshme nga shpretka e rupturuar dhe hematoma e
heparit. Ai i është nënshtruar splenektomisë dhe fiksimit të frakturës së femurit. Ai është i intubuar
dhe i seduar pas operacionit. Hemoglobina e tij pas operacionit është 5.2 g / dL. Saturimi me oksigjen
është 92%, dhe PaO2 e tij është 72 mmHg në FiO2 prej 0.6. Një kateter i arteries pulmonare u vendos
gjatë operacionit. Dalja e tij kardiake është 7.8 L / min. Niveli i laktatit është 4.8 mmol / L. Cili nga sa
më poshtë është faktori më pak i rëndësishëm që ndikon në shpërndarjen e oksigjenit te ky pacient?
A. Debiti kardiak
B. Përqendrimi i hemoglobinës
C. PaO2
D. SaO2
63. Një grua 67-vjeçare u pranua në ICU me pneumoni multilobare për shkak të Streptococcus
pneumoniae dhe COPD. Ajo kërkon intubim dhe ventilim mekanik. Të gjitha sa më poshtë janë
ndërhyrje të përshtatshme për të parandaluar ndërlikimet në ICU PERVEC:
A. Administrimi i enoksaparinës 40 mg në ditë
B. Administrimi i omeprazolit 20 mg në ditë
C. Kontrolli agresiv i glukozës në gjak
D. Mobilizimi i hershëm dhe terapia fizike ndërsa ajroset mekanikisht
E. Përdorimi i një terapie standarde për futjen e linjave qendrore venoze
64. Një burrë 68 vjeç pranohet në ICU me ethe, hipotension dhe hipoksemi. Ai është ndjerë i sëmurë
për 2 ose 3 ditët e fundit me dispne progresive në shtëpi. Ai ka një histori per COPD, sëmundje të
arterieve koronare që kërkon operacionin bypass të arterieve koronare SAK 3-vazal, dhe diabet
mellitus të tipit 2. Ai vazhdon të pijë një pako cigare çdo ditë dhe gjithashtu pi 6 birra çdo ditë. Në
prezantim, saturimi i O2 është 79%. Me një maskë non-rebreather, saturimi mbetet në 87%. Presioni
i tij i gjakut është 74/40 mmHg, dhe rrahjet e zemrës janë 124 bpm. Pas bolusit të likideve, presioni i

80
tij i gjakut mbetet i ulët në 86/53 mmHg. Radiografia e gjoksit të tij tregohet në Figurën VI-64. Brenda
12 orëve pas pranimit, kulturat e gjakut janë pozitive për S pneumoniae. Ai mori dozën e tij të parë të
antibiotikëve në departamentin e
urgjencës dhe mbetet në trajtim me
ceftriakson dhe moxifloxacin. Ai është
i intubuar, i seduar dhe aktualisht me
mbështetje të vazopresoreve. Astrupi
pas intubimit është pH 7.28, PaCO2
52 mmHg dhe PaO2 64 mmHg në FiO2
0.8. Cila nga më poshtë identifikon
më së miri diagnozën e pacientit?
FIGURA VI-64
A. Pneumoni akute intersticiale
B. ARDS e lehte
C. ARDS e moderuar
D. Pneumoni e fituar e
komunitetit multilobare
E. ARDS e rende

65. Nëse një biopsi e mushkërive do të merrej 4 ditë pas pranimit në pacientin e përshkruar në Pyetjen
VI-64, cila thënie identifikon saktë gjetjet e pritura?
A. Dëmtimi difuz alveolar me membrana hialine dhe likid edeme të pasur me proteina në alveola
B. Infiltrat i gjerë i pasur me eozinofile me likid edeme të pasur me proteina
C. Fibroza ekstensive e duktuseve alveolare me zhvillimin e bullave
D. Infiltrimi homogjen i neutrofileve dhe leukociteve që prekin të gjitha hapësirat alveolare
E. Proliferim ie pneumociteve të tipit II dhe prania e një infiltrati pulmonar të pasur me limfocite
66. Një grua 48-vjeçare pranohet në ICU kirurgjikale pas një aksidenti me automjet. Ajo ka pësuar një
tronditje te kokes, thyerje të brinjëve 4 deri 8 në të majtë me një hemopneumotoraks dhe një shpretkë
të laceruar që kërkonte splenektomi. Gjatë operacionit për të hequr shpretkën e saj, ajo kërkoi
transfuzionin e 6 njësive të eritrociteve, 6 njësive të trombociteve dhe 4 njësive të plazmës së freskët të
ngrirë. Pas pranimit në ICU pas operacionit, ajo mbetet e intubuar dhe e seduar. Një tub gjoksi është vene
në të majtë. Radiografia e saj e gjoksit tregon infiltrate bilaterale difuze. Mushkëria e majtë ka infiltrate
dense, dhe ka edhe infiltrate të gjera në të djathtë. Ajo diagnostikohet me një kontuzion në mushkëri e
majtë dhe ARDS. Ajo peshon 90 kg. Ajo është 167cm e gjatë. Pesha e saj ideale e trupit është 59 kg.
Saturimi me oksigjen në FiO2 1.0 është 92% me astrup që tregon pH prej 7.28, PaCO2 prej 48 mmHg dhe
PaO2 prej 68 mmHg. Cili është volumi tidal fillestar me i mire në këtë pacient?
A. 236 ml
B. 354 mL
C. 472 mL
D. 540 mL
E. 590 mL
67. Ju jeni duke trajtuar një pacient të pranuar në ICU për sindromën e distresit respirator akut të
rëndë si pasojë e pankreatitit nekrotizant. Pacienti ka peshë ideale 70 kg. Ventilatori I pacientit është
vendosur në kontroll volumi me frekuencë respiratore 28 për minutë, volum tidal 420 ml, FiO2 0.7,
PEEP 8 cmH2O. Pacienti është hipoksemik me saturime të oksigjenit 86%. Ti sheh statistikat e grafikut
presion-vëllim për sistemin respirator. Pika më e ulët është tek 12 cmH2O dhe më e larta është te

81
30cmH2O. Presioni i matur me mbajtje të frymës është 26cmH2O. Cila nga të mëposhtmet është
zgjidhja më e mirë për të përmirësuar oksigjenimin në këtë pacient?
A. Administro një agjent paralitik
B. Ul vëllimin tidal te 350 ml
C. Rrit FiO2 te 0.8
D. Rrit PEEP te 12cmH2O
E. Rrit volumin tidal te 560 ml
68. Një burrë 75 vjecar është duke u trajtuar në ICU për sepsis si pasojë e neutropenisë nga
kimioterapia që ka marrë për kancer gastrik. Ai ka sindromë të distresit akut të rëndë dhe kërkon
intubim. Gjatë 48 orëve të para në ICU, statusi I volumit të tij është pozitiv 6L. Cila nga phimet e
mëposhtme në lidhje më menaxhimin e likideve në sindromin e distresit respirator është e vërtetë?
A. Diureza agresive për të mbajtur presion të ulët mbushës në attriumin e majtë shoqërohet me risk
të shtuar për dëmtim akut të veshkave që kërkon diurezë
B. Mbajtja e një presioni të ulët mbushës në atriumin e majtë nëpërmjet diurezës rrit kompliancën
e pulmoneve dhe përmirëson oksigjenimin
C. Mbajtja e një presioni të ulët mbushës të atriumit të majtë nëpërmjet diurezës zvogëlon kohën e
qëndrimit në ICU dhe zvogëlon mortalitetin në ICU mjekësore, por jo në ICU kirurgjikale
D. Vendosja e një kateteri pulmonar arterial për matjen e saktë të presionit mbushës në atriumin e
majtë përmirëson saktësinë diagnostike dhe siguron përfitim të shtuar në përcaktimin e
strategjisë së menaxhimitë të likideve
69. Në cilat nga situatat konsiderohet ventilimi mekanik joinvaziv I kundërindikuar?
A. Përkeqësim akut i SPOK
B. Insuficiencë akute respiratore hipoksemike në pacient me sindromin e distresit respirator akut
C. Infarkt miokardi akut
D. Insuficiencë kardiake sistolike e dekompensuar pa ishemi akute miokardiale
E. Pas ekstubimit të një pacienti që ka qënë i intubuar për SPOK
70-73. Bashko mënyrën e ventilimit me përshkrimin.
70. Ventilimi I asistuar I kontrolluar C
71. Ventilimi I kontrolluar intermittent I sinkronizuar B
72. Ventilimi me presion të kontrolluar A
73. Ventilimi me mbështetje presion D
A. Në këtë mënyrë ventilimi përcaktohet frekuenca dhe presioni target I rrugëve ajrore. Volumi tidal
dhe fluksi inspirator janë të varura nga komplianca e pulmoneve.
B. Kjo mënyrë ventilimi siguron ventilim të bazuar në frekuencën respiratore dhe në volumin tidal.
Frymëmarrja spontane mbi frekencën respiratore të caktuar mbështetete nga modaliteti I
presionit.
C. Kjo është mënyra më e përdorur e ventilimit mekanik. Cdo frymëmarrje, si nga pacienti ashtu edhe
nga aparati, siguron një volum tidal të paracaktuar.
D. Kjo mënyrë ventilimi varet nga pacienti, flukset dhe është me presion të caktuar. Përdoret vetëm
atëherë kur pacienti merr frymë spontanisht.
74. Një grua 62 vjecare është intubuar për shkak të një pneumonie të marrë në komunitet dhe sepsis.
Në pranim në ICU pacientja është hipotensive dhe kërkon trajtim me vazopresor. Ajo u sedatua me
propofol për tu ventiluar. ICU ka një protokoll për ndalimin e sedatimit cdo ditë. Paciantja aktualisht
është e intubuar për 8 ditë. Cili nga të mëposhtmit është një kundraindikim për frymëshkëmbimin
spontan te kjo paciente?
A. Rritja e infiltrative në radiografi toraksi
B. FiO2 prej 0.45

82
C. Rritja e nevojave për support me vazopresor
D. Pacientja merr frymë mbi frekuencën respiratore të caktuar 8/min
E. PEEP of 8cmH2O
75. Të gjitha fjalitë për patofioziologjinë e shokut janë të vërteta duke përjashtuar njërën:
A. Hipotensioni frenon qendrën vazomotore, duke zvogëluar përgjigjen adrenergjike dhe duke
rritur aktivitetin vagal
B. Ndryshimet metabolike, duke përfshirë lipolizën dhe glikolizën, rrisin osmolaritetin ekstracelular,
duke cuar në një gradient osmotik që rrit volumin intersticial në kurriz të volumit interacelular.
C. Mediatorët proinflamator që stimulohen nga sistemi imun fillestar kontribuojnë dukshëm në
disfunksion organor të shumëfishtë dhe cojnë në shok.
D. Stresi I rëndë ose dhimbja shkaktojnë rritje të clirimit të ACTH, duke cuar në rritje të nivelit të
kortizolit, që rrit glukoneogenezën, ul kapjen perferike të glukozës dhe rrit lipolizën.
76. Një pacient është pranuar në ICU e neurologjisë me transeksion (shkëputje totale e trakteve të
lëndës së bardhë, pjesërisht e lëndës grid he rrënjë nervore) të C2-C3. Pacienti është në shok me
tension arterial 72/40 mmHg. Cilat janë rezultatet e pritshme në një kateter të arteries pulmonare?
Presioni venoz Output kardiak Rezistenca Saturimi venoz I
central dhe vaskulare O2
presioni kapilar sistemike
pulmonar
A ulet Ulet rritet Ulet
B rritet Ulet Rritet Ulet

C Nuk ndryshon Rritet Ulet Ulet

D Ulet Nuk ndryshon Nuk ndryshon Ulet

E Ulet Ulet ulet ulet

77. Të gjitha fjalite per epidemiologjisë së sepsisit dhe shoku spetik janë të vërteta përvec njërës:
A. Bakteret gram positive janë organizmat më të shpeshtë që janë zbualuar si shkaktarë të
sindromave septike.
B. Në pacientë me shok septik, kulturat e gjakut janë positive në 40-70 %
C. Shumica e rasteve me sepsis ndodhin në pacientë me një sëmundje sinjifikante shoqëruese
D. Infeksionet respiratore janë shkaqe kryesore të sindromave septike
E. Incidence vjetore e sepsisit të rëndë është rritur 30 vitet e fundit me një incidencë aktuale prej 3
raste për 1000 individë.
78. Një grua 62 vjec paraqitet në urgjencë për përkeqësim të temperaturës dhe simptomave
respiratore. Ajo filloi të ndihej e sëmurë rreth 2 ditë më parë dhe simptomat janë përkeqesuar që prej
atij momenti. Ajo ka një histori të kaluar për artrit rheumatoid dhe merr adalimumab 40 mg cdo javë.
Në prezantim pacientja duket në gjendje të renduar dhe me dispne. Saturimet e oksigjenit të pacientes
janë 84 % dhe frekuenca respiratore është 25/min, TA 82/44 mmHg, frekuenca kardiake 132/min,
temperature 101.9 F (38.8 C). Në auskultacion dëgjohen krepitacione në të gjithë gjysmën e poshtme
të pulmonit të djathtë. Grafia e toraksit paraqet konsolidime ne pjesën e poshtme të djathtë dhe lobet
e mesme pa efuzion pleural. Kultura e gjakut dhe sputumit janë marrë. Pacientja ka filluar hidratimin
iv dhe trajtimin me ceftriaxone dhe moxifloxacine. Pas 2 orësh, pacientja ka marrë 2 litra likide (normal
saline). TA është 98/70 mmHg, dhe frekuenca kardiake 125/min. Asaj I duhet 50% oksigjen me maska

83
nasale për të mbajtur saturime 92-94%. Që nga momenti I prezantimit në urgjencë ka prodhuar 100
ml urinë. Në astrupogramë ka Ph 7.46, PaCO2 32mmHg, PaO2 76mmHg. Si do të klasifikohej pacientja?
A. Shok septik refraktar
B. Shok septik
C. Sepsis
D. Shenja të një përgjigje të mundshme të dëmshme sistemike
E. Sindroma e përgjigjes inflamatore sistemike
79. Një burrë 68 vjec është pranuar në ICU me shok septik. Në fillim ka paraqitur temeperaturë 102.5F
(39.1 C) dhe hipotension. Pasi mori 3 litra likide saline bolus, ai vazhdon të jetë hipotensiv dhe ka
nevojë për support me norepinefrinë prej 10 mikrogram/min. Në prezantim, asnjë burim infeksionni
nuk është I dukshëm. Kultura e gjakut dhe urinës janë marrë. Pacienti e ka të pamundur që të
ekskretojë sputum. Historia e kaluar e pacientit është sinjifikante për SPOK, sëmundje të arterieve
koronare dhe abuzim me alkoolin. Ai po ashtu ka pasur një aksident me mjet para 20 vitesh që ka
rezultuar në splenektomi. Nuk ka alergji nga barnat. Incidenca locale për S.pneumoniae cefalosprorin
rezistent është më shumë se 1 %. Cili është antibiotiku empirik I rekomanduar për këtë pacient?
A. Cefotaxime
B. Cefotaxime dhe Vankomicinë
C. Levofloxacinë
D. Levofloxacinë dhe Vankomicinë
E. Vankomicinë
80. Pavarësishtë terapisë së saktë me antibiotikë dhe kujdesit suportiv, vdekshmëria nga sepsis
mbetet e lartë, me 40-60% të pacientëve me shok septik që vdesin brenda 30 ditëve. Terapi të
ndryshme janë munduar që të përmirësojnë këto vlera të mortalitetit, por shumë kanë rezultuar
joefektive, edhe pas rezultateve fillestare premtuese. Shembuj përfshijnë të gjitha përvec njnërës:
A. Proteina C e aktivizuar
B. Transfuzione me eritrocite për të mbajtur nivelin e hemoglobinës më shumë se 7 g/dl
C. Terapi me insulin për të mbajtur kontroll të ngushtë të glicemisë (100-120 mg/dl)
D. Kryerja e testeve të stimulimit të ACTH për insuficiencën adrenale
E. Frenimi i rrugës së faktorit indoor
81. Një 62 vjecar është sjellë në urgjëncë për dhimbje kraharori që u zhvillua ndërkohë që ai ishte duke
pastruar dëborën pas një stuhije dimërore. Dhimbja filloi 30 min më parë dhe u shoqërua me vështirësi
në frymëmarrje dhe nauze. Dhimbja klasifikohet si 10 nga 10, është shtypëse dhe përhapet poshtë në
krahun e majtë. Ai thirri ambulancën në 10 min e para të shfaqjes së dhiimbjes. Ai ka një histori për
hypertension, hiperlipidemi, duhanpirje dhe abuzim me alkoolin. Ai ka marrë candesartan 16 mg cdo
ditë dhe atorvastatinë 10 mg cdo ditë. Megjithatë ai nuk ka shkuar tek doktori për më shumë se 1 vit.
Ai vazhdon të pijë një paketë e gjysmë duhan në ditë dhe po ashtu pi 6- 10 birra në ditë. Në prezantim,
TA është 84/50 mmHg, frekuencë kardiake 132/min, frekuencë respiratore 28/min dhe SaO2 91%. Ai
paraqitet në distres si pasojë e dhimbjes dhe dispnesë. Ai është i mbushur me djersë dhe I zbehtë.
Ekzaminimi kardiak paraqet takikardi të rregullt me S1 të butë dhe me një galop S3 prezent. Degjohen
krepitacione bilateralisht në të dy pulmonet. Pulset I ka të dobëta. Radiografia e toraksit paraqet
edemë pulmonare. EKG shihet në figurë (VI-81 fq. 515). Ai është duke shfaqur ektopi frekuente dhe
fragmente të vogla të takikardisë ventrikulare. Pacientit iu dha nitroglicerinë dhe aspirinë. Laboratori
I kateterizimit kardiak po bëhet gati pët revaskularizim primar. Ndërkohë, pacienti vazhdon të
përkeqësohet, është intubuar, TA e fundit është 76/52 dhe frekuenca kardiake 122/min. Cili është
opsioni më I mirë për trajtimin fillestar të këtij pacienti?
A. Dopaminë 5mikrogram/kg/min iv me infuzion të vazhduar
B. Fillimi I oksigjenimit ekstrakropral venoarterial me membranë

84
C. Nitroglicerinë 10 mikrogram/min iv me infuzion të vazhduar
D. Vendosja e një pompe intra-aortike me balon
E. Vazopresinë 0.05U/min iv me infuzion të vazhduar
82. Një burrë 72 vjec me kardiomiopati të dilatuar ishemike të njohur dhe me fraksion ejeksioni prej
20 % është pranuar në ICU me insuficiencë kardiake të dekompensuar. TA është 74/56 mmHg dhe
frekuenca kardiake 108/ min. Saturimi I O2 është 85% me një maskë jo-frymëshkëmbyese. Radiografia
e toraksit paraqet edemë pulmonare difuze dhe efuzione bilaterale. Paraqitet një 4+ edemë bilaterale
në të dyja këmbët dhe në sacrum. Ka evidencë për ascit. Ekstremitetet janë të ftohta në prekje dhe
cianotike. Statusi mendor është I dobët. Ai është letargjik dhe nuk I përgjigjet emrit të tij. Të gjitha
indikohen si trajtime fillestare për pacientin përvëc njërës:
A. Dobutamine 5mikrogram/kg/min iv
B. Furosemide 60 mg iv
C. Fillimi I ventilimit mekanik
D. Morfinë 2mg iv
E. Norepinefrinë 5mikrogram/min iv
83. Një grua 52 vjec është në ICU koronare pas një infarkti miokardi në degën anteriore. Ajo
revaskularizua me një angioplasti perkutane të arteries proximale anteriore descendente sinister. 12
orë pas procedurës, ju thirreni në shtratin e të sëmurës për një rënie të presionit arterial dhe dispne.
Dy orë më parë TA ishte 115/72 mmHg. Tani TA paraqitet 90/78 mmHg. Frekuenca kardiake është
rritur nga 88 në 120/min. Saturimet e O2 kanë zbritur në 86 %. Është e djersitur dhe e zbehtë.
Paraqitet në distres. Shfaqet një murmur holosistolik e dëgjuëshme në apeks dhe që përhapet në
drejtim të aksilës së majtë. Ju dyshoni për regurgitim mitral akut. Cila nga të t do të pritej nqs do të
vendosni një kateter në arterien pulmonare?
Presioni në Presioni në Presioni në Presioni Indeksi
atriumin e ventrikulin e arterien kapilar kardiak
djathtë mmHg djathtë mmHg pulmonare pulmonar (l/min/m2)
mmHg mmHg
a 3 20/8 15/6 6 6.2
b 5 20/8 22/10 11 2.8
c 9 30/15 35/18 20 me valë v 2.5
prominente
d 14 22/15 22/10 12 2.1
e 15 32/15 33/15 15 1.8

84. Të gjitha pohimet e mëposhtme për vdekjen e papritur kardiake janë të vërteta përvec njërit:
A. Një pacient konsiderohet se ka bërë vdekje të papritur kardiake kur ka ndodhur pas një arresti
kardiak nga I cili nga shpëtuar, nqs vdekja ndodh gjatë hospitalizmit ose brenda 30 ditëve
B. Pothuajse 66% të pacientëve që kalojnë në vdekje të papritur kardiake kanë pasur evente
kardiake të mëparshme
C. Asistolia është ritmi më I zakonshëm në individë me vdekje të papritur kardiake
D. Historia prindërore e një vdekjeje të papritur kardiake rrit gjasat e eventeve kardiake të papritura
në pasardhësit.
E. Incidenca e vdekjeve të papritura kardiake është më e lartë në popullsinë Afrikano-amerikane
krahasuar me popullsinë e bardhë.
85. Cila nga gjendjet identifikon nje pacient ne koma?
A. mutismi akinetik
B. katatonia
C. pseudokoma (locked-in)

85
D. encefalopatia metabolike
E. gjendje vegjetative persistente
86. Një grua 74-vjeçare u shtrua në reaminacionin neurologjik me koshience të ulur e cila përparoi në
gjendje kome pas renies në një trotuar të akullt. Në ekzaminim fizik, pacientja I pergjigjet minimalisht
stimujve të dhimbshëm. Pupila e majtë është zmadhuar. Shenja e Babinski djathtas ne rritje dhe ka
dobësi të dukshme të anës së djathtë. Kryhet një MRI.
(Figura VI-86). Cila është etiologjia e komës në këtë pacient?
A. Komprimimi i arterieve cerebrale anteriore dhe posteriore
B. Komprimimi i mesencefalonit kundrejt buzës së kundërt tentorale
C. Komprimimi i pedunkulës së kundërt cerebrale kundrejt buzes tentorial
D. Zhvendosja e tonsilave cerebellare në foramen magnum
E. Bllokim i sistemit ventrikular duke shkaktuar hidrocefali

87. Një grua 44-vjeçare shtrohet ne reamiacion me koshience të ulur. Ajo u soll në spital pasi u gjet e
pandergjegjshme në shtëpi pasi nuk shkoi ne dreken me nënën e saj. Ajo është ndjerë në depresion
pasi humbi punën 2 muaj më parë. Ajo jeton e vetme pas një divorci 10 vjet më parë. Ajo nuk ka histori
mjekësore te rendesishme, por nëna e saj është e pasigurt nëse pacienti merr ndonjë ilaç. Presioni
arterial gjatë pranimit është 92/54 mmHg, frekuenca kardiake112 bpm, frekuenca respiratore 24
frymë/min, dhe SaO2 është 99% në ajrin e dhomës. Temperatura është 38.2°C. Ajo duket e skuqur
dhe ka frymëmarrje të thellë të shpejtë. Ajo nuk i përgjigjet stimujve zanor, por reagon ndaj stimujve
te dhimbshëm. Ajo ka spazma muskulore intermitente difuze. Pupilat janë 5 mm dhe reaktive. Cila
është etiologjia më e mundshme e komës së pacientit?
A. Traume e kokes
B. Tromboze e venave kortikale
C. Intoksikim
D. Infeksion sistemik
E. Infeksion meningeal
F. Infeksion meningeal
88. Një 48-vjeçar u pranua në reaminacion me një hemorragji të madhe intrakraniale pas njëaksident
me motor. Në CT fillestar, ka pasur zhvendosje të vijës se mesme dhe evidence të hernimit unkal. Në
vendin e aksidentit, pacienti fillimisht ishte minimalisht I ndergjegjshem, por nuk ka reaguar ndaj asnje
stimuli qe ne momentin e ardhjes ne spital. Pacienti u intubua nga shërbimet e urgjencës mjekësore
gjatë rrugës për në spital. Tashme ai ndodhet ne spital prej 24 orësh. Ju besoni se pacienti ka pesuar
vdekje cerebrale. Cila nga alternativat kontribuon në diagnostikimin e vdekjes cerebrale?
A. Mungesa e refleksit pupilar ndaj drites
B. Apnea
C. Humbja e reflekseve okulovestibulare
D. Mosreagimi ndaj të gjitha formave të stimulimit
E. Të gjitha më lart janë kritere për vlerësimin e vdekjes cerebrale

89. Monitorimi i presionit intrakranial intraparenkimal preferohet mbi ventrikulostominë në cilët prej
pacientëve te mëposhtëm?
A. Një burrë 24 vjeç me dëmtim traumatik të trurit pas një aksidenti me automjet dhe GCS 6
B. Një grua 35-vjeçare me hepatit autoimun me insuficence hepatike fulminante dhe INR 3.5
C. Një burrë 48 vjeçar në reaminacion me hemorragji subaraknoidale pas një rupture të një
aneurizme te arteries cerebrale anteriore
D. Një grua 68-vjeçare me stroke hemorragjike në regjonin e djathtë temporal me zhvendosje te
linjës së mesme ipsilaterale te dukshme ne CT
E. Të gjithe më lart janë kandidatë të përshtatshëm për ventrikulostominë

86
90. Një grua 55-vjeçare pranohet në reaminacion pas një hemorragjie subaraknoidale të shkaktuar
nga ruptra e arteries communicans posterior. Pacienti trajtohet me vendosje intravaskulare tënjë
spirale në aneurizëm dhe vendoset një ventrikulostomi për monitorimin e presionit intrakranial.Pas
mbërritjes në ICU pas operacionit, ajo është e intubuar dhe pergjigjet minimalisht ndaj stimujve të
dhimbshëm. Ajo nuk i përgjigjet stimujve zanor. Presioni intrakranial fillestare është 45 mmHg, dhe
presioni fillestar artrial 138/85 mmHg (presioni arterial mesatar, 103 mmHg) me kokën e shtratit të
ngritur në 30 gradë. Presioni intrakranial mbetet 45 mmHg gjatë 5 minutave të ardhshme. Çfarë do të
rekomandonit më tej në menaxhimin e këtij pacienti?

A. Administroni 30 ml solucin saline hipertonike 23.4%


B. Administroni deksametazon 4 mg çdo 6 orë
C. Mos bëni asgjë pasi pacienti ka një presion të perfuzionit cerebral normal
D. Zbrazni ventrikulostominë për të arritur një presion intrakranial <20-25 mmHg
E. Rritni frekuencen respiratore në aparatin e intubimit për të ulur PaCO2 në 30 mmHg

91. Një 56-vjeçar është shtruar në spital pas një arresti kardiak të dëshmuar. Pas mbërritjes se
personelit mjekësor të urgjencës, evidentohet fibrilacion ventrikular. Konvertohet ritmi ne ritem
sinusal me kthimin spontan te qarkullimit. Kohëzgjatja e arrestit vleresohet 10 minuta. EKG
demonstroi prova të infarktit akut të miokardit me perhapje anteriore. Pacienti trajtohet me
angioplastikë primare të një okluzioni 100% të pjeses proximale te arteries descendente anteriore te
majte. Ai është sjellë në ICU i intubuar dhe kërkon përdorimin e norepinefrines për tu ruajtur presioni
arterial mesatar më i madh se 60 mmHg. Ekzaminimi fillestar neurologjik tregon mungesë të reflexit
korneale dhe farigeal. Kur lëviz kokën anash, sytë lëvizin në përputhje me lëvizjen e kokës dhe nuk
qendrojne të fiksuar. Myoklonusi intermitent është i pranishëm. Ju po konsideroni përdorimin e
hipotermise. Cila nga pohimet karakterizon saktë përdorimin e hipotermisë tek ky pacient?

A. Një CT e kokës duhet të kryhet për të përcaktuar shkallën e edemës cerebrale para se të vendoset
përdorimi i hipotermise terapeutike
B. Menaxhimi i temperaturës duhet të drejtohet në parandalimin e etheve vetëm me një synim
temperatura prej 36°C.
C. Hipotermia terapeutike rekomandohet të synojë një temperaturë prej 32–34 ° C për 24 orë.
D. Nuk ka asnjë rol për hipoterminë terapeutike, sepse prognoza e pacientit është shumë e dobët.
E. Nuk ka asnjë rol për hipoterminë terapeutike sepse nuk ndryshon prognozen e pacientit.

92. Një burrë 67-vjeçar ka qenë në ICU për 3 javë pas pranimit për shkak të nje pneumonie
pneumokoksike. Ai aktualisht është në suport minimal ventilator me presion inspirues mbështetes
prej 15 cmH2O dhe presion ekspirues prej 5 cmH2O. FiO2 është 0.4. Vellimi Tidal spontan në këto
kushte eshte mesatarisht 450 ml. Ai nuk është më I sedatuar, duket alert dhe po ndjek komanda. Ai
duket i dobët dhe i lodhur. Ai nuk është në gjendje të ngrejë krahët ose këmbët. Radiografia torakale
tregon përmirësim të dukshëm te infiltratateve bilaterale. Gjate javës se kaluar, provat per
frymemarrje spontane kane deshtuar. Kur presionit inspirator I suportuar ulet në më pak se 10
cmH2O, frekuenca respiratore rritet ne 28 frymëmarrje / min, dhe vëllimi tidal bie në 220 mL. Cili është
shkaku më i mundshëm i paaftësisë së pacientit për tu deintubuar?

A. Miopatia kritike e sëmundjes


B. Polineuropatia kritike e sëmundjes
C. Progresi fibroproliferativ i sindromës akute të detresit respirator
D. Sindromi Guillain-Barré
E. Deliri i ICU

87
93. Një grua 56-vjeçare pranohet në ICU me një dhimbje koke thunderclap dhe rupture të një
aneurizme te arteries cerebrale anteriore. Ajo trajtohet me kateterizim endovaskular (endovascular
coiling). Me pranimin në ICU, ajo fillimisht ishte intubuar dhe sedatuar. Gjatë 4 ditëve, ajo ekstubohet,
por vazhdon të kete hemiparezë dhe afazi te djathtë. Shtatë ditë pas prezantimit, gjendja e saj
papritmas perkeqesohet me alterim te statusit mendor dhe përkeqësim të dukshëm të hemiparezës.
Një CT e kokës kryhet urgjentisht dhe nuk tregon ndonjë perkeqesim te hemorragjise ose zgjerim të
ventrikujve. Ajo eshte trajtuar me nimodipinë 60 mg çdo 4 orë që nga prezantimi. Cila është shkaku
me I mundshem I gjendjes se saj te përkeqësuar?
A. Edemë cerebrale
B. Hidrocefali
C. Hiponatremia
D. Rirupture e aneurizmit
E. vazospazëm
94. Çfarë trajtimi do të rekomandonit për pacientin në Pyetjen 93?
A. Konsultohuni me neurokirurgun për clipping te aneurizmës
B. restriktimi i ujit te lire
C. Vendosja e një ventrikulostomie
D. Intubimi me hiperventilim dhe trajtim me manitol
E. Norepinefrina intravenoze dhe konsulta për angioplastikën transluminale perkutane

95. Një burrë 64-vjeçar paraqitet në urgjencë duke u ankuar për dispne dhe ënjtje te fytyrës. Ai pi një
pako cigare çdo ditë që nga mosha 16 vjeçare. Ne ekzaminim fizik ai duket dispneik në një kënd prej
45 gradë ose më pak. Krekuenca kardiake 124 bpm, presioni arterial 164/98 mmHg, frekuenca
repiratore 28 frymëmarrje / min, temperatura 37.6 ° C, dhe SaO2 89% në ajrin e dhomës. Pa puls
paradoksal. Venat e qafës janë të zgjeruara dhe nuk kolapsohen gjate inspirimit. Dilatim I venave
kolaterale në toraksin e siperm. Ka edemë të fytyrës dhe 1+ edemë të ekstremiteteve të sipërme
bilaterale. Cianoza është e pranishme. Ka shurdhim ne perkusion dhe renie te tingujve respirator ne
auskultim ne gjysmën e poshtme të mushkërisë së djathtë. Duke pasur parasysh këtë skenar klinik, cili
do të ishte gjetja më e mundshme ne CT torakal?
A. Një lezion që pengon bronkun e djathtë kryesor
B. Një masë e madhe apikale që invadon murin e gjoksit dhe pleksusin brakial
C. Një efuzion i madh perikardial
D. Një efuzion pleural masiv me opacitet të hemitoraksit të djathtë
E. Nyjet limfatike mediastinale të zgjeruara duke shkaktuar bllokim të venës kava superiore
96. Në skenarin e përshkruar në Pyetjen 95, terapia fillestare e këtij pacienti përfshin të gjitha pervec:
A. Administrimi i furosemidit sipas nevojës për të arritur diurezën
B. Ngritja e kokës së shtratit në 45 gradë
C. Rrezatimi urgjent
D. Dieta me pak natrium
E. Oksigjeni

97. Një grua 58-vjeçare me kancer të gjirit të njohur në fazën IV paraqitet në urgjencë me paaftësi për
të lëvizur këmbët. Ajo ka pasur dhimbje mesi për 4 ditët e fundit dhe e ka patur te veshtire te shtrihet.
Nuk ka dhimbje rrezatuese. Sot pacienti humbi aftësinë për të lëvizur këmbët. Përveç kësaj, ajo ka
patur inkontinence urinare kohët e fundit. Ajo eshte diagnostikuar me metastaza pulmonare dhe
pleurale, por nuk dihet te kete metastaza medulare ose cerebrale. Ne ekzaminim fizik verehet
mungesë e lëvizjes në ekstremitetet e poshtme bilaterale të shoqëruara me ndjeshmeri te rene ose
mungese e plote ndjeshmerie nen umbilikus. Verehet tonus i shtuar dhe 3+ refleks I tendineve te thelle
në ekstremitetet e poshtme me aduksion të kryqëzuar. Tonusi I sfinkterit anal I rene dhe refleksi anal
mungon. Cili është hapi I pare në menaxhimin e këtij pacienti?

88
A. Administrimi i deksametazonit 10 mg IV
B. Konsultohuni me neurokirurgun për dekompresionin urgjent të shtyllës kurrizore
C. Konsultohuni me onkologun për rrezatimin urgjent kurrizor
D. Kryeni MRI të trurit
E. Kryeni MRI të të gjithë palcës kurrizore
98. Një 21-vjeçar trajtohet me kimioterapi induktive për leuceminë limfoblastike akute. Leukocitet
para trajtimit ishin 156,000 / μL. Të gjitha ndërlikime priten gjatë trajtimit pervec:
A. Dëmtimi akut i veshkave
B. Hiperkalcemia
C. Hiperkalemia
D. Hiperfosfatemia
E. Hiperuricemia
99. Të gjitha alternativat e do të ishin të rëndësishme për parandalimin e ndërlikimeve të përshkruara
në Pyetjen VI-98 pervec:
A. Administrimi i alopurinolit 300 mg / m2 në ditë
B. Administrimi i likideve IV në një minimum prej 3000 mL / m2 në ditë
C. Alkalinizimi i urinës në një pH më të madh se 7.0 me administrimin e bikarbonatit të natriumit
D. Monitorimi i shpeshtë i serumit çdo 4 orë
E. Hemodializë profilaktike para fillimit të kimioterapisë

Case files
7.1 Nje F 32vj, me nje histori 5 vjecare me HIV, rezultoi me CD4 100 qeliza/mm3. Ajo erdhi ne spital
me nje histori 2 javore me temperature, veshtiresi ne frymemarrje, dhe kolle te thate. Cila nga
ekzaminimet e eshte me I mundshmi per te konfirmuar diagnozen?
A. Silver stain of the sputum
B. Gram stain of the sputum showing gram-positive diplococcic
C. Acid-fast smear of the sputum
D. Serum cryptococcal antigen
7.2 Cili nga organizmat e meposhtem eshte me I mundshem per te shkaktuar nje pneumoni lobare ne
nje pacient me AIDS?
A. Pneumocystis jirovecii
B. Mycobacterium tuberculosis
C. Histoplasmosis capsulatum
D. Streptococcus pneumonia
7.3 Nje F 44vj, e infektuar me HIV u vleresua me CD4 180 qeliza/mm3. Cili nga trajtimet e eshte e
rekomanduar per ta perdorur si profilaksi ne kete pacient ne kete moment?
A. Fluconazole
B. Azithromycin
C. Trimethoprim-sulfamethazole
D. Ganiciclovir
7.4 Nje F 36vj, me HIV erdhi ne urgjence me “seizures”. CT i kokes tregon lezione unzazore te shpeshta
ne tru. Cili trajtim eshte terapia me e mire per kushtet e pacientit?
A. Rifampin, isoniazid, ethambutol
B. Ganciclovir
C. Penicillin
D. Sulfadiazine me pyrimethamie

89
31.1 Nje F 42vj, nga Pakistani trajtohet me infliximab per artrit rheumatoid. Pas terapise 6 mujore ajo
zhvillon temperature persistente, humbje ne peshe, djersitje gjate nates. Suspect tuberkuloz. Cila nga
zonat e eshte me e mundshme per vendosjen e tuberkulozit?
A. Zonat pulmonare te mesme dhe te poshtme
B. Hapesira pleurale
C. Segmenti apikal i lobeve te siperme pulmonare
D. Limfonodujt cervikal ose supraklavikular
31.2 Nje M 24vj, eshte trajtuar me isoniazid, rifampicin dhe pyrazinamide per tuberkuloz aktiv
pulmonar. Pas 3 muajsh, tregon qe ka mpirje dhe gudulisje te dy kembeve por jo dhimbje shpine.
Pacienti mohon te kete marre medikamente te tjera. Kush eshte hapi i rradhes me I pershtatshem?
A. CT te shtylles kurrizore ne zonen lumbare
B. Fillo pyridoxine
C. Vazhddo trajtimin per tuberkulozin dhe monitoro per problem neurologjike te mevonshme
D. Fillo punen per kompresionin ne nerving femoral te adenopatise tuberkulare
31.3 Nje F 25vj, erdhi ne klinike sepse babai i saj, i cili se fundmi imigroi nga Amerika e jugut, u
dignostikua dhe u trajtua per tuerkuloz. Ajo thote se nuk ka patur kolle dhe grafia e toraxit eshte
normale. Testi Mantoux tregon nje indurim prej 10 mm. Mediakamenti I vetem qe merr pacientja
eshte kontraceptivi. Cili eshte hapi I duhur ne vijim?
A. Isoniazid oral dhe kontraceptiv
B. Terapi e kombinuar: isoniazid, rifampin, pyrazinamide
C. Vezhgo pacienten
D. Merr tre kampione sputum
31.4 Cila nga ekzaminimet e eshte me I rendesishmi per te ndjekur nje pacient I cili merr isoniazid
dhe rifampin per trajtimin e tuberkulozit?
A. Test funksional renal
B. Test funksional I heparit
C. Slit-lamp examinations
D. Amilaza dhe lipaza
34.1 Cila nga alternativat jane gjetjet me te mundshme te ekzaminimit fizik pacient me emfizeme?
A. “wheezing” difuz ekspirator
B. Gishtat me “clubbing”
C. Kercitje respiratore bibazilare me presionin jugular venoz te rritur
D. Stridor inspiratory
E. Toni i 3 i zemres S3
34.2 Cila nga gjetjet e jane me te mudnshme te jene ten je zonje 80 vjecare me kifoskolioze?
A. TLC i rritur
B. FEV1/FVC i ulur
C. VC i ulur (kapaciteti vital)
D. VC I rritur (kapaciteti vital)
E. ABG me pH 7.48, PaCO2 32mmHg
34.3 Nje F 56vj, pranon qe eshte perdoruese e duhanit. Ajo ankon per dobesi, dispne me sforcim
minimal, dhe kolle productive cdo mengjes. Cili parameter i mundshem ne kete pacient?
A. DLCO normale
B. Volumi residual i ulur
C. FEV1 normal→ lehtesisht i rritur
D. FEV1/FVC i ulur

90
34.4 Cila nga terapite e eshte me e mundshme qe te jape benefite te mira tek nje pacient me
emfizeme kronike te stabilizuar dhe saturim te O2 86% ne pushim?
A. Tiotropium inhalador ditor
B. Albuterol inhalator aq sa eshte i nevoshem
C. Prednisone oral ditor
D. O2 suplementar te perdorur gjate nates
E. O2 suplementar te perdorur vazhdimisht
35.1 Nje pacient i cili eshte i njohur si astmatik merr terapi me kortikosteroide inhalatore dhe Beta2
agoniste me veprim te shkurter. Ka ankesa per zgjime nokturale te shoqeruara me kolle dhe
“wheezing” me raste. Ky episod perseritet 3-4 here ne dite. Testet pulmonare ne te kaluaren kane
treguar per semundje pulmonare obstruktive te lehte. Kush eshte hapi i rradhes?
A. Steroidet orale
B. Inhibitoret e leukotrieneve
C. Beta 2 agoistet me veprim te gjate
D. Theophylline
E. Terapi antirefluks
35.2 Cili pohim eshte me i sakte?
A. Kolla e shkaktuar nga kaptoprili mund te zgjidhet duke e zevendesuar me enalapril
B. Trajtimi fillestar I kolles kronike duhet te perfshije codeine ose derivate opiatesh te ngjashme per
te supresuar kollen
C. Kola e shkaktuar nga refluksi mund te largohet ne menyre efektive nqs pacienti nuk referon per
dispepsi apo djegie ne gjoks
D. Me shume sesa shkak eshte zakonisht pergjegjes per shkaktimin e kolles kronike tek nje pacient
35.3 Nje F 22vj Afro-Amerikane ka dobesi, artralgji, kolle e thate qe e bezdis per 6 javet e fundit por jo
veshtiresi ne frymemarrje. Ne ekzaminim fizik: pulmonet jane te pastra ne degjim. Pacientja ka noduj
te zmadhuar eritematoze dhe ndjeshme ne zonen pretibiale bilateralisht. Kush eshte hapi i rradhes?
A. Grafi toraxi
B. CT
C. Trajtim empiric per ‘postnasal drip’
D. Antitrupa antinukleare
E. Fillimi I terapise antituberkulare
35.4 Nje M 50vj, obez, me histori astmatike rikthehet me ankesa per dispne ne disa raste dhe kolle
nocturne. Zgjohet ne mengjes me nje shije te tharte ne goje. Mjekmi qe merr aktualisht perfshin
kortikosteroid inhalator dhe beta2 agonist me veprim te shkurter. Kush duhet te jete hapi I rradhes?
A. Monitorim i pH te ezofagut per 24h
B. Grafi toraxi
C. Fillo omeprazole
D. Kortikosteroide orale per periudhe te shkurter
E. Fillo desensitizimin e alergjise
37.1 Nje F 35vj, ankohet per ndjeshmeri te pulpes dhe dispne akute. Gazet e gjakut arterial: Po2
76mmHg. Kush eshte ekzaminimi fizik me i zakonshem per te zbuluar emboline pulmonare?
A. Wheezing
B. Forcim i komponentit pulmonary ne tonin e dyte te zemres
C. Takipne
D. Enjtje e pulpes
E. Rale pulmonare

91
37.2 Nje M 39vj, ka tromboze venoze te thelle pa asnje factor risku te njohur. Ai tregon se vellai I tij
gjithashtu ka patur emboli pulmonare ne moshe 45 vjecare, dhe mamaja e tij ka patur nje “mpiksje”
ne kembe kur ka qene ne te tridhjetat e saj. Kush eshte crregullimi I mundshem ne kete pacient?
A. Deficenca e proteines S
B. Deficenca e antitrombines III
C. Mutacioni: Faktori V Leiden
D. Sindromi i antitrupave antifosfolipidike
E. Sindroma malinje familjare
37.3 Nje F 54vj, vuan nga kanceri cervical. Paraqitet me hemorragji vaginale sinjifikative, me nivel Hb
7g/dL. Kemba e majte eshte e enjtur dhe pas nje ekzaminimi me Doppler zbulohet nje tromboze e
venave te thella/ Kush eshte trajtimi me I mire per trombin?
A. Heparine e pafraksionuar IV
B. Heparine e fraksionuar SC
C. Heparine e pafraksionuar SC
D. Varfarine orale (Coumadine)
E. Filter I venes cava
38.1 Nje pacient 67vj duhanpires per nje kohe te gjate, me SPOK, referon per dhimbje koke dhe enjteje
te madhe te fytyres dhe krahut te djathte. Kush eshte dianoza e mundshme?
A. Angioedema
B. Hypotiroidizmi
C. Sindromi I vena cava superior
D. Trichinosis
38.2 Nje F 64vj, ankohet per ze te ngjirur per 4 muaj. Nuk ka patur temperature, dhimbje fyti apo kolle.
Ne ekzaminim ajo ka zhurma ekspiratore ne fushat e mesme pulmonare te majta. Hapi i metejshem?
A. Bej recete me antibiotike per bronkitin
B. Thuaji qe te beje nje grafi toraxi
C. Keshillo qe te beje gargare me solucion uje-kripe
D. Trajtoje me albuterol inhalator
38.3 Nje F 33vj, e cila nuk eshte duhapirese, ka humbur 15 kg dhe ka kolle. Ne grafi toraxi eshte dalluar
nje mase pulmonare. Kush nga tipat qelizore eshte me e mundshme per kancerin pulmonar?
A. Qeliza skuamoze
B. Adenokarcinoma
C. Qelizat e vogla
D. Qelizate medha
38.4 Nje M 52vj prezantohet me dispne dhe grafia e toraksit tregon nje mase hilare me efuzion pleural
ipsilateral. Kush eshte hapi I rradhes?
A. CT i toraxit, kokes dhe abdomenit per te percaktuar stadin e tumorit
B. Teste funksionale pulmonare per te vleresuar rezervat pulmonare per pulmonektomine
C. Biopsi e mases hilare
D. Fillo radiacionin paliativ sepse pacienti nuk eshte kandidat per rezeksion
39.1 Nje pacient 65 vj, duhanpires me nje histori hipertensioni dhe insuficiance kardiake te lehte vjen
ne urgjence me kolle te perkeqesuar, temperature dhe dispne gjate puhimit. Semundja filloi 1 jave me
pare me temperature, dhimbje muskulare, dhimbje abdominale, diarre dhe me kolle joproduktive e
cila erdhi vone gjate jetes dhe me shejtesi u perkeqesua. Per cilin organizem atipik konsidero terapia
A. Chlamydia pneumonia
B. Mycoplasma pneumonia

92
C. Legionella pneumonia
D. Coccidiomycosis
E. Aspergillus fumigatus
39.2 Nje paciente 85 vj I cili qendron ne shtepin e te moshuarve, me nje histori te insuficiences
kardiake, ka demence nga e cila kerkon asistence per te gjitha aktivitetet ditore. Ajo ka 3 dite me
temperature dhe kolle productive. Grafia e toraxit tregon konsolidim te lobit te mesem te djathte.
Kush eshte zgjedhja me e pershtatshme e antibiotikut?
A. Amoxicillin orale
B. Linezolid IV
C. Cefepime IV
D. Azithromycin orale
39.3 Nje M 56vj, vjen ne urgjence I intoksikuar me alkool. Ai ka episode te perseritura te vjelljes me
intensitet te larte dhe eshte gjetur duke u asfiksuar. Ekzaminimi pulmonar tregon kercitje ne bazen e
pulmonit te djathte. Kush eshte trajtimi me i mire?
A. Fillo azitromicine
B. Fillo terapi me kortikosteride
C. Fillo terapi me haloperidol
D. Observim dhe ndjekje me grafi toraxi
45.1 Nje M 55vj, me insuficience kardiake kongjestive zhvillon efuzion pleural bilateral. Kush jane
karakteristikat e mundshme te fluidit pleural nese kryhet torakoceteza?
A. Pleural fluid LDH 39, LDH ratio 0.2, protein ratio 0.7
B. Pleural fluid LDH 39, LDH ratio 0.2, protein ratio 0.1
C. Pleural fluid LDH 599, LDH ratio 0.9, protein ratio 0.1
D. Pleural fluid LDH 599, LDH ratio 0.9, protein ratio 0.7
45.2 Nje M 39vj, zhvillon “free-flowing pleural effusion” i cili ndiqet nga pneumonia e lobit te poshtem
te majte. Torakocenteza zbulon fluid me ngjyre kashte, me diplokok gram pozitiv sipas Gramit, pH 6.9,
glukoze 32 mg/dL, LDH 1890. Kush eshte hapi i rradhes?
A. Dergo fluidin per kulture
B. Vazhdo trajtimin me antibiotike per infeksion nga pneumokoku
C. Vendos tub torakostome per te drenuar efuzionin
D. Bej follow up permes grafise se toraksit per dy jave, dokumentuar ndryshimet e efuzionit.
45.3 Nje M 69vj, ankohet per perkeqesim te dispense ne menyre graduale dhe kolle bezdisese per tre
muajt e fundit, por jo temperature. U zbulua efuzion pleural dexter, i cili eshte i crregullt me pranine
e gjakut. Kush eshte diagnoza e mundshme?
A. Efuzion parapneumonik
B. Malinjitet ne hapesiren pleurale
C. Rupture e disksionit te aurtes ne hapesiren pleurale
D. Emboli pulmonare me infarkt pulmonar

Pneumologji 18
VI-1. Cila prej alternativave lidhur me auskultimin e kraharorit është e vertete
1. Mungesa e zhurmave respiratore në një hemitorax thuajse gjithmonë lidhet me një
pneumotorax
2. Një klinicist i mprehtë duhet të diferencojë ralet e njoma nga ralet e thata
3. “Astmë kardiake” është wheezing i lidhur me edemën alveolare në IK kongjestive
4. Ronket janë manifestim i obstruksionit të rrugëve me përmasa të mesme
5. Prezenca e egofonisë përdoret për diferencuar fibrozën pulmonare nga alveolare

93
VI-2: Një burrë 72 vjeç me një histori duhanpirje vjen në klinikë për një dispne progresive në efort
prej 3 javësh. Ai ka pasur një kollë joproduktive të moderuar dhe anoreksi, por mohon të ketë pasur
temperaturë, frisone, apo djersitje. Në ekzaminimin fizik, ai ka shenja vitale normale, SpO2 normal
në temperaturën e dhomës. Presioni venoz jugular është normal, ekzaminimi kardiak tregon tone të
ulura të zemrës, pa anomali të tjera. Trakea është në vijën e mesit, nuk ka limfadenopati
shoqëruese. Në ekzaminimin pulmonar, tingulli në fushën pulmonare inferiore të majtë është mat,
fremitusi taktil është i ulur, nuk ka transmetim të zërit. Pas bërjes grafisë së kraharorit, cili do të ishte
hapi tjetër i përshtatshëm në këtë pikë?
1. ABiv
2. Torakocentezë
3. Bronkoskopi
4. Deep suctioning
5. Terapi me bronkodilatatorë

VI-3: Në çfarë volumi pulmonar tërheqja e jashtme e kafazit të kraharorit barazon tërheqjen elastike
të brendshme të pulmonit?
1. Volumi respirator rezervë
2. Kapaciteti rezidual funksional
3. Volumirezidual
4. Volumi Tidal
5. Kapaciteti total pulmonar

VI-4: Një burrë 65 vjeç vlerësohet për dispnenë progresive të efortit që i është shfaqur që prej muaj.
Historia e tij mjekësore është sinjifikative për një episod të pankreatitit nekrotizant që rezultoi në
insuficiencë multiorganore dhe ARDS. Para se të shërohej qëndroi 6 javë me ventilim mekanik. Ai ka
histori për duhanpirje (30 pack-years), por ka hequr dorë para 15 vitesh. Nuk njihet të ketë SPOK. Në
ekzaminimin fizik, dëgjohet një gulçim i ulët inspirator dhe ekspirator, i cili është është më i ulët në
mesin e kraharorit. FEV1 = 2.5l (parashikohet 78%), FVC = 4.0l (parashikohet 94%) dhe raporti
FEV1/FVC = 62.5%. Kurba e vëllimit të fluksit shihet në figurë. Cili është shkaku më i mundshëm i
simptomave të pacientit?
A. Trupi i huaj i aspiruar
B. SPOK
C. Fibrozëpulmonareidiopatike
D. Stenozë subglotike
E. Paralizëekordavevokaleunilaterale

VI-5: Një grua 32 vjeçare paraqitet në departamentin e urgjencës në javën e 36 të shtatzënisë duke u
ankuar për dispne me fillim akut. Ajo ka pasur një shtatzëni të pakomplikuar dhe nuk ka pasur
probleme të tjera. Nuk merr asnjë lloj medikamenti tjetër veç vitaminave të shtatzënisë. Në
ekzaminim paraqitet dispneike. Shenjat vitale janë si në vazhdim: TA 128/78 mmHg, FK 126
rrahje/min., FR 28/min., SpO2 96% në temperaturë dhome. Është afebrile. Ekzaminimi kardiak dhe
pulmonar janë normalë. Ka gjurmë të edemave pedale bilaterale. Një grafi kraharori me mbrojtje
abdominale del normale, EKG-ja tregon takikardi sinusale. Rezultatet e gaz-analizës janë si më
poshtë: pH 7.52, PaCO2 26 mmHg, PaO2 85 mmHg. Cili është hapi tjetër më i përshtatshëm në
menaxhimin e kësaj paciente?
1. Nis terapinë me amoxicilin për bronkit akut
2. Performo një CTA pulmonare (CT angiogramë)
3. Performo një EKO kardiake
4. Siguro pacienten se dispnea normale në këtë stad shtatzënisee dhe në testet e mësipërme
nuk shihen anomali
5. Trajto me clonazepam për atak paniku

94
VI-6. Lidh rezulatet e testeve të funksionit pulmonar me çrregullimin pulmonar të mundshëm:

1. TLC (total lung capacity) i rritur, VC (decreased vital capacity) i ulur, raporti FEV1/FVC i ulur
2. TLC i ulur, VC i ulur, RV (residual volume) i ulur, FEV1/FVC i rritur, MIP (maximum inspiratory
pressure) normal
3. TLC i ulur, RV i rritur, FEV1/FVC normal, MIP i ulur
4. TLC normal, RV normal, FEV1/FVC normal, MIP normal
1. Miastenia gravis
2. Fibrozë pulmonare idiopatike
3. HTP familjar
4. SPOK

Përgjigje: 1C, 2B, 3D, 4A

VI-7: Një grua 78 vjeçare shtrohet në REA me pneumoni multilobare. Në prezantimin fillestar në
departamenitin e urgjencës (DU), SpO2 ishte 60% në temperaturën e dhomës, dhe u rrit deri në 82%
kur pacientes iu vendos maskë oksigjeni. Ajo ishte në distres respirator dhe u intubua në DU. Gjatë
shtrimit në REA, ajo u qetësua dhe u paralizua. Ventilatori u vendos u vendos në modalitetin assist-
control me FR 24, volum tidal 6ml/kg, FiO2 1.0 dhe PEEP 12cmH2O. U marr një gaz-analizë në këto
kushte. Rezultatet janë: pH 7.20, PCO2 32mmHg, PO2 54mmHg. Cili është shkaku i hipoksemisë?
1. Hipoventilimi
2. Hipoventilimi dhe mospërputhja ventilim-perfuzion
3. Shunti
4. Mospërputhja ventilim-perfuzion
VI-8: Një burrë 65 vjeç vlerësohet për dispne progresive në efort dhe kollë të thatë që janë
përkeqësuar 6 muajt e fundit. Ai nuk ka dispne në qetësi dhe mohon të ketë wheezing. Nuk ka
dhimbje gjoksi. Ka histori për sëmundje koronare dhe Afib, dhe i është nënshtruar bypass-it të
a.koronare para 12 viteve. Medikamentet e tij përfshijnë: metoprolol, aspirin, ëarfarin, enalapril. Ai
ka qënë duhanpirës për 40 vjet (pinte një pako cigare/ditë) dhe e ka lënë para 5 vitesh. Shenjat e tij
vitale janë: TA 122/68 mmHg, FK 68 rrahje/min., FR 18/min., SpO2 92% në temperaturën e dhomës.
Ekzaminimi i kraharorit tregon rale bibazilare në rreth 1/3 sipër bilateralisht. Nuk dëgjohet wheezing.
Ai ka një ritëm çrregullisht të çrregullt me një zhurmë holosistolike me intesitet 2/6 në apex. Presioni
venoz jugular nuk është i rritur. Nuk ka edema, por vërehet zmadhim proksimal i gishtave (clubbing).
Testet fnk pulmonar tregojnë : FEV1 65%, FVC 67%, FEV1/FVC 74%, TLC 68%, DLCO 62%. Cili prej
testeve të mëposhtëm mund të vlerësojë etiologjinë e dispnesë së pacientit?
1. Bronkoskopi me biopsi transbronkiale
2. CTA (CT angiography)
3. EKO kardiake
4. HRCT
5. Testi i stresit me medikamente nukleare

95
9.Nje vajze 24 vjec eshte vizituar per dispnene dhe wheezing.Ajo raporton dhe simptomat
perkeqesohen kur ben aktivitet fizik ne natyre dhe ne prani te maceve.Vajza ka pasur rinit alergjik
gjate pranveres dhe veres per shume vite dhe pervec kesaj ka vuajtur edhe nga ekzema kur ishte
femije.Ne ekzaminimin objektiv u vu re te kete wheezing gjate ekspirimit.Provat funksionale pulm
tregojne FEV1=2.67 FVC=3.81 FEV1/FVC=70% .Pas administrimit te Albuterol FEV1 rritet ne 3.0L.
Cili nga pohimet e lidhur me gjendjen e pacientes eshte E VERTETE?
A.Per te konfirmuar diagnozen d=kerkohet prova me metakoline.
B. Mortaliteti per shkak te semundjes eshte rritur dekaden e fundit.
C. Faktori I riskut me I zakonshem ne individe me kete crregullim eshte predispozita gjenetike.
D. Prevalenca e ketij crregullimi nuk ka ndryshuar ne dekadat e fundit.
E.Gravideti I semundjes nuk ndryshon ne menyre te ndjeshme midis pacienteve me ket semundje.

10.Nje grua 38 vjec u dergua ne urgjence me status asmatik.Gjendja e saj perkeqesohet menjhere
dhe humbet jeten nga kjo semundje.Te gjitha te dhenat anatomo-patologjike mund te gjenden ne
kete paciente PERVEC:
A. Infiltrim I mukozes se rrugeve ajrore me eozinofile dhe LT active.
B. Infiltrim I hapsirave alveolare me eozinofile dhe neutrofile
C. Bllokim I rrugeve ajrore nga pllaka tapa mukusi
D. Trashje dhe edeme e murit te rrugeve ajrore
E.Trashje e membranes bazale te rrugeve ajrore me depozitim subepitelial te kolagjenit.
11.Nje vajze 25 vjec ndiqet per simptomat persistente te astmes,pavaresisht trajtimit me Flutikazone
inhalator 88μg 2 here ne dite per tre muajt e fundit.Sipas guideline te Programit Kombetar mbi
Edukimin dhe Parandalimin e Astmes (NAEPP Guideline),e miratuar nga Instituti Kombetar I
Shendetit(NIH), cili nga ndryshimet e ne mjekim mund te merret ne considerate ?
A. Shtim I nje antagonisti te leukotrieneve
B.Shtim I nje B-agonisti me veprim te gjate
C. Shtim I teofilines me doze te ulet.
D.Rritja e dozes se kortikosteroidit inhalator.
E. Cdo ndryshim I mesiperm mund te merret ne konsiderate.
12.Cili nga pacientet e eshte diagnostikuar su duhet me astma?
A.Nje vajze 24 vjec,e mjekuar me kortikosteroid inhalator per kollen dhe wheezing qe ka
persistuar edhe 6 jave pas nje infeksioni viral te rrugeve te siperme ajrore.
B. djale 26 vjec me kolle dhe me raste wheezing pas aktivitetit fizik ne mot te ftohte.
C. Grua 34 vj, kolle kronike dhe vlera te FEV1/FVC=68% dhe FEV1 rritur nga 1.68L ne 1.98L
pas administrimit te Albutelolit.
D. Nje burre 44 vjec qe punon ne nje laborator te eksperimenteve mjekesore mbi minjte me
ankesat e wheezing,dispense dhe kolles qe perkeqesohen ne fund te javes.
E. Nje burre 60 vjec ,I cili ka pire dy paketa ne dite per 40 vjet ,me dispne,kolle dhe
hiperaktivitet te rrugeve ajrore ndaj metakolines.
13.Nje grua 40 vjec me astme persistente te moderuar ka qene ne kontroll te mire per 3 muaj dhe
eshte duke perdorur Albuterol MDI per kontroll simptomatik 1 here ne jave.Ajo zgjohet naten nga
simptomat e astmes dy here ne muaj,por vazhdon aktivitetin fizik rregullisht pa asnje
veshtiresi.Medikamente te tjera perfshijne Flutikazone inhalator 88 μg /puff 2 here ne dite dhe
Salmeterol 50 μg 2 here ne dite.Aktualisht pacientja eshte me FEV1 83% .Cili eshte veprimi I
pershtatshem per kete gjendje?
A. Shto Montelukast 10 mg 1 here / dite,meqe perdorimi I Albuterolit kontroll te dobet
B. Te ulet Flutikazone ne 44 μg /puff 2 here ne dite
C. Te nderpritet Flutikazone
D. Te nderpritet Salmeterol.
E. Te mos behet asgje, meqe perdorimi I Albuterolit jep kontroll te dobet te astmes.

96
14.Ti po merr ne considerate terapine me Omalizumabin te nje pacient me astme persistente te
rende qe kerkon Prednizon oral 5-10mg ne dite vec kortikosteroideve inhalator me doze te
larte,bronkodilatator me veprim te gjate dhe montelukast per te kontrolluar simptomat. Cila nga
eshte e nevojshme te behet para fillimit te omalizumab-it
A.Nderprerja e Prednizonit oral
B. Te demostrohet rritja e Ig E mbi 1000IU/L
C. Normalizimi I FEV1 ose PEEF
D. Testimi I prezences se sensitivitetit ndaj nje aeroalergenti per gjate gjithe vitit.
E. Kthimi I Prednizonit oral ne Prednizolon iv.
15.Nje grua 76 vjec vleresohet nga mjeku per fillimin akut te dispense, kolles se thate ne dy ditet e
fundit dhe temperatures se larte 39.2C.Ne historine e saj mjekesore perfshihet DM dhe
hipotiroidizmi.Aktualisht eshte ne mjekim me Metformin 1000mg 2xdite.Doza e Levotiroxines I eshte
rritur ne 100 ne dite muajin e fundit dhe I eshte shkruar ne recete Nitrofurantoin 100mg 2xdite(per 3
dite) per infeksionin urinar qe ka.Shenjat vitale te saj trekojne TA=115/82mmHg FK=96rr/min
FR=24f/min,temperature 38.5C satO2=96%.Ne perkusion dhe auskultacion ka matitet(dullness) dhe
zhurma respiratore te dobta ne bazen e pulmonit te djathte,ndersa krepitacionet degjohen
bilateralisht.Radigrafia e pulmoneve tregon efuzion pleural ne pulomnin e djathte dhe pllaka(patchy)
iinfiltratesh ne te dyja anet.Pacientja shtrohet ne spitale dhe I behet torakocenteza prej te ciles u
morr likid eksudativ.Ne ekzaminimin citologjik te likidit u gjenden WBC=3500/mm3 me diferencim
60%PMN 30% EOS dhe 10% LYM.Ndersa ne bronkoskopi u dallua nje shkalle diferencimi 50%PMN
15% EOS dhe 35% makrofage alveolare. Cili eshte hapi I radhes per trajtimin e kesaj pacienteje?
1. A) Prit rezultatet e kultures se efuzionit pleural para se te besh rekomandimin.
2. B) Ulja e dozes se Levotiroksines.
3. C) Nderprit Nitrofurantoinen.
4. D) Rrit dozen e Levotiroksines.
5. E) Fillo terapine me steroide me doza te larta (Metilprednizolon 1g/d)
16.Nje grua 34 vjec kerkon te vizitohet per kollen dhe dispnene ne efort,te cilat kane ardhur duke u
renduar ne 3 muajt e fundit.Pacientja nuk vuan nga astma apo ndonje semundje tjeter
pulmonare.Para 6 muajsh nisi te punonte ne nje dyqan kafshesh duke pastruar kafazet e tyre.Ajo
raporton se ka pasur me raste temperature te lehte,kolle te thate ,pa fishellimia(wheezing).Tre
muajt e pare e perballonte mire sforzimin fizik,por me pas nisi ti shfaqej veshtiresia ne frymemarrje
menjehere pas ngjitjes se shkalleve.Gjendja fizike e saj duket e mire.Ne ekzaminimin objektiv satO2=
95% ne qetesi dhe 89% ne efort,temperature 37.7C.Nuk u gjet ndonje e dhene ne ekzaminimin e
pulmoneve(pa cianoze dhe pa thonjte e Hipokratit)Radigrafia e pulmoneve doli e paster.Ne CT u
dallua infiltrate difuz si ‘’xham I thyer’’(ground-glass) ne lobet e poshtme si dhe prezenca e nodujve
centrilobulare.Nga biopsia transbronkiale u gjend infiltrate alveolar intersticial me qeliza plazmatike,
limfocite,eozinofile (te rralla) si dhe granuloma jonekrotike.Te gjitha kulturat dolen negative per
patogjenet bakterial,viral dhe fungeal. Cila eshte diagnoza?
1. A) Aspergillosis
2. B) Pneumoni nga hipersensibiliteti
3. C) Pneumoni intersticiale jospecifike e ligjur me semundjen e kolagjenit te vazave
4. D) Psittacosis
5. E) Sarcoidosis
17) Femer 36 vjec vjen me kolle dhe dispne ne aktivitet fizik e cila kjo e fundit eshte
perkeqesuar 3 muajt e fundit .Pacientia nnuk ka histori te meparshme per semundje
pulmonare.Ajo punon ne nje dyqan kafshesh per rreth 6 muajt e fundit.Detyra e saj eshte te
pastroj kafazet e zvarranikeve dhe zogjeve.Ajo raporton se ndnjehere ka temperature te
ulet(occasional low grade fevers),por nuk ka wheezing(gulcoj,dihas,mer frym me zore).Kolla
eshte e thate dhe jo productive.Perpara 3 muajve nuk kishte limite per tolerance e
ushtrimeve(no limitation of exercise of tolerance) por tn raporton se behet dispneike kur
ngjite shkallet e 2 kateve. Examinim fizik :saturimet e O 2 jan 95% ne qetesi por ndodhe

97
desaturim deri ne 89% me ambulation.Temp 37.70c . Examinimi pulmonare eshte I
zakonshmen pa ndnje ndryshime(unremarkable),cianoza dhe clubbing nuk jan prezente.
Radiografitoraksi norml. CT me rezolucion te larte toraksi tregon infiltrate difuze ground
glas(zone qe seshte shum e qarte) ne lobet inferiore me presence te nodujve
centrilobulare.Biopsia transbronkiale tregon infiltrate intersticial alveolar me
limfocite(plazma cells) dhe eozinofile .Kulture negative per patogjenet
bacterial,virla,fungal.Cfare trajtimi do rekomandoni?
1. Amfotericine
2. Doxaciklin
3. Glukokortikoid
4. Glukokortikoid+azatioprin
5. Glukokortikoid+removal of antigen
18) Burre 75 vjec po vlersohet per effusion pleural ne anen e majte dhe veshtirssi ne
frymarrje .Ai ka punuar si punonjes izolimi ne nje kantier detar x 30 vjet dhenuk ka perdorur
mjete mbrojtese ,duhanpires me COPD te moderuar te njohur (FEV1 55%) +histori IAM para
10 vitesh.Mjekimet qe perdor aspirin,atenolol,benazepril,tiotropium,albuterol.EX,fizik:
efuzion ne anen e majte me shurdhesi (dullness)ne perkursion dhe reduktim te zhurmaave
respiratore e cila ndodh vetem ne gjysmen e hemitoraksit .Ne RTG toraksi ka nje efuzion
plerural te moderuar ne anen e majte ,me kalcifikime pleurale bilaterale dhe trashje pleurale
apikale te majte.Nuk shihet mase ne pulmon .CT toraksi konfirmon gjetjet n RTG e toraksit.
Gjithashtu ka atelektaze kompresive ne lobin e poshtem te majte .Behet torkocenteza e cila
tregon effusion eksudative me 65% limfocite ,25%qeliza mesoteliale dhe 10% neutrofile
.Citologjia nuk tregon malinjitet. Cili pohimi I vertet per shkakun e mundeshem te efuzionit ?
1. Duhanpirja rrit me teper mundesin per tu zhvilluar kjo situate
2. Vdekja ne kete semundje eshte e lidhur mesemundjen metastatike difuze
3. Ekspozimin ndaj agjenteve shkaktare mund te jete per rreth 1-2 vjet dhe gjendja
latente e expresionit te semundjes mund te jete me shume se 40 vjet
4. Citologjia e perseritur e likidit pleural te drejton te dg definitive
5. Terapia me kirurgji+kimioterapi adjuvante permirson ne enyre sinjifikative
mbijetesen long term
19)Cili prej kushteve meposhte rrit riskun per silikoze kronike?
1. Infeksion me aspergillus invasive
2. Infeksion me mycobakter tuberculosis
3. Kancer pulmonic
4. Artrit rheumatoid
5. Te gjitha mesiper
20)Te gjitha semundjet occupational pulmonare lidhen me faktorine ekspozues pervec
1. Berylliosis –paisjet elektronike hight teck
2. Byssinosis-bluarje e pambukut
3. Farmers lung –bari I mykur
4. Fibroze massive progressive –puntoret e kantierit detar
5. Metal fume fever-ethe tymi metalik (saldim)
21) Maskull 45 vjec vlersohet ne klinike per astem. Simptomat e tij filluan para 2 vitesh dhe
kkt nga kolle episodike dhe wheezing te cilat fillimisht I pergjigjeshin bronkodilatatoreve
inhalatore dhe kortikosteroideve inhalator por tn kerkon marrjen konstante te prednisonit .Ai
ka ven re se simptomat perkeqesohen gjate javes por nuk mund te percaktoj shkaktarin
specific. Medikamentet e tij jan : albuterol MDI,flutiasoneMDI,prednisolone 10mg/dite.
Pacienti nuk ka vese dhe eshte puntor tekstili. Ne ekzaminim fizik degjohen wheezing
ekspirator polifonik difuz e mesme,por pa anomaly te tjera .Cili hapi pasues I pershtatshem ?
1. Provaushtrimore(execisephysiologytesting)

98
2. Matja e FEV1 para dhe pas punes
3. Testi me methacholin
4. Teste alergjike te lekures
5. Kultura e sputumit per Aspergillus fumigatus
22) Nj burre 53 vjec paraqitet ne urgjence me fillim te papritur te temperatures, ethe,gjendje e
pergjithshme jo e mire dhe dispne por jo wheezing.Ai nuk ka histori sinjifikative dhe ai eshte
fermer. Ai ka punuar in the day stacking hay .Rtg e toraksit postero-anteriore dhe laterale
tregon infiltrate bilateral ne lobet e siperme .Cili nga organizmat pergjegjes per situaten ?
1. Noeardia asteroides
2. Histoplasmacapsulatum
3. Cryptococcus neoformons
4. Actinomyces
5. Aspergillus fumigatus
23) Të gjitha kushtet e mëposhtme shoqërohen me një rrezik të shtuar të stafilokokut aureus
rezistent ndaj meticilinës si një pneumoni e lidhur me kujdesin shëndetësor pervec
1. Antibiotiko terapia ne 3 muajt e fundit
2. Dializakronike
3. Kujdes I plages ne shtepi
4. Hospitalizim per me shume se 2 dite ne 3 muajt e fundit
5. Nursinghomeresidence
24)cili nga pohimet ne lidhje me diagnozen e pneumonis ne komunitet e vertet?
1. Terapi direkte specifike ndaj mikroorganizmit shkaktar eshte me shume efektive sesa
terapia emperike ne pacientet e hospitalizuar ne cilet nuk jan ne terapi intensive
2. 5% nga 15%e pacienteve te hospitalizuar me pneumoni ne komunitet do kene
culture gjaku positive
3. Ne pacientet qe kane bakteremi nga streptococcu pneumonie,kultura e sputumit eshte
positive ne 80% te rasteve
4. Testi I PCR per identifikimin e legionella pneumonia me mycoplasma pneumonia jan
te disponueshme dhe duhet te perdoren per diagnozen ne pacientet e hospitalizuar me
pneumoni ne komunitet
5. Etiologjia e pneumonis ne komunitet eshte identifikuar ne pothuajse 70% te rasteve

25. Një mashkull 55 vjeç paraqitet tek mjeku i familjes me një histori dy ditore kolle dhe
temperature. Kolla është produktive me përmbajtje sputumi të trashë në ngjyrë jeshile të
errët. E kaluara mjekësore përfshin hiperkolesterolemi të trajtuar me rosuvastatin. Nuk pi
duhan dhe është fizikisht aktiv, ushtrohet disa herë në javë. Nuk e mban mend kohën e fundit
kur ka marrë antibiotikë. Shenjat vitale të tij janë: temperatura 38.9 gradë C, presioni arterial
132/78 mmHg, frekuenca kardiake 87 rrahje/min, frekuenca respiratore 20/ min, saturimi i
oksigjenit 95%. Radiografia e kraharorit prezanton konsolidime segmentare të lobit inferior të
djathtë. Çfarë do të bësh më pas me këtë pacient?
A. Merr kulturë sputumi dhe prit rezultatet përpara se të nisësh trajtimin B. Bëji një
CT kraharori për të përjashtuar një pneumoni postobstruktive
C. Kaloje në pavionin e urgjencës për administrim dhe trajtim me antibiotikë IV
D. Trajtoje me doxyciklinë 100 mg dy here në ditë
E. Trajtoje me moxifloxacinë 400 mg në ditë

26. Femër 65 vjeçë sillet në reanimacion për trajtimin e shokut septik të lidhur me infektimin
e kateterit të hemodializës. Ditën e parë u intubua për shkak të detresit respirator akut. Arriti
të përmirësohej nga dita e parë, për shkak se fraksioni i inspiruar i oksigjenit në fillim ishte
0,4 ose 40%. Nuk ishte më febrile dhe nuk iu dhanë vazopresorë për shkak se nuk

99
nevojiteshin atë moment. Në ditën e shtatë, ajo zhvillon temperaturë 39.4 gradë celsius me
shtim të sputumin jeshil të errët. Ti dushon se mund të ketë zhvilluar një pneumoni prej tubit
endotrakeal. Cila është mënyra më e mirë për të vendosur diagnozën në këtë paciente?
A. Aspirim endotrakeal i sputumit për identifikimin e një mikroorganizmi tipik të
pneumonisë nozokomiale
B. Prezenca e një infiltrati të ri në një radiografi toraksi
C. Kultura e marrë me aspirim endotrakeal që tregon më shumë se 10 në fuqi të 6 të
mikroorganizmave tipikë të pneumonisë nozokomiale
D. Kulturë e një mostre të marrë me furçë të mbrojtur që tregon 10 në fuqi të 3
mikroorganizma tipikë të pneumonisë nozokomiale
E. Nuk ka një kriter fiks të besueshëm diagnostik për pneumoninë e marrë nga
ventilimi i një pacienti.

27. Cili aktore arrin të japë një klinikë të ngjashme me pneumoninë e fituar në komunitet?
A. Pneumonia aspirative: Streptokoku piogjen
B. Konsumimi i gjerë i alkoolit: Patogjenë atipikë dhe stafilokoku i artë
C. Higjiena e ulët dentare: Clamidia pneumonie, kliebsela pneumonie
D. Sëmundje pulmonare strukturore: Pseudomonas aeruginosa, Stafilokoku i artë
E. Udhëtimet në jugperëndim të SHBA: Aspergilusi saprofit

28. Cili nga më poshtë është shkaktari më i shpeshtë i bronkektazisë difuze në mbarë botën?
A. Fibroza cistike
B. Deficiencat imunoglobinike
C. Infeksionet me mycobacterium avium-intraqelizor
D. Infeksionet me mycobacterium tuberculosis E. Artriti rheumatoid

29. Femër 54 vjeçe që ankohet për kollë kronike të përkeqësuar në 6-12 muajt e fundit. Ajo
thotë se kolla është prezente gjatë ditës dhe natës, është kollë produktive me përmbajtje të
sputumit jeshil të errët. Gjatë gjithë ditës mund të kollitet dhe të nxjerrë mbi 100 ml sputum.
Dëgjohen rale bilaterale në zonat inferiore të mushkërive. Testet e funksionit pulmonar
tregojnë një FEV1 1.68L (53,3% e normës), FVC 3L (75% e normës), dhe raporti midis të
dyjavë 56%. Në radiografi nuk vihet re gjë. Çfarë do të rekomandoje ?
A. Bronkoskopi me lavazh bronkoalveolar
B. CT toraksi me kontrast intravenë
C. CT toraksi me rezolucion të lartë
D. Matjen e nivelit të imunoglobulinave në serum
E. Trajtimin bronkodilatator me kohë të gjatë veprimi, + kortikosteroide inhalatore
30. Mashkull 48 vjeç paraqitet me temperaturë dhe kollë në spital. Është i pastrehë dhe i
alkoolizuar. Nuk kujdeset për shëndetin e tij. Thotë se është ndjerë keq në 8 javët e fundit, ka
ndjerë lodhje, gjendje të rënduar dhe humbje në peshë pasi e ndjen se rrobat i ka më të gjera
tashmë, por nuk e vlerëson saktë sa ka qenë rënia në peshë. Disa herë ka pasur edhe
temperaturë. Gjatë kësaj kohe ka pasur edhe kollë të dhimbshme me sputum të errët.
Llogariten të paktën 3 lugë gjelle me sputum në ditë, disa herë edhe me përmbajtje gjaku.
Nuk merr medikamente, por thotë se në ditë mund të pijë rreth 1 litër vodka. Pi gjithashtu 1
pako cigare në ditë. Në ekzaminimin fizik duket i shpërqendruar. Shenjat fizike janë:
frekuenca kardiake 98 rrahje në minutë, presioni arterial 110/73 mmHg, frekuenca respiratore
20/min, temperatura 38.2 grade celsius dhe saturimi i oksigjenit 94%. Vihet re që nuk
kujdeset për shëndetin oral dhe ndihet një erë e keqe nga goja. Dëgjohen zhurma amforike në
pjesët fundore të mushkërive në kurriz. Një CT toraksi tregon një lezion kavitar 4 cm në lobin

100
inferior të djathtë. Pacienti hospitalizohet në kushte izolimi. Kërkohet kulturë e sputumit për
baktere, mykobaktere dhe funge. Cila është zgjedhja më e mirë për fillimin e trajtimit?
A. Ampicilinë-sulbactam 3g intravenë çdo 6 orë
B. Izoniazid, rifampinë, pirazinamid dhe etambutol oral
C. Metronidazol 500 mg oral katër herë në ditë
D. Drenim perkutan i kavitetit
E. Piperacilin-tazobactam 2.25 mg intravenë çdo katër orë në kombinim me
tobramicin 5mg/kg intravenë në ditë
31. Kontrollohet një mashkull 35 vjeçar për infertilitetin. Nuk ka pasur fëmijë të mëparshëm
dhe pas dy vitesh marrëdhëniesh të pambrojtura, e shoqja nuk ka mundur të mbetet shtatzënë.
Analiza e spermës tregon numër normal spermatozoidësh, por këta paraqiten të palëvizshëm.
E kaluara mjekësore tregon për infeksione rekurrente sinopulmonare dhe së fundmi pacienti
ka mësuar se vuante dhe nga bronkektazia. Çfarë do të tregojë radiografia e këtij pacienti?
A. Limfadenopati bihilare
B. Infiltrate bilaterale në lobet e sipërme C. Do të rezultojë normale
D. Situs inversus
E. Zemër në formë balloni
32. Një femër 28 vjeçe vlerësohet për shkak të infeksioneve rekurrente pulmonare dhe të
sinuseve. Duke filluar nga adoleshenca e saj, ajo thotë se thuajse çdo vit ka të paktën një
episod bronkiti. Në 5 vitet e fundit ajo thotë se së paku ka marrë antibiotikë 3 herë në vit për
infeksionet respiratore ose sinusale. Ajo gjithashtu raporton humbje në peshë, si dhe ndihet
më e shkurtër se sa bashkëmoshataret e saj. Në ekzaminimin fizik, ka BMI 18.5 kg/m2.
Saturimi i oksigjenit 94%. Vihet re prezencë e polipeve nazale. Rale dhe ronke dëgjohen
bilateral në lobët e sipërme. Radiografia e toraksit tregon bronkektazi bilaterale të lobeve të
sipërme me zona të bllokuara nga mukusi. Ti dyshon për mundësinë e një fibroze cistike të
padiagnostikuar. Cila nga testet mbështesë më tepër diagnozën e fibrozës cistike ?
A. Analiza e ADN që tregon një kopje të alelit delta F508.
B. Diferencat në potencialet bazike nazale në respiracion
C. Prezenca e pseudomonar aeruginosa në mënyrë të përsëritur në kulturën e sputumit
D. Klori në djersë më i lartë se 35 mEq/L
E. Klori në djersë më i lartë se 70 mEq/L

VI-33. Nje djale 22 vjev me fibroz kistike vjen per nje kontroll rutine. Aktualisht trajtohet me
ADNaze humane te rekombinuar dhe albuteron me nebulizant dy here ne dite. His primary
sputum clearance technique is aerobic exercise five times weekly and autogenic drainage. Ai
ndjehet mire pergjithesisht dhe ekzaminimet jane normale. Testet e funksioneve pulmonare
tregojne nje FEV1 4.48 L (97% e parashikuar), nje FVC 5.70 L (103% e parashikuar), dhe
nje FEV1/FVC 79%. Ne nje kulture rutine sputumi rritet Pseudomonas aeruginosa. I vetmi
organizem i izoluar ne kulturat e meparshme ka qene Staphylococcus aureus.
Cfare do te rekomandoni per kete paciente?
A. Oshilacione me frekuence te larte te kraharorit (High-frequency chest wall oscillation)
B. Saline hipertonike (7%) me nebulizante 2 here ne dite
C. tobramycin me inhalim 300 mg 2 here ne dite cdo muaj
D. Cefepime and tobramycin intravenoze per 14 dite.
E. Rikontroll pas 3 muaj me perseritje te kultures se sputumit dhe trajtim vetem nese ka
presence te P. aerugunosa

VI-34. Cili nga organizmat e ka me pak mundesi te gjendet ne sputumin e nje pacienti me
fibroz cistike. A. Haemophilus influenzae B. Acinetobacter baumannii
C. Burkholderia cepacia D. Aspergillus fumigatus E. Staphylococcus aureus

101
VI-35. Te gjitha t jane faktor risku per semundje pulmonare obstruktive PERVEC: A.
Hipersensitiviteti i rrugeve ajrore ( Airway hyperresponsiveness)
B. Ekspozimi nga pluhurat e qymyrit
C. Duhanpires pasiv
D. Infexione respiratore te perseritura
E. Perdorimi i biomass fluels (burrime energjie te rinovueshme) ne zonat pak te ajrosura

VI-36. Nje grua 65 vjece vleresohet per dispne ne efort dhe kolle kronike. Ajo ka nje histori
te gjate si duhanpirese, pi 1.5 paketa ne dite qe ne moshen 20 vjecare. Ajo eshte nje grua e
dobet dhe pa shenja ankthi te dukshme. Saturimi i oksigjenit ne kushtet e dhomes eshte 93%
dhe frekuenca respiratore 22/min. Pulmonet ne perkusion jane te hiperekspanduara me
dobesim te zhurmave pulmonare ne pjesen e siperme te mushkrise. Ju dyshoni per semundje
pulmonare obstruktive akute.

Cfare rezultatesh priten ne testet e funksionit pulmonar? Pergjigja: B


FEV1 FVC FEV1/FVC TLC DLCO
A. E ulur Normale ose e ulur E ulur E ulur E ulur
B. E ulur Normale ose e ulur E ulur E rritur E ulur
C. E ulur E ulur Normale E ulur E ulur
E rritur
D. E ulur Normale ose e ulur E ulur Normale ose e rritur

VI-37. Nje burre 70 vjec me semundje pulmonare obstruktive kronike vjen per nje kontroll
rutine. 6 muajt e fundit ka qene i qendrueshem klinikiste dhe pa ekzacerbime. Pergjithesishte
nuk ndihet mire dhe eshte i limituar ne ate cka mund te bej. Ai referon per dispne ne
aktivitetet e perditshme. Aktualishte trajtohet me Albutamol inhalatore metered-dose (doza
qe matet nga pompat ne e kemi me shkrepje, nje shkrepje) dy here ne dite dhe sa duhet. ai ka
nje hstori 50 vjecare (50 pack-year) si duhan pirese dhe e ka lene 5 vite me pare. Probleme te
tjera mjekesore perfshijne semundje vaskulare periferike, hipertension, dhe hiperplazi beninje
e prostates. Ai trajtohet me aspirin, lisinopril, hidroklortiazid dhe tamsulosin. Ne ekzaminim
saturimi i oksigjeni ne gjetesi ne kushtet e dhomes eshte 93 %. Pulmoni ne perkusion eshte i
hiperekspanduar me dobesim i zhurmave respiratore ne apeks dhe wheezing i zbehte. Testet e
funksionit pulmonar tregojne nje FEV1 55%, nje FVC 80%, dhe nje raport FEV1/FVC 50%.
Cili eshte hapi i radhe me i mire per trajtimin e ketij pacienti?
A. Fillo nje trajtim me glukokortikoid oral per nje periudh 4 javore dhe fluticason inhalatore,
vlereso nese ka permiresime ne funksionin respiratore.
B. Fillo trajtim me fluticasone 110μg/puff 2 here ne dite.
C. Fillo trajtim me fluticasone 250μg/puff e kombinuar me salmeterol 50 mg/puff 2x/ dite.
D. Fillo trajtim me tiotropium 18 μg/ne dite
E. Performo ushtrime dhe oksimeter nokturn, dhe fillo O2 terapi ne ka hipoxemi te thelle

VI-38. Nje grua 56 vjecare pranohet ne njesine e kujdesit intensive me nje histori 4 ditore te
rritjes se shkurtimit te frymemarjes dhe kolle me sasi te larta sputumi. Ajo njihet per SPOK te
rende me nje FEV1 42 %. Ne kushtet e dhomes nivel e gazeve te gjakut jane pH 7.26. PaCO2
78 mmHg dhe PaO2 50 mmHg. Ajo eshte payshim ne detres respirator me perdorim te
muskujve aksesor dhe retraksion. Breath sounds are quiet me wheezing ekspirator difuz dhe
ronke. Nuk ka prani te infiltrateve ne radiografin e toraksit.
Cila nga terapite me e dobishme per reduktimin e mortalitetit ne keta paciente?

102
A. Administrimi i bronkodilatatoreve inhalator B. Administrimi i glukokortikoidev IV
C. Fillimi i hershem i antibiotikeve me spekter te gjere qe perfshijne edhe Pseud.aeruginasa
D. Intubimi i hershem me ventilator mekanik.
E. Perdorimi i ventilatoreve joinvaziv me presion pozitiv (Use of noninvasive positive
pressure ventilation)

VI-39. Nje burre 63 vjec me nje histori te gjate si duhanpirese paraqitet tek ju per nje histori 4
mujore me shkurtim progresiv te frymemarjes dhe dispne ne efort. Simptomat jane te lehta
dhe pa perkeqesime. Pacienti nuk referon per temperature, dhimbje kraharori apo hemoptizi.
Ai ka nje kolle te perditshme me sputum te verdhe-sasia 3-4 luge (3–6 tablespoons of yellow
phlegm). Pacienti thote qe nuk eshte vizituar ne nje mjek famije per me teper se 10 vjet . Ne
ekzaminimin fizik senjat vitale jane normale, ekpiracion i zgjatur, ronke te shperndara, pulsi
venoz jugular i rritur dhe edem e moderuar e kembeve. Hematokriti eshte 49%.
Cila nga terapite e ka me shume gjasa te zgjas mbijetesen e tij?
A. Atenolol B. Enalapril C. Oksigjen D. Prednisone E. Theophiline

VI-40. Nje burre 62 vjec vleresohet per dispnen ne eforte e cila ka ardhur duke u perkeqesuar
ne nje periudhe 10 mujore. Ai ka nje histri duhanpires 50 pack-year dhe e ka lene 10 vite me
pare. Vleresimi fiziologjik dhe radiologjik tregojne per defekte restriktive respiatore me
fibroz difuze qe eshte me e avancuan ne regjonin subpleural dhe ne baze. Behet biopsi
pulmoni e cila rezulton ne pneumoni intersticiale. Nuk jane gjetur shkaktare autoimun apo te
lidhura me medikamentet. Cili eshte trajtimi i rekomanduar ne kete pacient.
A. Azatioprine 125 mg ne dite plus prednisone 60 mg ne dite
B. Ciklofosfamide 100 mg ne dite
C. N-acetilcistein 600 mg 2x/dite plus prednisone 60 mg ne dite
D. Prednisone 60 mg ne dite
E. Refero per transplant pulmoni.
41. Cfare pritet ne lavazhin broncoalveolar te nje pacienti me hemorragji difuze alveolare?
1. Pneumocite tip II hiperplastike atipike
2. Trupa ferrugios (ferruginous bodies)
3. Makrofageqembajnehemosiderine
4. Limfocite me rritje te raportit CD4:CD8
5. Dukje si qumesht me makrofag te bulezuar

42. Nje mashkull 42 vjec prezantohet me dispne progressive ne effort, temperature te ulet,
dhe humbje ne peshe gjate 6 muajve. Ai gjithashtu ankohet per kolle te thate fillimisht,
megjithese ndonjehere ai kollitet dhe nxjerr sputum te trashe mukopurulent. Nuk ka histori te
meparshme mjekesore. Nuk pi duhan. Ne ekzaminimin fizik, pacienti shfaqet dispneik ne
efort minimal. Temperature e pacientit 37.9oC. SaO2= 91% ne ajrin e dhomes ne qetesi. Ne
analiza laboratorike pacienti ka poliklonal hipergamaglobulinemi dhe hematokrit 52%. Ne
CT scan duket infiltrime alveolare bilaterale qe jane fillimisht me natyre perihilare me nje
model mozaiku (mosaic patern). Pacienti shkon per bronkoskopi me lavazh bronkoalveolar.
Permbajtja duket si qumesht. Citopatologjia tregon mbetje ( debris) amorfe me periodic acid-
Schiff (PAS)- makrofage pozitiv. Cila eshte diagnoza?

1. Pneumoni e organizuar bronkiolite obliterante


2. Pneumnitis deskuamativ intersticial
3. Nocardiosis
4. Pneumcystis carinii pneumonia
5. Proteinoz alveolare pulmnare ( pulmonary alveolar proteinosis)

103
43. Cili trajtim eshte me i pershtatshem ne kete kohe per pacientin ne pyetjen 42?
1. Doxycycline
2. Prednisone
3. Prednisonedhecyclophosphamide
4. Trimethoprim- sulfamethoxazole
5. Lavazh salin i gjithe pulmonit

44. Nje burre 68 vjecar prezantohet per vleresim te dispese ne effort. Ai fillimisht i vuri re
simptomat perreth 2 vjet me pare. Ne ate kohe atij iu desh ta ndalonte te ecuren ne kursin e
golfit dhe filloi te perdorte nje karroce nga ato qe perdoren e fushat e golfit (cart), por ai ishte
akoma i afte te perfudonte nje seksion 18 te plote. Gjate vitit te fundit, ai e ka ndaluar golfin
plotesisht per shkak te dispense dhe gjendjes se tij qe ka veshtiresi te eci edhe deri te kutia
postare, qe eshte perreth 50 yards nga shtepia e tij. Ai gjithashtu ka kolle te thate shumicen e
diteve. Nuk perkeqesohet naten dhe ai nuk identifikion kercitje (triggers). Mohon wheezing.
Ai nuk ka pasur temperature, ftohje ose humbje ne peshe. Ai mohon te kete pasur simptoma
ne artikulacione. Ai eshte ish duhanpires me perreth (50 pack-years) njesi matese e sasise se
duhanit, por e la duhanin para 8 vitesh pasi u diagnostikua me semundje te arterieve
koronare. Ne ekzaminimin fizik, ai duket qe i mbahet fryma pasi eci ne korridor per ne
dhomen e ekzaminimit, por e mori veten shpejte duke pushuar. Shenjat vitale jane: TA=
118/67 mmHg, FK=88 rrahje/ min, shkalla respiratore 20 frymemarrje/ min. SaO 2= 94% ne
qetesi dhe ulet ne 86% pasi ka levizur 300 ft. Ekzaminimi i pulmonit tregon perkusion dhe
ekspansion normal. Atje ka Velcro- like crackles ne te dy bazat, dhe ato jane shperndare
pergjysme pergjate te dyjave fushave pulmonare. Nuk verehet wheeing. Ekzaminimi
kardiovaskular eshte normal. Deformimi i gishtit (digital clubbig) eshte present. Nje CT
kraharori tregohet ne Fig. VI-44. Ai referohet per biopsy kirurgjikale te pulmoneve. Cili
pershkrim patologjik ka me shume te ngjare te shikohet ne semundjen e ketij pacienti?

1. Fluid dens amorf brenda alveolave difuze qe qendron positive me ngjyrosje periodike
acid-Schiff
2. Shkaterrim i alveolave me hapesira emfizematoze, predominon ne lobet e siperme
3. Demtimdifuzalveolar.
4. Formimi i granulomave jo nekrotizante
5. Depozitimi heterogjen i kolagjenit me fibroblast foci and honeycombing

45. Te gjitha t jane manifestime pulmonare te Lupusit eritematoz sistemik pervec:


1. Nodule te kavitetit pulmonar
2. Disfunksion diafragmatik me humbje te volumit pulmonar
3. Pleuritis
4. Hemorragji pulmonare
5. Semundjepulmonarevaskulare

104
46. Nje grua 56 vjecare paraqitet per vleresim te dispese dhe kolle per 2 muaj. Gjate kesaj
kohe aj gjithashtu ka pasur luhatje temperature, discomfort, dhe humbje peshe 5.5 kg. Ajo
mohon te kete patur kontakt me te semure dhe nuk ka udhetuar kohet e fundit. Ajo punon si
infermiere, dhe nje test i pervitshem PPD i bere 3 muaj me pare doli negative. Ajo mohon te
jete ekspozuar ndaj mbetjeve orgainike dhe nuk ka zogj as kafshe shtepiake. Ajo ka histori te
artritit reumatoid dhe tani po merr hiydroxychloroquine, 200 mg dy here ne dite. Nuk ka
pasur perkeqsim te simptomave artikulare. Ne ekzaminimin fizik degjohen rale krepitante dhe
fishkellima pulmonare difuze. Nje CT scan i kraharorit shfaq infiltrime te vogla alveolare dhe
mur brokial dense. Testimi i funksionit pulmonar tregon restriksion te lehte. Ajo i nenshtrohet
nje biopsie kirurgjikale te pulmonit. Patologjia tregon indin granular qe mbush rruget e vogla
ajrore, duktusin alveolar, dhe alveolat. Intersticiumi alveolar ka inflamacion kronik dhe
pneumoni te organizuar. Cila eshte terapia me e pershtatshme per kete paciente?
1. Azathioprine 100 mg ne dite
2. Nderprerja e hydroxychloroquine dhe observim
3. Infliximab IV nje here ne muaj
4. Methotrexate 15 mg cdo jave
5. Prednisone 1 mg/kg cdo dite
47. Ne cilin nga pacietet te prezantuar me dispne acute, do te shtoje nje test pozitiv D-dimer
per nje emboli pulmonare?
1. Nje grua 24 vjecare qe eshte 32 javeshe shtatzene
2. Nje burre 48 vjec pa histori mjekesore, i cili shfaq dhimbje te pjeses se pasme te
kembes (calf pain) nga udhetimi i zgjatur ajror. Gradienti i oksigjenit alveolar-
arterial eshte normal
3. Nje grua 56 vjece nen kimiterapi per kancer gjiri
4. Nje burre 62 vjec qe ka kryer kirurgji per zevendesim te pelvisit (hip) 4 jave me pare
5. Nje burre 72 vjecar i cili pati nje infarkt akut miokardi 2 jave me pare
48. Nje grua 62 vjece eshte hospitalizuar per te ndjekur nje emboli pulmonare acute. Te gjitha
t tipikisht do te indikojne nje emboli masive pulmonare pervec:
1. Nivele te ngritura te troponines ne serum
2. Shfaqje fillestare me hemoptizi
3. Shfaqje fillestare me sinkop
4. Prani e zmadhimit te ventrikulit te djathte ne CT scan te kraharorit
5. Prani e hipokinezise se ventrikulit te djathte ne echocardiogram
VI-49: Cili nga pohimet rreth diagnostikimit imazherik të embolisë pulmonare i VËRTETË?
A. Një skaner ventilim-perfuzion me probabilitet të lartë është një skaner që ka të paktën një
defekt perfuzioni në një ventilim normal
C. Angio-RM siguron rezolucion të shkëlqyer si për embolinë pulmonare segmentare të vogël
ashtu edhe për atë proksimale të madhe
D. CT spirale me multidedektorë në rradhë (kam përshtypjen që e ka për HRCT!) eshtë
suboptimale për dedektimin e embolive të vogla periferike, gjë që e bën të nevojshme
përdorimin e angiografisë invazive pulmonare
E. Asnjë nga teknikat rutinë imazherike nuk siguron vlerësim adekuat të ventrikulit të djathtë
për të ndihmuar në stadifikimin e riskut të pacientit
B. Nëse pacienti ka një skaner me probabilitet të lartë, ka probabilitet 90%
që pacienti ka emboli pulmonare
1. Vazhdimi me të njëjtin menaxhim
2. Vazhdimi i fluideve IV me 500mL/orë, në një total prej 4L për
shpëtimin(rivitalizimin) me fluide
3. Referimi për vendosjen e një filtri në venën cava inferior dhe vazhdimi i
menaxhimit të tanishëm D. Referimi për embolektomi kirurgjikale

105
 VI-51: Një grua 42 vjeçe vjen në urgjencë me marrje frymë të shfaqura në mënyrë të
mënjëhershme. Së fundmi ajo ka vizituar prindërit e saj jashtë shtetit dhe ka udhëtuar me
makinë për 9 orë vajtje dhe po aq edhe ardhje. Para dy ditësh, asaj iu shfaq një dhimbje e
lehtë në pulpë dhe edemë, por ajo mendoi se ishte normale për shkak të qëndrimit ulur gjatë
udhëtimit. Në ardhjen e saj në ER, u vu re që ajo ishte takipneike. Shenjat vitale:
PA=98/60mmHg, FK=114 rrahje/min, FR=28/min, satO2=92%, peshë 89 kg. Pulmonet janë
të pastra bilateralisht. Ka dhimbje në pulpën e majtë me dorsifleksion të këmbës dhe gjithë
këmba e djathtë është më e fryrë krahasur me të majtën. Një matje e gazeve arteriale në gjak
tregon: pH=7.52, PCO2=25mmHg dhe PO2=68mmHg. Funksioni renal dhe hepatik janë
normal. Një CT-scan konfirmon embolin pulmonar. Të gjithë agjentët e mëposhtëm mund të
përdoren veçmas për terapi fillestare në këtë pacient, PËRVEÇ:
A. Enoxaparin 1mg/kg SC 2 herë në ditë
E. Trajtimi me dopaminë dhe aktivizues rekombinantë të plasminogjenit indor (rTPA:
Alteplase, reteplase, tenecteplase) 100mg IV
2. Fondaparinux7.5mgSCnjëherënëditë
3. Tinzaparin 175U/kg SC një herë në ditë
4. Heparin e pafraksionuar IV për të mbajtur aPTT(kohën parciale të tromboplastinës
aktive), 2-3 herë mbi kufirin normal të saj
5. Warfarin 7.5mg po një herë në ditë për të mbajtur INR në vlerat 2-3

 VI-52: Një grua 62 vjeçe është pranuar në spital për pneumoni komuniteti me histori
4ditore të temperaturës, kollës dhe dhimbjes së kraharorit tip pleuritike në anën e djathtë.
Radiografia e kraharorit në pranim identifikon infiltrat në lobin e poshtëm dhe të mesëm
djathtas me një efuzion. Të gjitha karakteristikat e mëposhtme të efuzionit pleural indikojnë
për një efuzion të komplikuar që mund të kërkojë tub trakeostomie, PËRVEÇ:
1. Lokulimi i fluidit (loculated fluid/ i vendosur si në xhep/ i kyçur)
2. pH i fluidit pleural më pak se 7.20
3. Glukoza e fluidit më pak se 60mg/dL
4. Ngjyrim grami ose kulturë pozitive e fluidit
5. Rekurencë e fluidit pas torakocentezës fillestare

 VI-53: Një burrë 58 vjeç është vlerësuar për dispnenë dhe i është gjetur një efuzion
pleural i moderuar në anën e djathtë. Ai i nënshtrohet torakocentezës me këto karakteristika:
Parametri Vlera
pH 7.48
Proteina 5.8 g/dL (proteina e serumit 7.2g/dL
LDH 285 IU/L (LDH e serumit 320 IU/L)
Glukoza 66 mg/dL
Leukocitet (ËBC) 3800/mm3
Eritrocitet (RBC 24’000/mm3
PMNs 10%
Limfocitet 80%
Qelizat mezoteliale 10%
Citologjia Limfocitozë me inflamacion kronik dhe pa qeliza malinje të identifikuar
Cila nga të mëposhtëmet është një shkak jo i mundshëm për efuzioni pleural në këtë pacient?
1. Cirroza
2. Kanceri pulmonar 3.Mezotelioma 4.Embolia pulmonare 5.Tuberkulozi

106
 VI-54: Një grua 66 vjeçe është vlerësuar për dispnenë. Një muaj më parë, ajo i është
nënshtruar një ezofagotomie për adenokarcinomë të ezofagut. Në ekzaminimin fizik pacientja
paraqitet takipneike me vështirësi në të folurin me fjali të plota. Ajo ka FR=28/min dhe një
saturim prej 88%. Në perkusionin e kraharorit zhurma dëgjohen zhurma jointense, jo të
qarta(dullness) me zhurma munguese frymëmarrjeje në hemitoraksin e majtë. Një radiografi
pulmoni konfirmon një efuzion pleural të majtë me zhvendoje mediastinale nga e djathta. Një
torakocentezë heq 1.5L likid me pamje si qumësht. Proteina e fluidit 6.2mg/dL, LDH është
368 IU/L, dhe leukocitet 1500/μL (20% PMN, 80% limfocite). Trigliceridet 168mg/dL.
Kultura dhe citologjia janë negative. Cila është menaxhimi më i mirë për këtë paciente?
1. Vendosja e një dreni plus octreotide
2. Vendosja e drenit me aspirim (chest tube to wall suction) derisa drenazhi të ulet në më
pak se 100mL në ditë
3. Rieksplorimi i kraharorit me korrigjim kirugjikal të defektit të mundshëm
4. Referimi për kujdes paliativ
5. Përsëritja e torakocentezës për ekzaminim citologjik

 VI-55: Një burrë 28 vjeç paraqitet në urgjencë me marrje fryme që ka nisur menjëherë
dhe dhimbje kraharori pleuritike që filloi para 2 orësh. Ai është në përgjithësi i shëndetshëm
dhe nuk ka histori mjekësore. Ai ka tymosur 1 paketë cigaresh përditë që prej moshës
18vjeçare. Në ekzaminimin fizik, ai është i gjatë dhe i dobët me nje indeks të masës trupore
19.2kg/m2. Ai ka FR=24/min me saturim O2 95%. Ka zhurma respiratore lehtësisht të ulura
në apeksin e pulmonit të djathtë. Një radiografi e pulmonit demonstron një pneumotoraks
20% në anën e djathtë. Cila është E VËRTËTË lidhur me pneumotoraksin në këtë pacient?
A. Nje CT-scan ka më shumë gjasa të tregojë ndryshimet emfizematoze.
C. Shumica e pacientëve kërkojnë tub torakostomie për të zgjidhur pneumotoraksin.
4. Probabiliteti për pneumotoraks rekurent është gati 25%.
5. Një faktor risku primar për zhvillimin e pneumotoraksit spontan është një
habitus(konstrukt trupor) i gjatë dhe i dobët.

 VI-56: Shkaku më i zakonshëm i efuzioneve pleurale është:


1. Cirroza
2. Insuficienca e VM
3. Malinjiteti
4. Pneumonia
5. Emboliapulmonare
-57-Nje pacient me Skleroze amiotrofike laterale te bute ndiqet nga nje pneumolog per
disfunksion respirator te lidhur me semundjen e tij neuromuskulare.
Cila nga simptomat e qe i shtohet PaCO2 prej 45 mmHg ose me teper do e bente te
nevojshme terapine e ventilimit me presion pozitiv per hipoventilimin ?
1. Ortopnea
2. Cilesia e keqe e gjumit Pergjigja e sakte:E
3. Kolla e dobet
4. Dispnea ne aktivitetet e perditshme
5. Te gjitha me siper
-58-Nje djale 27 vjecar me distrofi muskulare diagnostikohet me hipoksemi. Ndihet mire dhe
nuk ka dispne, pulsoksimetria e gishtit 86% ne ajrin e dhomes, pulmonet te qarta dhe
pavaresisht distrofise muskulare normal. Radiografia torakale tregon volum te ulet pulmonar.
Burimi me i mundshem i saturimit te tij te ulet me oksigjen?
1. Atelektaza 2.Tapat mukotike 3.PaCO2 i rritur
2. Pneumonia 5.Methemoglobinemia

107
-59-Pacienti me crregullime te hipoventilimit kronik shpesh ankon per dhimbje koke pas
zgjimit. Cili eshte shkaku i ketij simptomi?
1. Zgjimet nga gjumi
2. Vazodilatacioni cerebral
3. Vazokonstriksionicerebral
4. Policitemia
5. Mikroaspirimet gjate nates(nokturne) dhe kolla
-60- Nje grua 47 vjecare me hipertension arterial pulmonar idiopatik nuk ka qene e
suksesshme ne terapine mjekesore perfshire ketu edhe Epoprostenol IV. Ajo ka insuficience
kardiake te djathte te avancuar me disfunksion sever ventrikulit te djathte ne ekokardiografi
dhe me nje Index kardiak prej 1.7L/min per m2. Pacientja eshte referuar per transplant
pulmonar. Cila nga pohimet eshte e vertete?
1. Asaj do t’i nevojitet transplanti zemer-mushkeri per Insuficiencen e kardiake te
djathte te avancuar.
2. Pacientet me hipertension arterial pulmonar idiopatik kane nje mbijetese 5 vjevare me
te keqe se marresit e tjere te transplantit.
3. Transplanti vetem i mushkerive eshte procedura e preferuar kirurgjikale per
hipertensionin arterial pulmonary idiopatik.
4. Funksioni ventricular i djathte do te rekuperohet pas transplantit pulmonar.
5. Ajo eshte ne risk per hipertension pulmonar rekurrent pas transplantit pulmonar.

-61-Nje grua 25 vjecare me Fibroze Cistike referohet per transplant pulmoni. Ajo eshte e
shqetesuar per rezultatet afatgjate. Pengesa me e madhe per mbijetes afatgjat pas transplantit
1. Sindroma e Bronkiolitit Obliterant
2. Infeksioni nga CMV
3. Semundja renale kronike
4. Disfunksioni i graftit primar
5. Crregullimi limfoproliferativ pas transplantit
-62- Nje burre 30 vjecar me Fibroze Cistike te fazes se fundit i nenshtrohet transplantit
pulmonary. 3 vite me vone, ai peson nje renie progresive ne funksionin renal.
Cila nga medikamentet me poshte eshte etiologjia e mundshme e situates?
1. Prednisone
2. Tacrolimus
3. Albuterol
4. Mycofenolate mofetili
5. Asnje nga me lart
-63-Nje 22 vjecar vuan nga Fibroza Cistike. Ai eshte hospitalizuar rreth 3 here ne vit per
ekzacerbime infektive. Eshte i kolonizuar nga Pseudomonas Aeruginosa dhe Staphylococcus
Aureus , por kurre nuk e ka patur kompleksin e Burkholderia Cepacia. Pts mbetet aktiv dhe
eshte ne kolegj ku studion Arkitekture. Ai kerkon 2 L oksigjen gjate efortit. Testet me te
fundit te funksionit pulmonar demonstrojne nje FEV1 qe eshte 28% e vleres se parashikuar
dhe nje raport FEV1/FVC 44%. Matja e presionit te tij ne gjakun arterial tregon per pH=7.38,
PCO2 36 mmHg, PO2 62 mm Hg. Cila nga karakteristikat eshte nje tregues per ta referuar
pacientin per transplant pulmonar?
1. Kolonizimi me Pseudomonas Aeruginosa.
2. FEV1 me i vogel se 30% e vleres se parashikuar.
3. FEV1/FVC <50%
4. PCO2< 40mmHg
5. Perdorimi i O2 gjate efortit

108
NEFROLOGJI
1. Cila nga alternativat eshte nje faktor etiologjik potencial per insuficencen renale akute?
A. Apoptoza dhe nekroza e qelizave tubulare
B. Ulja e vazodilatacionit glomerular ne pergjigje te oksidit nitrik
C. Rritja e vazokonstriksionit glomerular ne pergjigje te rritjes se nivelit te endotelinave
D. Rritja e adezionit leukocitar brenda glomerulave
E. Te gjitha me siper

2. Te gjitha jane faktore risku per demtim akut postoperativ te veshkave pervec?
A. Kirurgjia kardiake me bypass kardiopulmonar
B. Diabeti mellitus
C. Seksi femer
D. Hipotensioni intraoperative
E. Humbje operative sinjifikante e gjakut

3. Nje burre 57 vjec me histori te diabetit mellitus dhe semundje kronike te veshkave me kreatinine
1.8 mg/dL, iu nenshtrua kateterizimit kardiak per infarkt akut miokardi. Me pas ai eshte diagnostikuar
me demtim renal akut lidhur me kontrastin e jodizuar. Te gjithe theniet e jane te verteta ne lidhje
me demtimin renal akut ne kete rast pervec?
A. Fraksioni i ekskretimit te Na do jete I ulur
B. Kreatinina ka presispozite te arrij kulmin brenda 3-5 diteve
C. Diabeti mellitus e predispozoi te zhvillonte nefropati nga kontrasti
D. Obstruksioni kalimtar i tubujve me kontrast te jodizuar te precipituar kontriboi ne zhvillimin e
demtimit akut renal
E. Kastet e qelizave te bardha te gjakut ka gjasa te gjenden ne ekzaminimin mikroskopik te
sedimentit urinar

4. Cili nga pacientet me demtim renal akut ka me shume gjasa te kete evidence per hidronefrosis
bilaterale ne ekzaminimin me eko te veshkave?
A. Nje djale 19 vjec me purpura fulminans shoqeruar me sepsis gonococcal
B. Nje grua 37 vjec qe ka bere kimioterapi dhe radioterapi per kancer cerviksi te avancuar
C. Nje burre 48 vjec me insuficence renale kronike ne saje te hipertensionit qe merr kontrast te
jodizuar per angiogramen abdominale
D. Nje burre 53 vjec me Escherichia coli shoqeruar me purpura trombocitopenike
E. Nje grua 85 vjec e shtruar me pielonefritis dhe sepsis

5. Ne nje vleresim te demtimit renal akut ne nje pacient qe i eshte nenshtruar se fundi bypassit
kardiopulmonar pergjate zevendesimit te valvules mitrale, cila nga gjetjet e te urines sugjeron me
teper per embol nga kolesteroli si burim te insuficences renale?
A. Kristalet e kalcium oksalatit
B. Eozinofiluria
C. Kastet granulare
D. Sediment normale
E. Katet e qelizave te bardha te gjakut

6. Nje burre 54 vjecar eshte shtruar ne kujdesin intensiv mjekesor me sepsis shoqeruar me pneumoni
pneumokoksale. Ai eshte nen ventilin mekanik dhe norepinefrike per te mbajtur nje presion arterial
mesatar >60 mmHg. Hemodinamika invazive tregon presione te mbushjes adekuade te zemres se
majte dhe ai nuk njihet per disfunksion te ventrikulit te majte. Ne diten e trete te shtrimit kreatinina
e tij rritet ne 3.4 mg/dL. Eshte diagnostikuar demtim tubular akut. Cili nga agjentet e meposhtem
permireson rezultatet shoqeruar me demtimin tubular akut?

109
A. Furosemidi
B. Bosentani
C. Dozat e uleta te dopamines
D. Faktori i rritjes insulin like
E. Asnje nga te mesipermet

7. Nje pacient 65 vjecar ka 5 dite i shtruar ne spital me azotemi prerenale sekondare nga dehidratimi.
Kreatinina e tij ishte 3.6 mg/dL ne shtrim, por eshte rritur ne 2.1 sot. Ai ankon ne dhimbje te lehte te
fundshpines dhe ju i pershkruani naproksen menjehere. Me cfare mekanizmi mundet ky medikament
ta demtoje me vone veshken?
A. Vazokonstriksion arteriolar aferent
B. Vazodilatacion arteriolar afferent
C. Vazokonstriksion arteriolar eferent
D. Toksicitet tubular proximal
E. Obstruksion ureteral

8. Cila nga biomarkerat e demtimit te indit parashikon fillimin e demtimit akut renal pas nje ngjarje
hipotensive apo ishemike?
A. Nitrogjeni i urese se gjakut (BUN)
B. Interleukina-1 8
C. Molekula e demtimit te veshkave 1 (KIM-1)
D. Neutrofil xhelatina qe shoqeron lipokalinen (NGAL)
E. Asnje nga te mesipermet

9. Cili nga pacientet ka riskun me te madh per progresion drejt semundjes renale kronike?
A. Nje mashkull 30 vjec me rritje te shpejtesise se filtrimit glomerular (GFR) ne 50 mL/min/1,73m2
dhe albuminuri persistente 350mg/g
B. Nje burre 45 vjec me rritje te GFR ne 90 mL/min/1.73m2 dhe albuminuri persistente prej <30mg/g
C. Nje burre 55 vjecar me rritje te GFR ne 65mL/min/1.73 m2 dhe albuminuri persistente <30mg/g
D. Nje burre 75 vjec me rritje te GFR ne 35mL/min/1.73m2 dhe albuminuri persistente <30mg/g

10. Ne stadin e V te semundjes renale kronike, GFR eshte nen?


A. 90 mL/min/ 1.73 m2
B. 60
C. 25
D. 15
E. 0 (anuria)

11. Cili eshte shkaku kryesor i vdekjes ne pacientet me semundje renale kronike?
A. Semundja kardiovaskulare
B. Hiperkalemia
C. Infeksioni
D. Malinjiteti
E. Uremia

12. Te gjitha theniet lidhur me perdorimin e eritropoetines ekzogjene ne pacientet me semundje


renale kronike jane te verteta me perjashtim te?
A. Eritropoetina ekzogjene administrohet me target te perqendrimit te hemoglobines 100-115 g/L
B. Perdorimi I EPO ekzogjene shoqerohet me permiresimin e rezultateve kardiovaskulare
C. Perdorimi i EPO ekzogjene shoqerohet me rritje te riskut per stroke ne pacientet me DM 2
bashkeshoqerues

110
D. Perdorimi i EPO ekzogjene mundet te shoqerohet me progres me te shpejte dhe nevoje per dialize
E. Perdorimi i EPO ekzogjene shoqerohet me rritje te incidences te eventeve tromboembolike

13. Nje grua 63 vjecare me semundje renale kronike mbahet ne nje dialize ditore peritoneale. Historia
e saj e shkuar mjekesore permban hipertensionin dhe fibrilacionin atrial. Ajo merr losartan dhe
varfarine. 2 dite me heret ajo ndjeu nje nodul te vogel te dhimbshem ne abdomen qe ka progresuar
ne nekroze te lekures dhe ulceracion te murit abdominal. Te gjitha theniet e meposhte ne lidhje me
gjendjen e saj jane te sakta me perjashtim te
A. Suplementi i kalciumit oral mund te jete nje factor risku
B. Patologjikisht, aty eshte nje okluzion vascular
C. Bashke-infeksioni me pseudomonas eshte tipik
D. Hiperparatiroidizem i rende mund te mos jete prezent
E. Warfarina eshte nje faktor risku per zhvillimin e lezionit

14. Nje pacient po ndiqet nga nefrologu i saj per semundje renale kronike stadi IV shoqeruar me
glomeruloskleroze segmentale lokale. Cila eshte indikim per fillimin e hemodializes?
A. Acidoza e kontrolluar me administrimin e bikarbonatit ditor
B. Diateza hemorragjike
C. BUN> 110 mg/dL pa simptoma
D. Kreatinina >5mg/dL pa simptoma
E. Hiperkalemia e kontrolluar me polistiren Na

15. Nje grua 27 vjecare me semundje kronike renali po I nenshtrohet hemodializes dhe gjate trajtimit
zbulohet se eshte hipotensive. Mekanizmi I mundshem per hipotensionin gjate hemodializes
A. Agjentet antihipertensive
B. Ultrafiltrimi I shumte
C. Pergjigje e demtuar autoonomike
D. Shift osmolar
E. Te gjitha alternativat

16. Nje grua 35 vjec me semundje renale hypertensive progreson drejt semundjes renale te stadit te
fundit. 1 vit me pare ajo filloi dializen peritoneale dhe qe atehere 111 aka dale te kontrolloje siptomat
uremike. Ajo vjen ne urgjence me ethe, status te alteruar mental, dhimbje difuze abdominale dhe
dializant te turbullt. Likidi peritoneal I hiqet npm kateterit dhe cohet ne laborator per analiza.
Leulkocitet ne likid jane ne nivelin125/mm3 me 85% polimorfonukleare neutrofile. Cili nga
mikroorganizmat mund te gjendet me shume mundesi ne likidin peritoneal?
A. C. Albicans
B. E. Coli
C. M. Tuberculosis
D. P. Aeruginosa
E. S. Aureus

17. Nje grua 45 vjec fillon hemodializen per semundje renale te stadit te fundit te shoqeruar me diabet
mellitus. Cili eshte shkaku me I mundshem I vdekjes?
A. Demenca
B. Episode hemoragjik I rende
C. Infarkt miokardial
D. Uremi progresive
E. Sepsis

111
18. “Doza” e dializes perkufizohet si
A. Ritmi I rrjedhjes kunder rryme e dializantit c.Klirensi I fraksionuar I urese
B. Oret ne jave te dializes d.Numri mujor I sesioneve te plotesuara

19. Pacienti juaj me semundje renale te stadit te fundit dhe qe ben dialize ka hiperkalemi persistente.
Ai ka nje anamneze positive per stenoze te arterieve renale bilateral dhe kjo eshte shkaku qe ai ben
hemodialize. Kan ndryshime ne elektrokardiograme vetem kur kaliumi eshte mbi nivelin 6.0 meq/L gje
qe I ndodh pak here ne jave.ti e shtron ne spital per vleresim te metejshem. Vleresimi laboratorik
keshillimi nutricional dhe rregullimi I mjekimit nuk kane ndikuar mbi nivelin e kaliumit ne serum. Cili
eshte hapi I rradhes qe duhet te mare ky pacient?
A. Rregullo dializatin D.Jepi doze ditore te furosemidit
B. Bej“sodium modeling.” E.Implanto nje defibrillator automatik
C. Bej nefrektomi bilaterale
20. Cila nga pohimet e eshte e vertete per transplantin e veshkave?
A. Ritmet 5 vjecare te mbijeteses se maresit te transplantit jane te njejta si kur dhenesi eshte I gjalle
ashtu edhe kur ai eshte I vdekur
B. Mosha e dhuruesit te vdekur nuk ndikon ne mbijetesen e grafti
C. Transplanti renal nuk ofron ndonje perfitim ne kosto ne krahasim me hemodializen
D. Kur dhuruesit jane familjare te linjes se pare, mbijetesa e graftit per nje periudhe 1 vjecare eshte
5-7 5 me e madhe se kur dhuruesi nuk jeton
E. Kur niqet per me shume se 20 vjet jane shume te shpeshta komplikacionet ne veshken unike.

21. Te gjitha alternativat me poshte konsiderohen si kritere per dhurim per transplantin renal, pervec?
A. Dhurues i vdekur >60 vjec
B. Dhurues i vdekur >50 vjec, hypertension dhe nivel kreatinine >1.5 mg/dL
C. Dhurues I vdekur >50 vjec, hypertension dhe vdekje e shkaktuar nga aksident cerebrovascular
D. Dhurues I vdekur >50 vjec, vdekje e shkaktuar nga AVC dhe kreatinina >1.5 mg/dL
E. Presence e antitrupave kunder dhuruesit veshkes te marresi ne kohen transplantit parashikuar

22. Jane shkaktare te demtimit glomerular qe con ne insuficence renale pervecse:


A. Diabeti mellitus c. Nefriti lupik d.Hipertensioni malinj
B. Sindroma Fanconi e.Mutacioi i TRPC6
23. Nje djale 21 vjecar diagnostikohet me glomerulonefrit post streptokoksik. Cila nga alternativat e
ka me shume mundesi te gjendet ne urinen e tij?
A. Proteinuri me e madhe se 3 g/24-ore pa hematuri
B. Hematuri makroskopike dhe albumina 24 oreshe ne nivelin 227mg
C. Hematuri mikroskopike me leukocyte dhe albumin 24 oreshe ne nivelin 227mg
D. Urokulture positive per Streptococcus
E. Piuri sterile pa proteinuri
24. Gjendja e nje femre 50 vjecare obeze me nje histori 5 vjecare per hypertension te kontrolluar me
diuretike tiazidike po vleresohet pasi ne kontrollin e saj vjetor eshte ve re prania e proteinurise.
Ekzaminimi fizik tregon per nje gjatesi 167.6 cm, peshe 91 kg, tension arterial 130/80 mmHg dhe
edeme ne kembe. Vlerat e laboratorit jane si me poshte: Serum creatinine: 106 μmol/L (1.2 mg/dL)
BUN: 6.4 mmol/L (18 mg/dL) Creatinine clearance: 87 mL/min Urine protein excretion: 5.9 g/d
Urinalysis: pH 5.0; specific gravity 1.018; protein 3+; no glucose; occasional coarse granular cast
Ne biopsi renale glomerulet kane trashje ne mikroskopin optic dhe pjesa tjeter duket e pa prekur.

112
Diagnoza me e mundshme eshte:
A. Nefroskleroze hypertensive
B. Skleroze fokale dhe segmentare
C. Semundja me ndryshine minimale(Minimal-change (nil) disease)
D. Glomerulopatia membranoze
E. Glomenulonefriti kreshent

25. Te gjitha theniet ne lidhje me nefropatine dabetike jane te verteta me perjashtim te:
A. Afersisht 40% e pacienteve me DM tip 1 ose 2 do te zhvillojne nefropati
B. Nefropatia diabetike eshte shkaku i vetem me i zakonshem ne insuficencen renale kronike ne USA
C. Ne pacientet me DM1, kontrolli i perqendrimit te glukozes ne gjak ndikon zhvillimin dhe
progresionin e nefropatise
D. Ndryshimet patologjike jane predominuese ne tubulin distal dhe ansen Henle
E. Indikatori me I hershem klinik i detektueshem i nefropatise diabetike eshte albuminuria

26. Cila manifestim ekstrarenal I semundjes renale policistike autozomale dominante?


A. Regurgitacioni I aortes
B. Dilatacioni I bazes se aortes
C. Divertikuli I kolonit
D. Aneurizem intrakraniale
E. Te gjjitha te mesipermet

27. Nje student 21 vjecar vleresohet per lodhje te madhe e cila ka qene prezente per disa vjet me
rradhe por kohet e fundit po shtohet. Ai gjithashtu referon per disa spazma dhe krampe te kembes
dhe ndonjehere kontraksione te pakontrolluara te muskujve qe . Ai eshte I shendetshem, nuk mer
mjekime dhe mohon perdorimin e duhanit dhe alkoolit. Ne egzaminim ai tregon se ka zhvillim normal
me shenja normale vitale perfshire ketu dhe tensionin arterial. Analizat tregojne per nje nivel te
Na=138meq/L, kalium=2.8meq/L, klori=90meq/Ldhe bikarbonatet 30mmol/L. niveli I magnezit eshte
normal. Kontrolli I urines per diuretike eshte negative dhe klori ne urine eshte I rritur. Cila eshte
diagnoza me e mundshme?
A. Bulimia nervore
B. Abuzimi me diuretike
C. Sindromi Gitelman
D. Sindromi Liddle
E. Pseudohipoaldosteronizem tipi 1
28. Nje grua 28 vjecare eshte diagnostikuar me semundjen policistike te veshkave (autozomale
dominante) pas nje episodi me hematuri. Ajo eshte e shqetesuar per rriskun per aneurizem
intrakraniale. Cila nga fjalite e eshte e vertete persa I perket ketij rrisku?
A. Historia familjare per rupture te aneurizmes intrakraniale nuk errit rriskun per rupture
B. Hemorragjia e meparshme intrakraniale nuk e rrit rriskun per hemoragji tjeter pasuese

113
C. Masa e aneurizmes nuk ka lidhje me rrezikun per rupture spontane
D. Nuk ka rrezik te rritur per aneurizem intracraniale ne kete gjendje
E. Hipertensioni I pakontrolluar rrit mundesine per rupture spontane.
29. Nje pacient me histori te sindromit Sjogren ka analizat e : natriumi ne plazme=139 meq/L, klori=
112 meq/L, bicarbonate= 15 meq/L, dhe kaliumi= 3.0 meq/L; analiza e urines tregon nje pH
6.0,natriumi 15 meq/L, kaliumi10 meq/L, and klori 12 meq/L. Diagnoza me e mundshme eshte:
A. Acidoze tubulare renale tipi 1 (RTA)
B. Tipi 2 RTA
C. Tipi 3 RTA
D. Tipi 4 RTA
E. Diarre kronike

30. Nje vajze 16 vjecare gjimnaste vjen per konsulte pasi ankon per lodhje, dobesi te gjeneralizuar dhe
krampe muskulare. Nuk ka histori te meparshme mjekesore dhe mohon perdorimin e duhanit, alkoolit
dhe drogerave te tjera. Ne anamnezen familjare nuk ka ndonje te dhene sinjifikative. Ekzaminimi
tregon nje vajze te dobet me tension arterial normal. BMI eshte 18 kg/m2. Ne ekzaminim dentar
rezulton kujdes jo I mire. Tonusi muscular eshte normal dhe ekzaminimi neurologjik gjithashtu eshte
normal. Analizat tregojne nje hematokrit 38.5% kreatinint 0.6mg/dL bikarbonati ne serum 30meq/L
dhe kaliumi 2.7 meq/L. Vleresimi I metejshem duhet te perfshije:
A. Analize urine dhe urokulture
B. Nivelet e renines dhe aldosteronit ne plazme
C. Skrinim toksikologjik I urines per opiate
D. Skrinim toksikologjik I urines per diuretike
E. Nivelet e magnezit ne serum

31. Trajtimi me Gk per nefritin intersticial te pare ne biopsi me shume gjasa te jap sherim te veshkes?
A. Nje 37-vjecare me sarkoidoz
B. Nje 48 vjecar me nefrit intersticial me progres te ngadalte mbi 2 muaj me fibroze ne biopsi
C. Nje 54 vjecar me DM dhe infeksion salmonele
D. Nje 63 vjecar me nefrit alergjik pas perdorimit te cefalosporinave
E. Asnjera

32. Nje 58 vjecare ben histerektomi dhe pas operacionit zhvillon ARDS trajtohet me ventilim mekanik
dhe Ab me spekter te gjere, ka hipotiroidizem. Ne diten e 5 te shtrimit debiti urinar bie dhe
kreatininemia shkon nga 1.2-2.5 mg/dl. Dysho nefrit intersticial alergjik nga cefalosporik e konfirmon
A. Hematuri b.Eozinofili te gj periferik cEozinofile ne urine
B. Leukocite ne urine e.Asnjera

33. Nje burre 63 vjec eshte shtruar ne spital me insuficence renale akute. Ai ka nje histori diabeti qe
eshte kontrolluar mire per 10 vite me diete, ushtrime dhe metformine. 4 muajt e fundit, ai ankohet
per lodhje progresive, anoreksi, dhimbje kockore difuze. Ka marr menjehere ibuprofen per dhimbjen
e kockave dhe ka patur nje qetesim minimal. Ekzaminimi fizik i tij tregon per shenja vitale normale,
pulmone te pastra, ekzaminim kardiak dhe abdominal normal, diskomfort kockor difuz te bacinit,
kockave te gjata dhe shtylles kurrizore. Ekzaminimi laboratorik tregon BUN 68 mg/dL, kreatinine 5.8
mg/dL, Kaliumi 4.7 mEq/L, Kalciumi 13.4 mg/dL, hemoglobina 8g/dL, proteinat e serumit 9 g/dL,
albumina 2.8 g/dL. Funksioni renal ishte normal para 1 viti. Biopsi renale. Diagnoza e mundshme?
A. Nefropati diabetike
B. Glomeruloskleroza segmentale fokale
C. Sindroma Goodpasture
D. Nefropatia me kaste me zinxhire te lehte
E. Nefriti lupik

114
34. Nje obeze 44 vjecare ben kolecistektomi. Pas operacionit eshte mire. 2 dite pasi del nga spitali
zhvillon alterim mendor dhe temperature dhe sillet ne urgjence. Merr nje antidepresant por eshte e
shendetshme. T=103 F. Pulsi=127 TA=110/78 dhe SaO2 normal. Ne ekzaminim vihet re konfuzion.
Elektrolitet normale BUN=80 mg/dl, kreatinina=2,5, leukocite=17,3 mije/mikroliter, hematokriti 30%,
trombocite=25 mije/mikroliter. Ka shizocite ne gj periferik dhe trombocite te uleta. Si eshte gjendja ?
A. Aktivitet i ulet i metaloproteazez ADAMTS13
B. Plazma fereza s’ka gjasa te ndihmoj
C. Ndoshta ka ardhur nga nje infeksion E.Coli
D. Kjo gjendje eshte me e shpeshte ne burra se ne gra
E. Mortaliteti I patrajtuar eshte I ulet

35. Nje 35 vjecare ankon edema bilaterale te gjymtyreve , poliuri dhe dhimbje te flankut te majte qe
filluan papritur para 2 javesh. S’merr mjekim dhe nuk pi duhan, alkool dhe droge. Ne ekzaminim ka
edema 2+, urina 24 oreshe dhe ka 3,5 g proteine, kreatinina 0,7 mg/dl. Ne eko veshka e majte eshte
13 cm kurse e djathta 11,5 cm, dyshon per tromboze te venes renale. Cfare testi do te besh?
A. CT e venave renale C.Venografi me kontrast
B. Venografi MRI d.DPTA imaging e. Ekodopler I venave renale

36. Nje vajze 28 vjecare qe eshte ne javen e 30 te shtatzanise se dyte, ndiqet per arsye te hipertensionit
te lehte. Shtatzania e saj e pare u komplikuar nga preeklampsia. Ajo ankon per me shume lodhje ditet
e fundit. Presioni i gjakut 140/90 mmHg, FC 84 bpm dhe dat i O2 ne ajrin e dhomes 95%. Monitorimi
fetal nuk tregon distres. Studimet laboratorike tregonje nje Hb ne 6g/dL dhe trombocitet 180000/dL.
Strisho e gjakut periferik tregon per schistocites. Te gjitha jane te verteta pervec?
A. Glukokortikoidet jane efektive ne uljen e morbiditetit dhe mortalitetit
B. Enzimat e funksionit hepatik kane gjasa te rritetn
C. Preeklampsia predispozon kete gjendje
D. Insuficenca renale eshte e zakonshme
E. Gjendja do te zgjidhet pas lindjes se fetusit

37. Nje burre 48 vjec me DM, hiperlipidemi dhe fibrilacion atrial paraqitet ne urgjence per vleresimin
e dhimbjes se rende ne krahun e majte, prezente prej 3 oresh. Medikamentet e tij perfshijne
metforminen, atorvastatinen dhe varfarinen. Ka nje temperature prej 37C, FC 105 bpm, presionin e
gjakut 145/95 mmHg, FR 21 frymemarrje/min, sat O2 ne ajrin e dhomes 98%. Nuk ka organomegali
apo diskomfort lokal. Ne EKG tregon nje takikardi sinusi me ndryshime jo specifike te valeves ST-T.
funksioni renal eshte normal dhe analizat e urines tregojne shume qeliza te kuqe, pak te bardha, jo
baktere dhe jo kristale. Cili nga testet e meposhtem eshte me diagnostik per situaten?
A. Mbledhja e urines 24 oreshe b. Cistoskopia c.MRI d.CT e.Eko

38. CT abdominal. Cila eshte diagnoza e mundshme?

A. Apendicitis c. Karcinoma e qelizave renale


B. Nefrolitiasis d . Pielonefrit e. Hematoma retroperitoneale

115
39. Cili eshte lloji me i shpeshte i gureve te veshkave?
A. Kalciumi B.Cisteina C. Ac oxalik D. Struvik E. Acidi urik

40. Nje 54 vjecare me histori kancer koloni te trajtuar me rrezeksion 2 vjet me pare dhe me kimioterapi
pranohet ne spital pas analizave me BUN 65 dhe kreatinine 4,5 referon lodhje te lehte dhe dhimbje
kurrizi. Ajo merr AIJS ne dozen e duhur. Pervecse te ndaloj AIJS dhe te shmang nefrotoksina, Cfare
ekzaminimi duhet bere ne fillim?
A. CT I abdomeni/pelvis
B. Volumi rrezidual I vezikes
C. Urografi retrograde
D. Eko abdomeni/veshke
E. Fraksioni urinar I ekskretimit te Na

41. Nje 67 vjecar vjen ne urgjence me distendin abdominal dhe dhimbje. Ka vezik te palpueshme dhe
pas vendosjes se kateterit kalon 1,5 l urine. PSA normale, por referon veshtiresi ne kalimin e urines
duke cuar ne mosurinim dy dite. BUN=89 dhe kreatinina=6,4. Ne kater ditet ne spital BUN dhe Cr bien
por debiti urinar po rritet (6 l ne diten e 3 dhe te 4). Nuk merr likide IV. Shpjegimi rritjs debitit urinar?
A. Humbje cerebrale e kripes
B. Osmolaritet modular I ulet
C. Aktivizim I rritur I SRAA
D. Rritje e presionit tubular
E. Diureze postobstruktive.

42. Pacienti i mesiperm cfare komplikacioni ka rrezik?


A. Eritrocitoze B. Acidoze metab hiperCl C. HiperKalemi D.Azotemi pre renale E.HTA

43. Dhimbja e lidhur me obstruksion akut urinar eshte rezultat i?


A. Natriureze kompensatore
B. Presion I gjakut medular I uleti
C. Fluks gjaku veshkor i rritur
D. Pg vazodilatatore

44. Nje 28 vjecar nga Peru me dhimbje abd. Eko abd - hidronefroze e hidroureter bilateral. Ca ka?
A. Limfoma B. Stenoze meatal C. Fimozis D.Fibroze retroperitoneale

45. Cila nga alternativat eshte etiologjia e mundshme per insuficencen ishemike renale akute?
A. Apoptoza dhe nekroza e qelizave tubulare
B. Ulja e vazodilatacionit glomerular si pergjigje nga oksidit nitric
C. Rritja e vazokonstriksionit glomerular si pergjigje ndaj rritjes se nivelit te endotelines.
D. Rritje e adesionit lekukocitar ne glomerul
E. Te gjitha te mesipermet

46. Nje burre 42 vjec me histori te diabetit mellitus, hiperlipidemi, duhanpires, dhe me semundje te
arterieve koronare I nenshtrohet appendectomies urgjente. Cila nga gjendjet e e predispozon
pacientin per demtim postoperator te veshkes?
A. Procedura abdominale, kirugjia emergjente dhe hyperlipidemia.
B. Mosha me e madhe se 40, procedura abdominale dhe kirurgjia emergjente
C. Mosha me e madhe se 40,kirurgjia emergjente dhe diabeti meliturs
D. Semundja e arterieve korornare, duhanpirja,dhe procedura abdominale
E. Diabeti meliturs dhe nderhyrja emergjente E SAKTE

116
47. Vleresimi preoperator per nje pacient 55 vjecar I cili do beje angiografi te koronareve tregon per
nje rritje t GFR ne 33 ml/dL per1.73 m2 dhe diabet I pa kontrolluar. Aktualisht ai nuk mer ndonje
mjekim nefrotoksik dhe nefrologu ju siguron se ai nuk ka insuficience renale acute. Nderhyrja duhet
te filloje per 4 ore dhe ti do doje te parandaloje nefropatine nga kontrasti. Cila nga alternativat e do
te ule riskun per te zhvilluar nefropati prej kontrastit?
A. Dopamine
B. Fenoldopam
C. Indomethacin
D. N-acetylcysteine
E. Sodium bicarbonate

48. Nje pacient ndiqet nga nefrologu per semundje kronike renale ne stadin e 4 dhe qe shoqerohet
me glomeruloskleroze fokale segmentale. Cila indokon fillimin dhe mbajtjen e hemodializes?
A. Acidoza e kontrolluar me injektim ditor te bikarbonateve
B. Diateza hemoragjike
C. BUN me I madh se 110 mg/dL pa simptoma
D. Creati me I madh se 5 mg/dL pa simptoma
E. Hyperkalemia e kontrolluar me polistirene natriumi

49. Nje 58 vjecare ben histerektomi dhe pas operacionit zhvillon ARDS trajtohet me ventilim mekanik
dhe Ab me spekter te gjere, ka hipotiroidizem. Ne diten e 5 te shtrimit debiti urinar bie dhe
kreatininemia shkon nga 1.2-2.5 mg/dl. Dyshohet nefrit intersticial alergjik nga cefalosporinat. Cila
gjetje do konfirmoj diagnozen? A.Hematuri B.Eozinofili te gj periferik
A. Eozinofile ne urine D. Leukocite ne urine E. Asnjera

50. Nje grua 63 vjecare me histori te kancer cerviksi ka bere histerektomi dhe iradim pelvik. Tani
prezantohet me insuficence renale oligurike akute. Ne ekzaminimin fizik, ajo ka presion venoz jugular
normal, eshte normotensive, pa ortostaze dhe ka nje ekzaminim abdominal beninj. Analiza urologjike
tregojne nje gravitet normal prej 1.010, pa qeliza, pa kaste ne mikroskop. FE i Na urinar eshte 2% dhe
nivelet e Na jane 35 mEq/L. Cili eshte hapi me i mire i mevonshem?
A. Bolus me fluide intravenoze
B. Eko renale
C. CT i abdomenit me kontrast intravenoz
D. Administrim i furosemidit per te rritur output-in urinar

51. Nje burre 49 vjec me nje histori te gjate te insuficences renale kronike nga shkaku I nefropatise
diabetike ka ardhur ne urgjence me nauze, letargji dhe konfuzion. Ne ekzaminimin fizik ka nje presion
venoz jugular te rritur, fusha te pastra pulmonare dhe tinguj te rende sistik dhe diastolik mbi
prekordium. K ne serum eshte 5.1 mEq/L, CO2 eshte 17 mEq/L, BUN 145 mg/dL dhe kreatinina 9.8
mg/dL. Cila eshte terapia me e pershtatshme?
A. Administrimi IV i insulines dhe glukozes
B. Administrimi IV i bikarbonatit te Na
C. Administrimi IV i furosemideve
D. Hemodializa urgjente

52. Nje burre diabetik 62 vjecar iu nenshtrua riparimit te aneurizmes se aortes abdominale. Ka nisur
te trajtohet me gentamicine per nje infeksion te traktit urinar. Output-i urinar ka rene ne 300 mL ne
24 ore, dhe nivelet e kreatinines se serumit jane rritur nga 1.1 mg/dL ne shtrim ne 1.9 mg/dL. Cila nga
vlerat e laboratorit eshte me konsistente ne nje insuficence renale me etiologji prerenale?
A. FE Na 3% C. Presioni venoz central 10 mmHg
B. Nivelet e Na urinar 10 mEq/L D. Gentamicina 4mikrogram/mL

117
53. Nje djale 18 vjec, vrapues ne maratone, eshte trajnuar pergjate veres. U soll ne urgjence i
disorjentuar, pas kolapsit ne piste. Temperatura 38C. Ne kateterin e vendosur u ve re urine e kuqe, pa
qeliza te para mikroskopikisht. Cili eshte shpjegimi per urinen e tij?
A. Semundje renale
B. Azotemia prerenale
C. Mioglobinuria
D. Glomerulonefriti

54. Cila nga gjetjet laboratorike eshte me tipike ne glomerulonefritin poststreptokoksal?


A. Rritje e niveleve te komplementit ne serum
B. Titri pozitiv per antitrupa antinuklear
C. Rritje e tittrit ASO
D. Kulture pozitive e gjakut
E. Titer pozitiv i krioglobulines

55. Nje djale 22 vjec ankon per hemoptizi akute nga java e kaluar. Ai mohon se pi duhan ose per ndonje
semundje pulmonare. Presioni i gjakut eshte 130/70 mmHg dhe ekzaminimi fizik duket normal. Analiza
e tij e urine stregon per hematuri mikroskopike dhe caste te qelizave te kuqe te gjakut. Cila eshte
etiologjia e mundshme?
A. Metastaze e karcinomes renale ne pulmone
B. Tuberkuloz akut i veshkave dhe pulmoneve
C. Lupus eritematosus sistemik
D. Semundja Goodpasture ( Membrana bazale antiglomerulare)

56. Nje grua 49 vjec me diabet mellitus tip 2 prezantohet me edema ne kembe dhe fytyre qe nuk kane
qene me heret. Nuk ka histori tjeter mjekesore por raporton qe ne viziten e fundit te oftalmologu,
mjeku i kishte thene qe diabeti i ka prekur syte. Ajo mer gliburide cdo dite per diabetin. Ekzaminimi
fizik eshte normal me perjashtim te eksudateve te forta dhe hemorragjive me pika ne ekzaminimin e
fundus okuli. Presioni i gjakut eshte normal. Analiza e urine stregon 2+ proteine dhe 2+ per glukoze.
Cili eshte trajtimi me i mire?
A. Ti thuhet pacientit qe te rikthehet ne 6 jave dhe te kontrollohet urina ne ate periudhe
B. Fillo metoprololin
C. Ndrysho gliburidet ne glipizide dhe follw up per 6 jave
D. Fillo lisinoprilin
E. Refero pacientin ne nje kardiolog
57. Nje djale 19 vjec ankon per faringit dhe ndryshim ne ngjyren e urines. Presioni i gjakut 178/110
mmHg dhe ne analizen e urines dallohen kaste te RBC, RBC dismorfike dhe 1+ proteinuri. Diagnoza ?
A. Lupus eritematosus sistemik C. Glomerulonefrit poststreptokoksal
B. Amiloidosis D. Nefropati nga HIV E. Nefropatia diabetike

58. Cili eshte testi diagnostikues me i mire per nefropati diabetike te hershme?
A. Mikroalbuminuria e urines C.Biopsi renale D.Glukoza ne gjak esell
B. Strisho e analizes se urines E.Grumbullim i urines 24 oreshe per klirensin e kreatinines

59. Nje burre 58 vjec me DM tip II, normotensive, pa histori per semundje kardiake paraqitet me
kreatinine 1.8 mg/dL. Profili esell i lipideve tregon trigliceridet ne nivel 205 mg/dL, kolesterolin total
220 mg/dL, HDL 35mg/dL dhe LDL 148 mg/dL. Cili eshte trajtimi me i pershtatshem?
A. Niacin B. Dieta me proteina te uleta C.Gemfibrozil D.Simvastatin

118
REUMATOLOGJI DHE IMUNOLOGJI
1.Te gjitha jane vecori te imunitetit te lindur PERVEC:
A. Eshte tipar I vecante per vertebroret.
B.Qelizat kryesorete ketij imuniteti perfshijne makrofaget dhe limfocitet NK .
C.Nuk njohin molekulat e huaja beninje apo mikrobet
D. Njeh molekula embrionale me vargje te koduar te bujtesit (germ line–encoded host molecules). E.
Njeh faktoret kryesor te virulences mikrobiale ,por nuk njeh qelizat e vete organizmit

2.Nje burre 29 vjec me dhimbje abdominale episodike dhe edeme nga stresi I buzeve, gjuhes dhe me
raste edhe te laringut ka me shume mundesi ka nivele funksionale te ulta te ciles proteine?
A. C1 esterase inhibitor
B. C5A (complement cascade) C. Cyclooxygenase
D. IgE
E. T-cell receptor, α chain CASE REPORT 3

3.Cila nga pohimet e pershkruan me mire funksionin e proteinave te koduara nga gjenet e KMPI 1
dhe 2(kompleksi madhor I pajtueshmerise indore)?
A. Aktivizimi i sistemit komplementar
B. Lidhja me receptoret siperfaqesore te granulociteve dhe makrofageve per te filluar fagocitozen.
C. Lidhja jospecifike me Ag per t’ia prezantuar qelizave T
D.Lidhja Ag specifike pas aktivizimit te qelizave B,pr te ndihmuar neutralizimin dhe precipitimin

4.Nj burre 37 vjec eshtediagnostiguar kohet e fundit me HTA sistemik dhe I eshte dhene Lizinopril si
monoterapi fillestare.Ne diten e trete te mjekimit zoteria vuri re se dora e djathte ishte e
enjtur,kishte krurje dhe ndjente pickime.Ne darke vone te po asaj dite pati enjte te buzeve veshtiresi
ne frymemarrje.Cili nga pohimet e pershkruan me mire kete gjendje?
A.Simpomat e tij jane per shkak te rritjes direkte te mast cell nga lizinoprili.
B. Simptomat e tij jane si pasoje e degradimit te demtuar te bradikinines nga lizinoprili.
C. Simptomat e tij nuk do te rindodhin nese kalon ne enalapril.
D.Testet e gjakut periferik do te tregojne deficit te C1 inhibitor.
E. Niveli plazmatik I IgE jane me shume gjasa te rritura.

5.Nje grua 35 vjec shkon ne kliniken locale per lezionet rekurente te urtikaries ,te cilat lene shenja te
dekoloruara ne lekure qe prej 6 muajsh.Ajo ka pasur gjithashtu atralgji.Shkalla e sedimentimit eshte
85mm/h.Procedura e cila do te japi diagnozen korrekte te ketij rasti eshte:
A. Teste alergike te lekures(wheal and flare test)
B. Matja e perqendrimit total te IgE ne serum
C. Matja e aktivitetit te C1 esterase inhibitor
D. Biopsi lekure
E. Teste te lekures (patch)
6.Nje grua 28 vjec kerkon te vizitohet nga mjeku I familjes per episodet rekurente te urtikaries dhe
pohon se ‘’ajo eshte alergjike ndaj motit te ftohte’’.Ajo raporton se per me shume se 10 vjet I jane
zhvilluar zona me urtikari,kryesisht ne kembe e ne krahe kur ekspozohej ndaj temperaturave te
ulta.Zonja pranon se nuk eshte vizituar me pare per kete gjendje dhe se vitet e fundit shfaqja e e
ketyre shenjave eshte bere me e shpeshte.Pervec te ftohtit ,nuk ka vene re ndonje nxites tjeter per
zhvillimin e urtikaries.Nuk ka astme apo atopizem.Mohon te kete intolerance ndaj ushqimit dhe te
vetmet medikamente qe merr qe prej 5 vitesh jane kontraceptivet orale.Ne ekzaminimin objektiv,I
shfaqet nje gervishtje lineare pergjate parakrahut pas goditjes me spatulen orale .Pas vendosjes ne
uje te ftohte dora behet e kuqe dhe e enjtur.Pervec kesaj,mbi zonen e ekspozuar ndaj ujit te
ftohte,shfaqen zona me reaksione te gervishtjes dhe skuqjes(wheal and flare)ne lekure.Cili eshte
hapi I radhes per menazhimin e ketij pacient?

119
A. Vleresimi per pranine e antitrupave antitiroglobulin dhe antimikrozomal
B. Kontrolli I niveli te C1 inhibitorit
C. Nderprerja e kontraceptiveve oral
D. Trajtimi me cetirizine 10mg/d
E. Trajtimi me ciproheptadine 8 mg/d

7.Nje vajze 23 vjec kerkon te vizitohet per rinitin sezonal.Ajo tregon se I shfaqen simptomat cdo vit
ne pranvere dhe vjeshte.Gjate ketyre periudhave riniti I shfaqet me rrjedhje postnazale dhe kolle qe
e con nga gjumi.Pervec kesaj ka kruarje dhe lotim te syve,Kur I shfaqen simptomat vajza merr
Loratadine 10mg/d (te pashkruar nga mjeku)e cila e permireson dukshem .Cili ose cilet alergente
eshte/jane shkak I simptomave te kesaj pacienteje?
A. Barerat
B. Ragweed (bime me shume polen)
C. Pemet
D. A dhe B
E. B dhe C
F.Te gjitha te mesipermet
8.Cilet nga antitrupat e meposhtem ka me shume gjasa te jene te pranishme ne nje pacient me LES?
A. Anti-dsDNA B. Anti-RNP (anti ribonucler protein) C. Anti-Ro (anti Sjorgen)
D. Antiphospholipid E. Antiribosomal P(protein)

9) Nje grua 23 vjec vlerësohet nga mjeku i saj i kujdesit parësor sepse është e shqetësuar se mund të
ketë LES pasi ka dëgjuar një njoftim të shëndetit publik në radio. Ajo nuk ka histori të

PERGJIGJE: E

konsiderueshme mjekësore dhe ilaçi i vetëm i saj është ibuprofen i rastit.Ajo nuk eshte
seksualisht active dhe punon ne nje ushqimore.Ajo raporton se ka pasur ulcera intermitente
ne gojë dhe dhimbje në gju të djathtë.Examinimi fizik nuk tregon evidence te alopecis,rash
lekure,inflamacion dhe enjtje te( joint) .Testet e gjakut tregojn ANA positive me nje titer 1:40
por asnje anormalitet tjeter.Cili prej pohimeve me poshte eshte e vertet?
A. 4 kritere duhen per tu diagnostikuar me LES ,ky pacient ka 3
B. 4 kritere diagnostike duhen per tu diagnostikuar me LES ,ky pacient ka 2
C. Nese nje analize urine tregon proteinuri, pacientia do të përmbushë kriteret për LES D.
Ajo I permushe kriteret per LES sepse ajo ka 3 kritere per semundjen
E. Prania e ANA positive vetem mjafton per ta diagnostikuar me LES

10) Nje grua 32 vjec me një diagnozë të gjatë (long standing)me LES vlerësohet nga
reumatologu i saj si ndjekje rutinore.Nje zhurm e re kardiake degjohet dhe mjeku thote te
behet nje EKO.Ajo ndihet mire ,nuk ka temperature,renie ne pesh ose semundje kardiake
preekzistuese. Shihet nje vegetacion ne valvulen mitrale.Cili prej pohimeve eshte I vertet?
1. Kultura e gjakut nuk ka gjasa të jetë positive
2. Terapi me glukokortikoid është vërtetuar qe cojn ne permirsim te gjendjes
3. Pericarditis është shpesh i pranishëm njëkohësisht
4. lezioni ka një rrezik të ulët të embolizimit
5. pacienti ka qenë duke përdorur në mënyrë të vrazhdueshme injeksione drogrash

11) Nje grua 24 vjec është diagnostikuar rishtazi me LES.Cila nga ndërlikimet e mëposhtme
të sistemit organik ka shumë të ngjarë që ajo të ketë gjatë gjithë jetës së saj?
A. Kardiopulmonare B. Kutane C. Hematologjike D. Muskuloskeletike E. Renale

120
12) Nje grua 27 vjece me LES ne remission; mjekimi aktual me azathioprine 75mg/d dhe
prednisone 5mg/d.Vitin e kaluar ajo kishte nje perkeqesim te semundjes,jete kercenuese.Ajo
deshiron te ngelet shtatezane.Cili prej pohimeve tregon veprimin me pak te duhur ?
1. Informim per ti then qe rrisku per aborte spontan eshte I larte
2. Paralajmerim qe exacerbimi(perkeqesimi)mund te ndodhi ne tremestin e pare te
shtatzanis ose ne periudhen post partum
3. Ka te ngjare qe dhe bebi mund te kete lupus ne te ardhmen
4. Informim se rreziku i humbjes se fetusit eshte shume I larte nese antitrupat
antikardiolipin detektohen ne serumin e saj
5. Ndalo prednisonin perpara se te fillosh tentativat per te ngelur shtatzane
13) Nje grua 45 vjec afrikano - amerikane me LES vjen ne urgjence me ankesat dhimbje koke
dhe lodhje.Manifestimi I saj I pare I LES -it ka qene artralgjia,anemia hemolitike,ulcera ne
goje,rash malar,dhe ajo dihet se ka titer të lartë të antitrupave ndaj ADN-së me dy nivele
??(double stranded ADN..Aktualisht ajo mere prednisone 5mg ne dite dhe
hydroxychloroquine 200mg ne dite.PA190/110 mmHg te rrahurat e zemres 98 rrahje/min.Ne
nje analize urine shihet 25 RBCs për fushë me fuqi të lartë dhe 2+ proteinuria. azot ure ne
gjake 88mg/dl,creatinina 2.6mg/dl.Ajo nuk ka pasur me pare semundje renale dhe as nuk ka
mare me pare AIJS.Ajo nuk pohon te kete semundje te tjera,renie te oreksit ose diarre.Cili
eshte hapi tjter me korrekte qe duhet te ndermeret ne menaxhimin e ketij pacienti?
1. Fillimi I ciklofosfamidit 500mg/m2 siperfaqe trupore IV dhe plnifikohet te perdoret
cdo muaj per 3deri n 6 muaj
2. FillimiIhemodializes
3. Fillimi I dozave te lart steroid (IV metilprednisolon 1000mg/dite per 3 doza e
ndjekur nga prednisolone oral 1mg/kg ne dite ) + mykofenolat mofetil 2g/dite
4. Fillimi I plazmoferezes
5. Ndërpreje te terapise derisa te behet biopsia renale

14) Nje grua 25vjec afro-amerikane është ndjekur në klinikën e LSE që nga diagnostikimi i
saj 6 muaj më parë. në atë kohë ajo kishte prova të sëmundjes degjenerative te
artikulacioneve(mild join disease)fotosensitivitet,malar rash,ANA+,anti- dsADN. Funksini
renal dhe analiza urines nrml. Ajo perdorte hydrocycloroquin dhe acetominofen. Ajo u
paraqite ne urgjence pas nje shetitje me shoket ne plazhe. gjatë 2 ditëve të fundit ajo ka vene
re një rritje e dukshme te lodhjes dhe ngurtësim mëngjesore.Ajo gjithashtu ka urine me
ngjyre te kuqerremte. Ne examinimi fizik vihet re rash ne zonat e ekspozuara ne diell te
lekures. Rritje e densitetit te kyçit te dores, gju dhe kyçin e këmbës.Ka gjithashtu renien te
nivelit te trombociteve deri n 45,000 dhe nje leukopeni te re. Creatinina =2.5 , RBC ne
analizen e urines. Biopsia renale është në përputhje me nefritin lupik difuz aktiv. Pasi ka
mare metilprednisolon 1g IV per 3 dite,te gjitha trajtimet te pershtatshme me perjashtim te
1. .Prednisone 60mg/d
2. Prednisone 60mg/d plus azathioprine
3. Prednisone 60mg/d plus cyclophosphamide
4. Prednisone 60mg/d +mycophenolat mofetil
5. Rituximab

15) Nje 27 vjecare grua shtrohet ne terrapin intensive 3 dite pasi ka lindur nje bebe ne term.
Pacienti u zbulua se kishte hemipares te djathe dhe nje dore te majte blu.Examinimi fizik:
livedo reticularis . Examinimet lab: WBC =10.2/uL hematocrit =35% trombocitet=13,000/uL
BUN=36mg/dL Creatinina 2.3mg/dL. Kjo shtatzani ishte e qete pa problem duke pasur
parasysh dhe faktin qe kishte bere 3 aborte me pare.Cila prej exzaminimave me poshte
tregojn etiologjin themelore te gjendjes se saj ?

121
1. Anticardiolipin antibody panel
2. Antitrupat antinuclear
3. Examinimi Doppler I pemes arteriale te krahut te majte
4. Ekokardiografi
5. MRItruri
16) Nje grua 28 vjec paraqitet ne urgjence me ankesat me perkeqesim te dhimbjes dhe enjtje
ne kemben edjathte. ajo hipi në një makinë për 8 orë duke u kthyer nga një udhëtim hiking 2
ditë më pare dhe me pase ndjeu dhimbje ne kemb. Ne fillim mendoj se ishte nga lodhja por
ajo u perkeqesua me teper gjate dites.Historia mjeksore e me parshme veshtirsi per te ngelure
shtatezan me 2 aborte spontane te meparshme.Kemba e saj eshte enjtur nga mesi e poshte I
kofshes .Doppleri tregon nje tromboze te venave te thella ne venen femorale dhe iliace qe
shtrihet deri ne pelvis. Elektrolitet nrml,WBC=nrml,trombocitet dhe koha e protombines
nrml.testi shtatzani negative, Fillojn heparin me pesh te vogel molekulare ne urgjence.Terapia
pasuese duhet te perfshij
1. Rituximab 375m/m2 ne jave per 4 jave
2. Warfarin me vlera INR 2.0-3.0 per 3 muaj
3. Warfarin me vlera INR 2.0-3.0 per 12 muaj
4. Warfarin me vlera INR 2.5-3.5 per jete
5. Warfarin me vlera INR 2.5-3.5 per 12 muaj e ndjekur nga marrja e aspirines cdo dite
per gjithe jeten

17. Cili nga më poshtë është artikulacioni i prekur më shpesh gjatë artritit rheumatoid?
A. Artikulacioni interfalangeal distal B. Artikulacioni coxofemoral
C. Artikulacioni genus D. Artikulacionet interspinoze
E. Art. e kycit të dorës

18. Në pacientët me artrit rheumatoid, gjithë gjetjet e mëposhtme në ragiografinë pulmonare
mund të shpjegohen nga gjendja reumatologjike, përveç:
A. Infiltrate intersticiale bilaterale B. Bronkektazi
C. Infiltrate lobare D. Noduj pulmonar solitarë E. Efuzion unilateral pleural

19. Cila nga më poshtë, është gjetja më e qartë radiografikenë artritin reumatoid?
A. Osteopeni jukstaartikulare B. S’ka anormalitete C. Edemë e indeve të buta
D. Erozion subkondral E. Humbje e simetrisë artikulare

20. Cili nga pohimet lidhur me artritin reumatoid është i vërtetë?


A. Afrikanët dhe afrikano-amerikanët më së shumti kanë klasën e dytë të kompleksit madhor
të histokompatibilitetit, alelet HLA-DDR4.
B. Femrat preken rreth 3x > meshkujt dhe kjo diferencë mbahet e pandryshuar përgjatë jetës.
C. Lezioni më i hershëm në artritin reumatoid është shtimi i qelizave në lëngjet
sinoviale me reduktim të mikrovaskularizimit.
D. Nuk ka lidhje me klasën II të kompleksit madhor të histokompatibilitetit, alelet HLA-B27
E. Titrat e faktorit reumatid nuk janë përcaktuese të rëndesës së artritit reumatoid, apo të
shfaqjeve të tij ekstraartikulare

21. Një femër 46 vjeçare vjen në klinikën tënde me një sërë ankesash. Ajo ndihet e sëmurë
dhe e lodhur prej 2-3 muajsh. Oreksi i saj ka rënë në këtë kohë. Thotë se ka humbur rreth 4.5
kilogramë. Kohët e fundit ka ndjerë dhimbje dhe ngurtësim të gishtave të të dyja duarve më
të shprehur në mëngjes si dhe gjatë lëvizjeve të njëjta. Ka motrën dhe gjyshen që vuajnë nga

122
artriti reumatoid, dhe shprehet e sigurtë se edhe ajo mund të jetë prekur prej artritit. Cila nga
shprehitë e mëposhtme klinike është manifestimi më i zakonshëm i artritit reumatoid?
A. Lodhje dhe anoreksi për më tepër se 2 muaj bashkë me dhimbje kyçesh
B. Ngurtësim mëngjesor i kyçeve që zgjat më tepër se 1 orë
C. Dhimbje në kyçe që rëndohet me lëvizjet
D. Histori familjare pozitive me dy të afërm me artrit reumatoid
E. Humbja e peshës rreth 4.5 kilogramë gjatë ecurisë së sëmundjes

22. Gjithë shfaqjet e mëposhtme janë karakteristike për manifestimet ekstraartikulare të
artritit reumatoid përveç: A. Anemia B. Vaskuliti kutan
C. Perikarditi D. Sindromi Sjogren sekondar E. Trombocitopenia

23. Te gjithë agjentët e mëposhtëm se kanë efektivitet në trajtimin e artritit reumatoid,
përveç: A. Infliximab B. Leflunomide C. Methotrexate D. Naproxen E. Rituximab

24. Cila nga gjetjet klinike të mëposhtme është shprehja më tipike e ethes reumatoide akute:
A. Karditi B. Chorea C. Eritema marginatum D. Poliatriti E. Nodujt subcutan

IX-24. Cila nga shenjat e eshte me e zakonshme ne ethen reumatizmale akute (ARF)
A. Karditis (besoj e ka per endokarditin) B. Chorea (eshte cregullim neurologjik)
C. Eritema marginatum ( eritem ne forme unaze) D. Poliartritis E. Noduj subkutan

IX-25. Nje emigrante 19 vjecare nga Etiopia paraqitet ne kliniken tuaj per te marre sherbim
primar. Ajo aktualisht ndihet mire. Ka nje histori te kaluar mjekesore per fibrilacion atrial (i
cili u shfaq per here te pare- new onset). Ne femijeri ajo ka zhvilluar nje patologji qe
shkaktonte paralize (flailing) te pakontrolluar te gjymtyreve dhe te gjuhes, e cila zgjaste
afersisht 1 muaj. Ajo gjithasht ka kaluar tre epiode me artrit migrator te artikulacioneve te
medha gjate adoleshences se saj, i cili kalonte me disa medikamente qe i merte ne farmaci.
Ajo aktualisht mer Metoprolol dhe varfarin, dhe nuk ka alergji nga barnat. Ne ekzaminimin
fizik vihe re rrahje te cregullta te zemres (irregularly irregular heart beat) me tension normal.
Pika maximale e impulsit (PMI) eshte me e theksuar ne vijen e mesit te klavikules dhe eshte
ne norme. Nje zhurme diastolike dhe 3/6 holosistolike degjohet ne apeks. Gjithashtu nje
zhurme e lehte diastolike degjohet ne interkostalen e trete te majte. Ju e referoni tek
kardiologu per vleresimin e protezave valvulare dhe echokardiografi. Cfare nderhyrje tjeter
mund te beni tani? A. Glukokortikoid B. Aspirin cdo dite
C. Doxycycline cdo dite D. Penicilin G me injeksion cdo muaj
E. Penicilin G kur eshte e nevojshme per te gjitha dhimbjet e fytit

IX-26. Nje paciente me diagnoze sklerodermi, qe ka perfshirje difuze cutane me hipertension


malinj, oliguri, edem, anemi hemolitike dhe insuficience renal. Ju vendosni diagnozen SRC
(scleroderma renal crisis). Cili eshte trajtimi i rekomaduar?
A. Kaptopril B. Karvedilol C. Klonidin D. Diltiazem E. Nitroprusside

IX-27. Nje grua 57 vjece me depresion dhe dhimbje migrenoze kronike referon per tharje te
gojes dhe syve prej disa vitesh. Ankesa e saj primare eshte qe nuk mund te haj me crakersat e
saj te preferuar, megjithate ajo referon per fotosensitivitet dhe djegje te syve kur pyetet ne
vazhdim. Ajo nuk ka simptoma te tjera shoqeruese. Ne ekzaminim dallohet mukoza orale e
thare, eritematoze dhe ngjitese. Te gjitha testet e meposhte kane gjasa te dalin pozitive ne
kete paciente PERVEC: A. Antitrupat La/SS-B B. Antitrupat Ro/SS-A
C. Testi Schirmer’s I D. Antitrupat Scl-70 E. Sialometria (mate sasine e pershtymes)

123
IX-28. Cila nga alternativat e meposhte eshte nder manifestimet primare extraglandulare me
te zakonshme e sindromit sjogren? A. Artralgjia/ artrit.
B. Limfoma C. Neuropati periferike D. Fenomeni Raynaud E. Vaskulitis.

IX-29. Nje grua 44 vjece paraqite per vleresimin e gojes dhe syve te thara. Hera e pare qe i
vuri re keto simptomat ishte para 5 vitesh dhe ka ardhur duke u perkeqesuar. Ajo ndjen sikur
ka rere ne sy. Ndonjehere syte i djegi dhe e ka e veshtire te qendroje jashte nen driten e
diellit. Per me teper goja eshte lehtesishte e thare. Ne pune i kerkojne shpesh te bej
prezantime dhe gjithmon e me shume e ka te veshtire te bej nje prezantim 30-60 min. Mban
gjithmon uje me vete. Ajo gjithashtu referon per higjen te mire dentare, para tre vitesh
dentistit te saj iu desh ti ndryshonte mbushjen dy here per shkak te kariesit dentare. Histori te
kaluar mjekesore ka vetem trajtim per tuberkulozin me te cili u infektua ne moshen 20
vjecare kur ishte ne trupat paqesore ne Azine e jugut. Nuk mer medikamete rregullishte dhe
nuk eshte duhanpirese. “Ocular examination reveals punctuate corneal ulcerations on Rose
Bengal stain, and the Schirmer’s test shows greater than 5 mm of wetness after 5 minutes”.
Mukoza orale eshte e thare dhe me sekrecione mukozale te trashe, dhe zmadhim bilateral i
gjendrave parotide. Ekzaminimet laboratorike tregojne per Antitrupa Ro dhe La (SS-A dhe
SS-B) pozitive. Natriumi 142 meq/L, kaliumi 2.6 meq/L, Cl 1115 meq/L dhe bikarbonatet 15
meq/L Cila ka gjasa te jete shkaktari i hipokalemise dhe acidozes ne kete paciente?
A. Diarrea
B. Acidoze tubulare distale renal tip I
C. Hipoaldosteronizmi
D. Purging with underlying anorexia nervosa
E. Kompesim renal per Alkaloze kronike respiratore

IX-30. Nje paciente me sindrom sjogren primar e diagnostikuar perpara 6 vitesh trajtohet
simptomatikisht me lot artificial, referon per enjtje te vazdueshme 3 muajt e fundit. Ajo
dallon zmadhim te limfonodyjve cervikal posterio. Vleresimi tregon leukopeni dhe nivel te
ulet te komplementit C4. Cila eshte diagnoza e mundshme? A. Amiloidosis
B. Pankreatiti kronik C. HIV D. Limfoma E. Sindroma secondar sjogren

IX-31. Ne cfare perqindje gjendet antigjeni HLA-B27 ne pacientet me spondilit ankilozant.


A. 10% B. 30% C. 50% D. 90% E. 100%

32. Cila nga t eshte manifestimi ekstra artikular me i zakonshem i spondilitit ankilozant?
1. Uveiti anterior
2. Insuficiencaeaortes
3. Semundja inflamatore e zorres (inflammatory bowel disease)
4. Fibroza pulmonare
5. Bllok i zemres grada e trete

33. Nje djale 25 vjec takon mjekun e tij ne kujdesin primar per te vleresuar nje dhimbje ne
fund te shpines. Dhimbja eshte severe, eshte me keq ne mengjes, lehtesohet me ushtrime dhe
perkeqesohet kur ben pushim; ne vecanti gjumin i nates eshte i veshtire. Ai ndjen ngurtesim
ne mengjes per te pakten 30 min. MRI e fundshpines tregon inflamacion aktiv ne
artikluacionin sakroiliak. Ai raporton nje histori te skuqjes se njerit sy trajtuar me
kortikosteroid 2 vjet me pare. Nje test per HLA-B27 del pozitiv. Cila nga t eshte terapia e
linjes se pare per gjendjen e tij?
A. Influximab B. Naproxen C. Prednisone D. Rituximab E. Tramadol

124
34. Nje djale 27 vjec shkon te mjeku i familjes per te vleresuar nje artrit te dhimshem qe
perfshin gjurin e djathte qe shoqerohet me enjtje difuze te gishtit (finger welling diffusely).
Pervec kesaj ai eshte i shendetshem, por ai kujton nje episod sever te semundjes diarreike
perreth 3-4 jave me pare qe u zgjidh spontanisht. Ai nuk merr medikamente dhe raporton
perdorim te rralle te marijuanes. Duke pare sistemet, ai raporton per urinim te dhimbshem.
Ekzaminimet tregojne artrit inflamator, ne gjurin e djathte, daktilitis, dhe ekzaminimi
genitourinary normal. Ai eshte diagnstikuar me artrit reaktiv. Agjenti etiolgjik i diarrese ?
A. Campylobacter jejuni B. Clostridiumdifficile C. Escherichiacoli
D. Helicobacter pylori E. Shigellaflexneri

35. Nje grua 28 vjec i nenshtrohet vleresimit te humbjes se peshes dhe diarre me gjak qe
kohet e fundit eshte diagnostikuar si semundja Crohn. Ajo eshte diagnostikuar me daktilitis
dhe sakroilitis bilateral gjate 6 muajve te fundit. Eshte programuar qe ajo te filloje trajtim me
infliximab ne 2 jave per semundejn e Crohn. Cila nga fjalite e meposhtem eshte e vertete
lidhur me efektin e infliximab ne artritin e saj?
1. Megjithese infliximab ka me shume mundesi te permiresoje artritin e saj, AIJS duhet
te provohen fillimisht
2. Megjithese infliximab eshte terapi shume efektive per semundjen e Crohn, ai nuk do
te kete efekt ne artritin
3. Artriti i saj nuk ka lidhje me semundjen e Crohn, dhe per shkak te kesaj ajo duhet ti
nenshtrohet nje vleresimi per shkaqe infeksioze perpara se ti nenshtrohet terapise
imunosupresive
4. Infliximab eshte terapi shume efektive per kete lloj artriti.
5. Asnje nga te mesipermet

36. Cili na fjalite e lidhur me artritin e semundjes Whipple eshte e vertete?


1. Artriti gjendet rralle ne semundjen Whipple
2. Manifestimet artikulare jane zakonisht ne te njejten kohe me simptoma
gastrointenstinale dhe malabsorbim
3. Radiografia shpesh tregon erozion te artikulacioneve
4. Ekzaminimi i lengut synovial nuk ka mundsi te tregoje qeliza polimrfonukleare
5. Asgje nga te mesipermet

37. Nje burre 35 vjec ka spondilitis ankilozant sever qe nuk i pergjigjet terapise me AIJS. Atij
i eshte rekomanduar terapia me infliximab dhe ai po mendonte per efektet e mundshme
anesore. Te gjitha t jane efetke anesore potenciale te zakonshme, pervec:
1. Crregullim demielinizues
2. Tuberkuloz i diseminuar
3. Eksacerbim i insuficences kardiake kongjestive
4. Pneumoni hypersensitive ( hypersensitivity pneumonitis)
5. Pancitopeni

38. Cili nga perkufizimet e i pershtatet me shume termit enthesitis?


1. Alterim i artikulacionit gjatesor qe siperfaqet artikulare te bashkohen jo plotesisht me
njera tjetren
2. Inflamacion ne anen ku tendini ose ligament inserohen ne kocke
3. Inflamacion ne membranen periartikulare ne ane e brendshme te capsules artikulare
4. Inflamacion i nje kaviteti (saclike) afer nje artikulacioni qe ul ferkimin
5. Nje vibrim i palpueshem ose ndjesi kliku qe merret nga levizja e artikulacionit

125
39. Nje femer 35 vjec paraqitet te mjeku i familjes duke u ankuar per dhimbje trupi dhe
artikulacionesh. Kur pyetet per tu pershkruar se cila nga artikulacionet e saj eshte me e
prekur, ajo pergjigjet, “Te gjitha” Nuk eshte e shoqeruar me ngurtesim, skuqje ose enjtje te
artikulacioneve. Nuk eshte vleresuar fenomen Reyno. Ndonjehere ajo ve re humbje te
ndjeshmerise ne gishtat e dores dhe te kembes. Pacientja ankohet per dhimbje kronike dhe
cilesi te ulet te gjumit qe ajo ndjen se eshte per shkak te dhimbjes. Ajo me pare ka qene ne
klinike per dhimbje koke kronike qe ishin ndjere te lidhura me tensionin. Ajo eshte perpjekur
te marre ibuprofen pa rekomandimin e mjekut 2 here ne dite pa reduktuar dhimbjen. Ajo nuk
ka problem te tjera mjekesore. Ne ekzaminimin fizik pacientja shfaqet e rehatuar.
Artikulacionet e saj jane plotesisht te levishme pa evidence per artrit inflamator. Ajo ka
dhimbje ne palpim ne inserimet e muskujve bilateral suboksipital, ne C5, ne epikondilin
lateral, ne kuadratin e siperm te jashtem te gluteusit, medialisht kupes se gjurit proksimalisht
linjes se artikulacionit, dhe unilateralisht ne brinjen e dyte te djathte. Niveli sedimentit
eritrocitar eshte 12 sek. Antitrupat antinuclear jane positive me nje titer 1:40 ne nje ( speckled
pattern). Kjo paciente eshte HLA-B27 pozitive. Faktori rheumatoid eshte negativ.
Radiograma e spines cervikale, pelvisit dhe berrylit eshte normale. Diagnoza e mundshme?
1 . Spondilit ankilozant
2. Infeksion i diseminuar gonokoksal
3. Fibromialgji
4. Artrit rheumatoid
5. Lupus sistemik eritematoz

IX-40: Një burrë 42 vjeç paraqitet me ankesën e rashit dhe dhimbjes së artikulacioneve. Ai e
vuri re për herë të parë rashin para 6 muajsh. Rashi ishte fillimisht i pranishëm në duar,
sipërfaqen ekstensore të bërrylave dhe gjunjëve, fundshpinë dhe skalp. Edhe pse ai
shqetësohej për shfaqjen e këtyre lezioneve ato nuk i kruheshin apo dhembnin. Ai nuk kishte
bërë më parë ndonjë vlerësim në lidhje me këtë dhe kohëve të fundit ka vërejtur ndryshime në
shtratin e thonjve. Gjatë 2 javëve të fundit, pacienti ka pasur dhimbje që kanë ardhur duke u
rritur, të rënda në artikulacionet distale të duarve dhe këmbëve. Duart e tij janë kaq të
dhimbshme sa ai ka pasur vështirësi në shkrim dhe në mbajtjen e mjeteve të
kuzhinës/serviseve. Mohon të këtë pasur temperaturë, humbje peshe, dobësi, kollë, dispne
dhe ndryshime përsa i përket zorrëve apo fshikëzës. Diagnoza më e mundshme?
A. Artriti i shoqëruar me sëmundje inflamatore të zorrëve B. Guta C. Osteoartriti
D. Artriti psoriatik E. Artritirheumatoid

126
IX-41: Të gjitha sindromat vaskulare të mëposhtme mendohet se vijnë prej depozitimeve të
komplekseve imune, PËRVEÇ:
1. Vaskuliti kriglobulinik
2. PurpuraHenoch-Schönlein
3. Poliarteriti nodoz i shoqëruar me hepatit B
4. Sëmundja e serumit
5. Granulomatoza me poliangit ( të Wegener-it)

IX-42: Një burrë 53 vjeç paraqitet me sindromë vaskulitik. Antitrupat e tij citoplazmike
antineutrofil (c-ANCA) janë pozitivë. Cilën prej sindromave ka më shumë gjasë të ketë?
1. Sindromi Chung-Strauss
2. PurpuraHenoch-Schönlein
3. Poliangiti mikroskopik
4. Koliti ulçerativ
5. Granulomatoza shoqëruar me poliangit (të Wegeberit)

 IX-43: Një burrë 40 vjeç prezantohet në urgjencë me hemoptizi në sasi shumë të vogël.
Ai raporton që ka kollitur deri në sasinë 2-5 lugë gjelle gjak për dy ditë. Ai referon për
dhimbje kraharori të lehta, temperaturë jo shumë të lartë dhe rënie në peshë. Përveç kësaj ai
ka rreth një vit me simptoma të rënda të sipërme respiratore përfshirë këtu epistaksin e
shpeshtë dhe sekrecionet purulente trajtuar me kura të shumta antibiotikësh. Përveç edhe një
hiperlipidemia të lehtë, ai është i shëndetshëm. Të vetmet barna të përditshme që ai merr janë
aspirina dhe lovastatina. Në ekzaminimin fizik ka shenja vitale normale dhe në rrugët e
sipërme respiratore vërehet një defekt septal në formë shale i hundës. Pulmoni është i pastër.
Një CT-kraharori tregon nodule kavitare të shumta dhe analiza e urinës tregon eritrocite. Cili
testim ofron argumenta/ të dhëna më të mira diagnostike për vendosjen e diagnozës së saktë?
A. Biopsia e thellë e lëkurës
2. Biopsirenaleperkutane
3. Angiograma pulmonare
4. Biopsia kirurgjikale e pulmonit
5. Biopsia e rrugëve të sipërme ajrore

 IX-44: Një grua 84 vjeçe vizitohet tek mjeku i saj i familjes në kujdesin parësor për të
vlerësuar dhimbjen e kokës. Ajo i përjetoi këto dhimbje para disa javësh dhe tani ato po
përkeqësohen. Edhe pse ajo nuk ka pasur aura vizuale, ajo është e shqetësuar që herë pas here
ka humbur shikimin në syrin e majtë këto ditët e fundit. Ajo mohon për ndonjë dobësi apo
mpirje të re, por referon për një dhimbje nofulle gjatë ngrënies. Historia e saj e mëparshme
mjekësore përfshin SAK, për të cilin ajo ka kryer një bypass para 10 vitesh; diabet mellitus,
hiperlipidemi dhe depresion të lehtë. Përmbledhja e gjithë simptomave nxjerr në pah edhe
djersitjet gjatë natës dhe një dhimbje të lehtë ne pjesën e poshtme të kurrizit që është
veçanërisht e pranishme në mëngjes. Cili është hapi tjetër më i përshtatshëm?
1. Aspirin 975 mg/ditë po
2. MatjaeESR
3. Prednisone 60 mg/ditë
4. Referimi për biopsi të arteries temporale
5. Referimi për ekografi të arteries temporale

 IX-45: Një burrë 54 vjeç është vlerësuar për vaskulitin kutan dhe nefropatinë periferike.
Për shkak të disfunksionit konkomitant renal ai i nënshtrohet biopsisë renale qe tregon

127
glomerulonefrit. Krioglobulinat janë të pranishme në gjakun periferik të tij. Cila nga gjetjet e
mëposhtmë laboratorike mund të shërbejë për të vendosur etiologjinë?

1. Antigjeni i sipërfaqes për hepatitin B


2. ANCA citoplazmik
3. PCR për hepatitin C
4. Antitrupat HIV
5. Faktori rheumatoid

 IX-46: Një burrë 54 vjeç është shtruar për dhimbje abdominale të poshtme të
vazhdueshme dhe dhimbje pelvike që filloi para 7 muajsh. Dy javë para ardhjes së tij, ai
realizoi një laparoskopi eksploratore për dhimbjen akute abdominale dhe kolecistitin e
paramenduar. Ajo tregoi nekrozë të indit omental dhe perikolecistit me nevojë për
omentektomi dhe kolecistektomi. Gjithsesi, dhimbja vazhdoi e pandryshuar. Ai aktualisht e
përshkruan atë si periumbilikale dbe që shpërndahet drejt pelvisit dhe këmbëve. Ajo
përkeqësohet pas ngrënies. Pacienti ka pasur gjithashtu disa episode me dhimbje severe
testikulare episodike, urgjencë për të defekuar, të përziera, të vjella dhe diurezë. Ai ka
humbur afërsisht 2.7 kg brenda 6 muajve. Historia e tij e mëparshme mjekësore është
sinjifikante për hipertension që kohëve të fundit është bërë vështirë të kontrollohet. Mjekimi
në hyrje përfshiu aspirin, hidroklortiazid, hidromorfon, lansoprazol, metoprolol dhe kuinapril.
Në ekzaminimin fizik, pacienti shfaqet i rehatuar. Presioni arterial 170/100 mmHg, FK
88/min dhe ai është afebrile. Ai ka zhurma primare dhe sekondare normale kardiake pa
zhurma patologjike dhe një ton S4 prezent. Nuk ka zhurma (bruits) karotide, renale,
abdomianle ose femorale. Mushkëritë janë të pastra në auskultacion. Lëvizjet e zorrëve janë
normale. Palpimi abdominal shfaq butësi difuze pa mbrojtje muskulare. Nuk ka masa
prezente dhe jashtëqitjet janë negative për gjak okult. Gjatë ekzaminimit pacienti shfaq
fenomen Raynaud në dorën e djathtë që persiston për disa minuta. Ekzaminimi neurologjik
normal, pa dëmtime. Vlerat laboratorike tregojnë për ESR 72mm/h, BUN 17 mg/dL,
kreatininë 0.8 mg/dL. Pacienti nuk ka proteinuria ose hematuria. Testimet për antitrupa
antinuklear, anti-ds DNA, antineutrofilik citoplasmik janë negativë. Testet e funksionit
hepatik janë jonnormale me një AST 89 IU/L dhe ALT 112 IU/L. Antigjeni i sipërfaqes dhe
ai e për hepatitin B janë pozitivë. Angiografia mesenterike demonstron aneurizma të vegjël,
si rruaza(beaded) në venat mezenterike superior dhe inferior. Diagnoza më e mundshme?
1. Karcinoma hepatoqelizore
2. Koliti ishemik
3. Poliangiti mikroskopik
4. Krioglobulinemia mikse
5. Poliarteriti nodoz

 IX-47: Një djalë 18 vjeç është shtruar në spital me dhimbje të menjëhershme te kraharorit
shtypëse substernale qe filloi në mënyrë të papritur para 30 minutash. Ai referon që dhimbja
përhapet në qafë dhe krahun e djathtë. Në përgjithësi ai ka qenë i shëndetshëm. Ai aktualisht
luan trompetë në bandën marshuese të shkollës së mesme por nuk merr pjesë rregullisht në
aktivitetet aerobike. Në ekzaminimin fizik ai paraqitet disforik dhe takipneik. Presion arterial
është 100/48 mmHg dhe FK 110/min. Ekzaminimi kardiovaskular tregon ritëm të rregullt por
takikardik. Një zhurmë holosistolike A II/VI dëgjohet më mirë në apeks dhe përhapet në
aksilë. Pulmonet e tij kanë bilateralisht rale në baza. EKG demostron 4mm ngritje të ST në
lidhjet anteriore. Në anamnezë në lidhje me historinë e tij të mëparshme mjekësore ai kujton
t’i kenë thënë se është shtruar në spital për probleme me zemrën kur ka qenë 2 vjeç. Mamaja,
e cila e shoqëron referon që ai ka marrë aspirin dhe ƴ-globulin si trajtim. Që prej asaj kohe,

128
atij i është dashur të bëjë ndjekje intermitente me ekokardiografi. Cili është shkaku më i
mundshëm i sindromit koronar akut të këtij pacienti?
1. Disekacion i rrënjës së aortës dhe ostiumit koronar të majtë
2. Prezenca e urave miokardiale( myocardial bridge, është një gjendje në të cilën një ose
më shumë arterie koronare janë të futura në muskulin kardiak në vend që të
qëndrojnë në sipërfaqe) të përhapur në arterien descendente anteriore të majtë
C. Stenozë e një aneurizmi koronar arterial
4. Vazospazma që ndjek përdorimin e kokainës
5. Vaskuliti që përfshin arterien descendente anterior të majtë

 IX-48: Cila nga të mëposhtmet kërkohet për diagnozën e sëmundjes Behçet?
1. Vaskuliti i enëve të mëdha
2. Testi patergjik (pathergy test, më duket se është i ngjashëm me atë prick testin se
ashtu dukej nga përshkrimi në internet, por ky ishte standard për Behçetin)
3. Ulçeracionet rekurente orale
4. Ulçeracionet rekurente genitale
5. Uveiti

 IX-49: Një vajzë 25 vjeçe prezantohet me ankesën e ulçeracioneve të dhimbshme në
gojë. Ajo i përshkruan lezionet si ulçera të cekëta që zgjasin 1-2 javë. Ulçerat janë shfaqur 6
muajve të fundit. Për dy ditët e fundit, pacientja ka pasur njërin sy të kuq dhe të dhimbshëm.
Nuk ka ulçeracione genital, artrit, rashe lëkure ose fotosensitivitet. Në ekzaminimin fizik,
pacientja shfaqet e mirëzhvilluar dhe jo në distres. Ajo ka një temperaturë 37.6oC, FK
86/min, PA 126/72 mmHg dhe FR 16/min. Ekzaminimi i mukozës orale tregon dy ulçera të
cekëta me baze të verdhë në mukozën bukale. Ekzaminimi oftalmologjik është normal. Ajo
nuk ka artrit, por medialisht në kofshën e djathtë dallohet një kordon i palpueshëm në venën
safena. Gjetjet laboratorike zbulojnë një ESR prej 68 sekondash. Leukocitet janë 10.230/μL
me 68% polimorfonuklearë, 28% limfocite dhe 4% monocite. Antitrupat antinuklearë dhe
anti-dsDNA janë negativë. C3 është 89 mg/dL dhe C4 24 mg/dL.Diagnoza më e mundshme?
1. Sindromi Behçet
2. Pemfigoidicikatricial
3. Lupusi diskoid eritematoz
4. Sindromi Sjëgren
5. Lupusieritematozsistemik

48. Cila nga t kerkohet per diagnozen e semundjes Behcet? A. Vaskuliti i eneve te medha
B. Prova e Patergjise C. Ulcerac rekurrent oral D. Ulverac rekurrent genital E.Uveit

49. Nje 25 vjecare paraqitet me ankese te ulceracioneve te dhimbshme orale. Ajo i


pershkruan keto lezione si ulcera te ceketa qe zgjasin per 1-2 jave. Ulcerat jane shfaqur per 6
muaj. Ne 2 ditet e fundit,pacientja ka patur sy te kuq te dhimbshem. Ajo nuk ka patur
ulceracione genitale, artrite, skuqje lekure apo fotosensitivitet. Ne ekzaminim fizik, pts
paraqitet e zhvilluar mire dhe pa ankth. T=37.6 o C, pulsi=86 rr/min, TA=126/72 mm Hg,
FR=16 respiracione/min. Ekzaminimi i mukozes orale zbulon 2 ulcera te ceketa me nje baze
te verdhe ne mukozen bukale. Ekzaminimi oftalmologjik tregon Uveit anterior. Ekzaminimi
kardiopulmonar eshte normal.Nuk ka artrit ,por medialisht ne kofshen e djathte, ka nje
kordon te palpueshem ne venen Saphena. Studimet laboratorike zbulojne ESR= 68 sec.
Leukocitet= 10230 /μL me nje diferencim : 68% PMN, 28% Lymph, 4% Mon. ANA dhe
anti-dsDNA jane negative. C3 eshte 89 mg/dL dhe C4= 24 mg/dL.
Cila eshte diagnoza e mundshme?

129
1. Sindroma Behcet
2. Pemfigoidicikatricial
3. Lupusi diskoid eritematoz
4. Sindroma Sjögren
5. LES

50.Cila eshte terapia me e mire fillestare per pts ne pyetjen 49?


A. Kolkicine
B.INFα intralezional
C. Glukokortikoide sistemike dhe Azathioprine
D. Talidomide
E.GK topikale perfshire Prednisone oftalmik

51. Perikondriti relapsues mund te jete nje semundje primare ose i lidhur me semundje te
tjera reumatologjike. Te gjitha gjendjet lidhen me te pervec:
1. Sindroma mielodisplastike
2. Cirrozabiliareprimare
3. Skleroderma
4. Spondyloarthritides
5. LES

52. Nje 47 vjecar eshte vleresuar prej nje viti me episode rekurrente te enjtjes bilaterale
aurikulare. Veshi eshte i dhimbshem ne keto episode dhe veshi i djathte eshte demtuar pak.
Pts paraqitet i shendetshem dhe nuk ka zakone te substancave te paligjshme. Ai punon ne nje
zyre dhe sporti i tij i vetem eshte tenisi. Ne ekzaminim, veshi i majte ka ngjyre te kuqe dhe
aurikula eshte e enjtur dhe e forte. Earlobe-minimalisht i enjtur dhe paraqitet as i kuq as i
tendosur. Cila nga t eshte shpjegimi me i mire per kete gjetje?
1. Sindroma Behcet
2. Sindroma Cogan
3. Hemoglobinopati
4. Traume rekurrente
5. Perikondrit relapsues

53.Nje 25 vjecare afrikano-amerikane eshte vleresuar me nje limfadenopati hilare bilaterale


ne nje Rx toraksi rutine perpara Kolecistekomise laparoskopike. Ajo ben nje mediastinoskopi
ku zbulohen granuloma multiple jokazeoze ne limfonoduj. Te gjitha shpjegojne pervec:
1. Proteinoze alveolare
2. Mykobaktereatipike
3. EkspozimngaBeriliumit
4. Histoplazmoza
5. Tumor
6. Sarkoidoze

54.Nje 34 vjecare ka nje histori Sarkoidoze kutane qe eshte trajtuar me Hidroksikloroquine


per 5 vite. Pas nje episodi dhimbjeje ne flankun e djathte, u diagnostikua me Kalkuloze
renale. Cila nga pohimet e lidhur me kalkulozen eshte e vertete?
1. Vit.D ekzogjene dhe ekspozimi ndaj diellit ne pacientet me Sarkoidoze mund te
coje ne hiperkalcemi dhe kalkuloze te lidhur me te.
2. Hiperkalcemia e rralle ne Sarkoidoze ,e pamundur te kontribuoje ne Kalkulin e pts.
3. Hiperkalcemia ne Sarkoidoze ndodh permes prodhimit rritur te 25-OH vitD ne lekure

130
4. Nese pts do te niste terapine me Calcium oral per te trajtuar gurin renal,urina e 24
oreve per matjen e fosfatit duhet marre para dhe pas terapise
5. Asnjera

55. Te gjithe medikamentet e eshte pare te permiresojne simptomat ose funksionin ne


pacientet me Sarkoidoze pervec: A. Etanercept B.Hydroxychloroquine
C. Infliximab D.Methotrexate E. Prednisone

IX 56. Te gjitha pohimet e per manifestimet klinike te sarkoidozes jane te verteta pervec :
A. Perfshirja kardiake ndodh ne 25 % te pacienteve
B. Perfshirja e syve zakonisht eshte nje uveit anterior
C. Perfshirja e heparit zakonisht prezantohet me rritje te fosfatazes alkaline D. Perfshrirja e
mushkrive zakonsiht ndodh ne 90 % te rasteve
E. Perfshirja e lekures ndodh zakonisht ne 1/3 e pacienteve

IX 57 . Je duke vizituar nje grua 56 vjec me ankesat e dhimbjes se gyceve dhe ngurtesim .Te
gjitha shenjat ose simptomat e jane indicator per semundjen inflamatore pervec:
A. Rritja e nivelit te eritrosedimentimit
B. Lodhja termperatura ose humbja e peshes
C. Prezenca e me shume se 6 jave
D. Prezenca e enjtjess se indit lidhor rreth kyceve te prekura E. NgurtesimmengjesorIzgjatur

IX 58 . Nje djale 22 vjec paraqitet ne spital me nje demtim ne shpatull gjate nje loje
baseball.Ai pershkroi se ndjeu nje shkeputje pastaj nje dhimbje acute ne shpatullen e majte
kur hodhi topin. Cili eshte me shqetesuesee per rupturen e muskujve te manshetes rotatore :
1. Pamundesia per te mbajtur krahun ne abduksion passiv ne kend 90 grade
2. Pamundesia per ngritur aktivisht krahun > 90 grade me fleksion te meparshem
3. Dhimbje ne palpim ne kellefin biciptal teksa krahu ben rotation intern dhe ekstern
4. Dhimbje ne palpim tek aplikohet presion anteriorisht ne artikulacion dhe duke ber
rotacion intern dhe ekstern
5. Dhimbje me abduksion passiv te krahut .

IX 59 . Nje burre 62 vjec I bardhe prezantohet ankesen e dhimbjes ne gjurin e djathte dhe
enjtjes .Historia mjekesore e meparshme tregon per obezitet , me BMI 34 kg / m2 , diabet I
tipit 2 I kontrolluar nepermjet dietes , dhe hypertension . Medikamentet qe perdore jane
hidroclortiazid dhe acetaminophen sipas rastit per dhimbjen. Ekzaminimi fizik eshte
sinjifikant per efuzion te moderuar ne gjurin e djathte , me kufinj te levizjes deri 90 grade
fleksion dhe 160 grade ekstension. Ka nje nxhehtesi minimale dhe jo skuqje . Gjate levizjes
ka kercitje. Me gjithe peshen e trupit ai ka devijime nga jashte te kembeve bilateralisht .
Grafia e gjurit tregon osteofite dhe ngushtim te hapesires artikulare . Cila nga pohimet e ka
me shume mundesi te gjendet ne ekzaminimin e lengut artikular :
1. Nje njolle Gram qe tregon per koke gram poz ne klaustera
2. Nje nivel te rruazave te bardha 1110/mikroL
3. Ne nivel I rruazave te bardha 22000/mikroL
4. Kristale birefringent positive ne mikroskopi me drite te polarizuar
5. Kristale birefringent negative ne mikroskopi me drite te polarizuar

IX 60 . Nje grua 62 vjece ka dhimbje bilaterale te duarve e cila ka qene progressive vitin e
fundit .Ajo k punuar ne nje fabrike qe ben dorashka per 35 vite .Dyshon per osteoartrit Te
gjitha t ne histori dhe ekzaminim fizik jane karakteristika diagnoses se osteostritit pervec

131
A.Evidence e enjtjes bilaterale dhe e nxehtesise qe prek vetem kycet.
B.Ngushtim I hapesires artikulare dhe osteofite ne artikulacionet interfalangeale proksimale
dhe distale ne grafi
C.Dhimbje qe behet me keq kur pergatit ushqimin .
D.Prezenca e noduseve te Heberdens .
E.Ngurtesimi I cili behet me keq pas pushimit per njefare kohe me bllokim te me shume
artikulationeve te prekura .
IX 61 . Nje grua 73 vjec me nje histori mjekesore te obezitetit dhe diabetit mellitus paraqitet
ne me ankesen e dhimbjes se gjurit te djathte qe ka qene progressive dhe behet me keq me
ecjen ose duke ndenjur ne kembe .Ajo ka mare OTC AIJS por pa asnje permirsim.Do te dije
cfare nuk shkon me gjurin e saj dhe cfare e ka shkaktuar . Behet grafia e cila tregon per
humbje te kartilagos dhe formim te osteofiteve . Cil faktor riskut me potent per zhvillimin e
osteoartitit: A. Mosha B. Gjinia C. Predispozitagjenetika
D. Obeziteti E. Demtime te meparshme te artikulacionit .
IX 62 . Nje burre 53 vjec prezantohet ne kliniken tuaj me ankesen e dhimbjes bilaterale te
gjunjeve.Thote qe dhimbja behet me keq kur ecen dhe nuk eshte prezente kur nuk ecen. Ka
perjetuar dhimbje intermitente te gjunjeve per disa muaj dhe nuk I ka ikur dhimbja ka
perdorimi OTC analgjezikeve. Ka histori mjekesore per hypertension dhe obezitet .Kur ishte
ne gjimnaz dhe shkolle te larte ai ka luajtur football dhe basketball. Cili nga pohimet e
prezanton trajtimin fillestar me te mire per pacientin :
1. Shmang ecjen per disa jave
2. Ushtrime me ecje ditore te pakta
3. Narkotik me doza te ulta dhe me veprim te zgjatur
4. Oral steroid puls
5. Renia ng pesha

IX 63 . Nje burre 74 vjec po vleresohet nga mjeku I tij 6 jave pas nje ataku gutoz. Ai ka nje
histori te meparshme te gutes te ngjashme me 2 episode ne 6 muajt e fundit .Historia e tij
mjekesore tregon IKK, hiperkolesterolemia, dhe semundje renale kronike stadi 3 . Ai eshte
duke mar pravastatin , aspirin , furosemide, metolazone , lisinopril, dhe metoprolol XL.
Filtrimi glomerular eshte 38 mL/ min, kreatinina 2.2 mg/ dl , dhe niveli I acidit uric 9.3 mg
/dl . Ai eshte duke pyetur nese eshte ndonje terapi qe mund te lehtesoje ataket e tij te gutes .
Cili nga regjimet eshte me e pershtatme per trajtimin e ketij pacienti :
A Allopurinol 800 mg ne dite. B. Kolkicine 0.6 mg dy here ne dite C Febuxostat 40 mg
ne dite. D Indometacine 25 mg 2 here ne dite. E Probendecid 250 mg dy here ne dite.

IX 64. Nje burre 64 vjec prezantohet me IKK ne urgjence me dhimbje acute te rende ne
kemben djathte .Dhimbja filloi gjate nates dhe e zgjoi nga gjumi I thelle .E pershkruan
dhimbjen e tij te aq te rende saqe nuk mund te vishte dot corapen dhe kepucen per ne
urgjence . Mjekimi aktual permban furosemide 40 mg dy here ne dite , Karvedilol 6.25 mg dy
here ne dite , candesartan 8 mg nje here ne dite , aspirin 325 mg nje here ne dite .Ne
ekzaminim ai eshte febril me temperature 38.5*C . Gishti I madh I kembes se djathte eshte
eritematoz dhe nuk I ndjeshem ne prekje . Ka nje enjtje sinjifikante dhe efuzion te artik
metatarsofalangeal ne kemben e djathte . NUk ka prekje te artikulacioneve te tjera. Cila nga
keto gjetje do te pritej ne artrocentezis : A.niveli I glukozes me I vogel se 25 mg/dl
B. Gram stain positive C. Prezenca e forte negative e kristaleve birefringent ne forme
gjilpere ne mikroskopine me drite te polarizuar
D. Prezenc + kristaleve ramdoidale birefringent ne mikroskopine me drite te polarizuar
E. Rruazat e bardha te gjakut me shume se 100.000 / mikroL

132
IX 65 . Nje grua 24 vjece vjen ne spital me simptomat e temperatures dhe gjurit te djathte te
enjtur dhe te dhimbshem . 3 Jave para kesaj pacientja ka patur shenja sistemike qe perfshin
temperature , ethe, dhimbje artikulare migratore qe prekte duart , kycet e duarve. gjunjet, ijet
dhe berrylat.Ne ate kohe ajo dalloi disa papula te vogla siper kraharrorit dhe ne duar.Te gjitha
keto symptoma u zhduken vet. Ajo punon si arreduese ambientesh dhe nuk tregon te jep
pickuar nga ndonje insekt . Mjekimi I vetem qe merr jane kontraceptivet oral.Eshte e
pamartuar dhe ka shume partnere seksual.Ne ekzaminim fizik ka temp 38.4 *C , frekuenca
kardiake 124 rrahje/min, frekuenca respiratore 24 frymarrje /min , dhe P.A 102/68 mmHg .
Gjuri I djathte eshte I skuqur , I nxehte , I enjtur, dhe ka dhimbje ne levizje. Artrocenteza
demonstron nje numer nje rruazave te bardha te gjakut 66.000 / mikroL, nga te cilat 90
neutrofile. NUk shihet asnje Kristal ose mikoorganizem. Cili nga opsionet ka me shume
mundesi te zbuloj diagnozen e sakte te pacientes: A.Kultura bakterore e cervixit
B.Kultura bakteriale e lengut synovial C.Kulturaegjakut
D. IgG kunder borrelia burgdorferi E. Faktori rheumatoid

IX 66. NJe grua 66 vjec me histrori te artritit reummatoid dhe atake frekuente te pseudogutes
ne e majte , vjen me ankesen e djeresitjes gjate nates dhe dhimbje prej 2 ditesh ne gjurin e
majte .Medikamentet qe mer jane methotrexate 15 mg ne jave , folat 1 mg cdo dite ,
prednisone 5 mg , dhe ibuprofen 800 mg tre here ne dite sipas nevojes . Ne ekzaminim fizik
,temepratura eshte 38.6 *C , frekuenca kardiake 110 rrahje /min, P.A 104 / 78 mmHg , sat O2
ne ajerin e dhomes 97 % . Gjuri I majte eshte I enjtur , skuqur , I nxehte ,dhe I dhimbshem .
Me flektimin ose ekstensionin 5 grade pacientja perjeton dhimbje te forta. Ka evidence te
deformimeve kronike te artikulacioneve ne duar, gjunje , dhe shpine . rruazat e bardha te
gjakut jane 16.700 / mikroL, me 95% neutrofile. Ekzaminimi I gjurit te majte tregon rruaza te
bardha 168.000/mikroL, me 99 % neutrofile dhe kristale diffuse ne forme gjilpere
birefringent prezente.Gram stain tregon per koke gram positive te rralla ne klustera.
Menaxhimi perfshin te gjitha t pervec :
1. Kultura e gjakut
2. Glukokortikoidet
3. Marja e lengur artikular me aspirim me gjipere
4. Consult me kirurgun ortoped
5. Vankomycin

IX 67 . Nje grua 42 vjec shkon te mjeku me ankesat e dhimbjes difuze dhe lodhjes . Ajo e ka
te veshtire te lokalizoj dhimbjen ne nje artikulacion te vecante por thote qe shpresh dhimbja
prek ekstremitetet e poshtme dhe te siperme , ne qafe dhe ije .Eshte dhimbje e madhe dhe e
cakton me noten 10 nga shkalla 10 . Ndjen qe kycet e saj jane te ngurte dhe nuk tregon ti kete
me keq ne mengjes.Dhimbja ka qene prezente keto 6 muajt e fundit , dhe eshte rritur ne
intensitet.Ka konsumuar ibuprofen dhe paracetamol pa ndonje efektivitet . Pacientja ankon se
kjo dhimbje po I pengon gjumin e qete dhe ja ka beret e veshtire perqendrimin .Ka munguar
disa dite ne punen e saj si kamariere dhe frikesohet se mund te humbas punen . Ka histori
mjekesore per depression dhe obezitet . Mjekohet aktualisht me VenlaFaxine 150 mg ne dite .
Ka histori mjekesore familjare te artiritit rheumatoid te e ema.PI nje pakete cigare ne dite
.BMI eshte 36kg/ m2. Shenjat vitale jane normale. Ekzaminimi I kyceve nuk tregon eriteme,
enjtje ose efuzione . Ka nje dhimbje difuze ne palpimin e muskujve suboksipital, ne piken e
mesit te kufiritt te siperm te muskulit trapezius, pergjate linjes kostokondrale te dyte , ne
epikondilet laterale , dhe ne mesin e indit dhjamor ne gju. Te gjithe rreth shkakut te
syndromes se dhimbjes difuze ne kete pacient jane te verteta pervec :

133
1. Disfunksioni konjitiv , crregullimet e gjumit , ankthi , dhe depresioni jane crregullime
komorbide neuropsikologjike
2. Dhimbja ne kete syndrome lidhet me rritjen e ndjeshmerise ndaj dhimbjes
3. Dhimbja ne kete syndrome shpesh lokalizohet ne artikulacione specifike .
4. Kjo syndrome ndodh ne 2-5 % te populates se pergjithshme por rritet ne prevalence
me 20 % ose me shume ne paciente me crregullime reumatoide inflamatore ose
degjenerative
5. Femrat jane 9 here me shume te predispozuara se burrat per kete syndrome .

IX 68 . Nje grua 36 vjec vjen me ankesen e dhimbjes difuze ne trup te lidhur me lodhjen ,
pagjumesine , dhe veshtiresine per tu perqendruar . E ka te veshtire ta lokalizoj kete dhimbje
dhe e vlerson ne shkallen 7-8 nga 10 e cila nuk lirohet nga perdorimi I AIJS. Ka nje historike
te gjate mjekesore per crregullim te ankthit te gjeneralizuar qe e trajton me sertraline 100 mg
cdo dite dhe klonazepam 1 mg dy here ne dite .Ne ekzaminim fizik ajo ka dhimbje ne palpim
ne disa pika muskuloskeletale. Analizat lab tregojne per gjak komplet normal , paneli
metabolic normal, dhe eritrosedimentim dhe factor rheumatoid normal. I vihet diagnoza e
fibromialgjise .TE gjitha t perdoren per trajtimin e fribromialgjise pervec :
A. Nje program ushtrimesh qe perfshine trajnimin e forces , ushtrime aerobike , dhe joga B.
Terapi konjitive behaviorale per pagjumsine
C. Milnacipran
D. Oxycodone
E. Pregabalin

IX 69 . Nje grua 53 vjec prezantohet me lodhje dhe dhimbje te gjeneralizuar e cila eshte
perkeqesuar keto 2 vitet e fundit . Ajo gjithashtu ankon per irritabilitet dhe gjume te keq dhe
eshte shqetesuar se mund te jete ne depression. Tregon se kohet e fundit eshte ndare nga
bashkeshorti dhe ka qene shume e stresuar ne pune . Cili nga elementet ne historikun dhe
ekzaminimin fizik te saj do te plotesonte kriteret e Anerican college of Reumatology per
diagnozen e fribromialgjise : A. Dhimbje diffuse kronike dhe probleme me gjumin
B. Dhimbje difuze pa ndonje etiologji dhe evidence te depresionit major
C. Depression major, stresore te jetes , dhimbje kronike dhe gjinia femerore
D. Depression major dhe dhimbje ne palpim ne 6 nga 18 pike te ndeshmerise
E. Dhimbje kronike e perhapur dhe dhimbje ne palpim ne 11 nga 18 tender point sites

IX . 70 . Nje burre 42 vjec ne ekzaminim fizik ka kete pamje te gishtave (foto, gishta tamburi
.Te gjitha gjndjet te lidhura me kete pamje pervec ;
1. Semundje pulmonare obstructive kronike
2. Semundje kardiake kongenitale cianotike
3. Fibrozecistike
4. Carcinoma hepatocelulare
5. Hipertiroidizm

IX. 71 . NJe grua 64 vjec shkon te mjeku per dhimbjen ne ije qe ka nje jave prezente .Ajo e
lokalizon kete dhimbje ne aspektin lateral te ijes se djathte dhe e preshkruan si dhimbje te
mprehte qe perkeqsohet me ecjen dhe e ka te veshtire te shtrihet nga ajo ane. Kjo dhimbje
filloi pasi ai mbolli kopeshtin . Ka nje histori mjekesore per obezitet , osteoartrit te gjunjeve ,
dhe hypertension .Mjekimet qe mer perfshijne losartan 50 mg cdo dite dhe
hydrochlorthiazide 25 mh cdo dite . Per dhimbjen ka mar iibuprofen 600 mg sipas nevojes me
nje lirim te lehte deri mesatare te dhimbjes.Ne ekzaminim fizik ajo nuk eshte febrile dhe
shenjat vitale I ka normale .Ne ekzaminimin e ijes dhimbja eshte e perforcuar ne rotacion

134
ekstern dhe abduksion te kembes. Palpimi direkt ne aspektin lateral te pjeses se siperme te
femurit afer artikulacionit koksofemural jep dhimbje . Diagnoza me e mundshme ?
A. Nekroze avaskulare e ijes B. Iliotibial band syndrome C. Meralgia paresthetica
D. Artriti septic
E. Bursitis trokanterik

IX 72 . Nje grua 32 vjec paraqitet me dhimbje ne gjurin e majte .Ajo merret me vrapime ne
distanca te gjata DHE aktualisht po pergatitet per nje maratone .Mesatarisht vrapon 30-40
milje ne jave . Aktualisht eshte duke perjetuar dhimbje ne aspektin lateral te gjurit te majte .
Ka nje ndjesi djegie ne aspektin lateral te kofshes se majte . Nuk raporton per demtime te
gjurit dhe nuk ndjere te kete qene I nxehte ose I enjtur . Pervec kesaj eshte njeri I
shendetsshem dhe nuk mer asnje medicament pervec suplementeve bimore .Ekzaminimi fizik
I gjurit tregon per nje pike ndjeshmeria ne kondilin femoral lateral qe behet me keq kur
flektohet gjuri .I kerkohet pacientes te qendroj me kemben e djathte me ijen dhe gjurin e
flektuar ne 90*. Kembsa e saj e majte eshte e extenduar ne ije dhe u ul ngadal ne adduksion
pas kembes tjeter duke I shkarktuar dhimbje ne gjurin e majte TE gjitha trajtimet mund te
rekomandohen per kete pacient pervec :
A.Zevendesimi I kepuceve te vrapimit per te siguruar perputhje te plote
B.Injeksione glukokortikoide qe lejoj pacienten te vazhdoj pergatitjen per maratone
C.Ibuprofen 600-800 mg cdo gjashte ore sipas nevojes per dhimbjen
D.Referim per fizioterapi
E.Referim per trajtim kirurgjikal nese terapia konsevative deshton .

IX 73 Nje grua 58 vjec ankon per dhimbje ne shpatulllen e djathte . Nuk kujton ndonje
demtim te meparshem por thekson se shpatulla eshte bere progresivisht me e ngurte gjate
muajve te fundit .Me perpara ka patur disa episode te bursitit ne shpatullen e djathte qe jane
trajtuar suksesshem me AIJS dhe injeksione steroidesh.Hisoria mjekesore e meparshme
tregon per dabet mellitus , per te cilen trajtohet me metformine dhe glyburide. Ne
ekzaminimin fizike shpatulla e djathte nuk eshte e ngrohte ose e skuqur por eshte e ndjeshme
ne prekje .Levizja pasive ose aktive eshte e limituar ne fleksion , ekstension, dhe abduksion .
Grafia shpatulles se djathte tregon osteopeni pa evidence te erozionit te artikulacionit dhe
osteofiteve . Cila eshte diagnoza me e mundshme ?
1. Kapsiliti adeziv
2. Nekrozaavaskulare
3. Tendinitis biciptal
4. Osteoarthritis
5. Rupture e manshetes rotatore

IX 74 . Nje grua 32 vjec prezantohet me dhimbje te gishtit te madh dhe te kycit ted ores e cila
ka ardhur duke u perkeqsuar gjate javeve te fundit .Ajo ka dhimbje kur shtyn gishtin e madh
kundrejt gishtave te tjere . Ajo eshte nene e re me nje femije 8 javesh . Ne ekzaminim fizike
ka enjtje te moderuar dhe ndjeshmeri ne procesin stiloid radial dhe dhimbja shtohet kur ajo
vendos gishtin e madh ne pellemben e dores dhe e terheq me gishtat e tjere . Manovra e
phalenit eshte negative . Cila patologji ka me shume mundesi te jete prezente :
1. Sindromi I tunelit karpal
2. De Quervain tenosinovitis
3. Artrit gutoz ne artikulacionin metakarpofalangeal
4. Fasciitis palmar
5. Artrit rheumatoid

135
Harrison edition 19

1. Te gjitha meposhte jane tipare specifike te sistemit imunitar te lindur pervec:


A. Tipar ekskluziv i gjallesave vertebrore
B. Qelizat e rendesishme perfshijne makrofaget dhe limfocitet natural killer
C. Nuk njeh molekulat ose mikrobet huaja beninje
D. Njohje nga molekulat strehuese te koduara ne genome
E. Njohja e faktoreve kryesore te virulences se mikrobeve por jo njohje e vet molekulave

2. Nje djale 29 vjec me dhimbje episodike abdominale dhe edeme te induktuara nga stresi te buzeve,
gjuhes dhe me raste laringut, i prirur te kete nivele funksionale te uleta ose absoluta te ciles protein ?
A. Inhibitorit C1 esteraze
B. C5A (kaskada e komplementit)
C. Ciklooksigjenaza
D. Imunoglobulina (Ig) E
E. Receptori i qelizes T, zinxhiri alfa

3. Cila nga thenie pershkruan me mire funksionin e proteinave te koduara nga gjenet e MHC I dhe II?
A. Aktivizojne sistemin komplementar
B. Lidhen me receptoret siperfaqesore te granulociteve dhe makrofageve per te nxitur fagocitozen
C. Lidhje jo specifike e antigenit per ta prezantuar ne qelizat T
D. Lidhje specifike e antigenit ne pergjigje te aktivizimit te qelizave B per te nxitur neutralizimin dhe
precipitimin
4. Te gjitha theniet e ne lidhje me crregullimet imunodeficente primare jane te verteta pervec:

A. Infeksionet e traktit respirator te siperm ose te poshtem sugjerojne nje pergjigje te demtuar te
antitrupave
B. Shumica diagnostikohen nga prezenca e infeksioneve te perseritura ose rralle severe
C. Infeksionet e perseritura ne saje te specieve Candida sugjerojn imunitet te demtuar qelizave T
D. Jane semundje tipike gjenetike me trashegimi Mendeliane
E. Ndersa perfshihen shumica aspekteve imunitetit, imunit lindur nuk ndikohet nga crregullime

5. Nje 19 vjecar paraqitet ne kliniken e universitetit duke u ankuar per lezione delikate, te dhimbshme
te lekures ne aksile. Ai referon se ka patur episode te tilla te ngjajshme pergjate jetes se tij per te cilat
ai merr antibiotike. Ne ekzaminimet e fundit laboratorike ai ka rezultuar kulture pozitive per Serratia
marcescens. Te gjitha te dhenat te mundshme jane te verteta, pervec:
A. Transplanti i qelizes burimore humare eshte kurues
B. Infeksionet me organizma katalaze-negative jane tipike
C. Perdorimi profilaktik I trimetoprin/sulfametoxazoleefektiv ne ulje riskut per infeksione bakteriale
D. Semundja shkaktohet nga prodhimi i demtuar i specive oksigjen reaktive ne fagolizozoma
E. Semundja transmetohet nga trashegimia recesive e lidhur me X

6. Nje burre 37 vjec eshte diagnostikuar se fundi me hipertension sistemik. Atij i eshte pershkruar si
monoterapi fillestare lisinoprili. 3 dite pasi ka nisur medikamentin ai referon se dora e djathte eshte e
fryre, ka ndjesi kruarje te lehte dhe shpimi. Me vone kishte fryrje edhe te buzeve dhe veshiresi ne
frymemarrje. Cila nga theniet pershkruan me mire situaten?
A. Simptomat e tij vijne ne saje te aktivizimit direkt te qelizave mast nga lisinoprili
B. Simptomat e tij vijne nga degradimi i bradikinines se demtuar nga lisinoprili
C. Simptomat e tij nuk do perseriten nese kalon te enalaprili
D. Analizat e gjakut periferik do tregojne deficitet e inhibitorit C1
E. Nivelet plazmatike te IgE ka gjase te jene te rritura

136
7. Nje vajze 28 vjecare kerkon vleresim nga mjeku I saj te episodeve te perseritura te urtikaries dhe
thote se eshte alergjike nga moti I ftohte. Ajo referon se per 10 vite ka episode te tilla urtikarie ne
krahe dhe kembe kur ekspozohet ndaj motit te ftohte. Me perpara nuk eshte paraqitur asnjehere te
mjeku per keto episode, por vitet e fundit jane bere me frekuente. Nuk identifikon faktore nxites te
tjere pervecse te ftohtit. Nuk ka histori per astme ose atopi. Mohon intolerancen ushqimore.
Medikamentet e vetme qe merr jane kontraceptivet orale, te cilat I ka marre prej 5 viteve. Ne
ekzaminim ajo zhvillon nje vije lineare ne parakrah pasi nxitet nga jashte. Kur vendos doren ne uje te
ftohte, ajo fryhet dhe merr ngjyre te kuqe. Pervec kesaj ka disa zona qe reagojne ne forme vijash te
kuqe ne krah, mbi zonen e ekspozimit. Cili eshte hapi tjeter i menaxhimit te kesaj paciente?
A. Vleresim per prezence te antitrupave antitiroglobuline dhe antimikrosomale
B. Kontrollo per nivelet e inhibitorit C1
C. Nderprit kontraceptivet orale
D. Trajto me cetirizine 10mg ne dite
E. Trajto me ciproheptadine 8mg ne dite

8. Nje vajze 23 vjecare kerkon vleresim per rinitin sezonal. Ajo referon se ka simptoma cdo vit ne
pranvere dhe vjeshte. Pergjate kesaj kohe, ajo zhvillon rinit, rrjedhje postnazale dhe kolle qe i
nderpret gjumin. Gjithashtu ka lotim ne sy dhe ndjesi kruajtje. Kur i shfaqen simptomat ajo merr
loratadine 10mg ne dite qe I lehtesojne simptomat. Cilet alergenet e mundshem i shkaktojne keto ?
A. Bari
B. Polenet
C. Pemet
D. A dhe B
E. B dhe C
F. Te gjitha

9. Nje djale 3 vjecar paraqitet ne urgjence. Ndersa po konsumonte vaktin e darkes atij papritur i nisi
kolla, wheezing dhe pergjigjej me pak progresivisht. Prinderit e tij jane te sigurt qe ai nuk ka aspiruar.
Ne urgjence, presioni i tij i gjakut eshte i ulet dhe merr fryme me shume veshtiresi. Ne auskultim
degjohet nje fishkellime e lehte bilaterale. Mjeket e diagnostikojne me anafilaksi dhe nisin terapine e
pershtatshme. Cila nga alternativat eshte e vertete ne lidhje me anafilaksine
A. Nje histori atopike eshte faktor risku per anafilaksi ndaj terapise me peniciline
B. Anafilaksia me shpesh nis 1-2 ore pas ekspozimit ndaj antigjenit
C. Moshat me te medha kane rezultate me te mira ne anafilaksi
D. Mosperdorimi i epinefrines ne 20 minutat e para simptom-faktor risku per vjekjen anafilaksia
E. Glukokortikoidet intravene jane efektive ne anafilaksine akute

10. Ethet reumatike zhvillohet ne saje te nje procesi autoimun. Cili nga mekanizmat e meposhtem te
imunitetit eshte pergjegjesi kryesor per zhvillimin e tyre
A. Abnormalitetet endokrine
B. Rritja e funksionit te qelizave B
C. Imbalance intrinseke citokinash
D. Rritja e qelizave T ne saje te stimulimit te citokinave
E. Mimika molekulare

11. Cila pershkruan mekanizmin autoimun patofiziologjik pergjegjes per semundjen Graves?
A. Citotoksicitet qelizor i varur nga antitrupat
B. Autoantitrupat te aktivizuara nga komplementi
C. Autoantitrupat inaktivizuese
D. Autoantitrupat stimuluese
E. Citotoksiciteti qelizor i ndermjetesuar nga qelizat T

137
12. Cili nga antitrupat e meposhtem eshte me pak i mundshem te jete prezent ne LES?
A. Anti- dsDNA
B. Antitrupat antinuklear
C. Anti-La (SS-B)
D. Antiphospholipidet
E. Antieritrocitet

13. Nje 23-vjecare eshte vleresuar nga mjeku sepse ajo eshte e shqetesuar se mos ka lupus
eritematosus pas nje lajmerimi te shendetit publik ne radio. Ajo nuk ka histori mjekesore sinjifikuese
ne te shkuaren dhe medikamenti i vetem qe ka marr me raste ka qene ibuprofeni. Nuk eshte aktive
seksualisht. Raporton se ka patur ulcera orale intermitente dhe dhimbje te gjurit te djathte.
Ekzaminimi fizik nuk tregon evidence per alopeci, rash cutan ose inflamacion te kyceve. Ekzaminimi i
gjakut tregon se ajo eshte pozitive per ANA (antitrupat antinuklear) ne nje titer 1:40, por jo
abnormalitete te tjera. Cila nga theniet eshte e sakte
A. Kerkohen 4 kritere per diagnozen e lupusit eritematoz dhe pacientja ka vetem 3
B. Kerkohen 4 kritere per diagnozen e lupusit eritematoz dhe pacientja ka 2
C. Nese analiza e urine stregon per proteinuri, ajo do plotesoj kriterin per lupusin eritematoz
D. Ajo ploteson kriteret per LES sepse ka vetem 3 kritere per semundjen
E. Prezenca vetem e ANA positive eshte e pamjaftueshme per te diagnostikuar lupusin eritematoz

14. Nje 32-vjecare e diagnostikuar gjate me lupus eritematoz vleresohet nga reumatologu i saj per
follow up. Degjohet nje murmur kardiake dhe eshte kerkuar nje ekokardiograme. Ajo ndihet mire, nuk
ka temperature, humbje peshe ose semundje kardiake preekzistuese. Demonstrohet nje vegjetacion
ne valvulen mitrale. Cila nga theniet eshte e vertet?
A. Kultura e gjakut nuk mund te jete pozitive
B. Eshte provuar se terapia me glukokortikoide ka dhene permiresim per kete gjendje
C. Perikarditi shpesh mund te gjendet njekohesisht
D. Pacientja mund te kete perdorur drogera ne fshehtesi

15. Nje 24 vjecare eshte diagnostikuar se fundi me lupus eritematoz. Cili sistem ka gjasa te pesoje
komplikacione pergjate gjithe jetes se saj?
A. Cardio-pulmonar B.Lekura C.Hematologjik D.Muskuloskeletal E.Renal

16. Nje grua afro-amerikane 45 vjec me lupus eritematosus paraqitet ne urgjence me ankesat e
dhimbjes se kokes dhe lodhjes. Manifestimi i pare i LES ka qene artralgjia, anamia hemolotike, rashi
malar dhe ulcerat orale dhe njihet te kete titer te larte te antitrupave te ADN. Momentalisht ajo merr
prednisone 5 mg ne dite dhe hidroksiklorokine 200mg ne dite. Ne urgjence ajo e kishte PA = 190/110
mmHg me nje ritem kardiak prej 98bpm. Analiza e urine stregon 25 RBC per fushe dhe 2+ proteinuri.
Nuk ka cilindra RBC. Azoti i urese ne gjak 88mg/dl dhe kreatinina 2.6mg/dl (norma 0.8mg/dl). Nuk ka
patur semundje renale te meparshme te lidhur me LES dhe nuk po merr barna AIJS. Ajo mohon ndonje
semundje te koheve te fundit, ulje te medikamentit oral ose diarre. Cili eshte hapi pasues me I
pershtatshem ne menaxhimin e ketij pacienti?

A. Te niset ciklofosfamid, 500mg/m2 per siperfaqe trupi IV dhe te perseritet cdo 3-6 muaj
B. Te niset hemodializa
C. Te niset terapia e steroideve me doza te larta (IV metilprednisolon, 1000mg ne dite per 3 doza,
e ndjekur nga prednizoni oral, 1 mg/kg ne dite) dhe mykofenolati, 2g ne dite
D. Te niset plazmafereza
E. Mbani gjithe teraine derisa te vije biopsia renale

138
17. Nje 27 vjecare eshte pranuar ne terapine intensive pas lindjes se fundit te nje foshnje ne term 3
dite me pare. Pacientja kishte hemipareze te djathte dhe dore te majte ngjyre blu. Ekzaminimi fizik
tregon per livedo retikularis. Ne analizat laboratorike u pane WBC 10.2 per mikroL, hematokriti 35%
dhe trombocitet 13000 per mikroliter. Azoti i urese (BUN) eshte 36 mg/dl dhe kreatinina 2.3 mg/dl.
Shtatzania ka qene e paqendrueshme dhe erdhi pas 3 humbjeve. Strisho periferike nuk tregon
evidence per schistocites(pjese e fragmentuar e nje eritrociti). Cili ekz konfirmoje etiologjine ?
A. Paneli i antitrupave antikardiolipin
B. Antitrupat antinukleare
C. Ekzaminimi doppler i krahut te majte te pemes arteriale
D. Ekokardiografi
E. MRI e trurit

18. Nje 28-vjecare vjen ne urgjence me ankesen e perkeqesimit prej 1 dite te dhimbjes ne kemben e
djathte si dhe fryrje te saj. Ajo i kishte dhene makines 8 ore pas nje udhetimi per hiking 2 dite me pare
dhe prej asaj I lindi dhimbja e kembes. Fillimisht mendoi se ishte ne saj te sforcimit, por u perkeqesua
gjate dites. Historia mjekesore e shkuar lidhet me veshtiresi ne te mbeturit shtatzane me 1 ose 2
aborte spontane. Ekzaminimi fizik paraqet shenjat vitale normale, po ashtu edhe zemra me
mushkerine. Kemba e saj e djathte eshte e fryre nga mesi i kofshes dhe poshte dhe eshte e delikate.
Ekzaminimi Doppler demonstron nje tromboze te gjere venoze te thelle ne ne venat ilake dhe
femorale qe perhapet ne pelvis. Ekzaminimet laboratorike tregojne elektrolite normale, WBC normale,
koha e protrombines normale dhe nje kohe tromboplastine parciale te aktivizuar 3X normal. Testi i
shtatzanise eshte negativ. Niset ne urgjence terapia me heparine me peshe te ulet molekulare. Cila do
ishte terapia pasuese?
A. Rituximab 375 mg/m2/ jave per 4 jave
B. Varfarine me INR 2.0-3.0 per 3 muaj
C. Varfarine me INR 2.0-3.0 per 12 muaj
D. Varfarine me INR 2.5-3.5 per gjithe jeten
E. Varfarine me INR 2.5-3.5 per 12 muaj e ndjekur nga aspirine ditore gjithe jeten

19. Pacientet me sindromen antifosfolipid shpesh testojne ne menyre te gabuar pozitiv per cilen nga
semundjet infektuese?
A. Malaria
B. HIV
C. Schistosomiasis
D. Hepatiti C
E. Sifiliz

20. Cila eshte vendi me i shpeshte i perfshirjes se kyceve ne arteritin reumatoid?


A. Artikulacioni distal interfalangeal
B. Pelvisi
C. Gjuri
D. Shpina
E. Dora

21. Ne pacientet me artrit reumatoid te vendosur, te gjitha gjetjet radiografike pulmonare mund te
shpjegohen nga gjendja reumatologjike, pervec?
A. Infiltrate itersticiale bilaterale
B. Bronkiektasis
C. Infiltrat lobar
D. Nodul pulmonar solitar
E. Efuzion pleural unilateral

139
22. Cila nga alternativat e eshte gjetja radiologjike me e hershme ne artritin reumatoid?
A. Osteopenia juksta-artikulare
B. Jo abnormalitet
C. Fryrje/zmadhim i indit te bute
D. Erozione subkondrale
E. Humbje simetrike e hapesires se kycit

23. Te gjitha jane manifestime karakteristike ekstra-artikulare te AR pervec?


A. Anemia
B. Vaskuliti kutan
C. Perikarditi
D. Sindroma Sjogren sekondare
E. Trombocitopenia

24. Te gjithe agjentet e meposhtem kane patur efikasitet si medikamente antireumatoide semundje-
modifikuese pervec?
A. Infliximabit
B. Leflunomide
C. Metotrexate
D. Naproxen
E. Rituximab

25. Te gjitha jane karakteristike e sindromes Felty (AR + splenomegaly + neutropeni) pervec?
A. Neutropenia
B. Artriti rheumatoid nodular
C. Shfaqje ne stadet e vona te AR
D. Splenomegali
E. Trombocitopeni

26. Kundrejt cilave prej malinjiteteve jane pacientet me AR me shume te riskuar?


A. Kanceri I kolonit
B. Kanceri I pulmoneve
C. Limfoma
D. Melanoma
E. Glioblastoma multiforme

27. Cili eshte prezantimi klinik me I shpeshte I etheve reumatike akute


A. Karditi
B. Korea
C. Eritema marginatum
D. Poliarteriti
E. Nodujt subkutane

28. Nje 19 vjecare nga Etiopia vjen ne klinike. Se fundi ndihet mire. Historia e kaluar mjekesore flet per
nje dergim ne spital per fibrilacion atrial. Kur ishte femije ne etiopi, ka zhvilluar nje semundja qe i
shkaktoi levizje te pakontrolluara te gjymtyreve dhe gjuhes, qe zgjaten afersisht 1 muaj. Ka patur
gjithashtu edhe 3 episode te atritit migrator ne artikulacione te medha pergjate adoleshences qe
qetesoheshin me medikamente qe merrte ne farmaci. Momentalisht merr metoprolol dhe varfarine
dhe nuk ka alergji te njohur nga medikamentet. Ekaminimi fizik tregon nje ritem te parregullt kardiak
me PA normal. Ne apeks degjohet nje murmur holosistolike dhe zhurme e hershme diastolike. Nje
murmur e bute diastolike e hershme degjohet edhe ne hapesiren interkostale te trete majtas. Ju e

140
referoni ate ne nje kardiolog per vleresim per zevendesimin e valvules dhe ekokardiografi. Cfare
nderhyrje te tjera do te konsideronit ju ne kete kohe?
A. Aspirine ditore
B. Doxycikline ditore
C. Korikosteroide me doze te ulet
D. Injeksione te permuajshme te penicilines G
E. Injeksione te penicilines G per gjithe dhimbjet e fytit

29. Manifestimet me te shumta te ethevre reumatike akute shfaqen afersisht per 3 jave pas infeskionit
me streptokok gr A eshte manifestimi I etheve reumatizmale qe paraqitet disa muaj pas infeksionit?
A. Korea
B. Eritema marginatum
C. Ethe
D. Poliartriti
E. Nodujt subcutane

30. Nje pacient me diagnoze te sklerodermise I cili ka perfshirje kutane difuze shfaq hypertension
malinj, oliguri, edema, anemi hemolitike dhe insuficence renale. Ju beni nje diagnoze te krizes renale
nga sklerodermia. Cili eshte trajtimi i rekomanduar?
A. Kaptoprili B.Carvediloli C.Klonidina D.Diltiazemi E.Nitroprusidi

31. Cila nga alternativat eshte afersisht 2 here mbi normalen ne pacientet me sklerosis sistemike
cutane difuze sesa sklerosis sistemike kutane te limituar?
A. Perfshirje ezofageale
B. Hipertension arterial pulmonar
C. Fibroze pulonare
D. Fenomeni Raynaud
E. Perfshirje e lekures

32. Cilet nga autoantitruoat e meposhtem eshte prezent ne titra te larte ne paciente me semundje
mikse te indit lidhor?
A. Anti-centromere
B. Anti La
C. Anti Ro
D. Anti Scl 70
E. Anti U1 RNP

33. Nje grua 57 vjecare me depresion dhe dhimbje migrene kronike raporton ndjesine e gojes dhe syve
te thate prej vitesh. Ankesa kryesore eshte qe nuk mund te konsumoje me ushqimin si zakonisht edhe
pse ne pyetjet e metejshme raporton per fotosensitivitet dhe djegie te syve. Nuk ka simptoma te tjera
shoqeruese. Ekzaminimi tregon mukoze orale te thate, eritematoze, ngjitese. Te gjitha testet vijuese
jane te prirura te jene pozitive pervecse?
A. Antitrupat La/SS-B
B. Antitrupat Ro/SS-A
C. Testi Schirmer I
D. Antitrupi Acl-70
E. Sialometria

34. Nje pacient me sindrom primar Sjogren qe eshte diagnostikuar para 6 vitesh, qe qetesohet me
terapine e loteve zevendesues, shprehet per vazhdim te fryrjes se parotideve prej 3 muajve te fundit.

141
Shprehet per zmadhim te limfonodulave cervikale posteriore. Ekzaminimet vleresuese tregojne per
leukopeni dhe nivele te uleta te complementit C4. Cila eshte diagnoza e mundshme?
A. Amiloidosis
B. Pankreatiti kronik
C. Infeksioni me HIV
D. Sindroma Sjogren sekondare

35. Cili eshte manifestimi me i zakonshem ekstraglandular i sindromes Sjogren primare?


A. Artralgjia/artriti
B. Limfoma
C. Neuropatia periferike
D. Fenomeni Raynaud
E. Vaskulitis

36. Nje grua japoneze 43 vjecare prezantohet ne klinike me sy te thate te irrituar, goje te thate dhe
fryrje te faqeve prej 6 muajsh. Ne ekzaminime ajo ka zmadhim bilateral te gjendrave parotide.
Sialometria eshte abnormale. Biopsia e gjendrave salivare minor ne buze tregon inflamacion
granulomatoz. Serologjia tregon antitrupa negativ SS-A dhe SS-B. Cila eshte diagnoza e mundshme?
A. Sklerosis sistemike
B. Sarkoidosis
C. Sindroma Sjogren
D. HIV- shoqerues i sicca syndrome
E. Granulomatoza eozinofilike me poliangitis

37. Ne cfare perqindje eshte prezent antigeni i histokompatibilitetit HLA-B27 ne pacientet e Amerikes
se Veriut me spondilit ankilozant?
A. 10%
B. 30%
C. 50%
D. 90%
E. 100%

38. Cili eshte manifestimi me i zakonshem i spondilitit ankilozant?


A. Uveiti anterior
B. Insuficenca aortike
C. Semundja inflamatore bower
D. Fibroza pulmonare
E. Blloku i zemres i grades se trete

39. Nje pacient 25 vjecar paraqitet ne klinike per vleresim te dhimbjes se fundshpines. Dhimbja eshte
e rende, permiresohet me ushtrime, perkeqesohet ne mengjes dhe ne gjendje qetesie, kryesisht gjate
gjumit te nates. Ai ndihet shume i ngurte ne mengjes per se paku 30 minuta. MRI e fundshpines tregon
inflamacion aktiv ne artikulacionin sacro-iliac. Ai raporton histori per skuqje syri unilaterale e trajtuar
me kortikosteroide per 2 vite. Testi per HLA-B27 +. Cila eshte terapia e linjes se pare per kete gjendje?
A. Infliximab
B. Naproxen
C. Prednisone
D. Rituximab
E. Tramadol

142
40. Nje 27 vjecar paraqitet ne klinike per vleresim te artritit te dhimbshem qe perfshin gjurin e djathte
shoqeruar me fryrje bilaterale difuze te gishtit. Ai eshte kryesisht i shendetshem, por shprehet per nje
periudhe te rende diarreike rreth 3-4 jave para sesa te zhdukej spontanisht. Ne vendin e tij te punes
ai referon se edhe persona te tjere kane episode diarreike te ngjajshme. Ai nuk merr medikamente
dhe raporton per perdorim te marjuanes. Shprehet per urinim te dhimbshem. Ekzaminimi tregon per
atrit inflamator i gjurit te djathte, daktilitis dhe ekzaminim genitourinary normal. Ai diagnostikohet me
artrit reaktiv. Cili eshte agjenti etiologjik I mundshem I diarrese se pacientit
A. Campylobacter jejuni
B. Clostridium difficile
C. Escherichia coli
D. Helicobacter pylori
E. Shigella flexneri

41. Cila nga theniet e lidhura me artritine Whipple eshte e vertete?


A. Artriti eshte gjetje e rralle ne semundjen Whipple
B. Manifestimet e artikulacioneve zakonisht jane te njekohshme me simptomat gastrointestinale
dhe malabsorbimin
C. Radiografia shpesh tregon erozione te artikulacioneve
D. Ekzaminimi i likidit sinovial nuk ka shumegjasa te tregoje qeliza polimorfonukleare
E. Asnje nga te mesipermet

42. Cili nga perkufizimet eshte me i sakte per enthesitis?


A. Nje ndjesi vibruese ose kercitese e palpueshme e shkaktuar nga levizja e artikulacionit
B. Ndryshimi i shtrirjes se artikulacioneve ne menyre qe siperfaqet artikuluese te perafrohen ne
menyre jo te plote me njera-tjetren
C. Inflamacioni ne vendin e inserimit te tendines ose ligamentit ne kocke
D. Inflamacioni i membranes periartikulare perreth kapsules se artikulacionit
E. Inflamacioni i nje kaviteti si qese afer artikulacionit qe ul ferkimin

43. Alternativat qe bejne dallimi i artritit psoriatik nga crregullimet artikulare te tjera pervec?
A. Daktiliti
B. Enthesitis
C. Thonj me gropeza
D. Prezence te diarrese
E. Shkutim i gishtave

44. Cili lezion valvular kardiak eshte me i shpeshte ne pacientet me spondilit ankilozant?
A. Regurgitacion aortik
B. Regurgitacion mitral
C. Stenoze mitrale
D. Stenoze pulmonare
E. Regurgitacion trikuspidal

45. Te gjithe sindromat vaskulite ne saje te depozitimeve te komplekseve imune pervec?


A. Vaskulitit crioglobulinemik
B. Granulomatosis me poliangiitis
C. Purpura Hench-Schonlein
D. Poliarteriti nodoz i shoqeruar me hepatitin B
E. Semundja e serumit (serum sickness)

143
46. Nje 40 vjecar paraqitet ne urgjence me hemoptizi me volum te ulet prej 2 ditesh qe del me kollen.
Ai raporton dhimbje kraharori, temperature subfebrile dhe humbje peshe. Pacienti thote se prej nje
viti ka pasur epistaksis te shpeshta dhe rrjedhje purulente qe i ka trajtuar me disa antibiotike. Kryesisht
eshte i shendetshem por ka nje hiperlipidemi te lehte. Medikamentet qe merr jane aspirina dhe
lovastatina. Ne ekzaminimin fizik ka parametra vitale normale, pulmone te qarte dhe ne rruget e
siperme respiratore verehet nje deformim te septumit te hundes. Ne CT te kraharorit shikohen disa
nodule kavitare. Ne analizen e urines shikohen RBC. Cili ekzaminim do I pershtatet me shume per tiu
afruar diagnoses ne kete rast?
A. Biopsi e thelle e lekures
B. Biopsi perkutane e veshkave
C. Angiograme pulmonare
D. Biopsi kirurgjikale e pulmoneve
E. Biopsi e rrugeve te siperme

47. Nje 84 vjecare vleresohet nga mjeku I saj per dhimbjet e renda te kokes.
Ajo raporton se dhimbjet i jane perkeqsuar ditet e fundit. Nuk ka patur aura visuale, por eshte e bindur
se i eshte ulur shikimi ne syrin e majte keto dite. Ka djersitje naten dhe dhimbje te lehte shpine ne
mengjes. Mohon per dobesi ose mpirje, por thote se ka dhimbje te nofulles teksa konsumon ushqim.
Historia e shkuar mjekesore perfshin semundje koronare e pasuar me bypass para 10 vitesh, diabet
mellitus, hiperlipidemia dhe depresion i lehte. Cili eshte hapi pasues me i pershtatshem ne kete rast?
A. Apirine 975 mg orale ne dite
B. Matja e ritmit te eritosedimentimit
C. Te nisen menjehere glukokortikoidet
D. Biopsi e arteries temporale
E. Ultrasound i arteries temporale

48. Nje burre 54 vjecar vleresohet per vaskulit kutan dhe neuropati periferike. Per shkak te
disfunksionit renal, atij i behet nje biopsi renale, qe tregon glomerulonefrit. Ne gjakun periferik kemi
demonstrim te krioglobuinave. Cili test laboratorik do percaktoje etiologjine ne kete rast?
A. Antigjeni siperfaqesor i hepatitit B
B. Antitrupi citoplazmik antinutrofilik
C. Reaksioni zinxhir i hepatit C polimeraze
D. Antitrupi i HIV
E. Faktori reumatoid

49. Nje djale 18 vjec eshte derguar ne spital per fillim akut te dhimbjes se kraharorit substernale para
30 minutash. Ai raporton qe dhimbja i perhapet ne qafe dhe krahun e djathte. Ne ekzaminimin fizik
paraqitet diaforetik dhe takipneik. Presionii i gjakut eshte 100/48 mm Hg dhe FC 110 bpm. Ne
ekzaminimin kardiovaskular ka ritem te rregullt por eshte takikardik. Nje apeks degjohet nje murmur
holosistolike qe perhapet ne apeks. Pulmonet kane rale bilaterale ne baza. EKG tregon per rritje te ST
me 4 mm ne lidhjet anteriore. Ne historine e shkuar mjekesore ai raporton se ka qene i shktruar ne
spital per disa probleme me zemren kur ka qene 2 vjec. Mamaja e tij thote qe ka marr si trajtim aspirine
dhe gama globuline. Qe nga ajo kohe i eshte kerkuar follow up me ekokardiografi. Cili eshte shkaku
me I mundshem I sindromes koronare akute te pacientit?
A. Diseksioni i rrenjes se aortes dhe ostia koronare e majte
B. Prezenca e nje ure miokardiale qe mbingarkon arterien e majte anteriore descendente
C. Tromboza e nje aneurizme te a koronare
D. Vazospazme nga gelltitja e kokaines
E. Vaskulit qe perfshin arterien e majte anteriore descendente

144
50. Nje burre 46 vjec paraqitet per vleresim te gjendjes ne spital. Para 6 muajsh u paraqit me hemoptizi
akute, infiltrate pulmonare nodulare difuze dhe glomerulonefrit. Ne ekzaminimet serologjike doli
pozitiv per antitupat kunder citoplazmike ANCA dhe u diagnostikua eventualisht me granulomatoze
me poliangiitis. Trajtimi iu nis me glukokortikoide me doza te larta dhe ciklofosfamide ditore, ndaj te
cilave pati nje reagim klinik te shkleqyer. Tani duhet bere kalimi nga trajtimi i induksionit me
ciklofosfamide ne trajtimin mbajtes me azatioprine. Cilin test gjaku duhet te kontrolloni perpara sesa
te nisni azatioprine?
A. Titrat e ANCA
B. Krioglobulinat
C. Genotipin CYP3A4
D. Nivelet e enzimes glukoze 6 fosfoat dehidrogjenaze
E. Aktivitetin e enzimes tiopurine metiltransferaze

51. Te gjitha shtratet vaskulare preken tipikisht nga poliarteriti nodoz pervec?
A. Arterieve cerebrale
B. Arterieve koronare
C. Arterieve pulmonare
D. Arterieve renale
E. Arterieve splanknike

52. Ne cilen sindrome vaskuliti bopsia pulmonare ka rendimentin diagnostik me te mire?


A. Vaskuliti krioglobulinemik
B. Vaskuliti cutaneus
C. Poliangiitis me granulomatosis (Wegener)
D. Vaskuliti IgA (Henoch-Scho:nlein)
E. Poliarteriti nodoz

53. Ne nje pacient suspekt granulomatoze me poliangiitis (Wegener), cila gjetje radiologjike
pulmonare eshte me pak e mundshme?
A. Bronkektazia
B. Stenoza endobronkiale
C. Nodulet kavitare multiple
D. Infiltratet nodulare
E. Noduli kavitar solitar

54. Cila nga alternativat kerkohet per diagnozen e semundjes Behcet?


A. Vaskuliti i vazave te medha
B. Pathergy test
C. Ulcer orale rekurente
D. Ulcer genitale rekurente
E. Uveiti

55. Nje 25 vjecare ankon per ulcerime te dhimbshme orale. Ajo i pershkruan lezionet si ulcera te ceketa
qe zgjasin 1-2 jave. Ulcerat jane shfaqur 6 muajt e fundit. Prej 2 ditesh pacientja ka patur skuqje te
dhimbshme te syve. Nuk ka ulcerime genitale, artrit, rash kutan ose ndjeshmeri ndaj drites. Ne
ekzaminimin fizik pacientja paraqitet ne gjende te mire. Temperatura eshte 37.6 C, FC 86 bpm, PA
126/72 mmHg, FR 16 frymemarrje/min. ekzaminimi i mukozes orale tregon per 2 ulcera te cekelta me
baze te verdhe ne mukozen bukale. Ekzaminimi oftalmologjik paraqet uveit anterior. Ekzaminimi
kardiopulmonar eshte normal. Nuk ka artrit por ne kofshen e djathe ka nje korde te palpueshme ne
venen safene. Testet laboratorike tregojne shkallen e ERS 68 sec. WBC 10.230/mikro L me 68% qeliza

145
polimorfonukleare, 28% limfocite dhe 4% monocite. Antitrupat antinuklear dhe anti- dsDNA jane
negative. C3 eshte 89mg/dl DHE c4 24mg/dL. Cila eshte diagnoza e mundshme?
A. Sindroma Behcet
B. Pemfigoidi buloz
C. Lupus eritematoz diskoid
D. Sindroma Sjogren
E. Lupusi eritematoz sistemik

56. Te gjitha njihen si komplikacione te sindromes Behcet pervec?


A. Trombozes arteriale
B. Perfshirje e SNQ
C. Tromboze e venave te thella
D. Vaskulit i arteries pulmonare
E. Te gjithe me siper

57. Rritja ne serum e ciles enzime eshte treguesi me sensitiv i miozitit?


A. Aldolaze
B. Kreatinine kinaze
C. Transaminaza oksaloacetik glutamike
D. Glutamat piruvat transaminaze
E. Laktat dehidrogjeza

58. Nje grua 64 vjecare vleresohet per gjendje dobesie. Ka patur veshtiresi ne larjen e dhembeve dhe
kujdesit per floket javet e fundit. Ka vene re nje rash nje fytyre. Ne ekzaminim dallohet rash heliotrop
dhe dobesi te muskujve proksimal. Ketinine kinaza e serumit eshte e rritur dhe pacientja
diagnostikohet me dermatomiozit. Pas vleresimit nga reumatologu, u gjet antitrupo anti-Jo-1. Cfare
mund te kete tjeter pacietja?
A. Spondilit ankiloz
B. Semundjen inflamatore te zorreve
C. Semundjen intersticiale te pulmoneve
D. Cirroze primare biliare
E. Psoriasis

59. Nje grua 63 vjecare vleresohet per rash ne syte dhe lodhje prej 1 muajsh. Ajo raporton per ulje te
forces ne krahe dhe kembe, lodhje te vazhdueshme, por jo temperature apo djersitje. Raporton se ka
nje cngjyrim te kuq perreth syve. Pacientja vuan edhe nga hipotiroidizmi. Ne ekzaminim ka rash
heliotrop dhe dobesi te muskujve proximale. Eshte vendosur diagnoza e dermatomiozitit pasi u gjet
ne serum rritje e kreatinine kinazes e konfirmuar edhe me elektromiograme. Cili ekzaminim pasues
duhet te behet per te pare gjendje te tjera shoqeruese?
A. Mamografia
B. Matja e antitrupave antinukleare ne serum
C. Ekzaminim i feceve per veze dhe parazite
D. Imunoglobulinat tiroid-stimuluese
E. Titrat e antitrupave. Per varicela zoster

60. Pacienti i paraqitur per vleresim ne klinike ka nje histori te gjate mjekesore. Ai ka semundje te
arterieve koronare dhe ka vuajtur nga nje infarkt lateral miokardi prej 1 viti. Ne ate kohe ai nis
simvastatine, apirine, metoprolol dhe lisinopril. Para 2 muajsh ai nisi te ndjeje dhimbje te kofshes dhe
shpatullave. Me vone dhimbja u rrit dhe pacienti pershkruan edhe dobesi. CK eshte rritur 8x mbi limitin
e normes. 3 jave me pare ka nderprere simvastatinen. Sot raporton se dhimbja ka vazhduar dhe qe
cdo gje eshte me keq sesa 1 muaj me pare. CK eshte 12x me larte se norma. Cili test vendos diagnoz

146
A. Antitrupat kunder 3-hifroksi-3-metilglutaril coenzime A reduktaze (HMGCR)
B. Antitrupat antinukleare (ANA)
C. Antitrupi anti Jo 1
D. Antitrupi kunder pjeses njohes te sinjalit (SRP)
E. Nivelet e aldolazes

61. Nje burre 47 vjecar eshte vleresuar para 1 viti me enjtje bilaterale te vesheve. Veshet jane te
dhimbshem dhe i djathti fillon te behet i flashket. Gjendja e pergjithshme e pacientit eshte e mire dhe
nuk ka ankesa te tjera. Gjate ekzaminimit veshi i majte ka nje ngjyre te kuqe dhe kerci eshte i bute dhe
i fryre. Lobi I veshit duket minimalisht i fryre, por seshte i kuq. Shpjegimi I mundshem per kete rast?
A. Sindroma Behcet
B. Sindromi Cogan
C. Hemoglobinopatia
D. Trauma e perseritur
E. Polikondriti i perseritur

62. Te gjitha perfshihen neopatogjenezen e propozuar te sarkoidozes me perjashtim te?


A. Ekspozim ndaj mykut
B. Ndjeshmeri gjenetike
C. Pergjigje imune ndaj proteinave mykobakteriale
D. Infeksioni me Propionibacterium acnes
E. Zgjerim malinj e qelizave T helper

63. Cila nga theniet ne lidhje me sarkoidozen pulmonare eshte e vertet?


A. Perfshirja pulmonare eshte manifestimi i dyte me i zakonshem i sarkoidozes, pas perfshir kutane
B. Semundja obstruktive eshte manifestim i rralle i sarkoidozes pulmonare
C. Hipertensioni pulmonar nuk i pergjigjet terapise ne pacientet me sarkoidoze
D. Prezenca e kolles kerkon vleresim te shpejte per shkak tjeter pulmonar pervec sarkoidozes

64. Nje burre 55 vjecar me histori per sarkoidoze, ka mbaruar mjekimin me prednisone para 2 muajsh
dhe pervec konstipacionit, ndihet mire. Paneli metabolik tregon nje kalcium 12.2mg/gL (normal
10.5mg/dL). Sarkoidoza shoqerohet me hiperkalcemi. Mekanizm lidhjes
A. Perfshirja granulomatoze direkte e skeletit aksial shkakton clirim te Ca nga kockat
B. Stimulim direkt i rritjes se thithjes se Ca intestinal
C. Rritje e prodhimit te hormonit paratiroid
D. Rritje e prodhimit te 1.25-dihidrovitamin D
E. Rritje e prodhimit te 25-hidroksivitamin D

65. Ne cilen popullate eshte sarkoidoza kardiake me e zakonshme?


A. Afro amerikane
B. Europiane lindore
C. Japoneze
D. Amerikane jugore
E. Australiane

66. Cili eshte manifestimi kardiak i sarkoidozes?


A. Kardiomiopatia e dilatuar
B. Blloku kardiak
C. Stenoza valvulare
D. Takiaritmite ventrikulare
E. Te gjitha me siper

147
67. Nje 34 vjecar prezantohet me pankreatit te pashpjegueshem para 2 javesh. Imazheria e pankreasit
tregon per zmadhim pankreatik difuz. Ai mohon te kete perdorur alkool dhe nuk kishte gure ne
koleciste. Gjithashtu ne ekz: zmadhim i glandulave lakrimale dhe submandibulare. Behet biopsia e
glandulave submandibulare dhe qelizat rezultojne + per IgG4, CD19, CD138. terapia e pershtatshme?
A. Talidomide dhe deksametazone
B. Imunoglobulina e citomegalovirus dhe ganciclovir
C. Kemioterapi sistemike
D. Prednisone
E. Anakinra

68. Nje 19 vjecare ankon per episode temperature te perseritura, prej vitesh ne nje mesatare 2-3
muajsh. Episodet jane te paparashikuara, por ajo mendon se shfaqen ne periudha te stresit psikologjik.
Cdo episod febril zgjat 2-3 dite. Ajo ka gjithashtu edhe episode te perseritur te dhimbjes abdominale.
Kulturat e gjakut te perseritura jane rezultuara negative edhe gjate episodeve febrile akute. Edhe CT
abdominal nuk tregon etiologji specifike per dhimbjen e saj. Pergjate nje episodi, iu nenshtrua nje
laparotomie eksplorative, e cila tregoi adezione peritoneale dhe eksudat peritoneal neutrofilike
sterile. Ajo raporton gjithashtu se edhe kur ben ushtrime, ajo ka nje dhimbje muskulore qe zgjat per
disa dite. Ekzaminimet serologjike rezultuan negative, perfshire edhe ato per antitrupa antinukleare.
Cila eshte diagnoza me e mundshme?
A. Ethe mediterrane familjare
B. Limfoma
C. Ethet e perseritura
D. Endokarditi bakterial subakut
E. Lupusi sistemik eritematoz

69. Nje 19 vjcare e diagnostikuar me ethe mediterrane familjare. Cili medikament redukton sulmet
dhe ndihmon ne parandalimin e zhvillimit te amiloidozes sistemike?
A. Colchicine
B. Ciklosporine
C. Diflunisal
D. Prednisone
E. Talidomide

70. Cili nga artikulacionet nuk preket nga osteoartriti?


A. Kolona cervikale
B. Artikulacionet interfalangeale distale
C. Pelvisi
D. Artikulacionet interfalangeale proksimale
E. Dora

71. Cila eshte e vertet per osteoartritin?


A. Gjate vendosjes se diagnozes per nje artikulacion te dyshuar per osteoartrit, MRI siguron
vleresim per shkaqe te tjera.
B. Humbja e kercit shkakton dhimbje ne saje te stimulimit te receptoreve direkt te dhimbjes ne kecin
e artikulacionit.
C. Osteoartrit eshte shkaku i dyte i artritit, pas artritit rheumatoid.
D. Qelizat e bardha te gjakut ne likidin sinovial jane zakonisht <1000 qeliza/mL ne osteoartrit.
E. Severiteti i ndryshimeve radiografike ne osteoartrit korrelon mire me simptomat.

148
72. Nje grua obeze 60 vjecare me osteoartrit bilateral ne artikulatio genus, ankon per dhimbje
shumicen e diteve te javes. Ajo ka ulur aktivitetin (ecen me pak) por pa sukses per simptomat. Te gjitha
terapite kane treguar efektivitet ne trajtimin e simptomave te osteoartritit pervec?
A. Acetaminofenit
B. Injeksionet intra-artikulare me steroid glukokortikoid
C. Glukosamine-konfroitine
D. Naproxen
E. Atroplasti totale e artikulacionit

73. Nje burre 72 vjec me histori hipertensioni nen hidroklortiazide, ankon per dhimbje akute
tortureuese te gjurit. Ne ekzaminim, gjuri eshte i ngrohte, lehtesisht eritematoz, i enjtur dhe i bute ne
prekje ose levisje pasive. Eshte bere ekzaminim mikroskopik i likidit te artikulacionit. Cili eshte
crregullimi metabolik i mundshem ne kete rast?
A. Infeksion akut bakterial i artikulacionit
B. Antitrupa ndaj antigjeneve antinukleare
C. Degjenerim i kartilagos hialine
D. Rritje e prodhiit e pirofosfatit inorganik
E. Mbiprodhim i acidit urik

74. Cili strategji dozimi me e pershtatshme per trajtimin me alopurinol ne diagnozen e artriti gouty
A. Perdorimi i alopurinolit dhe azatioprines se bashku
B. Dozimi i alopurinolit duhet rregulluar sipas funksionit hepatic
C. Alopurinoli duhet dozuar per te arritur nje nivel te acidit urik ne serum <6mg/dL
D. Alopurinoli duhet te shmanget kur pacientet marrin colchicine
E. Toksiciteti I alopurinolit eshte me I zakonshem ne paciente qe kane te shprehur HLA-B27

75. Nje grua 42 vjecare paraqitet ne spital me ankesat e dhimbjeve difuze dhe lodhjes. Ajo ka veshtiresi
ne lokalizimin e dhimbjeve ne nje artikulacion specifik, por thote qe i perfshin ekstremitetet e siperme
dhe te poshtme, qafen dhe pelvisin. Dhimbja eshte e hidhur dhe e forte 10/10. Ajo thote qe
artikulacionet e saj jane te ngurte por nuk raporton qe jane te perkeqesuara ne mengjes. Ajo ka
perdorur ibuprofen dhe acetaminofen pa permiresim sinjifikant. Pacietja thote qe dhimbja e pengon
te flejegjume qete dhe te perqendrohet. Ka histori per depresion dhe obezitet. Momentalisht merr
venlafaxine 150 mg ne dite. Ka histori familjare per artrit reumatoid nga nena e saj. Konsumon nje
pakete cigare ne dite. Ne ekzaminimin fizik, shenjat vitale jane normale. BMI 36 kg/m2. Ekzaminimi i
artikulacioneve nuk demonstron per eritema, fryrje ose efuzion.
Ka dhimbje difuze ne palpacion ne piken e inserimit te muskujve suboksipitale, ne buzen e siperme te
muskulit trapesius, ne epikondilet laterale etj. Te gjitha theniet per shkakun e dhimbjes difuze te
pacientes jane te verteta me perjashtim te?
A. Diskufunksion kognitiv, crregullim I gjumit, ankth, depression jane zakonisht gjendje
neuropsikologjike shoqeruese.
B. Dhimbja lidhet me rritjen e ndjeshmerise.
C. Dhimbja ne kete syndrome lokalizohet shpesh ne artikulacione specifike
D. Kjo sindrome eshte prezente ne 2-5% e popullates se pergjitheshme, por me prevalence te rritur
me 20% me shume ne pacientet me crregullime reumatologjike inflamatore ose degjenerative.
E. Femrat jane 9 here me te prirura sesa meshkujt te preken nga ky sindrom.

76. Nje 36 vjecare prezantohet ne klinike me dhimbje difuze ne gjithe trupin qe shoqerohet nga lodhja,
insomnia dhe veshtiresi ne perqendrim. Nuk e lokalizon dot drejt dhimbjen por e raporton ate si
dhimbje me intensitet te larte 7-8/10 dhe qe nuk qetesohet nga AIJS. Ka histori te gjate te
crregullimeve te ankthit dhe trajtohet me sertraline 100 mg ne dite si dhe clonazepam 1 mg 2 here ne
dite. Ne ekzaminim ka dhimbje ne palpimin e nje sere muskujve. Ne ekzaminimet laboratorike paraqet

149
gjak komplet, panel metabolik, ritem eritrosedimentimi dhe factor rheumatoid normal.
Diagnostikohet me fibromyalgia. Te gjitha trajtimet e rekomandohen per fibromyalgia pervec?
A. Program ushtrimesh qe perfshin stervitje per forcen, ushtrime aerobike dhe yoga
B. Terapia cognitive e sjelljes per insomine
C. Milnacipran
D. Oxycodone
E. Pregabalin

77. Nje grua 53 vjecare paraqitet ne klinike me ankesen e lodhjes dhe dhimbjes se gjeneralizuar qe
eshte perkeqesuar 2 vitet e fundit. Pershkruan gjithashtu irritabilitet, veshtiresi ne frymemarrje dhe
depresion. Thote qe eshte divorcuar nga i shoqi dhe ka stres ne pune. Cilat elemente te historise se
saj takojne kriteret e diagnostikimit te American College of Rheumatology per fibromialgji?
A. Dhimbje kronike difuze dhe gjume jo normal
B. Dhimbje difuze pa etiologji dhe te dhena per depresion madhor
C. Depresion madhor, elemente stresore ne jete, dhimbje kronike, seksi femer
D. Depresion madhor dhe dhimbje ne palpacion te 6-18 vende te inserimit te muskujve.
E. Dhimbje kronike e shperndare dhe dhimbje ne palpacion nga 11 ne 18 vende inserimi te muskujve.

78. Nje burre 42 vjec ka gishta si ne foto ne ekzaminim fizik.


Semundjet qe shoqerohen me kete gjetje jane te gjitha pervec?
A. Semundje pulmonare obstructive kronike
B. Semundje kongenitale cianotike e zemres
C. Fibrose kistike
D. Carcinoma hepatocelulare
E. Hipertiroidizem

79. Nje burre 52 vjec ankon per fillim te dhimbjeve ne nyjet e gishtit tregues dhe gishtave te mesit te
te dy duarve. Ne ekzaminim, dora e djathte dhe artikulacioni i trete metakarpofalangeal te te dy
duarve jane te fryra dhe te buta. Historia e kaluar mjekesore perfshin vetem hiperlipidemi te
kontrolluar me atorvastatine. Ne studimet laboratorike ka rritje te ferritines dhe mutacion te genit
HFE. Diagnostikohet me hemokromatoze. Cila lidhet me abnormalitet artikulacioneve eshte e sakte
A. Gishti i dyte dhe i trete jane ne menyre tipike te perfshire ne osteoartit.
B. Artropatia nuk ka gjasa te lidhet me hemokromatozen.
C. Artropatia mund te progredoje me flebotomi.
D. Artropatia ndodh ne me pak se 20% te pacienteve me hemokromatoze
E. Ne radiografi ka gjase te duken erozione te metakarpofalangealeve.

80. Nje grua 64 vjecare ankon per dhimbje pelvisi per rreth 1 jave. Ajo e lokalizon dhimbjen me aspekt
lateral djahtas dhe e konsideron ate si therese, qe perkeqesohet me levizjen dhe e gjen te veshtire te
qendroje ulur ne krah te djathte. Dhimbja filloi pas disa puneve ne kopsht. Ka histori mjekesore per
obezitet, osteoartrit te gjurit dhe hipertension. Mjekimet e saj perfshijne lorsartan 50 mg ne dite dhe
hidroklortiazide 25 mg ne dite. Per dhimbjen merr ibuprofen 600 mg. Ne ekzaminimin objektiv

150
paraqitet afebrile dhe parametrat vitale normale. Ne ekzaminim e pelvisit dhimbja fillon ne rotacion
extern te tij dhe ka rezistence ndaj abduksionit. Palpacioni ne aspektin lateral te porcionit te siperm
te femurit afer pelvisit riprodhon dhimbjen. Cila eshte diagnoza e mundshme per kete paciente?
A. Nekroza avaskulare e pelvisit
B. Sindroma e brezit iliotibial
C. Meralgia paraesthetica
D. Artrit septik
E. Bursit trokanterik

81. Nje 32 vjecare ankon per dhimbje te gjurit te majte. Asaj I pelqen vrapi dhe se fundi po stervitet
per nje maratone. Vrapon zakonisht 30-40 milje ne jave, por se fundi ka patur dhimbje te madhe te
gjurit te majte. Ka nje ndjesi djegese qe zgjerohet lateralisht edhe ne kofshe. Mohon per ndonje
demtim te gjurit dhe nuk e ndjen ate te ngrohte apo te fryer. Eshte ne gjendje te pergjithshme te mire
dhe nuk merr medikamente pervecse suplementeve bimore. Ekzaminimi fizik i gjurit zbulon
ndjeshmeri te pikes mbi kondilin lateral te femurit qe perkeqesohet me perkuljen e gjurit. Pacientit i
thuhet te shkrihet ne krahur e djathte me gjurin e djathte dhe pelvisin e kthyer ne 90 grade. Kemba e
saj e majte shtrihet poshte pelvisit dhe ulet ngadale ne aduksion prapa kembes poshte, duke
riprodhurar dhimbjen e gjurit te majte te pacientes. Trajtimi rekomandohen per pacienten pervec?
A. Vleresim i kepuceve te vrapimit te pacientes per te siguruar nje pershtatje me te mire
B. Injeksione glukokortikoide ne menyre te tille qe te mos interferoje me peratitjen e pacientes
per maratonen
C. Ibuprofen 600-800 mg cdo 6 ore per dhimbjen
D. Te referohet per terapi fizike
E. Referohet per kirurgji nese terapia konservative deshton

82. Nje grua 58 vjec ankohet per dhimbje ne shpatullen e djathte. Nuk shprehet mbi ndonje demtim
por raporton qe shpatulla per disa muaj eshte bere me e ngurte. Me pare ka patur episode te bursitis
ne shpatullen e djathte qe jane trajtuar ne menyre te suksesshme me AIJS dhe injeksione steroidesh.
Pacietja eshte diabetike, per te cilen merr metformine dhe gliburide. Ne ekzaminimin fizik, shpatulla
e djathte nuk eshte e ngrohte ose e kuqe, por e bute ne prekje. Levizjet pasive ose aktive jane te
limituara ne fleksion, ekstension dhe abduksion. Radiograma e shpatulles se djathte tregon osteopeni
pa te dhena per erozione ose osteofite te artikulacionit. Cila eshte diagnoza e mundshme e ketij rasti?
A. Kapsulit adeziv
B. Nekroze avaskulare
C. Tendinit bicipital
D. Osteoartrit
E. Rotator cuff tear

83. Nje vajze 32 vjece ankon per dhimbje ne gishtin e madh te dores qe eshte perkeqesuar ne javet e
fundit. Ajo ka dhimbje kur pickon gishtat e tjere. Ne ekzaminimin fizik pacientja ka fryrje te lehte dhe
diskomfort mbi procesin stiloid radial dhe dhimbja nxitet kur ajo e vendos gishtin e madh ne pjesen e
brendshme te dores dhe e kap ate me gishtat e tjere. Manovra Phalen eshte negative. Cila eshte
gjendja e mundshme per kete rast?
A. Sindromi i tunelit karpal
B. Tendosinoviti De Quervain (inflamacioni i abduktor policis longus dhe ekstensor policis brevis
ne tendinen e procesit stiloid)
C. Artriti gouty i artikulacionit te pare metacarpofalangeal
D. Fascit palmar
E. Artrit reumatoid

151
84. Nje grua 42 vjec ankon per dhimje ne pjesen e poshtme te thembres se saj te djathte qe eshte
torturuese ne mengjes kur shkon nga shtrati per ne tualet. Dhimbja permiresohet disi gjate mengjesit
por perkeqesohet gjate dites kur ngjit shkallet. Ka histori te kaluar mjekesore per hipertension, pi
duhan nje pakete ne dite dhe punon si kamariere ne darke. Medikamentet qe merr jane
hidroklortiazidi dhe kontraceptivet orale. Ekzaminimi fizik eshte normal, pervec shputave te sheshta
dhe ndjeshmerise ne pjesen e poshtme te kembes se djathte. Nuk ka disfomfort ne kycin e kembes.
Diametrat e kembeve jane ekuivalente. Ne radiografi te thembres dhe kycit te djathte paraqiten
vetem spure. Te gjitha theniet jane te sakte pervec? (Diagnoza: fascit plantar)
A. Spuret e thembres nuk jane diagnostike
B. Injeksione lokale te glukokortikoideve kane nje risk per rupture plantare
C. Kontraceptivet orale dhe duhanpirja jane faktore riskante
D. Kepucet ortopedike mund te jene perfituese ne permiresimin e gjendjes
E. Prognoza per permiresim eshte e mire

Case files 85. Nje vajze 18 vjec ankon per nje dhimbje ne pjesen dorsale te dores se majte dhe te kycit
te djathte, temperature, rash pustular ne siperfaqen e jashtme te te dy parakraheve. Ka fryrje te lehte
dhe eritema te kycit si dhe dhimbje ne fleksionin pasiv te dores. Apirimi i likidit te kycit tregon 8000
polimorfonukleare per fushe po jo organizma ne strisho Grami. Cili eshte trajtimi me i mire?
A. Indometacin orale
B. Ampiciline intravenoze
C. Colchicine orale
D. Prednizon intraartikular
E. Ceftriakson intravenoz

86. Cili test diagnostik eshte i mundshem te jape diagnozen e sakte per rastin e mesiperm?
A. Analiza e kristaleve e likidit te artikulacionit
B. Kultura e likidit te artikulacionit
C. Kulture gjaku
D. Kulture cervikale

87. Nje burre 30 vjec ankon per fryrje dhe skuqje akute te gjurit. Aspirimi I artikulacionit tregon per
nje sere leukocitesh dhe leukocite polimorfonukleare, por jo organizma ne strisho Grami. Analizat
tregojne pak kristale bifrinues negativisht. Cili eshte trajtimi me i mire?
A. Kortikosteroidet orale
B. Kortikosteroidet intraartikulare
C. Terapi e antibiotikeve intravenoze
D. Colchicine orale

88. Nje burre 72 vjec ankon per dhimbje te rende dhe fryrje te te dy gjunjeve, pak kohe pasi iu
nenshtrua interventit per ripariminin e hernies abdominale. Ne ekzaminimin fizik gjunjte jane te
ngrohte, te fryre si dhe me efuzione. Artrocenteza e gjurit te djathte tregon prezencen e kristaleve
intra dhe ekstra celulare ne likidit sinovial. Strisho e Gram negative. diagnoza me e pershtatshme?
A. Gout
B. Artriti septik
C. Semundja e depozitimit te oksalatit te kalciumit
D. Artriti reaktiv
E. Pseudogout

89. Nje burre 65 vjec me histori per hipertension kronik, diabet mellitus dhe semundje degjenerative
te artikulacioneve prezantohet me fillim akut te nje dhimbje te rende te artikulacionit
metatarsofalangealeve dhe fryrje ne anen e majte. Ne ekzaminimin fizik dallohet nje diskomfort i lehte

152
i artikulacionit, e fryre, e ngrohte dhe me eriteme. Pacienti nuk ka histori per trauma ose probleme
mjekesore te tjera. Analiza e likidit sinovial dhe aspiracioni jane te prirura me teper per te treguar?
A. Likid hemorragjik
B. Kristale ne forme gjilpere, birefinues negativ
C. Organizma gram negative
D. Likid jo-inflamator
E. Kristale romboide, birefinuese pozitive

90. Nje adoleshent 17 vjecar, seksualisht aktiv, prezantohet me nje histori 5 ditore per temoerature,
ethe, dhimbje dhe fryrje te kycit te majte te kembes. Ne ekzaminimin fizik u pane lezione te lekures
makulopapulare dhe pustulare ne trup dhe ekstremitete. Ai mohon per simptoma te lidhura me
infeksione te traktit genitourinar. Analiza e likidit eshte e prirur te tregoje?
A. WBC 75000/MM3 ME 95% leukocite polimorfonukleare
B. RBC 100000/ mm3, WBC 1000/mm3
C. WBC 48000/mm3 me 80% limfocite
D. WBC 500/mm3 me 25% leukocite polimorfonukleare

91. Nje 22 vjecar ankon per dhimbje te fundshpines prej 3-4 muajsh dhe ngurtesi te zones lumbare, e
cila perkeqesohet me inaktivitetin. Ai raporton veshtiresi ne ngritjen nga krevati ne mengjes dhe duhet
te manovroje levizjet ne menyre qe te minimizoje dhimbjen. Nje grafi e pjeses lumbosakrale tregon?
A. Semundje artikulare degjenerative me formim spuriesh
B. Sacroileit me rritje te sklerozes perreth artikulacionit sacroiliac
C. Shkaterrim I trupave vertebrale me fraktura
D. Osteoporoze me fraktura te L3-L5
E. Osteonekroze difuze e kolones LS

92. Nje grua 36 vjec ankon per dhimbje ne duar dhe gjunj. Ajo eshte diagnostikuar me artrit reumatoid.
Cili nga trajtimet redukton inflamacionin e artikulacioneve dhe ul progresionin e semundjes?
A. NSAIDs
B. Aspirimi i artikulacioneve
C. Metotreksate
D. Kortikosteroidet sistemike

93. Cili me i mundshem per te qene kandidat per matjen e densitetit te mineraleve te kockave
A. Nje grua 65 vjecare, e dobet, e cila pi duhan dhe eshte 15 vite postmenopauze
B. Nje grua 40 vjec me cikel, qe ben ushtrime perdite
C. Nje grua 75 vjecare qe ben jete sedentare
D. Nje grua 60 vjec, mbi peshe, me origjine afro-amerikane
E. Nje grua astmatike 35 vjec, ka marr 40m dite prednizon 2 jave, 1 jave me perpara

94. Pergjate ciles periudhe ne jeten e nje femre, akumulohet pjesa me e madhe e mases kockore
A. 15-25 vjec B.25-35 vjec C.35-45 vjec D.45-55 vjec

95. Nje grua 60 vjec prezantohet me rezultatin e skanimit DEXA. Ajo ka shenuar -1.5 SD ne pelvis dhe
-2.5SD ne shpine. Cili eshte interpretimi i duhur i ketij rezultati?
A. Ajo ka osteoporoze te shpines dhe osteopenia te pelvisit
B. Ajo ka osteoporoze ne te dy zonat
C. Eshte ekzaminim normal
D. Ajo ka osteoporoze ne pelvis dhe osteopeni ne shpine
E. Duhet te dihet studimi i zones Z

153
96. Nje grua 70 vjec vjen ne klinike per check-up dhe skanimi DEXA per te matur denisitetin mineral te
kockave tregon nje T -2.5 SD ne shpine dhe -2.6 ne pelvis. Cili eshte interpretimi i duhur?
A. Kjo paciente ka osteopeni
B. Terapia e zevendesimit te estrogjenit duhet te nise me nje rindertim te parashikuar te mases
kockore gati brenda 1 viti
C. Noti do te ndihmoje ndertimin e e mases kockore
D. Bifosfonatet do te ulin riskun e frakturave te bacinit me 30-50%

97. Cila nga rastet ka prirje te shoqerohet me osteoartrit te avancuar?


A. Paaftesia me renie te perseritura dhe pamundesi per te jetuar vetem
B. Artikulacione me skuqje dhe efuzione
C. Te trajtuara me steroide orale (nuk ndihojne)
D. Permiresim gjate dites pas afersisht 1-2 oreve ne “unfreezing joint”

98. Cila semundje e pershkruan me mire nje devijacion ulnar bilateral simetrik ne te dy duart ne nje
grua 42 vjecare? Artriti reumatoid (jep devijacion ulnar te gishtave)

99. Cila semundje e pershkruan me mire ne nje mashkull 45 vjec, nje dhimbje dhe fryrje te
metatarsofalangut te madh (unilateral) me skuqje dhe nxehtesi lokale pasi hengri nje darke me biftek
dhe karkaleca’
- Artriti Gouty (kryesisht kap artikulacionin e pare metatatsofalangeal dhe precipiton pas
ushqimeve te ndryshme ose alkoolit)

100. Cila semundje e pershkruan me mire nje fillim akut unilateral me fryrje, ngrohtesi lokale,
diskomfort te articulatio cubitis dhe cervical discharge ne nje vajze 25 vjec?
- Artrit genococcal (cervical discharge + artikulacion I inflamuar)

101. Nje burre 72 vjec ankon per dhimbje ne artikulacionet e bacinit dhe gjunjeve, te cilat jane
diagnostikuar si osteoartrit. Cili eshte medikamenti me i mire per pacientin?
A. Naproxen sodium
B. Celecoxib
C. Prednizon oral
D. Prednizon intra-artikular
E. Acetaminofen (agjenti i pare i zgjedhur per trajtimin e osteoartriti te hershem)

102. Nje grua 35 vjecare, obeze ankon prej 1 jave per dhimbje ne fundshpine. Ekzaminimi nuk tregon
per ndonje simptome te rrezikshme dhe eshte komplet normal, pervecse peshes.hapi pasardhes
A. Doza te rregullta te NSAID dhe aktivitet fizik
B. 6 muaj qendrimi ne krevat
C. MRI e spines lumbare
D. Radiografi e spines lumbosakrale

103. Nje vajze 28 vjecare paraqitet me nje histori 6 mujore te dhimbjes persistente te fundshpines,
lumbare, e shoqeruar me temperature te lehte dhe djersitje naten. Ajo mohon te kete patur dobesi
te ekstremiteteve ose te kete faktore risku per e HIV. Ekzaminimi fizik eshte normal pervec
diskomfortit lumar L4-L5. Cila eshte diagnoza e mundshme
A. Osteomieliti i stafilokokus aureus
B. Osteomieliti tuberkular
C. Nga mosha, dhimbje e shpines idiopatike
D. Kancer metastatik i gjirit
E. Mieloma multiple

154
104. Nje grua 70 vjec paraqitet me nje histori te dhimbjes se shpines prej 4 javesh. Shoqeruar me
dobesi dhe humbje peshe 2 muajve te fundit. Ne ekzaminimin fizik, paraqitet normale. Ekzaminimi
laboratorik paraqet rritje te ritmit te sedimentimit, anemi te lehte, nivel kreatinine 1.8 mg/dL dhe
niveli i Ca 11.2 mg/dL. Cila eshte diagnoza e mundshme?
A. Osteoporosis me fraktura kompresive
B. Insuficence renale me osteodistrofi
C. Mieloma multiple
D. Tendosje lumbare (lumbar strain)
E. Osteomielitis

105. Nje burre 45 vjec ankon per ulje te ndjeshmerise ne bacin dhe pamundesi per te arritur ne
ereksion. Ne ekzaminim ka ulje te tonit te sphinkterit anal dhe ulje te reflekseve bilaterale te kycit te
kembes. Cili eshte hapi i rradhes i menaxhimit?
A. Qendrim ne shtrat dhe follow up per 4-6 jave
B. Radiografi e spines lumbosacrale
C. Ritem te sedimentimit dhe gjak komplet
D. Referim te menjehershmen per imazheri te avancuar dhe vleresim kirurgjikal

ENDOKRINOLOGJI HARRISON 19
X-1. Te gjitha hormonet prodhohen nga hipofiza anterior pervec:
A. Hormoni adrenocorticotrop B..Hormoni I rritjes
C. Oxytocina D. Prolaktina E. TSH

X-2. Nje burre 45 vjec tregon te mjeku qe gruaja e tij ka vene re ashprsim te tipareve te fytyres se
pacientit pergjate disa viteve.Vete pacienti raporton ulje te libidos dhe shterim energjie. Ekzaminimi
fizik tregon dalje e ballit (frontal bossing) dhe duar te zmadhuara.MRI konfirmon mase hipofizare. Cili
nga testet duhet kerkuar per te diagnostikuar shkakun e mases?
A. Kortisol I lire ne urinen 24 oreshe
B. Analize e Adrenocorticotropic hormone (ACTH)
C. Niveli I GH (Growth hormone)
D.Niveli I insulin-like growth factor-1 (IGF-1) serike
E. NIveli I prolaktines serike

X-3. Cila nga pohimet eshte e vertete per anatomine e hipofizes


A. Hormoni I rritjes derivon nga prekursori proopiomelanocortin (POMC).
B. Qelizat sekretuese te prolaktines perbejne pjesen me te madhe te qelizave te hipofizes anteriore.
C. Hipofiz ant sekreton hormone sintetizuar dprejtperdrejt ne qel neuroendokrine hypotalamike.
D.Gjendra hipofizare formohet embriologjikisht nga Rathke.
E. Hipofiza posteriore furnizohet me gjak nga 2 arterie

X-4. Nje burre 58 vjecar peson traume te rende kraniale dhe zhvillon insficience hipofizare. Pas
sherimit, ai merr hormone tiroideje, testosteron, glukokortikoide dhe vazopresine. Ne vizite rutine
pyet mjekun nqs ka mundesi te kete deficit te hormonit te rritjes. Te gjitha me poshte jane shenjat
apo simptomat e mungeses se hormonit te rritjes, PERVEC:
A. Profil anormal lipidesh
B. Ateroskleroza
C. Densitet mineral kockor I rritur
D. Rritje e raportit bel-ije (waist-hips)
E. Disfunksion I ventrikulit te majte.

155
X-5. Nje 75 vjecar paraqitet me obezitet abdominal, miopati proksimale dhe hiperpingmentim lekure.
Vleresimi laboratorik tregon alkaloze metabolike hipokalemike. Dyshohet per sindromen Cushing. Cili
nga pohimet eshte i vertete per kete sindrome?
A. Niveli basal I ACTH eshte I ulur.
B. Hormoni kortikotrop qarkullues eshte I rritur.
C. MRI hipofizare vizualizon te gjithe tumoret sekretuese te ACTH.
D. Referohet per marrjen urgjente te kampionit venoz petros inferior.
E. Niveli I K serik<3.3 mmol/L sugjeron per prodhim ektopik ACTH .

X-6? Cila nga t eshte e zakonshme ne pacientet me sindromen KALLMAN:


A. Anosmia
B. Nje baluke e thinjur/e bardhe
C. Pubertet I hershem te femrat
D. Sindaktilia te meshkujt
E. Obezitet hiperfagik

X-7. Nje vajze 22vjecare qe eshte e shendetshme I nenshtrohet nje lindje vaginale. Nje dite pas lindjes
ankon per crregullime pamore dhe dhimbje te forte koke. Pas 2 oresh ajo eshte e pavetedijshme dhe
thellesisht hipotensive. Ajo intubohet dhe eshte ne ventilim mekanik. Presioni arterial 68/28 mmHg,
frekuenca kardiake 148 bpm, saturimi oksigjenit 95%, FiO2 0.40. Ekzaminimi fizik nuk eshte tregues.
Testet laboratorike tregojne glukoze 49 mg/dL, hematokrit normal dhe numer normal te qelizave te
bardha te gjakut.Cila ka te ngjare ti ktheje hipotensionin?
A. Drotrecogin alfa I aktivizuar
B. Hidrokortisone
C. Piperacillin/tazobactam
D. Tiroksina (T4)
E. Transfuzioni eritrocitar

X-8. Po kujdesesh per Mr. Gelston, nje 19 vjecar qe me pare ka pasur tumor ne tru dhe iu nenshtrua
rrezatimit cranial. Ai eshte I shkurter dhe ska kaluar akoma ne pubertet. Ju dyshoni per insuficience
pituitare nga rrezatimi. Cila eshte e vertete per hipopituitarizmin nga rrezatimi?
A. Per nje doze 50 Gy rrezatim, vetem 5% e pacienteve manifestojne hipopituitarizem.
B. Shumica e pacienteve zhvilljne hypopituitarizem pas rrezatimit cranial brenda vitit 1 te trajtimit.
C. Growth hormone eshte hormoni deficitar me I shpeshte.
D. Nuk ekziston nje lidhje midis dozes se rrezatimit me zhvillimin e hypopituitarizmit.
E. Moshat e medha kane risk me te larte per te pasur hipopituitarizem nga rrezatimi.

X-9. Nje vajze 23vjecare shoqerohet ne qendren shendetesore te studenteve per menaxhimin
mjekesor te panhipopituitarizmit pas heqjes se nje kraniofaringeome ne femijeri. Ajo raporton qe
efekti I mjekimit aktual eshte I moderuar, por ndihet ne pergjithesi mire. TSH eshte ne kufinjte e
normes. Cili eshte veprimi I duhur?
A. Pergjysmo dozen e levotiroksines se pacientes.
B. Asgje
C. Kerko nivelin e T4 se lire
D. MRI koke
E. Kerko kontroll per rritjen e funksionit te tiroides

X-10.Nje paciente u vizitua ne urgjence para 1 jave per dhimbje koke, beri MRI koke ku nuk u gjet
ndonje shkak per simptomat e saj, por raporti final u mbyll me: sella turcika eshte bosh, beni
korrelacionin klinik. Pas largimit nga urgjenca pacientes iu rekomandua vizita te mjeku I saj sa me

156
shpejt. Ajo nuk ka as dhimbje koke as simptoma te tjera. Pas 1 dite vjen ne zyren tuaj e shqetesuar per
pergjigjen e MRI. Cili eshte hapi I radhes
A. Diagnostiko pacienten me panhypopituitarizem subklinik, dhe fillo zvd hormonal doza te uleta.
B. Mbajeni ne vezhgim dhe ndiqni rezultatet laboratorike.
C.Mbajeni ne vezhgim dhe kerkoni MRI pas 6 muajsh.
D. Mund te paraqese malinjitet endokrin, prandaj duhet (PET)/(CT) e gjithe trupit .
E. MRI tregon fillimin e nje adenoma prandaj duhet referuar te nje neurokirurg per rezekim.

X-11. Adenomat pituitare zhvillohen tipikisht ne drejtimin?


A. Anterior
B. Inferior
C. Lateral
D. Posterior
E. Superior

X-12. Ne MRI te hipofize cila nga gjetjet eshte anormale te te rriturit?


A. Nje aspekt konkav I siperm I hipofizes.
B. Intensitet I rritur I hipofizes posteriore
C. heterogjenitet I hipofizes anteriore
D. madhesia hipofizare 8–12 mm
E. Intensiteti me I ulet se indi trunor perreth ne T1, dhe me I rritur ne T2.

X-13. Mr. Jones ka adenoma hipofizare qe eshte zhvilluar superiorisht dhe ka komprimuar kiazmen
optike. Cili nga deficitet vizuale ka me shume mundesi te jete I pranishem?
A. deficiti inferior bilateral I fushes pamore
B. deficiti superior bilateral I fushes pamore
C. hemianopia bitemporale
D. skotoma centrale bilaterale
E. hemianopia homonime e djathte

X-14. Jane te pranishme ne sindromen Carney te gjitha pervec


A. Akromegalia
B. Adenoma adrenale
C. Miksoma atriale
D. Kardiomiopatia hipertrofike
E. Njolla ne lekure

X-15. Cila semundj eshte shkaku me I shpeshte I deficences mendore te parandalueshme ne bote?
A. Beriberi
B. Kretinizmi
C. deficenca e folateve
D. skorbuti
E. deficenca e vitamines A

X-16. Te gjitha kane rritje te T4 totale ne plazme, me T4 te lire normal pervec:


A. Cirroza
B. shtatzania
C. Euthyroid sick syndrome
D. hipertiroksinemia familiare disalbuminemike
E. Familial excess thyroid binding globulin

157
X-17. Cili eshte shkaku me i shpeshte e hipotiroidizmit?
A. semundja Graves
B. tiroiditi Hashimoto
C. hipotiroidizmi jatrogjenik
D. deficiti I jodit
E. ekspozimi ndaj rrezatimit

X-18. Nje 75-vjecare diagnostikohet me hipotiroidizem.Ajo ka semundje koronare prej nje kohe te
gjate dhe do te dije pasojat e hipotiroidizmit ne sistemin kardiovaskular. Cila eshte e vertete per
nderveprimin mes hipotiroidizmit dhe sistemit kardiovaskular:
A. Ne hipotiroidizem eshte pare ulje e hedhjes kardiake
B. Ne hipotiroidizem devijohet rrjedhja e gjakut drejt lekures.
C. Kontraktiliteti I miokardit rritet ne hipotiroidizem.(Harrison 19 kete ka te sakte)
D. Efuzioni pericardial eshte I rralle ne hipotiroidizem.
E. Rezistencat periferike ulen ne hipotiroidizem dhe mund te shoqerohen me hypotension.

X-19. A nje nene 38vjecare e 3 femijeve paraqitet me lodhje. Ajo referon qe ka 3 muaj qe ndihet jo
energjike. Ka qene e shendetshme dhe nuk merr asnje mjekim. Ka shtuar 10ib dhe ka kosntipacion te
rende prandaj ka marre laksative. TSH eshte 25 mU/L dhe T4 I lire eshte I ulur. Ajo ka hipotiroidizem.
Cili nga testet ndihmon ne diagnostikimin e etiologjise se hipotiroidizmit te saj?
A. Antitrupat antithyroid peroxidase (TPO)
B. Antitrupat antithyroglobulin
C. Radioiodine uptake scan
D. Niveli ne serum I tiroglobulines
E. Eko Tiroideje

X-20. Nje 54vjecare ka hipotiroidizem prej shume kohesh dhe vjen per rikontroll. Ka lodhje dhe
konstipacion. Nga kontrolli I fundit qe ka pasur, hiperkolesterolemia dhe hipertensioni jane stabel. Ajo
eshte diagnostikuar me fibromiome uterine dhe ka filluar merr Fe. Mjekimet e tjera perfshijne
levothyroxine, atorvastatin, dhe hydrochlorothiazid. TSH eshte 15 mU/L. Shkaku I rritjes se TSH eshte:
A. Celiakia
B. Kancer koloni
C. Mosrespektim i mjekimit
D. Absorbim I ulet i levothyroxines nga sulfati I Fe
E. Adenoma hipofizare TSH-sekretuese

X-21. Nje 87 vjecare sillet ne reanimacion me vetedije te ulur, hipotermi, bradikardi sinusale dhe
hipoglicemi. Zonja ka hipotiroidizem dhe hipertension sistemik. Familjaret referojne qe nuk merrte
mjekimin per arsye ekonomike. Ne ekzaminim urine dhe Rx kraharor nuk ka evidence per infeksione.
Ne analiza vihet re hiponatremi e lehte, glukoze 48 mg/dL, TSH >100mU/L. Te gjitha te verteta pervec:
A. Ngrohja e jashtme eshte kyce ne terapi per pacientet me temperature <34°C.
B. Solucionet hipotonike IV duhet te shmangen.
C. Levothyroxina (IV) duhet administruar me glucocorticoide IV.
D. Sedacioni duhet shmangur nqs mundet.
E. Kjo gjendje ndodh te te moshuarit dhe shpesh vje nga shkaqe qe nuk lidhen me semundje.

X-22. Nje 29vjecare ka ankth, palpitacione dhe diarre si pasoje e semundjes Graves. Para nisjes se
terapise ajo ka nje dhimbje akute kraharori dhe paraqitet ne urgjence. Radiologu njofton qe CT
angiograma do te jete potencialisht e rrezikshme te kryhet. Cili eshte shpjegimi me I mire per
rekomandimin e radiologut:

158
A. Ekspozimi ndaj kontrastit te jodizuar ne Graves mund te perkeqesoje hipertiroidizmin.
B. Embola pulmonare eshte shume e rralle ne Graves.
C. Ekspozimi ndaj rrezatimit ne pacientet hipertiroidien rrit riskun per malinjitet.
D. Takikardia me Graves limitojne kualitetin e imazheve ne CT angiografi dhe nuk lejojne vleresimin e
embolise pulmonare.
E. Radiologu eshte gabim. CT angiografia eshte e sigurt per pacientet me Graves.

X-23. Nje pacient ka kryer neurokirurgji per rezekim hipofizes pas nje tumori hipofizar. Cili funksion
duhet te ruhet menjehere pas operimit?
A. Piku mengjesor I kortizolit ne plazme.
B. Clirimi i cortisolit si pergjigje ndaj stresit.
C. Retensioni I Na si pergjigje ndaj hipovolemise.
D. Asnjera

X-24. Shkaktari kryesor I sindromes Cushing?


A. adenoma hipofizare ACTH prodhuese
B. adenoma adrenokortikale
C. carcinoma adrenokortikale
D. sekretimi ekopik I ACTH
E. sindromi McCune-Albright

X-25. Lidhen me sindromen Conn te gjitha pervec:


A. Alkaloza
B. Hiperkalemia
C. krampet muskulare
D.Na normal ne serum
E. hipertension sistemik I rende

X-26. Te gjitha masat asimptomatike mbiveshkore (incidentalomas) jane te verteta pervec:


A. Te gjithe pacientet me incidentaloma duhet te bejne screening per feokromocitomen.
B. Fine-needle aspiration ben dallimin midis masave beninje dhe malinje tumorale te mbiveshkores.
C. Shancet qe nje pacient me mase malinje te kene metastaze mbiveshkore jane 50%.
D. Shumica e incidentalomas nuk jane sekretuese.
E. Pjesa derrmuese e incidentalomave adrenale jane beninje.

X-27. Kulmi ditor I nivelit te kortizolit arrihet ne:


A. mesnata (12:00 AM)
B. 4:00 AM
C. 8:30 AM
D. mesdita (12:00 PM)
E. 8:30 PM

X-28. Mr. McTrap sillet ne spital pas nje aksidenti automibilistik. Nuk dihet historiku mjekesor, sepse
ai eshte I trullosur dhe nuk mund te tregoje. CT scan tregon laceracion splenik, dhe futet direkt per
splenektomi, qe perfundon pa komplikime. Pas operimit hemorragjia ka ndaluar dhe cohet ne kujdes
intensiv. Gjithsesi ai eshte hipotensiv me presion arterial 70/50 mmHg qe shkon ne 82/52mmHg pas
nje bolus 2 L normal saline IV. Eshte afebril me numer normal te rruazave te bardha. CT scan I
perseritur I kraharorit, abdomenit, dhe pelvisit tregon qe ska hemorragji. Presioni venoz jugular nuk
ndjehet mbi klavikul. Ai eshte me fytyre rrumbullake e obeze, dhe ne ekzaminim fizik gjeni shenja si
ne figure. Nuk ka hiperpigmentim te duarve. Hapi I radhes eshte:

159
A. Kryeni laparotomi exploratore
B. Administro hidrokortizon 100 mg IV
C. Administro vankomicin dhe piperacilin/tazobaktam
D. Vendos intra-aortic balloon pump per kontrapulsion.
E. MRI spinale

X-29. Nje burre 43vjecar me hipertension episodik sever referohet per vleresim te shkaqeve dytesore
te hipertensionit. Ai ndihet pergjithesisht mire, pervec disa episodeve me ankth, palpacione dhe
takikardi qe shoqerohen me rritje tensioni. Keto episode I shfaqen gjate ushtrimeve. Pacienti vuan nga
depresioni I lehte dhe aktualisht merr sertraline, labetalol, amlodipine, dhe lisinopril. Urina 24oreshe
tregon metanephrinat totale te rritura me 1.5 nga norma. Cili eshte hapi I radhes me I pershtatshem?
A. Vazhdo labetalol per 1 jave dhe perserit testimet.
B. Vazhdo sertraline per 1 jave dhe perserit testimet
C. Refero per nderhyrje kirurgjikale te menjehershme
D. Mat ne urinen 24-ore nivelin e acidit vanillylmandelic
E. MRI abdomeni

X-30. Nje burre 45vjecar diagnostikohet me feokromocitome pas prezantimit klinik me konfuzion,
hipertension 250/140 mmHg, takikardi, dhimbje koke, dhe skuqje. Metanefrinat plazmatike te
fraksionuara tregojne nje nivel normetanephrinash prej 560 pg/mL dhe nje nivel metanephrine prej
198 pg/mL (norma: normetanephrine 18–111 pg/mL; metanephrine 12–60 pg/mL). CT abdomeni me
kontrast IV tregon mase 3-cm ne mbiveshkoren e djathte. MRI koke me gadolinium tregon edeme te
substances se bardhe ne kryqezimin parieto-occipital me leukoencefalopati posteriore te
rikthyeshme. Cila nga pohimet eshte I sakte per menaxhimin e feokromacitomes te ky pacient?
A. β-Bllokadae kundraindikuar ne takikardi even after adequate α-blockade has been attained.
B. Indikohet heqje kirurgjikale imediate e mases of the mass is indicated, sepse pacienti ka krize
hypertensive dhe encefalopati.
C. Ndalimi marrjes se kripes dhe lengjeve per te parandaluar perkeqesimin e hipertensionit.
D. Trajtimi me phenoxybenzamine fillohet me doze telarte (20–30 mg 3 here ne dite) per te
kontrolluar presionin e gjakut dhe kirurgji brenda 24–48 oreve.
E. Fentolamine IV indikohet si trajtim ne krizen hipertensive. Fenoxybenzamina duhet filluar me
doze te ulet dhe titrohet per dozen maksimale te tolerueshme per 2-3 jave. Kirurgjia planifikohet
pasi presioni I gjakut eshte konstant nen 160/100 mmHg.

X-31. Mr. Robinson rikthehet per follow-up pas nje qendrimi te gjate ne spital me hpertension ne
kushtet e feokromacitomes dhe pasi kreu nje adrenalektomi te majte. Qe nga ajo kohe referon qe
eshte ndier mire dhe hipertensioni ka qene nen kontroll. Deshiron te informohet nqs feokromacitoma
e tij eshte majlinje. Cfare do I thoni:

160
A. MesatariSht 50% e feokromacitomave jane majlinje.
B. Malinjiteti I feokromacitomes definohet nga prania e qelizave atipike dhe invazionit vaz gjakut.
C. Matja e 23I-metaiodobenzylguanidine nuk eshte I dobishem per testimin meta ne distanc
D. Mungesa e metastazave ne distance perjashton malinjitetin e semundjes.

X-32. Nje vajze 18vjecare vjen ne spital per kontroll rutine. Ajo eshte aktualisht e shendetshme. Ka
histori familjare te atin dhe 2 halla me MEN1, dhe pasi u nenshtrohet testeve gjenetike rezulton
mbartese e gjeneve MEN1. Prezantimi I pare dhe me I shpeshte I pacienteve me kete mutacion?
A. Amenorrhea
B. Hiperkalcemia
C. Hipoglicemia
D. Ulcer peptike
E. Hipertensioni sistemik I pakontrolluar

X-33. Nje burre 35vjecar vitin e kaluar iu nenshtrua tiroidektomise parciale pas nje karcinome
medulare tiroidiene. Ai vjen ne spital dhe diagnostikohet me feokromocitome, dhe pas 2 javesh ne
terapi intensive I nenshtrohet adrenalektomise unilaterale. Rikuperimi eshte I mire. Gjate rishikimit te
karteles vini re qe gjate tiroidektomise, eshte verejtur nje tumor paratiroidien dhe per kete arsye eshte
hequr njekohesisht dhe gjendra paratiroide e prekur. Cfare do I thuash pacientit ne vizite?
A. Screening familjar dhe gjenetik per kancere te ngjashme nuk eshte I nevojshem, sepse keto lloje
kanceri zhvillohen rastesisht dhe te palidhura me shkaqe gjenetike.
B. Une dyshoj qe keni MEN1.
C. Une dyshoj qe keni MEN2.
D. Tiroidektomia parciale nuk ishte trajtimi I duhur per rastin tuaj.
E. Keta tumore shkaktohen me shume gjasa nga mutacion ne gjenin Menin.

X-34. Nje 4vjecar sillet me hipotension, letargji dhe hiponatremi. Gjithashtu ka K te rritur 5.7 mEq/dL.
Ai eshte afebril dhe ka vlera normale te elementeve te gjakut. Gjithsesi vereni nje inflamacion (thrush)
te gjere oral. Ne te dhenat e pacientit vini re qe ka ndjekur disa trajtime per inflamacionin oral dhe
infeksionin kutan te shkaktuar nga kandidoza. Testimi I antitrupave (HIV) rezultoi negativ. Ne moshen
1 vjecare ka pasur nje episod tetanie dhe eshte paraqitur ne urgjence. U pa qe ishte hipokalemik.
Diagnostikohet me hipoparatiroidizem. Cili mendoni qe eshte mjekimi I duhur?
A. Kalcium IV
B. Hydrocortisone IV
C. Kayexalate, insulin IV, and albuterol per te trajtuar paralizen periodike hiperkalemike
D. Ketoconazole
E. EchoKardiogram urgjente per tamponade kardiake te dyshuar.

X-35. Mr. David paraqitet ne urgjence me mpirje dhe dobesi te kembes. Ne ekzaminim kuptoni qe ai
ka mpirje ne gju dhe dobesi ne kyc ne dorsifleksion dhe ne fleksion plantar. Para 2 vitesh u
diagnostikua me diabet dhe vitin e kaluar u zbulua qe ka hipotiroidizem te theksuar. Ai ka splenomegali
dhe lekure te erret jo nga ekspozimi ne diell. Cili test do ju ndihmonte per diagnozen e pacientit?
A. Matje e titrit te antitrupave anti–nuclear
B. Matje e titrit te antitrupave anti–thymoglobulin
C. Kulture gjaku
D. Elektroforeza e proteinave serike
E. Biopsi lekure per klone qelizore T intravaskulare

X-36. Nje burre 37vjecar ka infertilitet. Sebashku me te shoqen kane 2 vite qe duan te kene femije,
por pa rezultat. Pas vizitave te andrologu, ai I referon konsulte me nje endokrinolog pasi ne analizat e

161
spermes nuk kishte sperme. Ai nuk ka semundje te tjera dhe merr vetem multivitamina. Ne ekzaminim
fizik shenjat vitale jane normale. Eshte I gjate dhe ka teste te vogla, gjinekomasti dhe pak floke e qime
aksilare. Analiza kromozomale konfirmon sindromen Klinefelter. Cili pohim eshte I vertete:
A. Suplementet androgjene perdoren pak ne kete rast.
B. Nuk eshte I rrezikuar per kancer gjiri.
C. Prani e perqendrimit te rritur plazmatik te estrogjenit.
D. Shumica e rasteve diagnostikohen para pubertetit.
E. Perqendrimet plazmatike (FSH) dhe (LH) jane te ulura ne kete rast.

X-37. Nje vajze 17vjecare paraqitet me amenorre. Duket sikur nuk ka hyre ne pubertet pasi nuk ka
pasur menstruacionet e para dhe ka shume pak qime aksilare apo pubike. Eshte 150 cm, vije flokesh
te ulet (balli I vogel) dhe qafe te trashe. FSH 75 mIU/mL dhe estradiol 2 pg/mL. Dyshoni per sindromen
Turner. Indikohen te gjitha testet pervec:
A.Testimi bukal per trupat Barr (buccal smear barr body)
B. Echocardiogram
C. Analize kariotipi
D. Eko renale
E. TSH

X-38. Nje infant lindi me ambiguitet genital. Amniocenteza pergjate shtatzanise tregoi gjenotipin 46,
XX. Infanti paraqitet me phallic-appearing genitalia dhe partically fused labia. Nuk mund te palpohen
testet. Pervec analizes standarte te gjakut, cfare testimi biokimik indikohet?
A. Citometria e gjakut periferik
B. Niveli serik i kortizolit
C. Niveli serik i 17-hidroksiprogesteron
D. TSH serik
E. Niveli serik prolaktines

X-39. Nje 58vjecar, burre, paraqitet per zmadhim bilateral te gjinjve. Para disa muajve eshte paraqitur
per dhimbje te lehta ne te dy gjinte. Nuk ka simptoma te tjera. Ne historikun e jetes tregon qe ka
semundje koronare dhe insuficence kardiake kongjestive, fibrilacion atrial, obezitet dhe DM2. Mjekimi
aktual perfshin lisinopril, spironolactone, furosemide, insulin, dhe digoxin. Nuk perdor drogera dhe ka
3 femije. Gjate ekzaminimit konfirmohet zmadhim I gjinjeve dhe gjendra te palpueshme 2cm
bilaterale. Cili pohim per gjinekomastine eshte I vertete:
A. Duhet kryer mamografi per te perjashtuar kancerin e gjirit.
B. Gjinekomastia e tij eshte nga obeziteti me ind adipoz te pranishem ne gjinj.
C. Testosteroni serik, LH, and FSH duhen matur per vleres mungesen e sensitivitetit ndaj adrogjeneve.
D. Nderprit spironolaktonin dhe observo per regres.
E. Testet e funksionit hepatik duhen kryer per te pare ka cirroze.

X-40. Cdo mjekim interferon me funksionin testikular pervec:


A. Ciklofosfamid B.Ketokonazol
C. Metoprolol D.Prednizon E. Spironolakton

X-41. Shenjat dhe gjetjet klinike per prezencen e ovulimit perfshijne te gjitha pervec:
A. Zbulimi I vlerave te rritura urinare te LH.
B. Kulmi I estrogjenit eshte gjate fazes sekretore te ciklit menstrual.
C. Temperature e rritur trupore>0.5°F ne gjysmen e dyte te ciklit menstrual.
D. Prezence e mittelschmerz.
E. Niveli I progesteronit >5 ng/mL 7 dite para menseve.

162
X-42. Ne zhvillimin progresiv femijeri-pubertet-menopauze cili pohim nuk eshte I vertete per nivelet
e FSH dhe LH:
A. FSH suprimohet nga lindja deri 20 muajsh.
B. LH rritet pergjate vitit te pare (lindje - 20muajsh).
C. Nivelet e LH dhe FSH ulen pergjate femijerise para pubertetit.
D. Ne fillim te pubertetit GnRH stimulon hipofizen per nivelet FSH dhe LH.
E. Nivelet e LH dhe FSH rriten tmerresisht pas menopauze.

X-43. Shfaqja e pare e nje puberteti normal ne shumicen e vajzave eshte:


A. Arritja e shpejte e pikut/kulmit te gjatesise se saj
B. Menarke
C. Zhvilimi I gjinjve
D. Qimet pubike
E. Qimet aksilare

X-44. The Women’s Health Initiative kreu nje studim per terapine hormonale postmenopauze. Studimi
u ndal ne faze te hershme per shkak te riskut te rritur qe ekziston nga marrja e estrogjenit per:
A. Tromboze te venave te thella
B. Kancer endometri
C. Infarkt miokardi
D. Osteoporoze
E. Strok

X-45. Te gjitha kundraindikacione tradicionale te terapise hormonale orale postmenopauzes pervec:


A. semundje hepatike aktive.
B. koagula
C. kancer gjiri
D. risku per semundje koronare pas 10 viteve shkon 5-10%
E. gjakrrjedhje vaginale e papritur

X-46. Nje cift kane 5vite te martuar kane 12 muaj tentojne per koncepsion. Edhe pas mardhenieve
seksuale te rregullta nuk kane arritur ne shtatzani. Te dy jane 32 vjec dhe skane probleme
shendetesore, smarrin as mjekime. Shkaku me I mundur per infertilitetin e tyre?
A. Endometrioza
B. Shkaku te mashkulli
C. Disfunksion I ovulimit
D. Defekt I tubave
E. I pashpjegueshem

X-47.Nje cift keshillohen mbi fertilitetin. Partnerja eshte 35 vjec dhe ska qene kurre shtatzene, dhe
nga mosha 20-34 vjec ka perdorur kontraceptivet oral. Jane bere tashme 16 muaj qe I ka ndaluar dhe
ka cikle menstruale cdo 35 dite, qe ndonjehere ndodh te kalojne edhe 60 dite. Ajo ka dhimbje gjinjsh
2-3 javet pas fillimit te ciklit menstrual. Ne universitet eshte diagnostikuar me Neisseria gonorrhoeae
pas episodeve me temperature dhe dhimbje pelvike. Nuk ka pasur histori tjeter mjekesore. Punon 60
ore ne jave si avokate korporate dhe ben aktivitet fizik cdo dite. Pi kafe cdo dite dhe alkool vetem ne
takime shoqerore. Ka BMI 19.8 kg/m2.
I shoqi, 39 vjec, ska pasur femije edhe pse ka qene I martuar me pare ne moshen 24-28vjecare. Me
pare ka konsumuar marijuane cdo dite dhe ka 9vite abstinence totale. Pikerisht kete fakt e kane pare
si shkak te mungeses se shtatzanise ne cift. Gjithashtu ai ka hipertension dhe trajtohet me lisinopril
10 mg cdo dite. Ka BMI 23.7 kg/m2. Ata duan te vleresojne infertilitetin e tyre dhe shkakun e mungeses
se koncepsionit. Cili pohim eshte I vertete ne kete rast?

163
A. Percaktimi I ovulacionit te femra nuk eshte I nevojshem pasi ciklet e saj janene shumice te rasteve
te rregullta dhe ne mes cikel zhvillon dhimbje gjiri, indikacione ovulimi.
B. Lisinopril duhen ndaluar se perdoruri per rreziqet e defekteve te lindjes te lidhura me te.
C. Partnerja duhet ti nenshtrohet histerosalpingogrames. Nqs gjenden dhembeza fibrotike
sinjifikante, duhet te konsiderojne fekondimin invitro per te ulur rrezikun e barres ektopike.
D. Perdorimi I kontraceptiveve oral >10vite rrit riskun e mungeses se ovulacionit dhe infertilitetit.
E. Marijuana eshte toksike per spermen dhe ka gjasa qe ky te jete shkaku I infertilitetit te tyre.

X-48.Cilat forma te kontracepsionit kane efikasitet teorik >90%?


A. Kondomet
B. Pajisjet Intrauterine
C. Kontraceptivet oral
D. Spermicidet
E. Te gjitha

X-49.Nje burre, 30 vjecar, baba i 3 femijeve, ka zmadhim progresiv te gjinjeve ne 6 muajt e fundit. Nuk
perdor drogera. Testet laboratorike tregojne LH dhe testosteron te ulur. Ca vleresimi do kryer?
A. Urine 24oreshe per matjen e 17-ketosteroideve
B. Marrje gjaku per matjen e transaminazen glutamike oxaloacetike serike (SGOT) dhe fosfatazen
alkaline serike dhe nivelin e bilirubines.
C. Biopsi gjiri
D. Analize kariotipi per Klinefelter
E. Matje estradioli dhe hCG

X-50. Nje grua 36 vjecare raporton qe nuk ka semundje dhe nuk merr mjekim.Ajo dhe I shoqi prej 7
muajsh po mundohen per nje shtatzani, por pa sukses. Cila eshte pergjigja e pershtatshme?
A. Ka gjasa te jeni futur ne menopauze dhe smund te keni femije.
B. Nuk rekomandojme te besh vleresime per feritilitetin pa u bere 12 muaj qe tentoni per shtatzani.
C. Do ju referoj te nje mjek specialist fertiliteti.
D. Shkaqet me te shpeshta te infertilitetit lidhen me meshkujt. Duhet te vleresohet gjendja e
bashkeshortit.
E. Mosha e madhe e nje gruaje nuk zvogelon mundesine e shtatzanise derisa te futet ne menopauze.

X-51. Cila etni ne US ka riskun me te larte per diabet mellitus?


A. Hebrenjte Ashkenazi
B. Amerikanet aziatik
C. Hispanike
D. Zezaket jo-Hispanik
E.Te bardhet jo-Hispanik

X-52. Cila percakton tolerance normale ndaj glukozes?


A. Glukoza plazmatike esell <100 mg/dL
B. Glukoza plazmatike esell <126 mg/dL qe pasohet nga testi I ngarkeses me glukoze orale (following
an oral glucose challenge)
C. Glukoza plazmatike esell <100 mg/dL, glucoza plazmatike <140 mg/dL following an oral glucose
challenge, dhe hemoglobin A1C <5.6%
D. Hemoglobina A1C <5.6%, glukoza plazmatike esell <140 mg/dL
E. Hemoglobina A1C <6.0%

164
X-53. Nje grua obeze, 37vjecare, raporton se gjate vitit te kaluar ka pasur 2 infeksione vaginale/vulvare
qe eshte mjekuar me ilace pa recete dhe ndihet vazhdimisht e etur. Ngrihet naten per te urinuar. Cilin
test eshte me I nevojshem per te vleresuar pranine e diabetit mellitus:
A. Hemoglobin A1C
B. Testi I tolerances te glukozes orale
C. Niveli plazmatik i peptidit C
D. Niveli plazmatik i insulines
E. Niveli I glukozes se rastesishme plazmatike

X-54. Nje vajze obeze, 27vjecare paraqitet me etje te shtuar dhe poliuri. Dyshohet per DM dhe glukoza
plazmatike rastesore eshte 211 mg/d qe konfirmon diagnozen. Test I nevojshem per percaktuar DM1?
A. Antitrupat anti-GAD-65
B. Testimi i polimorfizmit per Peroxisome proliferator-activated receptor γ-2
C. Niveli I insulines ne plazme
D. Testim per antigenin (HLA) DR3 te leukociteve humane
E. Nuk ka test laboratorik detektues per DM1.

X-55. Nje vajze, 18vjecar vjen ne urgjencen e te rriturve per ketoacidoze diabetike (DKA). Ajo nuk
njihej si paciente diabetike, por e ema rrefen qe ajo shkonte shpesh ne banjo se fundmi dhe ishte
shume e etur. Ka BMI 44 kg/m2 dhe nuk ka histori familjare per diabet. Pasi e trajtoni DKA e pacientes
me sukses, vereni qe antitrupat anti-GAD serike dhe antitrupat anti-qelizave te ishujve pankreatik
(ICA) nuk detektohen. E ema dhe pacientja duan te dijne tipin e diabetit. Ca duhet ti thoni ?
A. Bazuar ne moshen e re te shfaqjes, me shume mundesi keni DM1
B. Bazuar ne prezantimin tuaj me ketoacidoze diabetike, gjasat jane te keni DM1
C. Dyshoj qe keni fillim matur te diabetit te te rinjve.
D. Me shume gjasa keni DM2
E. Mendoj qe diabeti juaj ka shkak viral.

X-56. Nje pacient vjen ne urgjence per te vleresuar komplikimet e diabetit si pasoje e nje situate
stresuese ne jeten e tij.Te gjitha testet laboratorike jane ne perputhje me diagnozen e ketoacidozes
diabetike (DKA) pervec:
A. pH arterial 7.1
B. Glukoze 550 mg/dL
C. Ketonet plazmatike pozitive
D. K serik normal
E. Osmolaliteti I plazmes 380 mOsm/mL

X-57. Zgjidh kombinimin korrekt te fillimit dhe kohezgjatjes se vepritmit per insulinat:

D-pergjigja e sakte

X-58. A Nje grua, 54vjec ka DM2 dhe ne kontrollin rutine per glukozen esell u pa hemoglobin A1C
7.6%. Ajo eshte munduar te beje aktivitet fizik qe te dobesohet, por hemoglobin A1C nuk eshte
normalizuar dhe rekomandohet tashme terapi mjekesore. Ajo ka hipertension te lehte te kontrolluar
mire dhe nuk ka semundje te tjera. Terapia e linjes se pare me e pershtatshme:

165
A. Akarboze
B. Exenatide
C. Glyburide
D. Metformin
E. Sitagliptin

X-59. Nje vajze, 21vjecare me histori te DM1 vjen ne spital me nauze, te vjella, letargji dhe dehidrim.
E ema thote qe nuk ka marre insulinen 1 dite me pare. Ajo eshte letargjike dhe ka mukoza te thata
dhe eshte e merzitur, presionin arterial 80/40 mmHg dhe rrahjet e zemres 112 bpm, zhurmat kardiake
normale, pulmone te pastra, abdomen te bute dhe ska zmadhim organesh. Eshte e pergjegjshme dhe
orientohet x3,
Na serik 126 mEq/L, K 4.3 mEq/L, Mg 1.2 mEq/L, BUN 76 mg/dL, creatinina 2.2 mg/dL, bikarbonatet
10 mEq/L, Cl 88 mEq/L. Glukoza serike eshte 720 mg/dL. Te gjithe hapat e meposhtem jane te
pershtatshme per menaxhimin e pacientes pervec:
A. tretesire natriumi 3%
B. gazet ne gjakun arterial
C. insulin IV
D.K IV
E. lengje IV

X-60.Nje paciente 28vjecare me DM1 me trajtim me insuline. Ajo dhe I shoqi kane vendosur te
tentojne per nje koncepsion. Cili pohim eshte I vertete per riprodhimin ne kushtet e diabetit:
A. Grate me diabet kane ulje te aftesise riprodhuese
B. Insulina kalon placenten dhe ndikon negativisht te fetusi
C. Pritet qe kerkesat per insulin ne shtatzani te rriten
D. Glukoza e larte e nenes sjell rritje te riskut per anomali te fetusi
E. Periudha me e rendesishme per kontrollin glicemik eshte tremestri I 3te I shtatzanise per te
shmangur malformacionet fetale.

X-61. Cila nga t ne lidhje me kujdesin spitalor ndaj pacienteve diabetik eshte i vertete:
A. Anestezia e pergjithshme con ne sensibilitet ndaj insulines dhe rrit riskun per hipoglicemi.
B. Hiperglicemia me e theksuar gjate qendrimit ne spital nuk komplikohet me infeksione.
C. Kontrolli strikt I glicemise (goal 81–108 mg/dL) eshte superior perkundrejt kontrollit te glicemise se
moderuar (goal, 140 mg/dL).
D. Fillimi I ushqimit parenteral vjen si pasoje e rritjes se nevojes per insuline.
E. Nepacientet ne gjendje kritike, insulina subcutane preferohet me shume se insulina IV.

X-62. Gjetja e pare ne fundusin e pacientit:

166
A. Arteriovenous nicking
B. Mikroaneurizem
C. Neovaskularizim
D. Papiledema

X-63. Cili pacient duhet te trajtohet me ACE inhibitor ose ARB?


A. Nje vajze 24vjecare, me diabet 1 me 2 teste pozitive per mikroalbuminuri me 1 jave diference.
B. Nje grua 32vjecare, me diabet 1 me glukoze ne gjak 328 mg/dL dhe me nje test pozitiv per
mikroalbuminuri.
C. Nje burre 48vjecar, me diabet 2, me nje test pozitiv mikroalbuminurie 1 jave pas nisjes se nje
program ushtrimor.
D. Nje burre 56 vjecar, me diabet 2 me 2 teste pozitive per mikroalbuminuri brenda 3 muajve.
E. Nje burre 62vjecar, me diabet 2, hipertension, test pozitiv per microalbuminuri dhe tension 190/118
mmHg ne diten e testimit.

X-64.Nje 58vjecare femer, me DM2 prej 15 vitesh, vjen ne spital me ndjesi shpimi ne duar dhe ne
kembe. Hemoglobina A1C e fundit e matur eshte 7.9%. Aktualisht eshte ne perdorim te detemir 40
unite ditore dhe metformin 1000 mg dita. Ne ekzaminimin neurologjik ka humbje te reflekseve ne te
dy kycet. Reflekset e tendineve te thella jane 2+ ne gju, biceps dhe triceps. Ndjesia ndaj prekjes
pikesore dhe asaj te lehte eshte e ulur bilateralisht ne kycet e kembeve dhe duarve. Ajo gjithashtu ka
te veshtire te konstatoje nqs gishti I madh eshte I ngritur lart apo I ulur kur eshte me sy mbyllur. E ka
te veshtire te fleje gjume, ndonjehere shkak eshte dhimbja e kembeve. Ajo eshte diagnostikuar me
polineuropati distal sensoriale nga diabeti. Cili mjekim eshte I aprovuar nga U.S. Food and Drug
Administration per trajtimin e neuropatinse diabetike
A. Duloxetine
B. Gabapentin
C. Pregabalin
D. A dhe C
E. Te gjitha

X-65.Glukoza plazmatike eshte normalisht e rregulluar/kontrrolluar ne trup, me nivel eseell midis 70-
110 mg/dL. Kur glukoza ne gjak bie nen 80–85 mg/dL, cili ndryshim fiziologjik ndodh I pari:
A. ulje e GH
B. ulje e sekretimit te insulines
C. rritje cortisoli
D. rritje epinephrine
E. rritje glucagoni

X-66. Nje punonjese e shendetesise, 25 vjec ka hipoglicemi. Ka pasur disa episode vitin e fundit ku ka
pasur marrje medsh (lekundje), ankth dhe djersitje. Testi I shpejte I glukozes (fingerstickglucose)
tregon vleren 40–55 mg/dL. Pergjate nje episodi niveli plazmatik I glukozes eshte 50 mg/dL dhe pasi
pi leng portokalli ndihet me mire. Keto episode nuk kane ndodhur jashte ambientit te punes. Pervec
kontraceptiveve oral s’merr mjekim tjeter. Cila nga keta matje do percaktoje shkakun e hipoglicemise
A. Matje e IGF-1.
B. Matje e insulines esell dhe nivelit te glukozes.
C. Matje e insulines dhe glucozes esell, dhe niveli i C-peptidit.
D. Matje e insulines, glukozes, dhe nivelit te C-peptidit pergjate nje episodi simptomatik.
E. Matje e kortizolit plazmatik.

167
X-67. Te gjitha pohimet per hipoglicemine ne diabet mellitus jane te verteta pervec:
A. Individet me DM 2 kane me pak shance per hipoglicemi sesa ata me DM 1.
B. 2-4% e vdekjeve ne DM1 jane nga hipoglicemia.
C. Episodet e perseritura te hipoglicemise predispozojne demtime autonome me deficit te rregullimit
te nivelit te glukozes dhe hipoglicemi pavetedije.
D. Nje person me DM 1 ka mesatarisht 2 episode hipoglicemie ne jave.
E. Tiazolidinedionet dhe metformina shkaktojne hipoglicemi me shume se sulfonilurea.

X-68. Nje djale 18vjecar vjen me dhimbje te lehte abdominale qe irradiohet ne shpine. Ne ekzaminim
fizik vihet re temperature prej 38.0°C, tensioni 95/55 mmHg, frekuenca 110 bpm, frekuenca
respiratore 18 breaths/min, saturimi I oksigjenit 96% . Ka zmadhim te heparit dhe splenit. Ka gjithashtu
ksantoma eruptive ne duar, kembe. Lipaza eshte 2300 U/L, niveli i triglicerideve esell eshte 1019
mg/dL. Pas vleresimit u arrit ne perfundimin qe ka pankreatit dhe deficence te lipazes lipoproteinike.
Ai eshte stabel dhe pa kompikime dhe gati per tedale nga spitali pas 4 ditesh. Cfare rekomandoni:
A. kufizimi I yndyrnave ne diete 15 g/d
B. vaj peshku supplementar
C. Gemfibrozil 600 mg 2 here ne dite(bid)
D. Nicotinic acid sustained-release 250 mg bid
E. Simvastatin 20 mg ditore

X-69.Nje burre, 32vjecar vizitohet per faktoret e riskut per semundje koronare. Nuk konsumon duhan
dhe presionin sistemik te gjakut e ka normal, nuk ka diabet dhe semundje te tjera. Ka histori familjare
per hiperkolesterolemi, te emen, gjyshin dhe gjyshen nga mami. Nga ekzaminimi fizik ka prani te
ksantomave. Kolesteroli esell tregon nivel te (LDL) prej 387 mg/dL. Cili problem gjenetik mund te kete
A. deficience e apolipoproteines (apo) A-V
B. defekt familjar apoB-100
C. deficience familjare e lipazes hepatike
D. hiperkolesterolemia familjare
E. deficienca e lipazes Lipoproteinike

X-70. Te gjitha jane shkaqe potenciale te rritjes se LDL pervec:


A. anoreksia nervore
B. cirroza
C. hipotiroidizem
D. sindromi nefrotik
E. diuretiket tiazidik

X-71. Pacienti 60vjecar me gamapati monoklonale, ka creatininen te rritur se fundmi 2.0 mg/dL, K
eshte 3.7 mg/dL, Ca eshte 12.2 mg/dL, LDL jane 202 mg/dL, triglyceridet jane 209 mg/dL. Ka 3muaj
me sy te enjtur dhe urinen e ka te shkumezuar. Ne ekzaminim vini re anasarka. I shqetesuar per
mielomen multiple dhe sindromin nefrotik, ju kerkoni matje te raportit proteina/ kreatinine urinare
dhe del 14:1. Cili trajtim eshte I pershtatshem per te trajtuar nivelin anormal te yndyrnave te tij:
A. Cholesterol ester transfer protein inhibitor
B. Dietary management
C. 3-Hydroxy-3-methylglutaryl–coenzyme A (HMG-CoA) reductase inhibitors
D. Lipid apheresis
E. Niacin and fibrates

X-72. Sindromi metabolik eshte percaktuar nga World Health Organization in 1998 si sindrom qe
perfshin: obezitet qendror, hipertrigliceridemi, HDL te ulura, hiperglicemi dhe hipertension. Cila nga
pohimet ne lidhje me epidemiologjine e sindromit metabolik eshte e vertete:

168
A. Pas te 60tave, meshkujt zhvillojne sindrom metabolic me shume se grate.
B. Per pacientet me diabet mellitus, prania e sindromit metabolik rrit riskun per semundje
kardiovaskulare.
C. BMI eshte treguesi me I rendesishem I rezistences insulinike dhe I riskut te diabetikeve per sindrom
metabolik.
D. Prevalencen me te larte per sindrom metabolik ne US e kane grate meksikane.
E. Japonezet jane populli me pak I riskuar.

X-73. Nje burre kinez, 47 vjec vjen per kontrollin e pervitshem.ai nuk ka ankesa por ben jete sedentare.
Punon si llogaritar dhe puna e tij eshte ulur perballe ekranit. Nuk ben aktivitet fizik dhe ka nje diete te
varfer ushqimore. Eshte I divocuar dhe jeton vetem, prandaj ha kryesisht jashte (4 here ne jave) dhe
ditet e tjera preferon ushqim te shpejte. Historia mjekesore: vuan nga hypertensioni dhe obeziteti dhe
merr hydrochlorothiazide 25 mg dita. Tensioni sot eshte 148/92 mmHg, perimetri I belit 93 cm,
gjatesia 177.8 cm, pesha 105 kg, BMI 32.3 kg/m2. Gjetjet e tij laboratorike esell jane cholesteroli220
mg/dL, HDL 28 mg/dL, triglyceridet 178 mg/dL, dhe LDL 103 mg/dL, glukoza plazmatike esell 98 mg/dL.
Cila eshte e vertete ne lidhje me diagnozen e sindromit metabolik te pacientit
A. Smund te kete sindrom metabolik sepse glukoza esell e tij eshte normale.
B. Ai ka sindrom metabolik sepse ka 3 nga 5 kriteret: triglyceride te larta, HDL ulura dhe hypertension.
C. Ai ka sindrom metabolik sepse ka 4 nga 5 kriteret diagnostike:BMI e larte, triglyceride te larta, HDL
te ulura dhe hypertension.
D.Ai ka sindrom metabolik sepse ka 4 nga 5 kriteret diagnostike: perimeter te gjere beli, , triglyceride
te larta, HDL te ulura dhe hypertension.
E. Sindromi metabolik nuk mund te diagnostikohet me nje vleresim te vetem, testet duhen perseritur
pas 3-6 muajsh.

X-74. Nje burre 55vjecar sillet ne reanimacion pas 1 jave me temperature dhe kolle. Ai ka qene mire
deri para 1 jave kur u pa qe ka frymemarrje te shkurtuar progresive dhe sputum produktiv. U soll ne
reanimacion pasi u intubua sepse e shoqja vuri re qe ai ishte letargjik dhe stafi spitalor pa qe pacienti
ishte I papergjegjshem. Mjekimet e vetme qe ai merr jane insulin glargine 20 unite dita dhe insulin
aspart me vaktet. Ai ka diabet mellitus dhe ne historikun e jetes abuzim alkooli me me shume se 12
birra ne dite. Me te shtruar pacientin, temperatura e matur eshte 38.9°C (102°F), tensioni 76/40
mmHg, dhe saturimi I oksigjenit 86%, volumi tidal 420 mL, frekuenca respiratore 22 breaths/min,
presioni fundekspirator pozitiv me vlere 5, FiO2 eshte 1.0. Pacienti eshte ne ventilim mekanik, presioni
jugular eshte normal, zhurmat respiratore te zbehura ne bazen e pulmonit te djathte me egofoni.
Zhurmat kardiake normale. Abdomen I bute. Nuk ka edema periferike. Rx kraharori tregon infiltrim ne
lobin e poshtem te djathte dhe efuzion pleural te moderuar. Ecg eshte normale. Kultura e sputumit
treon prani te diplokokeve gram pozitiv. Leukocitet jane 23 × 103/μL me 70% polymorphonuclear cells
dhe 6% bands, BUN eshte 80mg/dL, creatinina eshte 3.1 mg/dL, Glukoza plazmatike 425 mg/dL. I
fillohen antibiotic me specter te gjere, lengje IV, omeprazol dhe insulin. Ushqehet me tub nazogastrik
dhe ne diten e 2te kreatinina shkon 1.6 mg/dL, por fosfatet plazmatike jane 1.0 mg/dL (0.3 mmol/L)
dhe calcium eshte 8.8 mg/dL. Hipophosfatemia te ky pacient shkaktohet nga te gjitha pervec:
A. Demtim renal akut B. Alkoolizmi C.Insulin D. Kequshqyerj E. Sepsisi

X-75. Hipofosfatemia te pacienti me lart korrigjohet duke:


A. Administruar gluconat calcium 1 g IV pasuar nga infuzion IV I 8 mmol/hr fosfate per 6 ore.
B. Administruar vetem fosfate IV 2 mmol/hr per 6 ore.
C. Administruar vetem fosfate IV 8 mmol/hr per 6 ore.
D. Duke vezhguar nivelin e fosfateve te cilat pritet te normalizohen per 24-48 oret ne vijim.
E. Filluar fosfate orale zevendesuese 1500 mg/d.

169
X-76. Po kujdesesh per nje burre 72vjecar qe ka 3 vite ne azil. Ai vuan nga SPOK I rende dhe I nevojitet
oksigjen I vazhdueshem 3 L/min. Me pare ka pasur strok dhe si pasoje ka hemipareze te djathte.
Mjekimi actual perfshin aspirin, losartan, hydrochlorothiazide, fluticasone/salmeterol, tiotropium,
dhe albuterol. BMI e tij eshte 18.5 kg/m2. Ju jeni te shqetesuar nqs ai ka apo jo deficence vit D. cili test
eshte percaktues nqs ka apo jo deficence te vit D?
A. 1,25-Hydroxy-vitamin D
B. 25-Hydroxy-vitamin D
C. Fosfataze alkaline
D. Hormonin paratiroid
E. Serum total dhe nivelin e calciumit te jonizuar

X-77. A Nje grua 72vjecare vjen nespital me nje frakture ne ijen e djathte. Pas trajtimit kirurgjikal
mbahet per rehabilitim. Niveli i 25-hydroxyvitamin D te saj eshte 18.3 ng/ L. Cfare rekomandon
A. Vitamin D3 800 unite dita
B. Vitamin D3 800 unite dita plus carbonat calcium 1500 mg dita
C. Vitamin D3 2000 unite dita
D. Vitamin D3 2000 unite dita plus carbonat calcium 1500 mg dita
E. Vitamin D3 50,000 unite java per 4 jave, pastaj 800 unite java, plus calcium 1500 mg dita

X-78.Nje grua 60vjecare vjen me hiperkalcemi. Kalciumi serik 12.9 mg/dL eshte gjetur gjate ohesqe
ishte e shtruar per kolecistoektomi. Edhe pas dhenies se lengjeve, kaciumi serik ishte 11.8 mg/dL.
Pacientja eshte asimptomatike dhe niveli I PTH se saj eshte 95ng/L (reference value 10–65 ng/L).
Ajo nuk ka semundje te tjera dhe ka kryer screening per kancer (te rekomanduar per moshen e saj).
Nuk ka konstipacion ose dhimbje kockash. Ka 8 jave qe nga kryerja e operimit dhe kalciumi serik sot
eshte 12.6 mg/dL, fosfatet 2.3 mg/dL. Hematokriti dhe gjithe gjetjet e tjera perfshire dhe kreatininen
jane normale. Cili do perbente indikacion kirugjikal per pacienten:
A. mosha >50
B. rritja e kalciumit ne urinen 24-oreshe
C. Nephrolithiasis
D. Osteopenia ne nje testim te densitetit kockor
E. Serum calcium >1 mg/dL mbi normen

X-79. Nje burre 42vjecar vjen me dhimbje akute ne flankun e djathte. Ai pershkruan dhimbje 10/10 qe
irraidohet ne ije. Ka pasur nje episode hematurie. Ct pa kontrast konfirmon kalkul renale ne anen e
djathte aktualisht ne ureterin distal. Me pare ka pasur sarkoidoze pulmonare dhe nuk mjekohet
aktualisht per te. Kjo diagnoze I eshte vene me biopsi me bronkoskopi dhe nuk ka pasur granuloma
kazeoze. Rx kraharori tregon adenopati hilare bilaterale. Niveli serik I kalciumit eshte 12.6 mg/dL.
Mekanizmi I hiperkalcemise te pacienti eshte
A. Rritjaktivizimit 25-hydroxyvitamin D ne 1,25-hydroxyvitamin D nga makrofaget ne Granuloma.
B. Rritje e aktivizimit te 25-hydroxyvitamin D ne 1,25-hydroxyvitamin D nga veshka
C. Rritje e aktivizimit te vitamines D ne 25-hydroxyvitamin D nga macrophaget ne Granuloma.
D. Missed diagnosis of lymphoma with subsequent bone marrow invasion and resorption of bone
through local destruction
E. Prodhim I peptidit te lidhur PTH nga macrofaget ne granuloma

X-80. Nje burre 52vjecar ka semundje renale stad I fundit, hipertension dhe DM.ka 8 vite qe kryen
hemodialize. Gjate kesaj periudhe ai nuk eshte treguar I kujdesshem ndaj mjekimit dhe hemodializes
duke humbur ndonje seance javore. Tani ka dhimbje kockash dhe dispne. Saturimi I o2 eshte 92% dhe
Rx tregon infiltrime bilaterale te mjegullta. CT kraharori tregon infiltrime bilaterale si xham jo I
tejdukshem. Te dhenat laboratorike: calcium 12.3 mg/dL, phosphate 8.1 mg/dL, parathyroid hormone
110 pg/mL. cili perben trajtimin me te pershtatshem actual per pacientin:

170
A. Calcitriol 0.5 μg IV dhe hemodialysis me sevelamer 3 here ne dite.
B. Calcitriol 0.5 μg oral ditor dhe sevelamer 1600 mg 3 here ne dite.
C. Hemodialize me agresive per te arritur balance elektrolitike dhe per te normalizuar lengjet.
D. Paratiroidektomi
E. Sevelamer 1600 mg 3 here ne dite

X-81.Nje grua 54vjecare I nenshtrohet tiroidektomise per karcinome folikulare. 6ore pas kirurgjise
pacientja ka ndjesi shpimi ne mukozen orale qe pasohet nga ndjesi shpimi dhe ne kembe e gishta.
Infermjerja kerkon mjekun per vleresim te gjednjes se pacientes pasi ajo ka dhimbje te forte ne dore
gjate kohes qe I matet tensioni. Pergjate vleresimit ajo ankohet per dhimbje te vazhdueshme te dores.
Qe pas kirurgjise pacientja merr sulfat morfine per dhimbjet dhe metoklopramid per nauzene. Ajo ska
ndryshime te shenjave vitale dhe eshte afebrile. Prekja e pjeses inferiore te harkut zigomatik 2cm para
veshit dha nje dridhje te cepit te buzes. Ekg shfaqi nje interval QT 575msec. Hapi tjeter vleresues dhe
A. Administro benztropine
B. Administro gluconat calcium
C. Administro sulfat magnezi
D. Matje e nivelit te calciumit, magnezit, P, dhe K.
E. Matje te capacitetit vital te sforcuar

X-82. Nje grua 68vjecare me karcinome pulmonare me qeliza skuamoze stadi 3B vjen me gjendje
mendore te alteruar dhe e dehidruar. Matja e kalciumit rezulton 19.6 mg/dL dhe phosphate 1.8 mg/dL,
PTH 0.1 pg/mL (normal 10–65 pg/mL), and a screen for parathyroid hormone related peptide was
positive. Ne 24 oret e fundit pacjentja la marr 4 L fiziologjik NaCl dhe furosemid. Te nesermen calcium
eshte 17.6 mg/dL dhe phosphate 2.2mg/dL. Ajo vazhdon te jete e alteruar menderisht. Trajtimi I
nevojshem I hiperkalcemise se pacientes:
A. Vazhdo terapine me dhenie te sasive te medha te lengjeve, diureze me furosemide
B. Vazhdo terapine me dhenie te sasive te medha te lengjeve, ndalo furosemidin dhe jep
hydrochlorothiazide
C. vetem kalcitonin
D. vetem pamidronat
E. calcitonin dhe pamidronat

X-83. Cila eshte e vertete per epidemiologjine e osteoporozes dhe frakturave kockore
A. Cdo 5 vite pas moshes 70, incidencca e frakturave te ijeve rritet me 25%.
B. Frakturat e radiusit distal kane frekuence me te larte para te 50tave dhe te pandryshueshme pas
te 60tave, dmth nje rritje modeste te lidhur me moshen.
C. Shumica e grave shfaqin kriteret diagnostikuese per osteoporoze rreth te 60 dhe 70.
D. Grate e bardha dhegrate zezake jane njelloj te riskuara per frakture ijesh.
E.. Raporti femer mashkull me osteoporoze eshte 10:1.

X-84. Nje grua 50vjecare vjen dhe pyet per riskun e frakturave nga osteoporoza. Ka histori familjare
per osteoporoze te emen por qe nuk ka pesuar kurre ndonje frakture. Gjithashtu as pacientja akoma
nuk ka pasur ndonje frakture. Ka konsumuar duhan 20 vite dhe ka 10 vite qe e ka lene. Ne moshen 37
vjecare kreu nje histerektomi me salpingo-oophorectomy bilaterale per endometrioze. Eshte
intolerante ndaj glukozes dhe nuk konsumon produkte qumeshti. Merr carbonate calcium 500 mg
dita. Peshon 115 lb, eshte e gjate 66 in dhe ka BMI 18.6 kg/m2. Shkaqe ostoeporozete gjitha pervec
A. menopauze e hershme B. gjinia femerore
C. duhanpirja D. pesha e ulet trupore
E. marrje e paket e calciumit

171
X-85. Nje grua 54 vjecare vjen te endokrinologu per te vleresuar nqs ka osteoporoze pas dhimbjes se
shpines pas nje frakture kompresuese te vertebres T4. Pacientja eshte premenopauze dhe ka cikel jo
te rregullt dhe ndjenje te nxehti. Nuk pi duhan, dhe ska shqetesime te tjera. Peshon 70kg, eshte e
gjate168cm dhe ka humbur 5cm gjatesi. densitet kockor tregon nje T-score -3.5 deviacion standart
dhe Zscore -2.5 SD. Indikohen per vleresimin e osteoporozes te pacientja te gjitha pervec:
A. 24-Hour urine calcium
B. FSH and LH levels
C. Serum calcium
D. TSH
E. Vitamin D levels (25-hydroxyvitamin D)

X-86. ANje grua e bardhe 45vjec, kerkon keshilla per rrezikun e osteoporozes dhe nevojne e kontrollit
te densitetit kockor. Ajo nuk ka konsumuar duhan dhe alkool vetem me raste. Qe nga adoleshenca ka
astme persistente te moderuar dhe perdor flutikazon 44 mg/puff 2 here ne dite ne menyre te rregullt.
Hera e fundit e marrjes se prednizonit oral ishte para 6 muajsh si pasoje e infeksionit me influenza qe
iu komplikua me atak asmatik. Mori prednizon per 14 dite. Ka pasur 3 shtatzani dhe vetem 2 lindje 1
ne moshen 39, 1 ne moshen 41. Aktualisht ka perioda jo te rregullta cd0 42 dite. Ne diten e 12 te ciklit
niveli I FSH eshte 25 mIU/L dhe 17β-estradiol 115 pg/mL. Mamaja dhe tezja kane vuajtur nga
osteoporoza, nderkohe qe e ema ka pasur dhe artrit rheumatoid per te cilen merr 5 mg prednizon dhe
nje frakture kompresuese te vertebres spinale ne moshen 68vjecare.
Ne ekzaminim fizik duket e shendetshme, me gjatesi 168 cm, peshe 66.4 kg dhe ekzaminimet e
kraharorit, zemres, abdomenit, muskujve dhe neurologjike tregojne qe eshte normale. Cfare do I thoni
pacientes ne lidhje me nevojen e vleresimit te densitetit kockor
A. Sepse eshte perimenopausale dhe duhet te beje vleresimin e densitetit kockor cdo vit derisa te
futet ne menopauze dhe pastaj do kryeje densitometry te pervitshme.
B. Bazuar ne historine familjare ka indici kryeje kontroll densitetit kockor cdo vit duke filluar qe tani.
C.Vleresimi densitometrik nuk rekomandohet te kryhet pa arritur menopauzen.
D. Shtyrja e lindjes per ne dekaden e 4 dhe te 5te te jetes rrit riskun per osteoporoze, prnj
denisotmetria nuk duhet kryer.
E. Perdorimi I glukokortikoideve te inhaluara ne doze te ulet rrit 3 here riskun per osteoporoze prandaj
duhet te beje kontroll te denistetit kockor.

X-87 to X-91. Lidh medikamentet e perdorura per osteoporozen me mekanizmin e veprimit:


87. Calcitonin:
88. Denosumab:
89. Raloxifene:
90. Teriparatide:
91. Zoledronic acid

A. Recombinant parathyroid hormone (1-34hPTH) with direct stimulation of osteoblast activity (90)
B. Polypeptide hormone that suppresses osteoclast activity through a specific receptor for the
Hormone (87)
C. Bisphosphonate drug given on an annual basis that impairs osteoclast function and reduces
osteoclast number (91)
D. Selective estrogen receptor modulator (89)
E. Human monoclonal antibody to RANKL, a protein necessary for osteoclast maturation (88)

X-92. Nje grua 38vjecare me fibroze cistike dhe deficence te vit D ka T-score of –2.8 ne pjesen lumbare
te shpines dhe ne ije (hips). Ajo ka filluar mjekimin me alendronate 70 mg ne jave, cholecalciferol
5000 unite dita, dhe carbonat calciumi 1500 mg dita. Kur duhet perseritur densitometria per te pare
pergjigjen ndaj mjekimit?

172
A. 1 vit
B. 3 vite
C. 5 vite
D. 10 vite
E. Nuk duhet perseritur. Duhet bere nje MRI.

X-93.Nje vajze 19vjecare ka pasur fraktura te perseritura ne kockat e gjata, 2 here femurin dhe 3 here
humerusin. Ajo rrezohet dhe mavijoset shume kollaj. Pervec ketyre ajo ska pasur dhe nuk ka asnje
shqetesim tjeter. Ne ekzaminim fizik kockat duken te deformuara lehtesisht, dhembet teresisht te
verdhe, sclera blu. Dyshohet per osteogenezis imperfecta. Cila fjali eshte e vertete per kete patologji:
A. Mutacion ne prokolagjenin e tipit 1 eshte I pranishem ne kete paciente.
B. Biopsia kockore nevojitet per diagnoze definitare.
C. Bisphosphonates kane treguar sukses afatgjate ne parandalimin e frakturave kockore ne kete rast.
D.Frakturat te femrat rriten pas pubertetit.
E. Densiteti mineral I rritur I kockave demostrohet ne x-ray absorptiometry.

X-94.Nje djale 20vjec, I nenshtrohet kontrollit rutine ne skuadren e basketbollit. Me origjine eshtenga
Nigeria dhe ka ardhur neUS per edukim. Ne moshen 13 vjec eshte kuruar per tuberkuloz. Nuk merr
medikamente dhe nuk ka alergji. I ati ka vdekur ne moshen 46 vjec per shkaqe kardiake dhe nuk iu
nenshtrua autopsise. Gjithashtu te tjere familjare nga ana babait kane vdekur per shkaqe kardiake. E
ema eshte e shendetshme. Djali eshte 200.6cm, 89.8kg dhe ka BMI 22.3kg/m2. Vereni qe trupi eshte
me I shkurter se gjymtyret. Krahu I shtrire eshte 83in. ai ka pectus excavatum dhe arachnodactyly.
Qellza e tij eshte shume e harkuar, ka miopi dhe lente ektopike ne syrin e djathte. Ne ekzaminimin
kardiovaskular vihet re nje murmur diastolike II/VI nehapesiren interkostale te 3te. Ai eshte ne ankth
per fillimin e stervitjes se basketbollit. Cfare i keshilloni kete here:
A. Ai nuk duhet te filloje basketbollin dhe sportet e tjera te mundueshme.
B. Nuk ka nevoje per vleresime te tjera dhe mund te vazhdoje aktivitetin fizik.
C. Mund te vazhdoje te stervitet nderkohe qe duhet te beje vleresime te metejshme si ecg, ekzaminim
e syve dhe teste gjenetike.
D. Duhet te marre beta bllokues dhe pastaj te vazhdoje stervitjen.

X-95.Nje burre 40vjecar ka lexuar ne internet per disa procedura screening ne internet dhe eshte I
interesuar per tu testuar per hemokromatozen. Eshte I shendetshem dhe merr multivitamina ditore.
I ati ka vdekur 56 vjec nga cirroza pasi ka pire shume alkool gjate jetes. Nuk ka histori per familjare me
semundje hepatike te tjera. Cili eshte hapi I pare testues per screening te hemokromatozes:
A. Genetic testing for C282Y mutation
B. shqyrtim i HFE activity C. Liver MRI
D. Screening per hemochromatosis nuk eshte efektiv apo I keshillueshem
E. Saturimin e transferines dhe ferritinen serike

X-96. Nje burre I bardhe 55vjecar me diabet ankohet ne zyren tuaj per dobesi te pergjithshme, humbje
peshe, dhimbje difuze abdominal, disfunksion erektil. Ka histori te hiperkolesterolemise dhe merr
atorvastatine. Ekzaminimi tregon hepatomegali jo te bute ne palpim, atrofi testikulare dhe
gjinekomasti. Nuance gri difuze me e perhapur ne fytyre dhe ne qafe. Ekzaminimi I artikulacioneve
tregon artikulacionin metakarpofaringeal te trete te dores se djathte lehtesisht te enjtur. Dg duhur:
A. Anti–smooth muscle antibody
B. Ceruloplasmin
C. EkoDoppler hepatike
D. Antitrupa siperfaqesor te hepatitit B
E. HFE gene mutation screen

173
X-97. Nje djale 28vjecar shtrohet ne reanimacion per insuficence fulminante hepatike dhe hemolize.
Familjaret deshmojne qe ka 5 vite me depresion dhe pati 2vite me pare hepatit akut. Ne ate kohe
asparat aminotransferaza arriti kulmin prej1200 U/L dhe alanine aminotransferase kulmin ne 1900
U/L. ai kishte verdhez te lehte me bilirubine totale 7.2 g/dL. Nuk u gjet shkak per hepatitin paavaresisht
se perfshiheshin shkaqe virale dhe autoimune. Funksioni hepatic I tij u kthye ne normalitet. Merr
antidepresnat dhe me raste ibuprofen, dhe jo mjekime te tjera. Ekzaminimi fizik deshmon prani asciti
dhe gjendje mendore te alteruar me distoni. CT abdomen tregon qe ka cirroze hepatike por jo
obstruksion biliar. Cila ka me shume gjasa te konfirmoje diagnozen themelore:
A. 24-Hour urine level of iron
B. Brain MRI showing damage to the basal ganglia
C. Genotype for HFE mutation
D. Schistocytes on peripheral blood smear
E. Slit-lamp ocular examination showing Kayser-Fleischer rings

X-98. Cili nga mjekimet me I pershtatshmi si trajtim fillestar per pacientin ne pyetjen 97?
A. Cholestyramine
B. d-Penicillamine
C. Transplant hepari
D. Trientine
E. Zink

X-99.Nje vajze 22 vjecare vjen me dhimbje abdominale te nivelit 10/10 te dhimbjes. Ky episode ka 5
ore qe ka filluar dhe eshte dhimbje difuze, konstante me krampe te lehta. Ka nauze dhe ka vjellur 1
here. Ndjen qe abdomen eshte I distenduar. Para 6 vitesh eshte paraqitur ne urgjence per kete
problem dhe eshte trajtuar me fluide IV, antiemetic dhe opiate IV. Ajo shprehet qe eshte trajtuar si
konsumatore droge sepse analizat laboratorike, urinare dhe Ct abdominal me kontrast oral dhe IV
rezultuan negative per ndonje patologji. Simptomat kane kaluar pa nderhyrje spitalore brenda 24-48
oreve. Ajo ka pasur episode te meparshme 4-5 here ne vit , por nuk ka ardhur me pare ne spital nga
eksperienca e fundit. Kur ka dhimbje ka gjithashtu ankth dhe pagjumesi. Ka pasur 2 episode
halucinacionesh auditive gjate atakut te dhimbjes qe jane shkaktuar nga severiteti I dhimbjes. Ajo
eshte ne vit te fundit per inxhinieri mekanike dhe eshte e dalluar. Nuk ka histori te kaluar mjekesore.
ka ndaluar se perdoruri kontraceptivet oral pasi ndjen qe I perkeqesojne dhimbjen episodike
abdominale. Edhe e ema ka pasur episode te njejta dhe mendonte qe ishin nga endometrioza. E ema
nuk ka kerkuar ekzaminim specific per dhimbjen.
Shenjat vitale te vajzes: frekuenca karsiake 120bpm, temp 37.2°C, presioni I gjakut 138/88 mmHg,
frekuenca respiratore 18 breaths/min, SaO2 99%. Ekzaminimi I kokes syve vesheve hundes dhe fytit
nuk eshte tregues. Kraharori eshte I lire, ekzaminimi kardiovaskular tregon takikardi te rregullt pa
murmur. Ekzaminimi abdominal tregon zhurme hipoaktiviteti te zorreve dhe distension te lehte. Nuk
ka butesi te lokalizuar. Rx abdomen tregon ileus. Analiza urinare dhe toksikologjike jane negative.
Elementet e gjakut jane te gjithe ne norme, pervec hiponatremise se lehte 132 mmol/L. Hapi tjeter I
rendesishem per te vleresuar pacienten:
A. Endoscopy and colonoscopy
B. Plasma HMB synthase mutation analysis
C. Measurement of urine porphyrobilinogen and 5-aminolevulinic acid during attack
D. Measurement of urine porphyrins
E. Prescription for lubiprostone

X-100. ANje burre 39vjecar vjen per lekuren e dores dhe krahut te lezionuar dhe me flluska qe jane te
dhimbshme. Nxiten nga dielli dhe sherohen duke lene shenja. Ata persistojne pas konsumit te alkoolit
te tepruar. Duart dhe parakrahet kane shenja te hipopigmentuara nga episodet e kaluara. Pjesa dorsal
e dores ka lekure te trashur dhe te ashper. Sipas ekzaminimit fizik pacienti eshte normal. Testimi

174
konfirmon diagnozen e dyshuar. Cili nga trajtimet eshte me I dobishem per te permiresuar ne afatgjate
gjendjen e pacientit:

A. Evito ekspozimin ne diel dhe per lezionet acute hem IV.


B. Hydroxychloroquine 200 mg 2 here ne dite.
C. Phlebotomy of 450 mL of blood every 1–2 weeks
D. Prednison oral 0.5 mg/kg dita
E. Triamcinolone 0.5% topically twice daily

HARRISON 18
X-1. Jane shembuj te feedback-ut negativ hipotalam-hipofize pervec:
A. Cortizoli ne aksin CRH-ACTH
B. Steroidet gonadike ne aksin GnRH-LH/FSH
C. IGF-1 ne aksin (GHRH)-GH
D. Renin-angiotensin-aldosterone
E. Hormonet e tiroides ne aksin TRH-TSH

X-2. Disfunksioni endokrin ndahet ne hiper dhe hipofunksion, ose rezistence ndaj hormonit.
Rezistence ndaj hormonit ka:
A. Semundja Graves
B. Tiroiditi Hashimoto
C. Feokromacitoma
D. Sindromi Sheehan
E. DM 2

X-3.Sekretimi I GnRH stimulon clirimin e LH dhe FSH, qe mundesojne prodhimin e testosteronit dhe
estorgjenit. Cili mekanizem shpjegon me se miri se si leuprolidi (agonisti I GnRh) ul nivelet e
testosteronit ne menaxhimin e kancerit te prostates?

A. GnRH- agonist nxit prodhimin e proteines lidhese (binding-globulin) te hormoneve seksuale, qe ul


testosteronin e disponueshem per perdorim.
B. Feedback negativ midis GnRH dhe LH/FSH.
C. Sensitiviteti I LH dhe FSH ndaj GnRH.
D. Translokacioni I receptorit nuclear citoplazmik me aktivizim te GnRH.

X-4. Receptori mineralokortikoid ne tubujt renal eshte pergjegjes per retensionin e Na dhe per
eleminimin e K gje qe eshte vene re ne rastin e mineralekortikoideve te teperta nga prania e tumoreve
aldosteron sekretues. Gjths keto gjendje mund te paraqiten edhe me retension Na dhe hipokalemi.
Cila e shpjegon kete gjetje:
A. Afiniteti I larte I receptoreve mineralocorticoid per glucocorticoidet.
B. Mbingarkesa e rruges se degradimit te glukokortikoideve nga tepria e glukokortikoideve.
C. Sitet DNA-binding prodhojne te njejtat efekte metabolike.
D. Upregulation I mineralocorticoid-binding ne kushtet e shtimit te glucocorticoideve.

175
X-8. Hyperprolactinemia shkaktohet nga te gjitha pervec:
A. Cirroza
B. Hirsutizmi
C. Thithja e gjirit
D. Opiatet
E. cisti I Rathke

X-9. Nje vajze 28 vjec ka 1 vit me amenore. Thote qe ka galaktore minimale dhe dhimbje koke. Edhe
pse eshte seksualisht aktive, testimi I urines per shtatzani eshte negativ. Prolaktina ne serum eshte e
rritur dhe diagnostikohet me prolaktinome mikroskopike. Terapia me bromokriptine ka si qellim
kryesor:
A. Kontrollon hiperprolactinemine
B. Zvogelon permasat e tumorit
C. Zgjidh galaktorene
D. Rikorrigjon menset dhe fertilitetin
E. Te gjitha

X-13. Nje vajze 23vjecare referon qe ka shtuar shume ne peshe muajt e fundit, ka lodhje, amenore
prej disa muajsh dhe akne te perkeqesuara. Nuk mban mend kur I filuan simptomat por per 6 muaj ka
shtuar 12.3-kg. pas ekzaminimit mund te themi qe ka obezitet trunkal dhe strija lejla ne flanke.
Dyshohet per sindrome Cushing. Cili test duhet per te vene diagnozen:
A. Kortizoli I lire ne urinen 24 oreshe.
B. Matje e ACTH bazal
C. Niveli I CRH ne 8 a.m.
D. Marrja e monstres nga vena petrose inferior.
E. Testi I suprimimit me 1mg dexametazon gjate 1 nate.
X-15. Nje grua 31 vjecare vjen perseri ne spital pas nje apendektomie nga apandesiti akut. Kirurgjia
rezultoi e pakomplikuar por pas nderhyrjes pati 6L/dite urinim dhe etje te theksuar. Diten e trete
postoperatore Bun dhe kreatinina jane te rritura. Pacjentja pyetet dhe kuptohet qe gjate kesaj kohe
ajo ka pasur etje ekstreme, urinim te shpeshte, episode enureze por I vinte turp te tregonte. Pervec
kontraceptiveve oral nuk merr gje tjeter dhe nuk ka pasur semundje me perpara. Cili eshte testi I
radhes per te konfirmuar diagnozen:
A. Osmolariteti dhe volumi urinar 24 oresh.
B. Osmolariteti plazmatik esell ne mengjes
C. Testi I ndalimit te lengjeve
D. MRI truri
E. Niveli plazmatik I ADH

X-16.Nje burre 63vjecar paraqitet per te filluar kimioterapine per leucemine mielomonocitare acute.
Eshte afebril dhe pervec lodhjes dhe nxirjeve nuk ka shqetesime te tjera. Shenjat vitale jane normale
dhe nuk ka gjetje fokale te reja pervec 3 nodujve subkutan 1- × 2-cm. Naten e pare te shtrimit e shoqja
kerkon ndihme pasi statusi mendor i pacientit alterohet, eshte konfuz dhe somnolent. Ju shihni qe
jane 4 mbajtese urine te mbushura anash shtratit te tij. E shoqja raporton qe per 6 oret e fundit ka
urinuar shpesh dhe ka pire uje gjithashtu. Oren e fundit nuk ka mundur te oije uje nga gjendja e
renduar por ka vazhduar te urinoje. Gjetjet laboratorike tregojne neutrofile prej 400, trombocitet
35,000 dhe Na serik 155. Cila terapi duhet filluar menjehere:

A. All-trans retinoic acid (ATRA)


B. Desmopresine
C. Hidroklortiazid
D. Hidrokortizon
E. Litium

176
X-18. Cila nga proteinat eshte burimi paresor I T4 ne plazme:
A. Albumin
B. Gamma globulinat
C. Transtiretin
D. Peroxidaza tiroidiene
E. Tiroksin-binding globulin

X-26. Sa eshte perqindja e pacienteve me hipertiroidizem dhe fibrilacion atrial qe mund te kalojne ne
ritem normal pas trajtimit vetem te tiroides:
A. 20%
B. 30%
C. 50%
D. 70%
E. 90%

X-27. Pershkruan me se miri oftalmopatine ne Graves:


A. Pervecse nje problem estetik, kjo oftalmopati nk eshte ndonje komplikim madhor ocular.
B.Diplopia mund te ndodhe nga enjtja e muskujve periorbital.
C.Nuk eshte gjendur kurre pa pranine e hipertiroidizmit.
D. Abrazioni corneal eshte komplikimi kryesor.
E. Nuk eshte gjetur si semundje unilaterale.

X-28. Mekanizmi I veprimit te propiltiouracilit per trajtimin e Graves:


A. Demton prodhimin e transtiretines
B. Inhibim I prodhimit te immunoglobulinave thyroid-stimuluese
C. Inhibim I funksionit te thyroid peroxidazes.
D. Redktim I shnderrimit periferik te T4 ne T3
E. Reversal of iodine organification

X-29. A Nje 44 vjecar ben aksident me motorr dhe ka demtime ne fytyre kraharor dhe pelvis. Eshte I
pandergjegjshem I intubuar dhe I eshte vene linje IV. Ka demtime te shumta dhe pasi stabilizohet 2
dite ne spital ben nderhyrjet kirurgjikale ne femur dhe humerusin e djathte. Ne gjetjet laboratorike
TSH eshte 0.3 mU/L, dhe total T4 eshte normal. T3 eshte 0.6 μg/dL. Hapi I radhes ne menaxhimin e
pacientit:

A. fillimi I levothyroxines
B. fillimi i prednisonit
C. Observim
D. jod radioaktiv
E. eko tiroideje

X-30. Nje vajze 29vjecare vjen ne klinike me ankesat: veshtiresi ne gelltitje, dihmbje fyti dhe enjtje te
bute te qafes. Ka pasur dhe ethe dhe perpara se ti shfaqe keto simptoma ka pasur nje infeksion te
traktit te siperm respirator.Nuk ka semundje te tjera. Ne ekzaminimin fizik ka shfaqje te gushes te
dhimbshme ne prekje dhe te vogel ne permasa. Orofaringu eshte I lire. Leukocitet jane 14,100 cells/μL,
(ESR) eshte 53 mm/h, dhe (TSH) 21 μIU/mL.Antitrupat tiroidien jane negativ. Diagnoza me e
mundshme:
A.hipotiroidizem autoimun
B. Cat-scratch fever
C. Graves
D. angina Ludwig
E. tiroiditi subakut

177
X-31. Trajtimi per pacietin me lart:
A. largim I jodit nga tiroidja
B. aspirina ne doza te larta
C. radioterapi lokale
D. nuk ka nevoje per trajtim
E. Propiltiouracil

X-32. Cila perputhet me diagnozen etiroiditit subakut?


A. Nje grua 38vjecare me 2 jave me dhimbje tiroideje, T4i rritur, T3 I rritur, TSH e ulur, dhe nje rritje
e thithjes se jodit radioaktiv.
B. Nje burre 42vjecar me 4 muaj histori te dhimbjes se tiroides, lodhje, pafuqi, T4 I lire I ulet, T3 I
ulur, dhe rritje e TSH.
C. Nje vajze 31vjecare me tiroide te zmadhuar te padhimbshme, me TSh te ulur, T4 te rritur, T4 e lire
te rritur, dhe perthithje e rritur e jodit.
D. Nje burre 50vjecar me dhimbje tiroideje, T4 paksa te rritur, TSH normal, dhe nje mase e dukshme
ne eko.

X-34. Nje pacient me kirurgji per nje tumor pituitary I nenshtrohet rezeksionit te gjendres. Cili funksion
duhet shqyrtuar menjehere pas nderhyrjes:
A. Niveli I pikut te kortizolit serik ne mengjes
B. Clirimin e kortizolit si pergjigje ndaj stresit
C. Retensioni I Na si pergjigje ndaj hipovolemise
D. Asnjera

X-43. Faktore risku per DM2 jane te gjitha pervec:


A. BMI > 25 kg/m2
B. Lindja e nje bebi > 3.5 kg
C. HDL< 35 mg/dL
D. Hemoglobin A1C 5.7–6.4%
E. Hipertension sistemik

X-45. Ne pacientet me glukoze esell te demtuar te gjitha nderhyrjet ngadalesojne progresionin e DM2,
pervec
A. modifikime dietike
B. ushtrime
C. Glyburide
D. Metformin

X-47. Te gjitha jane konsistente me/ne retinopatine nonploriferative diabetike, pervec:


A. njolla hemorragjike
B. Cotton-wool spots
C. Neovaskularizim
D. shfaqet ne dekaden e pare ose te dyte te jetes
E. mikroaneurizma vaskulare retinale

X-48. Nje burre 68vjecar me DM2 shume pak te kontrolluar, sillet ne spital me ulcer ne spierfaqen
laterale te kembes se djathte, qe eshte e dhimbshme dhe purulente. Ka pasur 3 dite me ethe. Te gjitha
nderhyrjet rekomandohen per permiresimin me te shpejte te plages se pacientit, pervec:
A. perdorim I duhur I antibiotikeve
B. Debridement
C. O2 hiperbarik
D. Off-loading
E. Rivaskularizimi

178
X-51. The Diabetes Control and Complications Trial (DCCT) provided definitive proof that reduction in
chronic hyperglycemia:
A. permireson komplikmet mikrovaskulare te DM1
B. permireson komplikmet makrovaskulare te DM1
C. permireson komplikmet mikrovaskulare te DM2
D. permireson komplikmet makrovaskulare te DM2
E. permireson komplikimet microvascular dhe macrovascular ne DM2

X-52. A Nje pacient me DM2 vjen per nje follow-up. Hemoglobin A1C eshte jo mire e kontrolluar rreth
9.4%. pacienti duhet te informohet qe sebashku me permiresimin e glicemise do jene te pritshme te
gjithe te tjerat, pervec:
A. ulje microalbuminurie
B. ulje e riskut per nefropati
C. ulje e riskut per neuropati
D. ulje e riskut per semundje vaskulare periferike
E. ulje e riskut per retinopati

X-54. Cili detekton me mire nefropatine diabetike/ eshte me sensitiv:


A. klirensi I kreatinines
B. testi I tolerances ndaj glukozes
C. niveli serik I kreatinines
D. Ultrasonografi
E. albumina urinare

X-55. Alterimi I nivelit te ciles prej substancave eshte mbrojtja e pare kundrejt gipoglicemise:
A. kortizol
B. Epinefrine
C. Glukagon
D. Insuline
E. IGF-I

X-60. Shenjat klinike dhe gjetjet e prezences se nje ovulacioni perfshijne te gjitha t pervec:
A. Zbulimi I LH urinar
B. Piku I estrogjenit gjate fazes sekretore te ciklit menstrual.
C. Rritja e temperatures bazale trupore me shume se 0.5°F ne gjysmen e dyte te ciklit menstrual.
D. Prania e mittelschmerz
E. Niveli I progesteronit> 5 ng/mL 7 dite perpara menseve

X-68. Nje burre 35 vjecar vjen ne urgjence me dhimbje epigastrike, diarre dhe refluks. Ka pasur disa
episode te ngjashme dhe I shte nenshtruar disa endoskopive. Ne cdo rast eshte diagnostikuar me ulcer
duodenale. Ai eshte shume I indinjuar pasi I eshte thene qe shkaku I ulcers eshte bacterial, por
asnjehere nuk ka rezultuar pozitiv ne biopsy per prani te Helicobacter pylori. Mjekimi actual eshte
omeprazol me doza te larta dhe oxycodone/acetaminophen. Ka ardhur ne spital, sepse ka dhimbje.
Hapi I radhes per vleresimin e pacientit:
A. CT abdominale.
B. Ndalo omeprazolin per 1 jave dhe mat nivelin e gastrines serike
C. Mat ph gastrik
D. Niveli I gastrines serike
E. Screening per hiperplazi paratiroideje

179
X-69. Nje 48 vjecare, femer, vjen me diarre dhe skuqje. Ekzaminimi fizik eshte normal me perjashtim
te hepatomegalise nodulare. CT e abdomenit tregon noduse multiple ne dy lobe te heparit, me
metastaza hepatike dhe nje mase 2cm ne ileum. Urina 24 oreshe tregon ekskretim te rritur te 5-HIAA.
Jane trajtime te pershtatshme te gjitha pervec:
A. Diphenhydramine
B. Interferon α
C. Octreotide
D. Ondansetron
E. Phenoxybenzamine

X-70. Ne nje ekzaminim fizik , pacienti I pyetjes 69 , paraqet skuqje , wheezing , nauze dhe marrje
mendsh. Presioni I gjakut eshte 70/30 mmHg dhe frekuenca kardiake eshte 135 rrahje/min . Cila nga
te eshte trajtimi I duhur?
A. Albuteroli
B. Atropina
C. Epinferina
D. Hidrokortizoni
E. Octreotide
X-71. Nje 49 vjecar eshte sjelle ne spital nga familja per shkak te konfuzionit dhe dehidrimit . Familja
thote qe per 3 javet e fundit ka patur fryrje , diarre te ujshme qe nuk I eshte pergjigjur trajtimit
mjekesor. Diarre nuk ka lidhje me konsumin e ushqimit dhe ka persistuar. Diarreja eshte jo e
yndyrshme dhe pa ere te keqe. Pacienti eshte avokat, vegjetarian dhe nuk ka udhetuar se fundmi. Ne
familje nuk I ka njeri tjeter keto simptoma . Para diarrese , ai ka patur anoreksi dhe nje humbje peshe
prej 5-lb. Presioni I gjakut eshte 100/70 mmHg, rrahjet 110 rrahje/min dhe temperatura 36.8°C. Pervec
turgorit te ulut te lekures , konfuzionit dhe dobesise muskulare difuze, ekzaminimi fizik del normal.
Ekzaminimi laboratorik tregon: Na 146 meq/L, K 3.0 meq/L, Cl 96 meq/L, HCO3 36 meq/L, BUN 32
mg/dL, kreatinina 1.3 mg/dL. Mbledhja e feceve 24 orareshe tregon 3 L te ngjyrosura me ngjyre caji.
Sodiumi eshte 50 meq/L dhe osmolaliteti eshte 170 mosm/L. Cili nga testet do jape diagnozen?
A. Kortizoli serik
B. TSH serik
C. VIP serik
D. 5-HIAA urinare
E. Metanefrina urinare
X-72. Nje 18 vjecare po ben nje vizite rutine. Ajo eshte e shendetshme. Babai I saj dhe dy halla kane
patur MEN 1 dhe pacientja ka bere testin gjenetik dhe e ka genin e MEN 1. Kush nga t eshte paraqitja
me e zakonshme ne kete rast?
A. Semundja e ulceres peptike
B. Hipercalcemia
C. Hipoglicemia
D. Amenorrea
E. Hipertensioni sistemik I pakontrolluar

X-73. Nje 35 vjecar referohet te ju per nje hiperkalcemi te pare gjate nje screening. Ai ka lodhje dhe
nje humbje peshe prej 4- lb ne 2 muajt e fundit. Ai ka konstipacion dhe “djegie ne zemer “. Ai ka nauze
pas vakteve te medha dhe shije te hidhur ne goje. Ai nuk ka te vjella , disfagi apo odinofagi. Ka ulje te
libidos dhe depresion. Nuk ka mase tiroidiene, limfadenopati. Presioni jugular eshte normal. Zemra
eshte e paster. Gjoksi po ashtu. Ekzaminimi neurologjik eshte normal. Ca ne gjak eshte 11.2 mg/dL,
fostatet jane 2.1 mg/dL, Mg 1.8 mg/dL, albumina 3.7 g/dL dhe proteina totale eshte 7.0 g/dL. TSH
eshte 3 unite /mL, prolaktina 250 mikrog/L, testosteroni 620 ng/dL dhe IGF -1 normal. Hormoni
paratiroidien eshte normal. Ju beni nje CT qe tregon nje lezion 2×2 cm ne koken e pankreasit. Cila
eshte diagnoza?

180
A. Multiple endocrine neoplasia (MEN) type 1
B. MEN tip 2a
C. MEN tip 2b
D. Sindroma autoimune poliglandulare
E.Sindroma Von Hipel Lindau

X-76. Nje 35 vjecare eahte ne spital ne javen e 37 gestacionale me nje presion gjaku 190/96 mmHg.
Po mjekohet me sulfat magneziumi IV per eklampsine 1g/h per 24 ore. Planifikohet nje cezariane
urgjence. Niveli I Mg do matet cdo 6 ore. Cilat nivele do ishin shqetesuese per depresion te SNQ,
paralize te muskujve te frymemarrjes dhe aritmi kardiake?
A. 0.5 mmol/L
B. 1.0 mmol/L
C. 2.5mmol/L
D. 3.0 mmol/L
E. 5.0 mmol/L

X-83. Te gjitha jane veprime te PTH pervec:


A. Stimiulimit te osteoblasteve per formimin e kockave
B. Stimulimit te osteoklasteve per rezorbimin e kockave
C. Riperthithjes se shtuar te Ca nga tubuli distal I veshkes
D. Ndalim I ripethithjes se fosfatit ne tubulin proksimal te veshkes
E. Stimulim I 1-α-hidroksilazes per te prodhuar 1,25-hidroksikolekalciferol

X-86. Te gjitha semundjet e jane te lidhura me riskun e rritur per osteoporoze pervec:
A. Anoreksia nervore
B. SPOK
C. Insuficienca kardiake
D. Sindromat malabsorbuese
E. Hiperparatiroidizmi

X-89. Cili eshte perkufizimi i testit duke perdorur energji dhe perthithjen e rrezeve X per osteoporoze
(densitometria)?
A. Nje pacient me densitet kockor me pak se norma nisur nga mosha, raca dhe seksi
B. Nje pacient me densitet kockor me pak se 1.0SD poshte normave nisur nga raca dhe seksi
C. Nje pacient me densitet kockor me pak se 1.0SD poshte normave nisur nga mosha, raca dhe seksi
D. Nje pacient me densitet kockor me pak se 2.5SD poshte normave nisur nga rraca dhe seksi
E. Nje pacient me densitet kockor me pak se 2.5SD poshte normave nisur nga mosha, raca dhe seksi

X-90. Nje grua aziatike 66 vjecare kerkon trajtim per osteoporoze. Ajo ka rene dhe ka pesuar frakture
ne ijen e djathte, duke kerkuar nje nderhyrje kirurgjikale 3 muaj me pare. Gjate hospitalizimit, u zbulua
qe vuante nga osteoporoza dhe zhvilloi treomboze te venanve te thella me emoli pulmonare, per te
cilen mori varfarine. Ne moshen 52 vjec perfundoi menopauzen. Ka qene duhanpirese e rregult para
6 vitesh me BMI 19.5 kg/m2. Vleresimet laboratorike tregojne nje Ca 8.7 mg/dL, P 3 mg/dL, kreatinine
0.8 mg/dL dhe 25-hidroksivitamine D 18 ng/mL. desnitometria e kockave tregon nje T-score prej -3.0.
cila do te ishte terapia me e mire per t’iu filluar kesaj pacienteje?
A. Calcitonin 200 IU IV ne dite
B. Karbonat kalciumi 1200 mg dhe vit.D 400 IU ne dite
C. Etinil estradiol 5 μg dhe acetat medroksiprogesteron 625 mg ne dite
D. Raloksifen 60 mg ne dite
E. Risedronate 35 mg 1 here ne jave, karbonat kalciumi 1200 mg dhe vit.D 400 IU ne dite

181
X-91. Nje burri 52 vjecar I eshte zbuluar nje fosfataze alkaline e larte pas nje check-up rutine. Ai kanje
histori hipertensioni dhe hiperlipidemie. Gjithashtu ka kryer kolecistektomi per shkak te gureve ne
temth. Mjekimi I tij eshte losartan 25 mg ne dite, hidroklortiazid 25 mg ne dite dhe rosuvastatin 20 mg
ne dite. Ai eshte fizikisht aktiv me nje BMI 25.2 kg/m2. Ai ankon per dhimbje shpine dhe ne ekzaminim
fizik duket normal. Hepari 10 cm nga perkusioni dhe e palpueshme ne inspirim te thelle ne harkun
brinjor te djathte. Nuk dallohen ndryshime ne palpimin e vertebrave te kolones lumbosakrale. Vlerat
laboratorike jane: ALP 468 U/L, ALT 22 U/L, AST 32 U/L, bilirubine totale 1.0 mg/dL, Ca 9.4 mg/dL, P
3.2 mg/dL dhe γ-glutamyl transferase 20 U/L. Cila eshte diagnoza me e mundshme?
A. Reaksion i kthyer nga rosuvastatin
B. Semundja Paget
C. Ciroza biliare primare
D. Gure te mbetur ne duktusin biliar te perbashket
E. Osteomieliti vertebral
X-92. Cila nga testet e meposhte con drejt diagnozes se pacientit ne pyetjen X-91?
A. Kolangiopankreatografia me MRI
B. MRI per kolonen lumbosakrale
C. Radiografi te kolones lumbosakrale
D. Eko abdominale e kuadrantit te siperm te djathte
E. Osteocalcin ne serum
X-93. Cili test biokimike eshte me e mundshme te jete ne vlerat normale ne semundjen Paget aktive?
A. Fosfataza alkaline ne serum
B. C-telopeptidi ne serum
C. Ca ne serum
D. N-telopeptidi ne serum
E. Osteocalcina ne serum
X-94. Nje grua paraqitet ne pavion 1 jave pasi ka rene ne siperfaqe akulli. Ajo kishte qene ne urgjence
menjehere pas renies dhe ne grafi nuk dukeshin fraktura ose zhvendosje te kockave. Grafia tregonte
fuzion te artikulacioeve sakroiliake dhe trabekula te ashpra te iliumit te lidhura me semundjen Paget.
Ne panelin biokimik verehet nje fosfataze alkaline 257 U/L, me nivel te Ca dhe P ne norme. Mori nje
doze analgjezikesh dhe iu kerkua te ndiqej nga mjeku i familjes per menaxhim te metejshem te
situates. Ajo eshte ne gjendje te mire qe prej rrezimit dhe mohon dhimbje ose imobilitet te
artikulacioneve nga qendrimi ne kembe per nje kohe te gjate. Ajo referon se i ati vuante nga nje
semundje kockore qe I shkaktoi dhimbje koke dhe humbje te degjimit ne fund te jetes se tij. Eshte e
shqetesuar per shpjegimin e grafive te saj. Cili do te ishte strategjia me e mire per trajtim ne kete pike?
A. Te filllohet nje terapi fizike dhe ushtrime force per te forcuar ijet e saj
B. Te fillohet nje terapi me vit.D dhe Ca
C. Te fillohet nje terapi me bifosfonate
D. Te fillohet nje terapi me prednizon 1mg/kg, duke e ulur dozen ne mbi 6 muaj
E. Asnje trajtim sepse eshte asimptomatike

X-95. Nje 32 vjecar vleresohet ne klinike per faktore risku ne lidhje me semundjet e aa. koronare. Ai
eshte i shendetshem dhe jo-duhanpires, me presion gjaku ne norme dhe nuk ka diabet. Ne historine
familjare, nena e tij dhe gjysherit kane patur kolesterol te larte. Ekzaminimi fizik tregon per ksantoma
te tendineve. Ne lipidograme dallohet nje LDL ne vlera 387 mg/dL. Cila nga anomalite genetike eshte
me e mundhshme ne kete pacient?
A. Hiperkolesterolemia dominante autosomale
B. ApoB-100 defektuoze familjare
C. Deficienca e lipazes hepatike familjare
D. Hiperkolesterolemia familjare
E. Deficienca e lipazes se lipoproteinave

182
X-96. Te gjitha jane shkak I LDL se larte pervec:
A. Anoreksise nervore
B. Cirroza
C. Hipotiroidizmi
D. Sindromi nefrotik
E. Diuretiket tiazidike

X-97. Nje 16 vjecar vjen ne klinike me prinderit per shkak te peshes se tij. Ai referon se ka fituar peshe
per shkak te aktivitetit te ulet fizik si pasoje e dhimbjes se gjoksit ne efort. Nuk merr mjekim dhe eshte
i adoptuar. Ekzaminimi fizik tregon hipertension te stadit 1 dhe BMI 30 kg/m2. Ka ksantoma ne duar,
thembra dhe ne te pasme. Testet laboratorike tregojne nje LDL prej 210 mg/dL, kretinine ).7 mg/dL,
bilirubine totale 3.1 mg/dL, haptoglobin nen 6 mg/dL, hemoglobin te glukozuar 6.7%. Ju dyshoni per
lipoproteinemi hereditare sipas ketyre te dhenave. Cili do te ishte testi diagnostic per crregullimet
primare te lipoproteinave ne kete pacient?
A. Biopsi me te kuqen e Kongos
B. Skaner i heparit
C. Analize e genetikes familjare
D. Gas chomatography
E. Biopsi lekure per funksionin e receptoreve LDL

X-98. Nje 60 vjecar me gamopati monoklonale idiopatike kryen nje check-up dhe kerkon shpjegim te
analizave. Kreatinina eshte rritur ne 2.0 mg/dL, K 3.7 mg/dL, Ca 12.2 mg/dL, LDL 202 mg/dL dhe
treigliceridet jane 209 mg/dL. Pacienti referon edema rreth syve ne 3 muajt e fundit dhe nje urine me
“shkume”. Ne ekzaminim ai ka anasarke. I shqetesuar per mielome multiple dhe sindrom nefrotik, ju
kerkoni nje raport protein/kreatinine ne urine, e cila eshte 14:1. Cili do te ishte trajtimi I duhur per te
kuruar crregullimet e tij lipidike?
A. Inhibitor te proteines transportuese te esterit te kolesterolit
B. Menaxhim diete
C. Inhibitor te reduktazes HMG-CoA
D. Afereze lipidike
E. Niacin dhe fibrate

X-102. Burre 58 vjec kryen nje ekzaminim fizik si pjese te sigurimit te jetes. Raporton qe ndihet mire
dhe nuk ka ankesa. Anamneza e kaluar mjekesore tregon per nje hiperlipidemi te lehte dhe nje episod
apandesiti disa vite me pare. Aktiviteti i tij perfshin tenisin 2 here ne jave dhe nuk eshte duhanpires.
Medikamenti i vetem qe perdor eshte atorvastatina. Ne ekzaminimin fizik: obezitet i lehte, shenjat
vitale dhe pjesa tjeter e ekzaminimit eshte normale. Te dhenat laboratorike jane normale me
perjashtim te nivelit te ac. urik 12mg/dl.
Cila nga theniet e lidhur me keto te dhena eshte e vertete?
A. Duhet dhene allopurinol
B. Duhet bere nje vleresim i kujdesshem per rezistencen insulinike
C. Ka rrezik te larte per nefrolitiaze nga ac.urik
D. Shumica e pacienteve me hiperuricemi prodhojne me shume ac.urik se popullsia e pergjithshme
E. Gjate 10 viteve pasardhes shumica e pacienteve me hiperuricemi do te zhvillojne Artrit

X-103. Te gjitha t lidhen me hiperuricemine pervec?


A. Semundja kardiovaskulare
B. Artiti
C. Nefrolitiaze
D. Neuropatia periferike
E. Nefropatia e ac urik

183
X-104. Grua 28 vjec kerkon te kryeje keshillimin prekoncepsional. Ka patur nje vella qe vdiq ne moshen
9 vjecare nga Sd. Lesch-Nyhan dhe njihet si bartese e ketij defekti gjenetik. Nuk ka histori mjekesore
te kaluar te rendesishme dhe burri i saj nuk ka te dhena sinjifikante ne anamnezen familjare. Cila nga
pohimet eshte e vertete?
A. Femijet e saj nuk do kene rrezik per semundjen pasi ajo nuk eshte simptomatike
B. Te behet kontroll nese burri i saj eshte bartes i ketij defekti gjenetik.
C. Nese lind nje vajze do te kete shansin 50% per te qene bartese
D. Nese lind djale i prekut fillo perdor allopurinolin pas lindjes per te parandaluar manifestimet
klinike te semundjes
E. Ajo duhet te filloje te perdori allopurinol per te ulur rrezikun e artitit dhe nefropatise uremike

X-107. Eshte e vertete per semundjen Wilson:


A. Diagnostikimi I hershem eshte kyc per efikasitetin e trajtimit.
B. Eshte semundje autozomale dominate.
C. Niveli I bakrit serik eshte 2-3 here me I larte se norma.
D. Frekuenca esemundjes ne popullaaten e pergjithshme eshte rreth 1%.
E. Organet me te prekura jane heparin dhe pancreasi.

X-109. Me shume se 90% e pacienteve me sindromin Marfan kane mutacion ne gjenin:


A. BMPR2
B. COL1A1
C. Fibrillin
D. TGFb
E. Type IV collagen

X-110. Femer 21 vjec paraqitet ne klinike me keshillen e instruktorit te saj te yoges. Kohet e fundit
filloi klasat oer te rritur nivelin e aktivitetit dhe instruktori i saj tha qe artikulacionet e saj duken shume
flexible, vecanerisht duke patur parasysh pervojen e saj. Gjate ushtrimit te "Akrepit" ishte ne gjendje
te zgjatej me shume se shoket e saj. Raporton per nje shendet te mire dhe shumicen e kohes jete
sedentare. Gjate jetes se saj ka qene e afte te performoje veprime te tolerances artikulare. Ekz.fizik
tregon nje lekure prej kadifeje dhe artikulacione fleksibile. Eshte ne gjendje ti shtrije kycet e dores te
pakten 90°. Cila nga pohimet lidhur me kete pc eshte e vertete?
A. Ka rrezik per disekacion ose rupture aorte ne 20 vitet e ardhshme
B. Ka rrezik per dislokacion te pelvisit ne 20 vitet e ardhshme
C. ka rrezik per rupture uterine gjate shtatzanise
D. ka te ngjare te kete nje mutacion te gjenit te elastines dhe te gjenit te fibrilines.

Case files internal medicine fifth edition

Nje grua 65 vjec sillet ne urgjence nga familjaret me konfuzion dhe letargji qe prej 1 jave. Eshte
diagnostikuar sefundmi me kancer pulmonary me qeliza te vogla, por nuk ka filluar akoma mjekimin.
Nuk ka qene febrile dhe ska patur ndonje semundje tjeter sefundmi. Nuk merr asnje medicament.
Presioni arterial eshte 136/82 mm Hg, frekuenca kardiake 84 bpm, frekuenca respiratore 14 bpm. ne
ekzaminimin fizik zonja ngjan me e vjeter qe e ka te veshtire te qendroje zgjuar apo te reagoje, dhe
reagon vetem ndaj stimujve te dhimbshem. Mund te levize gjymtyret pa fecicite motore te dukshme
dhe refleksi e tendineve te thella jane te ulura simetrikisht. Pjesa vijuese e ekzaminimit eshte normale,
me presio venoz jugular normal dhe pa edema ekstreme. Ne testet laboratorike, niveli I Na eshte 108
mmol/L, K 3.8 mmol/L, bicarbonate 24 mEq/L, BUNeshte 5 mg/dL, creatinina 0.5 mg/dL, osmolaliteti
serik 220 mOsm/kg dhe osmolaliteti urinar eshte 400 mOsm/kg. CT nuk shfaq masa trunore ose
hidrocefali. » Cila mund te jete diagnoza?
» Hapi I radhes per te diagnostikuar:

184
» Cfare komplikimesh ka trajtimi?
Diagnoza me e mundshme: Coma/ letargji sekondare nga hiponatremia se rende se, e cila ka shume
gjasa te jete shkaktuar nga sindroma paraneoplazike nga SIADH.
Hapi I radhes per te diagnostikuar: Hiponatremia me tretesire hipertonike.
Komplikimet qe ka trajtimi: demielinizimi osmotic cerebral, e njohur ndryshe si mielinoza centrale
pontine.

47.1 Nje djale 24 vjecar peson nje aksident automobilistik dhe nevojitet splenektomi. Niveli I Na serik
fillimisht eshte 116 mEq/L, pastaj korrigjohet ne 120 mEq/ L per 3 oret e ardhshme. Cfare e ka
shkaktuar hiponatremine
A. Vazopresina serike e rritur
B. Administrimi I solucioneve hypertonike
C. Humbja volumore
D. krize induktuese hyponatremike

47.2 Nje burre 56 vjec paraqitet te mjeku me lodhje dhe humbje peshe. Nuk ka pasur problem
shendetesore dhe per 35 vite ka pire nga 1 pakete ne dite. Ai eshte punetor dhe punon me dite, per
momentin eshte I pastrehe dhe jeton ne qendra strehimi. Ne ekzaminim fizik vihet re nje presion
arterial disi I ulur-normal, hiperpigmentim lekure dhe clubbing digital (thonj govate). Pacienti eshte
euvolemik, dhe ju I thoni qe nuk jeni akoma I sigurt cfare problem ka pacienti prandaj do behen disa
analiza laboratorike dhe nje vizite pas nje jave. Laboratori ju telefonon ate nate dhe ju informon qe
niveli I Na eshte 126 mEq/ L, kaliumi eshte 6.7 mEq/ L, creatinina eshte normale, and bicarbonatet
dhe klori jane te ulura. Cfare ka shume gjasate kete shkaktuar hiponatremine?
A. SIADH
B. hipotiroidizem
C. humbje gastrointestinal
D. insuficence adrenale
E. insuficence renale

47.3 Nje 83vjecare vjenne klinike me ankesat konfuzion I lehte dhe dhimbje koke. Ne historikun e
semundjeve, zonja vuan nga hipertensioni dhe mjekohet me hidroklortiazid. Nuk ka shenja infeksioni
ne testimet laboratorike, por niveli I Na eshte 119mEq/L dhe osmolariteti plazmatik is 245 mOsm/ kg.
Klinikisht zonja eshte hipovolemike. Terapia fillestare me e mire pershin:
A. ndalim lengjesh
B. Infusion NaCl 0.9%
C. Infusion NaCl 3%
D. Infusion NaCl 3% me furosemid

47.4 Nje 58 vjecar I eshte nenshtruar nje kirurgjie per kancer te kolonit. Diten e pare postoperatore
pacienti ka hiponatremi me nivel Na 128 mEq/l. Dyshoni per hiponatremi nga infuzioni IV I solucioneve
hipotonike. Cila gjetje laboratorike mbeshtet kete diagnoze?
A. Na urinar > 20 mmol/ L
B. osmolaliteti urinar > 200 mOsm/ L
C. osmolariteti serik<280 mOsm/ kg
D. kaliumi serik>5 mEq/ L

A 38-year-old woman presents to your clinic for evaluation of menstrual irregularity. She states that
her periods started when she was 12 years old, and they have been fairly regular ever since, coming
once every 28 to 30 days. She has had three previous uncomplicated pregnancies and deliveries.
However, approximately 9 months ago, her cycles seemed to lengthen, and for the last 3 months she
has not had a period at all. She stopped breast-feeding 3 years ago, but over the last 3 months she

185
noticed that she could express a small amount of milky fluid from her breasts. She had a bilateral tubal
ligation after her last pregnancy, and she has no other medical or surgical history. She takes no
medications except multivitamins. Over the last year or so, she thinks she has gained about 10 lb, and
she feels as if she has no energy despite adequate sleep. She has noticed some mild thinning of her
hair and slightly more coarse skin texture. She denies headaches or visual changes. Her physical
examination, including pelvic and breast examinations, are normal. She is not obese or hirsute. Slight
whitish nipple discharge is elicited from her breasts. Her pregnancy test is negative.
» What is the most likely diagnosis?
» What is the most likely etiology for the condition?
Most likely diagnosis: Oligomenorrhea and galactorrhea due to hypothyroidism.
Most likely etiology: Primary hypothyroidism is the most likely diagnosis, most often due to
autoimmune (Hashimoto) thyroiditis.

48.1 Nje gua 42 vjecare vjen per kontrollet vjetore te saj. Ne palpim te diroides, dalloni qe eshte e
zmadhuar, e bute. Pacientja eshte asimptomatike.ne testimet e funksionit tiroidien T4, T4 I lire dhe
T3 jane ne norme, por TSH eshte pak I rritur. Diagnoza e mundshme:
A. Deficit jodi
B. Kancer tiroideje
C. Hashimoto
D. Graves
E. Gushe multinodulare

48.2 Cila analize laboratorike nevojitet per te konfirmuar diagnozen per pacientin ne pyetjen 48.1?
A. perserit testet funksionale te tiroides
B. Eko tiroideje
C. Nuclear thyroid scan
D. testi I antitrupave antiperoksidaze TPO
E. Complete blood count with differential

48.3 Nje gjimnaste 19 vjec qe garonte per konkursin kombetar, sillet ne klinike nga e ema sepse nuk
ka pasur cikel menstrual per 3 muaj. Kjo eshte hera e pare qe I ndodh dhe vajza ka pasur cikel te
rregullt. Mohon te kete mbajtur diete, edhe pse stervitet per 3 ore ne dite. Ekzaminimi fizik I saj tregon
qe eshte normale, pervec BMI ne vlere 20 kg/ m2. Cila analize laboratorike duhet bere e para?
A. Testet e funksionit tiroidien
B. Gjak komplet
C. Luteinizing hormone /follicle-stimulating hormone (LH)/ (FSH)
D. Prolaktina
E. Beta-hCG

48.4 Nje grua 35 vjec e diagnostikuar me hipotiroidizem 4 jave me pare, paraqitet ne klinike me lodhje
dhe plogeshti. Pasi rikonfirmoni diagnozen duke matur TSH, filloni mjekimin me levothyroxine 50 μg
doze ditore. Ajo ka lexuar per diagnozen e saj ne internet dhe deshiron te provoje desiccated thyroid
extract (tiroide qe thahet dhe pluhuroset pastaj perdoret per hipotiroidizem) ne vend te mjekimit qe
ju I dhate. Ne ekzaminim fizik, pasha e saj eshte 175 lb, frekuenca kardiake 64 bpm ne qetesi, presioni
arterial normal. Hapi I radhes eshte:
A. Tregoni pacientes qe eshte normale qe simptomat mos te largohen qe ne fillim te trajtimit, dhe
kerojini te kthehet per konsulte pas 2 muajsh.
B. Ndryshoi mjekimin ashtu sic kerkon pacientja.
C. Rrit dozen e levotiroksines dhe kerkoji te kthehet pas 4 javesh per vizite.
D. Thuaji te nise multivitamina me hekur, pervec levotiroksines.

186
A 58-year-old woman comes to the office after she experienced a near-fainting spell 1 day ago. She
was outside playing tennis when she vomited and felt lightheaded. She spent the rest of the day lying
down with mild, diffuse, abdominal pain and nausea. She had no fever or diarrhea. She reports several
months of worsening fatigue, mild, intermittent, generalized abdominal pain, and loss of appetite with
a 10- to 15-lb unintentional weight loss. Her medical history is significant for hypothyroidism for which
she takes levothyroxine. She takes no other medications. On examination, her temperature is 99.8°F,
heart rate is 102 bpm, blood pressure is 89/62 mm Hg, and normal respiratory rate. She does become
light-headed, and her heart rate rises to 125 bpm upon standing with a drop in systolic blood pressure
to 70 mm Hg. She is alert and well tanned, with hyperpigmented creases in her hands. Her chest is
clear, and her heart rhythm is tachycardic but regular. On abdominal examination, she has normal
bowel sounds and mild diffuse tenderness without guarding. Her pulses are rapid and thready. She
has no peripheral edema. Initial laboratory studies are significant for Na 121 mEq/L, K 5.8 mEq/L, HCO3
16 mEq/L, glucose 52 mg/dL, and creatinine 1.0 mg/dL.
» What is the most likely diagnosis?
» What is your next step?
Most likely diagnosis: Primary adrenal insufficiency.
Next step: After drawing a cortisol level, immediate administration of intravenous saline with glucose
and stress doses of corticosteroids.

49.1 Shkaku me I zakonshem I insuficences adrenale:


A. proces autoimun
B. ekscizioni kirurgjikal
C. shoku hemorragjik
D. corticosteroidet ekzogjene
E. mungesa e ACTH nga panhipopituitarizmi

49.2 Nje grua 30 vjecare merr prednisone 15 mg/d per lupusin eritematoz. Vjen ne spital per
kolecistoektomi. Nderhyrja me e volitshme per te:
A. Hydrocortisone IV perpara kirurgjise dhe cdo 6 ore per 24 ore.
B. Dyfishim I prednizonit naten para nderhyrjes dhe steroide diten e kirurgjise.
C. Jep cyclophosphamide ne vend te corticosteroideve per 2 jave pas nderhyrjes per sherim te shpejte.
D. Anullo nderhyrjen dhe kerko litotripsi per thyerjen/shkrirjen e kalkulave.

49.3 Nje 30 vjecare grua 12 jave postpartum ka insuficence adrenale dhe lekure te nxire edhe pse nuk
eshte ekspozuar ne diell. Etiologjia me e mundshme:
A. Perdorimi afatgjate I steroideve
B. Sindromi Sheehan (pituitary insufficiency)
C. Tumor truri
D. Shkaterrimi/demtimi autoimun i gjendres adrenale

49.4 Testi me I mire per diagnostikimin e syndromes Cushing:


A. Random cortisol level
B. ACTH-stimulation test
C. Overnight 1-mg dexamethasone suppression test
D. Pituitary MRI

A 63-year-old African-American woman is brought to the emergency center for upper arm pain and
swelling following a fall at home. The family has noted that for approximately the past 2 months, the
patient has become progressively fatigued and absentminded, and she has developed loss of appetite
and weight loss. She has been getting up to urinate several times per night and complains of thirst;
however, a glucose test for diabetes in her doctor’s office was negative. This morning, she lost her

187
balance because she felt “light-headed” and fell, landing on her left arm. Physical examination is
notable for an elderly, thin woman in mild distress as a result of pain. She is afebrile, and her blood
pressure is 110/70 mm Hg and heart rate is 80 bpm. Her thyroid gland is normal to palpation. Her
mucous membranes are somewhat dry and sticky. Heart and lung examinations are normal, and
carotid auscultation reveals no bruits. Examination of her extremities is significant
only for deformity of the left mid-humerus with swelling. The left radial pulse is 2+ and symmetric.
The radiologist calls you to confirm the fracture of the mid-left humerus but also states that there is
the suggestion of some lytic lesions of the proximal humerus and recommends a skull film (Figure 50–
1). Serum creatinine level is 2.1 mg/dL, with normal electrolyte and glucose concentrations, but serum
calcium level is 13 mg/dL and hemoglobin level is 9.2 g/dL.
» What is the most likely diagnosis?
» What is the most likely underlying etiology in this patient?
» What is your next therapeutic step?
Most likely diagnosis: Hypercalcemia with pathologic fracture of the left humerus.
Most likely underlying etiology: Multiple myeloma.
Next therapeutic step: Initial therapy of the hypercalcemia with intravenous (IV) fluids could be
started in the emergency room.

50.1 Ne nje analize gjaku ten je grua 48 vjec ne premenopauze u gjet nivel I kalciumit 12 mg/ dL
(normal = 8.8-10.4 mg/ dL) dhe fosfati 2 mg/ dL (normal = 3.0-4.5 mg/ dL). Historiku mjekesor I zonjes
eshte tregues per osteoporze, e zbuluar me DEXA vitin e kaluar.shkaku per kiperkalcemi te zonja eshte:
A. Multiple myeloma
B. Parathyroid adenoma
C. Familial hypocalciuric hypercalcemia
D. Sarcoidosis
E. Undiagnosed breast cancer

50.2 Nje grua 62 vjecare ka mielome multiple dhe hiperkalcemi, por pa lezione kockore ose demtime
organore. Terapia immediate per trajtimin e hiperkalcemise eshte:
A. Bisphosphonates.
B. Erythropoietin.
C. Dexamethasone + thalidomide.
D. Interferon-alpha.
E. mbaj ne observim dhe mos fillo trajtim per aq kohe sa eshte asimptomatike.

50.3 Nje vajze 22 vjecare vjen me kolle qe ka ardhur duke u perkeqesuar ne 6 jave, edhe pse ka
perdorur antibiotike dhe antitusive. Kalciumi serik eshte 12.5 mg/ dL dhe Rx kraharori tregon
limfadenopati hilare bilaterale. Ne kembe ka eriteme nodoze. Ka shume gjasa te kete:
A. Sarcoidosis
B. Mycoplasma pneumonia
C. Acute lymphoblastic leukemia
D. Squamous cell carcinoma of the lung
E. Pulmonary embolism

50.4 Nje burre 66 vjec memetastaza ezofagale me qeliza skuamoze sillet ne urgjence me konfuzion
dhe letargji. Eshte I dehidruar dhe kalciumi serik I tij eshte 14 mg/ dL, ndersa kreatinina eshte 2.5 mg/
dL edhe pse para nje muaji ishte 0.9 mg/ dL. Cila eshte terapia fillestare per hiperkalcemine n
A. bisphosphonate IV B. furosemide IV C. Glucocorticoids
D. Intravenous normal saline
E. Chemotherapy for squamous cell carcinoma

188
While seeing patients in your preceptor’s clinic, you have the opportunity to meet and examine one
of her long-time patients, a 52-year-old woman who presents for her yearly physical examination. She
has been fine and has no complaints today. Her medical history is notable only for borderline
hypertension and moderate obesity. Last year her fasting lipid profile was acceptable for someone
without known risk factors for coronary artery disease. Her mother and older brother have diabetes
and hypertension. At prior visits, you see that your preceptor has counseled her on a low-calorie, low-
fat diet and recommended that she start an exercise program. However, the patient says she has not
made any of these recommended changes. With her full-time job and three children, she finds it
difficult to exercise, and she admits that her family eats out frequently. Today her blood pressure is
140/92 mm Hg. Her body mass index (BMI) is 29 kg/m2. Her examination is notable for acanthosis
nigricans at the neck but otherwise is normal. A Papanicolaou (Pap) smear is performed, and a
mammogram is offered. The patient has not eaten yet today, so on your preceptor’s recommendation,
a fasting plasma glucose test is performed, and the result is 140 mg/dL.
» What is your diagnosis?
» What is your next step?
Most likely diagnosis: Given her obesity, family history, and the finding of acanthosis nigricans, this
patient most likely has type 2 diabetes. Diagnostic criteria for diabetes as defined by the American
Diabetes Association (ADA) include (1) symptoms of diabetes and (2) fasting plasma glucose of 126
mg/ dL or greater.
Next step: Dietary counseling, assess for end-organ disease, and check hemoglobin A1C (A1c).

51.1 Nje pacient vjen per testin e glukozes esell. Ne dy matje te ndryshme rezultatet jane 115 mg/ dL
dhe 120 mg/dL. Cili eshte hapi I radhes?
A. Keto jane nivele normale te glukozes ne gjak.
B. Rekomando te humbasi peshe, diete ADA (dieta e diabetikeve), dhe ushtrime.
C. Diadnostiko pacientin me DM dhe fillo trajtimin me sulfanilure.
D. Rekomando testin e stresit kardiak.
E. Mat gazin arterial dhe nivelin e ketoneve serike.

51.2 Nje grua 54 vjecare vjen per kontroll rutine te diabetit. Merr metformine 1000 mg 2 here ne dite
dhe glukoza esell e saj eshte midis 170 -200 mg/ dL. HbA1C e fundit e matur eshte 7.9. Ajo thote qe I
permbahet dietes dhe ecen 30min-1 ore nedite. Hapi I radhes:
A. Refero te endokrinologu per nje pompe insulinike.
B. Stop metforminen dhe starto glimepiridinen.
C. Shto 1 here ne dite injeksion insulin glargine (Lantus).
D. Hospitalizo menjehere pacienten.

51.3 Nje grua 75 vjecare diabetike prej 20 vitesh, ka retinopati dhe nefroati diabetike, me kreatinine
2.2 mg/ dL. Ajo sillet nga e bija per kontroll. Aktualisht merr sulfanilure dhe Ace-I per proteinuri. E bija
raporton qe ne 2 javet e fundit e ema ka pasur 3 raste konfuzioni, me dridhje dhe djersitje qe
zgjidheshin me dhenien e nje lengu portokalli. Nga se mund te kene ardhur keto episode?
A. Marrja e tepert kalorike (orale)
B. Nderveprimi ACE inhibitor dhe sulfonylurea
C. Perkeqesim I funksionit renal
D. Amnezi hiperglicemike

51.4 Nje grua 42 vjecare me diabet gestacional nga shtatezania e fundit para 10 vitesh, po
ekzaminohet per diabet tip 2. Ka sensitivitet me te larte per te:
A. glukoza esell B. testi I tolerances se glukozes pas 2 oresh
C. Hemoglobin A1C D. Random glucose

189
51.5 Nje burre 36 vjecar sefundmi eshte diagnostikuar me diabet tip 2. Nqs nuk eshte vaksinuar me
pare, cili imunizim eshte me I rendesishmi te behet:
A. Pneumococcal
B. Human papilloma virus
C. Hepatitis B
D. Rubella
E. Toxoplasmosis

An 18-year-old woman is brought to the emergency department by her mother because the daughter
seems confused and is behaving strangely. The mother reports the patient has always been healthy
and has no significant medical history, but she has lost 20 lb recently without trying and has been
complaining of fatigue for 2 or 3 weeks. The patient had attributed the fatigue to sleep disturbance,
as recently she has been getting up several times at night to urinate. This morning, the mother found
the patient in her room, complaining of abdominal pain, and she had vomited. She appeared confused
and did not know that today was a school day.
On examination, the patient is slender, lying on a stretcher with eyes closed, but she is responsive to
questions. She is afebrile, and has a heart rate of 118 bpm, blood pressure of 125/84 mm Hg, with
deep and rapid respirations at the rate of 24 bpm. Upon standing, her heart rate rises to 145 bpm, and
her blood pressure falls to 110/80 mm Hg. Her funduscopic examination is normal, her oral mucosa is
dry, and her neck veins are flat. Her chest is clear to auscultation, and her heart is tachycardic with a
regular rhythm and no murmur. Her abdomen is soft with active bowel sounds and mild diffuse
tenderness, but no guarding or rebound.
Her neurologic examination reveals no focal deficits.
Laboratory studies include serum Na 131 mEq/L, K 5.3 mEq/L, Cl 95 mEq/L, CO2 9 mEq/L, blood urea
nitrogen (BUN) 35 mg/dL, creatinine 1.3 mg/dL, and glucose 475 mg/dL. Arterial blood gas reveals pH
7.12 with PCO2 24 mm Hg and PO2 95 mm Hg. Urine drug screen and urine pregnancy test are
negative, and urinalysis shows no hematuria or pyuria, but 3+ glucose and 3+ ketones. Chest
radiograph is read as normal, and plain film of the abdomen has nonspecific gas pattern but no signs
of obstruction.
» What is the most likely diagnosis?
» What is your next step?
Most likely diagnosis: Diabetic ketoacidosis (DKA).
Next step: Aggressive hydration to improve her volume status and insulin therapy to resolve the
ketoacidosis

52.1 Cila mund te coje ne acidoze pa ndryshim te gapit anionic?


A. Diarrhea B. Lactic acidosis C. Diabetic ketoacidosis D. Ethylene glycol ingestions

52.2 Nje djale 18 vjecar, ka ketoacidoze diabetike me pH 7.20 dhe glukoze ne serum 400 mg/ dL. Eshte
e vertete per nivelin e Na te pacientit:
A.Na serik <3 mEq/ L.
B.Na serik >5 mEq/ L.
C.Ka deficit te Na total ne trup para se te kete ulje serike te Na.
D.Me korrigjimin e acidozes do normalizohet niveli serik I Na.

52.3 Hapi I pare per trajtimin e ketoacidozes diabetike:


A. Zevendesim I K
B. Zevendesim I lengjeve IV
C. Zevendesim I P
D. Terapi me antibiotike

190
52.4 Nje burre 59 vjecar me diabet prej nje kohe te gjate, me IRK nga nefropatia diabetike, vjen ben
nje kontroll laboratorik. Eshte asimptomatik , glukoza e rritur ne 258 mg/ dL, dhe nivelet e tjera jane:
Na 135 mEq/ L, K 5.4 mEq/ L, Cl 108 mEq/ L, dhe bicarbonate 18 mEq/ L ndersa creatinina eshte stabel
2.1 mg/ dL. Shkaku I acidozes eshte:
A. Diabetic ketoacidosis
B. Lactic acidosis
C. Type 4 renal tubular acidosis
D. Accidental salicylate overdose

A 37-year-old previously healthy woman presents to your clinic for unintentional weight loss. Over the
past 3 months, she has lost approximately 15 lb without changing her diet or activity level. Otherwise,
she feels great. She has an excellent appetite, no gastrointestinal complaints except for occasional
loose stools, a good energy level, and no complaints of fatigue. She denies heat or cold intolerance.
On examination, her heart rate is 108 bpm, blood pressure 142/82 mm Hg, and she
is afebrile. When she looks at you, she seems to stare, and her eyes are somewhat protuberant. You
note a large, smooth, nontender thyroid gland and a 2/6 systolic ejection murmur on cardiac
examination, and her skin is warm and dry. There is a fine resting tremor.
» What is the most likely diagnosis?
» How could you confirm the diagnosis?
» What are the options for treatment?
Most likely diagnosis: Thyrotoxicosis/ Graves disease.
Confirming the diagnosis: A low serum thyroid-stimulating hormone (TSH) level and an increased free
thyroxine (T4) level with this clinical presentation would be confirmatory of hyperthyroidism.
However, other tests that would define the etiology would be thyroid-stimulating immunoglobulins
or diffusely elevated uptake of radioactive iodine on thyroid scan.
Treatment options: Antithyroid drugs, radioactive iodine ablation, or less commonly, surgical removal
of the thyroid.

53.1 Nje 44 vjecare eshte nervos dhe intolerant ndaj nxehtesise. Tiroidja e saj eshte e zmadhuar, jo Ie
bute dhe me audible bruit. TSH eshte shume e ulur. Etiologjia mund te jete:
A. Lymphocytic thyroiditis
B. Hashimoto thyroiditis
C. Graves disease
D. Gushe multinodulare toksike

53.2 Dallon hyperthyroidizmin nga stuhia tiroidiene


A Tachycardia me frekuence kardiake 120 bpm
B. Humbja e peshes
C. Ethe dhe delir
D. Gusha e zmadhuar

53.3 Nje grua 58 vjecare diagnostikohet me Graves dhe ka gushe jo shume te zmadhuar. Terapia me
e mire eshte:
A. Long-term propranolol
B. Lifelong oral propylthiouracil (PTU)
C. Radioactive iodine ablation
D. Surgical thyroidectomy

191
LICENSE

1. Te gjitha gjendjet e mund te sjellin hipoglicemi pervec njeres:


A) Insulinoma
B) Deficiti i glukoze-6-fosfatazes
C) Morbus Addison
D) Feokromacitoma
E) Administrimi i insulines

2. Te gjitha pohimet e mbi sindromen e dehidrimit jane te sakta pervec njeres:


A) Ne komen diabetike ketoacidozike, dehidrimi ndodh si pasoje e humbjeve urinare te Na+ dheujit
B) Gjate diarrese mund te verehet dehidrim hipotonik i shoqeruar me hiponatremi
C) Te vjellat profuze provokojne hipernatremi me acidoze metabolike
D) Abuzimi me diuretike mund te provokoje dehidrim hipernatremik, ose me rralle hiponatremik
E) Ne diabetin e dekompensuar ka humbje urinare te Na+

3. Nje ikter i lehte, i zbuluar gjate ekzaminimit te sklerave ne drite natyrale, zakonisht
evidentohetkur vlerat e bilirubines plazmatike jane ndermjet:
A) 1.5-2 mg/dl
B) 3.5-4.5 mg/dl
C) 2.5-3 mg/dl
D) 1-1.5 mg/dl
E) > 5 mg/dl

4. Cili nga pohimet e ben pjese ne rekomandimet e parashikuara nga guidelinet nderkombetare?
A) Kryerja e mamografise duke filluar nga mosha 40 vjec
B) Kryerja e kolonoskopise cdo 5 vjet duke filluar nga mosha 50 vjec
C) Matja e kolesterolemise totale ne moshen 35 vjec
D) Kryerja e ekzaminimit oftalmologjik cdo vit duke filluar nga mosha 50 vjec
E) Matja e presionit arterial nje here ne vit nga te gjitha moshat

5. Ne demtimin akut pulmonar raporti PaO2/FiO2 (ku FiO2 eshte perqendrimi i O2 ne ajrin
einspiruar) eshte:
A) Me pak se 100
B) Midis 100-150
C) Midis 150-200
D) Me pak se 200
E) Midis 200-300

6. Si paraqitet dispnea?
A) Gjithmone inspiratore
B) Gjithmone ekspiratore
C) Gjithmone inspiratore dhe ekspiratore
D) Varet nga presionet ne qarkullimin e vogel
E) Pavaresisht neqoftese eshte inspiratore ose ekspiratore

7. Gjate sinkopit vaso-vagal vihet re:


A) Hipertoni vagale akute
B) Stimulum i refleksit simpatik
C) Reduktim i prurjes kardiake
D) Te gjitha opsionet e mesiperme
E) Asnje nga opsionet e mesiperme

8. Cila nga patologjite e mund te jete shkak i insuficences renale akute postrenale?
A) Pielonefrit

192
B) Diureze e shtuar
C) Obstruksion i arterieve renale
D) Hipertrofi e prostates
E) Kalkuloze vezikale jo obstruktive

9. Cianoza, eshte e zakonshme ne te gjitha gjendjet e pervec:


A) Poliglobuli
B) Emboli pulmonare masive
C) Methemoglobinemia
D) Ekspozimi i zgjatur ne lartesi te medha mbi nivelin e detit
E) Anemia grave

10. Etiologjia e anoreksise nervore eshte pasoje e:


A) Reduktimit te nivelit te LH-RH
B) Reduktim te nivelit te LH
C) Reduktim te nivelit te IGF-I (somatomedina
D) Ulje e funksionit te tiroides
E) Crregullim i sjelljes se ushqyerjes mbi baze Psikike

11. Cilat jane alterimet hemodinamike ne shokun hipovolemik?


A) Hipotension, bradikardi, rezistenca periferike normale ose te uleta, ulje te presionit venozqendror
B) Hipotension, takikardi, rritje te rezistencave periferike, ulje te presionit venoz qendror
C) TA normal ose i rritur, frekuence kardiake normale ose te ulur, rezistenca periferike tereduktuara,
rritje te presionit venoz qendror
D) Hipotension, takikardi, rezistenca periferike te reduktuara, presion venoz i ulet
E) Nuk ka alterime hemodinamike se vihen ne funksion mekanizmat kompensatore

12. Cila nga alternativat e ka nje funksion fiziologjik te rendesishem te kriprave biliare?
A) Konjugim i tyre me toksinat ne menyre qe te favorizohet ekskretimi i tyre
B) Lejimin e eliminimit te produkteve si pasoje e shkaterrimit te hemoglobines
C) Lehtesimin e absorbimit te yndyrnave ushqimore
D) Lejon absorbimin e Vit B12
E) Ruajtja e nivelit normal te pH intenstinal

13. Versamenti pleural hemoragjik, gjendet ne te gjitha situatat e pervec:


A) Emboli pulmonare
B) Kancer pulmoni
C) Dekompensim kardiak
D) Mezotelioma
E) Kancer gjiri

14. Te gjitha rastet e jane shkak i hipokalcemise pervecse:


A) Hipoparatiroidizmi
B) Sindroma Cushing
C) Rritje e sekretimit te kalcitonines4
D) Hiperparatiroidizmi
E) Hipovitaminoza D

15. Embolia gazore nen ujore favorizohet nga:


A) Hiperventilim nga frymemarrja e drejtuar
B) Hiperventilimi me O2
C) Zhytje ne uje ne periudha kohore te shkurtra
D) Aktivitet fizik i pamjaftueshem
E) Raca kaukaziane

193
16. Osteoporoza e tipit te I eshte pasoje e deficitit te hormonit:
A) Kalcitonine
B) Testosteronit
C) Estrogjenit
D) Parathormonit
E) Hormonit tireotrop

17. Niveli plazmatik i peptidit natriuretik atrial ne nje pacient me dekompensim kardiak kronik
eshte i lidhur me:
A) Moshen e pacientit
B) Seksin dhe moshen e pacientit
C) Graden e demtimit funksional
D) Vlerat e presioni sistolik
E) Nivelin e atherosklerozes te arterieve koronare

18. Cila nga barnat e eshte zgjedhja e pare ne parandalimin e fibrilacionit ventricular gjate
infarktit akut te miokardit?
A) Lidokina
B) Digoksina
C) Kinidina
D) Flekainidi
E) Propanololi

19. Gjate polisonografise mund te vleresohen te gjithe parametrat e meposhtem pervec nje:
A) Elektroencefalograma
B) Elektrokardiograma
C) Saturimi i O2
D) Diureza
E) Elektromiografia tibiale

20. Si konstatohet prania e edemes ne nje gjymtyre?


A) Palpimi siperfaqesor i zones se dyshuar
B) Kompresioni digital mbi zonen kockore
C) Vleresimi manual i temperatures lokale
D) Manovra e Perth
E) Palpimi i pulsit arterial

21. Cila nga anomalite elektrolitike vihen re me shpesh ne morbus Addison?


A) Hiperglicemi
B) Hipokalcemi
C) Hipokalemi
D) Hipernatremi dhe hipokalemi
E) Hiponatremi dhe hiperkalemi

22. IgA prodhohen nga:


A) Pneumocitet e tipit te I
B) Pneumocitet e tipit te 2
C) Qelizat endoteliale
D) Qelizat e epitelit bronkial
E) Makrofaget alveolare

23. Acidoza respiratore e dekompesuar, ne hemogazanalize, karakterizohet nga:


A) Rritje e Ph, rritje PaO2, ulja e PaCO2
B) Ulja e Ph, rritje e PaCO2
C) Ulja e Ph, ulja e PaCO2

194
D) Rritje e Ph, ulja PaCO2, BE negative
E) Ph normal, ulje e PaCO2, BE negative

24. Cila nga shenjat e nuk eshte tipike e arrestit cirkulator?


A) Frymemarrje agonike (gasping)
B) Mungese te toneve kardiake ne askultim
C) Kriza mioklonike
D) Midriaza
E) Mungese e pulsit arterial periferik

25. Rreziku kardiovaskular rritet mbi te gjitha kur:


A) Rritet e indit adipoz subkutan
B) Rritet e indit adipoz visceral
C) Rritet e indit adipoz kaf
D) Ulja e indit adipoz te bardhe
E) Ne asne nga rastet e mesiperme

26. Ne cilen nga rastet e , ka me shume probabilitet, shfaqja e edemes vetem ne nje nga gjymtyret
inferiore?
A) Arteriopatia kronike obliterante
B) Flebiti
C) Nefriti
D) Insuficenca kardiake
E) Hipoproteinemia

27. Te gjitha pohimet e rreth semundjes se Alzheimer jane te verteta, pervec:


A) Gjetjet patologjike perfshijne nderthurjen neurofibrilare dhe degjenerimin neuronal
B) Pjesa me e madhe e rasteve jane familjare
C) Aktiviteti kortikal cerebral i colin-acetiltransferazes, eshte i reduktuar
D) Simptomat fillestare shpesh stimulojne depresionin
E) Nuk ka hetime specifike laboratorike patologjike

28. Ne sindromen hepatorenale:


A) Urina eshte padyshim hematurike
B) Kolesterolemia eshte e rritur
C) Diureza i kalon 1500ml ne dite
D) Diureza eshte me pak se 300 ml ne dite
E) Glukozuria eshte gjithmone e pranishme
29. Ne oftalmopatine Bazedoviane te gjitha pohimet e jane te verteta pervec:
A) Mund te paraprije ose te ndjeke disfunksionin e tiroides
B) TSH eshte i ulur ne morbus Graves
C) Normalizimi i funksionit te tiroides ka si rezultat permiresimin e oftalmopatise Bazedoviene
D) Fibroblastet prodhojne acid hialuronik dhe si rezultat kemi trashje ne indit retrobulbar
E) Prek me shpesh femrat se sa meshkujt

30. Ne fazen paraprirese te okluzionit intestinal mekanik te kolonik kuadri klinik kkt nga:
A) Te vjella fekaloide
B) Dhimbje abdominale ne forme krampesh
C) Mbrojtje abdominale e gjeneralizuar
D) Mungese e peristatikes
E) Temperature e larte

31. Te gjitha shkaqet e jane shkaktare te arrestit kardiak, pervec:


A) Blloku atrio-ventrikular i grades III-te
B) Disfunskion elektrik

195
C) Asistoli
D) Takikardi ventrikulare
E) Shoku kardiogjen

32. Te gjitha indikimet e ne terapine e flebotrombozes se thelle ne nje grua shtatzane janete
gabuara pervec: (e sakte)
A) Heparina me peshe molekulare te ulet eshte e kunderindikuar ne tremujorin e trete teshtatzanise
B) Terapia me warfarine eshte e kunderindikuar vetem ne tremujorin e pare te shtatzanise
C) Terapia me warfarine eshte e kunderindikuar gjate gjithe shtatzanise, por jo gjate periudhes se
laktacionit
D) Terapia me warfarine eshte e kunderindikuar si ne periudhen pre-partum dhe ne ate post-partum
E) Te gjitha masat terapeutike qe aplikohen ne nje grua jo shtatzane perdoren dhe ne nje grua shtatzane

33. Cilat nga t, eshte nje faktor risku per kancerin e vesikes urinare?
A) Ekspozimi kronik ndaj alkoolit
B) Trajtim i zgjatur me ciklofosfamid
C) Karcinome renale ne antecedent
D) Histori familjare per karcinomen vesikale
E) Infeksion nga Schistosoma Mansoni

34. Nje pacient paraqitet me likid te lire peritonal, splenomegali, hemoroide, hiper-
aldosteronizem, deficit te faktoreve te koagulacionit dhe hipoalbuminemi. Rreth ciles prej
patologjive te duhet te orientoheni?
A) Dekompesim kardiak kongjestiv
B) Neoplazi abdominale
C) Sindromi nefritik
D) Cirroze hepatike
E) Semundje limfoproliferative

35. Nje grua e re 33-vjec eshte diagnostikuar me anemi hipokrome mikrocitare (me Hb 7,3 g/dl,
hipoferitinemi), pas dy shtatzanive ne tre vitet e fundit dhe mestruacione ne sasi te madhe
prejdhjete muajsh. Ne dy muajt e fundit mjekohet per os me 40 mg Fe te thjeshte ne formen
ekriperave te hekurit. Perse ne kete rast kemi nje deshtim te pergjigjes ndaj terapise?
A) Prania e humbjeve te tjera hematike
B) Absorbim i demtuar intestinal i hekurit
C) Doze e pamjaftueshme e hekurit te dhene.
D) Formula kimike e hekurit te dhene
E) Indikimi terapeutik i gabuar

36. Cila eshte menyra me korrekte ne menaxhimin e shokut?


A) Njohja e shpejte e simptomave te para- ruajtja e funksioneve vitale- identifikimi i menjehershem
ishkakut primar- korrigjimi i shkakut- menaxhimi i komplikacioneve
B) Te mos nderhyhet me asnje trajtim terapeutik, pa u identifikuar shkaku primar
C) Mbajtja nen vezhgim e pacientit deri sa te shfaqen komplikacionet e para per te cilat duhet
trajtuarmenjehere
D) Kufizimi ne nje trajtim simptomatik per te evituar demtimet e hipoperfuzionit
E) Transferimi i menjehershem ne reanimacion

37. Insuficenca renale akute funksionale mund te jete pasoje e:


A) Kalkulozes se rrugeve urinare
B) Obstruksion i rrugeve urinare
C) Abuzim me aminoglukozidik
D) Ulja e volumit ekstra qelizor
E) Nekroze kortikale bilaterale

196
38. Granulomatoza e Inflamuar eshte karakteristike e:
A) Arteriti Takayasu
B) Lupusi eritematoz sistemik (LES)
C) Panarteriti nodoz
D) Sklerodermia
E) Sindromi Sjogren

39. Ne pacientet me emfizeme pulmonare nga SPOK gjate spirometrise verehen alterimet e ,
pervec:
A) Rritja e kapacitetit pulmonar total
B) Reduktim i FEV 1
C) Rritja e volumit residual
D) Rritja e kapacitetit vital
E) Demtim i difuzionit gazor

40. Nje pacient 41 vjecar i shendetshem deri 5 dite me pare kur iu shfaq nje dhimbje therese ne
1/3 eposhtme te hemitoraksit te djathte, qe forcohet me teper nga inspirimi i thelle, kolle
dhetemperature. Ritmi dhe frekuenca kardiake normale, pa zhurma kardiake ose vaskulare. Pa
turgorvenoz jugular. Pas disa ditesh dhimbja ne hemitoraks zbutet dhe ne ekzaminim fizik
verehetreduktim i FVT-se, mbytje dhe reduktim i zhurmave respiratore ne hemitoraksin e
djathte, hemitoraksi i majte normal. Cila nga situatat patologjike pershtatet me teper me
ekzaminiminfizik te pershkruar me siper?
A) Pneumotoraks
B) Emboli pulmonare
C) Empieme pleurale
D) Versament pleural
E) Bronkopneumoni

41. Te gjithe faktoret e meposhtem favorizojne formimin e edemave, pervec:


A) Rregjim shtrati i zgjatur
B) Rritja e presionit arterial
C) Rritja e presionit venoz
D) Clirimi lokal i kininave
E) Staza limfatike

42. Ne rastet me angina pectoris te paqendrueshme, cila duhet konsideruar terapia baze, pervec
atyreqe jane te nevojshme per te kontrollaur simptomat e angines?
A) Aspirin 100 mg/dite
B) Aspirin 325 mg/dite + kalciparina 0,5 mg/ dy here ne dite
C) Diltiazem 360 mg/ dite
D) Verapamil 240 mg + nifedipine 40 mg/ dite
E) Trinitrine sublingual sipas nevojes

43. Cila nga teknikat e sherben per ekzaminimin invaziv te aparatit kardiovaskular?
A) Angiokardiografia
B) RMI
C) Ekokardiografia
D) EKG
E) CT me 64 shtresa

44. Te gjitha klasat e perdoren ne trajtimin e angina pectoris te paqendrueshme, pervec:


A) Nitrateve
B) Beta bllokuesve
C) O2 terapi
D) Digoksina

197
E) Kalci bllokuesit

45. Nje grua 52 vjec paraqitet me Psoriazis te gjeneralizuar. Kesaj situate mund t’i bashkangjiten
tegjithe patologjite e , pervec:
A) Uveit anterior
B) Artrit
C) Pleurit
D) Hiperuricemi
E) Carje e thonjve

46. Jane shkaqe te shpeshta te temperatures, interpretuar ne fillim si temperature ndd rastet e ,
pervec:
A) Tiroidit subakut
B) Limfoma Hodgkin
C) Granulomatoza Vegener
D) AIDS
E) Hepatit viral akut i tipit B

47. Oligouria hipertonike, acidoza metabolike, hiperkalemia, konfigurojne kuadrin klinik-


laboratorikte:
A) Diabetit mellitus te dekompesuar
B) Diabetit insipid
C) Insuficences renale kronike
D) Fazes funksionale te insuficences renale akute
E) Faza poliurike e insuficences renale akute

48. Cila nga pohimet e prezanton informacionin me te sakte mbi prognozen e fazes malinje te
hipertensionit?
A) Niveli presionit sistolik
B) Niveli i presionit diastolik
C) Ndryshimet ne ekzaminimin e fundus okulit
D) Alterimet e funksionit renal
E) Dimensionet kardiake

49. Fraksioni i eskretimit te natriumit te filtruar normalisht eshte:


A) Me i vogel ose e barabarte me 1%
B) 1,5-3%
C) 3-4,5%
D) 4,5-6%
E) 6,5-8%

50. Prej disa muajsh pacienti ankon per dispne nga sforcimet fizike te moderuara fillimisht te
toleruaramire por qe me pas agravohen. Prej disa javesh zgjohet shpesh naten me ndjesine e
mungeses se ajrit. Prej disa kohesh fle me dy jasteke te medhenj. Cilat nga sindromat e
korrespondon me kuadrin klinik te pershkruar?
A) Insuficence ventrikulare e majte
B) Dekompesim kardiak i djathe
C) Kriza paniku
D) Astma bronkiale
E) Insuficence e arterieve koronare

51. Ne shokun hipovolemik, ne baze te fispatologjise te tij, cilat nga parashikimet e eshtejo i
sakte?
A) Administrim i oksigjenit
B) Vendosja e pacientit ne pozion me koken poshte

198
C) Administrimi i vazokonstriktoreve per mbajtjen e presionit arterial
D) Administrimi i solucioneve koloidale endovenoze
E) Monitorimi i presionit venoz qendror

52. Cili do te ishte menaxhimi korrekt ndaj nje subjekti qe ekspozohet aksidentalisht ndaj virusit
te hepatitit viral B, i cili nuk eshte i vaksinuar?
A) Administrimi i nje doze te imunoglobulinave specifike
B) Fillimi i vaksinimit
C) Administrimi si i nje doze te vaksinimit dhe i nje doze te imunoglobulinave specifike
D) Administrimi i nje doze antibiotiku
E) Administrimi vetem i nje doze te vaksinimit

53. Morbus Graves


A) Eshte nje forme e hiperfunksionimit surrenalien
B) Eshte nje forme e hipotiroidizmit me etiologji autoimune
C) Eshte nje forme e hipertiroidizmit
D) Asnje nga pergjigjet e mesiperme
E) Eshte nje forme e hipotiroidizmit me etiologji infektive

54. Cila nga variantet e t do te ishte procedura me urgjente ndaj nje subjekti 21 vjecar meastme
bronkiale, i trajtuar ambulatorisht me beta agonist, teofiline, steroid i/v, simptomatik prejdisa
oresh, qe prezantohet ne urgjence me djerse profuze, teper i axhituar, i paafte per te folur, qeheq
shpesh masken e oksigjenit, me hipoventilim, dhe dispne sibilante te dobesuar?
A) Trajtimi sistematik me kortikosteroid
B) Kromoglikat te natriumit me rruge inhalatore
C) Kontakt telefonik me mjekun personal mbi ndryshimet e fundit ne anamneze
D) Pergatitje per intubim endotrakeal
E) Ipatropium bromidi ne rruge inhalatore

55. Per vendosjen e diagnoses te Aspergilozes bronkopulmonare alergjike, ne nje person me astme
bronkiale dhe infiltrat pulmonar rekurente, cilet nga testet e meposhtem do te ishte me i
pershtatshem?
A) Testi kutan i vonuar (i ngjashem me tuberkulinen) Aspergilus Fumigatus
B) Prania e eozinofileve
C) Reaksion kutan i shpejte i Aspergilus fumigatus
D) ANA-pozitive
E) Prania e neutrofileve ne lavazhin bronko-alveolar

56. Cilat nga grupet e markuesve eshte tipik ne periudhen fillestare te hepatit B?
A) HBsAg, HBeAg; anti HBc-IgM
B) HBsAg; anti HBc IgG
C) HBsAg, HBcAg; anti HBe
D) HBsAg; anti HBe
E) Anti HBs; anti HBc

57. Tamponada kardiake eshte emergjence mjeksore qe ndodh si pasoje:


A) Nga nje goditje e forte torakale
B) Nga nje stenoze mitrale severe
C) Nga akumulimi i likidit nen presion, ne kavitetin perikardial
D) Aksident rrugore
E) Stenoze aortale severe

58. Ne mungese te antibiogrames, cilin nga keta antibiotike do te zgjidhni si me efektiv kunder
stafilokokut?
A) Ceftazidime

199
B) Piperacilline
C) Amoksiciline + Ac. Klavulonik
D) Klaritromicine
E) Metronidazol

59. Feokromocitoma eshte nje neoplazi e ciles prej strukturave :


A) Korteksit surrenal
B) Medules renale
C) Medules surrenale
D) Hipofizes
E) Timus

60. Fenomeni Raynaud mund te lidhet me


A) Erisipelen
B) Sklerodermine
C) Dermatitin
D) Tromboflebitin
E) Limfadenitin

61. Nga zbulimi ne nje analize rutine i rritjes se perqendrimit plazmatik te CPK-se, cila nga
gjendjet e duhet te konsiderohet ne plan te pare?
A) Konstipacion te rende
B) Praktimi i body building
C) Diete hiperproteinemike
D) Atak paniku
E) Kolelitiaza

62. Hipertiroidizmi subklinik lidhet me:


A) Nivel te larta te TSH-se
B) Nivel te ulet te rT3
C) Nivel te ulet te TSH-se dhe nivel normal te FT4
D) Nivel te ulet te TSH-se dhe nivle te larte te FT4
E) Rrritje te titrit te antikorpeve

63. Cilat nga t, nuk eshte shkak i drejtperdrejte i anemise?


A) Hemoragjia
B) Hemoliza e shtuar15
C) Insuficence e eritropoezes
D) Gastrektomia totale
E) Dekompesimi kardiak

64. Cili nga hormonet e mund te jape hiponatremi ne qofte se eshte shtuar sekretimi tij?
A) Angiotensina II
B) ANP (peptidi natriuretik atrial)
C) Vazopresina
D) Aldosteroni
E) Testosteroni

65. Cili nga faktoret e riskut per semundje kardiovaskulare eshte edhe faktor risku per
disfunfuksion erektil:
A) Duhanpirja
B) Dislipidemia
C) Diabeti Melitus
D) Te gjitha pergjigjet
E) Jeta sedentare.

200
66. Cila nga demtimet e karakterizon cor-in pulmonar?
A) Hipertrofi e ventrikulit te djathte
B) Dilatacion i ventrikulit te majte
C) Shunti interventrikular nga e majta ne te djathte
D) Persistenca e Ductus Botali apertum
E) Sindromi Claude-Bernard-Horner

67. Shkaku kryesor i obstruksionit te rrugeve te frymemarrjes ne pacientin pa koshience eshte:


A) Renia e gjuhes
B) Fleksioni i epiglotisit
C) Bronkospazma
D) Obstruksioni i peshtymes
E) Kolapsi i trakese

68. Cila nga pohimet e ne lidhje me interesimin renal ne dekursin e LES eshte i sakte:
A) Semundja renale klinikishtb e rendesishme prek gati 90% te pacienteve me LES
B) Nefriti eshte nje gjetje e rralle ne biopsine renale
C) Biopsia renale mund te mos jete e nevojshme ne pacientet me sediment te rritur dhe meperkeqesim
progresiv te funksionitn renal
D) Interesimi renal eshte i rralle ne pacientet me titer te larte te antitrupave anti ADN mehelikedyfishe
E) Ekzaminimi urines ne LES zakonisht zbulon proteinuri por jo hematuri ose crregullime tetjera te
sedimentit urinar

69. Ne insuficiencen respiratore te obezit:


A) Demtimi funksional eshte kryesisht gjate fazes inspiratore
B) Demtimi funksional eshte kryesisht gjate fazes ekspiratore
C) Te dyja fazat e ventilimit jane te kompromentuara
D) Eshte i demtuar shkembimi alveolo-kapilar i gazeve
E) Rritet volumi residual

70. Hipokalcemia ne insuficencen renale kronike shkaktohet nga:


A) Hiperkalciuria
B) Marrjet e pakta te kalciumit nga ushqimi
C) Mungesa e vitamines D aktive
D) Mungesa e PTH
E) Hiperparatiroidizmi

71. Te gjithe antihipertensivet e meposhtem jane te kunderindikuar ne shtatzani me perjashtim


te:
A) Beta-bllokues
B) Frenuesit e enzimes se konversionit
C) Calci antagonistet
D) Nitratet
E) Diuretiket

72. Hipertensioni arterial ne insuficencen renale kronike shkaktohet nga:


A) Stenoza e arteries renale unilaterale
B) Stenoza e e arterieve renale bilaterale
C) Rritja e volumit jashteqelizor nga retensioni Na
D) Hiperaldosteronizmi primar
E) Sindromi Barter

73. Balloni aortik:


A) Eshte nje pajisje qe mban qarkullimin ne kushtet e debitit te ulur

201
B) Permireson perfuzionin coronar gjate diastole
C) Redukton pasngarkesen ne sistol
D) Vendoset ne aorten torakale
E) Te gjitha pergjigjet e mesiperme
74. Pasojat klinike me te shpeshta te Tiroiditit te Hashimotos:
A) Hipertiroidizmi subklinik
B) Galaktorrea
C) Hipoparatiroidizmi
D) Hipertiroidizem ne fazen e pare dhe hipotiroidizem me pas
E) Semundje te tjera autoimune

75. Te gjitha pohimet e jane karakteristike te detresit respirator te te rriturit meperjashtim te:
A) Reduktimi i pO2 me rritje pCO2
B) Demtimi alveolar
C) Dilatacioni rrugeve te frymemarrjes
D) Rritja e debitit kardiak
E) Takikardia

76. Te gjitha shkaqet infektive jane shkaqe te mundshme per hemoptizi vec:
A) Murtaja
B) Abcesi pulmonar
C) TBC miliar
D) Bronkoekatzine
E) Pneumonia stafilokoksike

77. Pankreatiti akut me nekroze te infektuar:


A) Ka prognoze te njejte me pankreatitin infeksioz
B) Nuk kerkon mjekim me nderhyrje kirurgjikale
C) Eshte rasti qe kerkonvetem mjekim me inhibitore te proteazes
D) Eshte rasti qe lejon ushqyerjen normale
E) Asnje pohim nuk eshte i vertete

78. Nje djale 23 vjec ka patur hematuri te perseritura vitin e fundit. Cdo episod eshte paraprire
ngainfeksion i rrugeve te siperme te frymemarjes. Egzaminimi objektiv dhe ai i funksionit renal
janenormal. Egzaminimi i urines tregon hematuri e gjurme proteine. Cfare ka me shume mundesi
tetregoje biopsia renale:
A) Proliferim extrakapilar te gjere
B) Proliferim mezangial difuz
C) Semundje polikistike renale
D) Depozitim difuz te IgA ne imunofluoreshence
E) Depozitim ne kapilar te C3 ne imunofluorereshence

79. “Semundja nga djegja” percaktohet nga prania e:


A) Perfshirje ne masen mbi 20% te trupit te adultit
B) Perfshirje ne masen mbi 5% te trupit te femijes
C) Shoku septik
D) Hemoragjia gastrike
E) Komplikacione Pulmonare

80. Cili nga ptogenet e meposhtem eshte me shpesh pergjegjes per Pyelonephritin Acut?
A) Klebsiella
B) Chlamidia
C) Escherikja Coli
D) Pseudomona
E) Candida

202
81. Edemat e gjymtyreve manifestohen ne te gjitha keto raste klinike me perjashtim te:
A) Hepatit alkoolik
B) Cirhozis Hepari
C) Insuficienca Kardiake
D) Sindroma nefrotike
E) Insuficience venoze e ekstremiteteve inferiore

82. Ne cilin nga rastet e me insuficience respiratore eshte gjithnje i rritur PaCO2:
A) Ne insuficiencen ventilatore te pompes
B) Ne insuficiencen e shkembimit te gazeve ne mushkkri
C) Ne papershtatshmerine e arritjes/transportit/perdorimit te oksigjenit
D) Ne te gjitha keto raste
E) Ne asnje nga rastet e mesiperme

83. Nje mase pulmonare e zbuluar nga Ra-Grafia Torakale mund te jete shprehje e nje prejketyre
patologjive, pervec:
A) Tuberkulozi post-primar
B) Mycetoma
C) Pneumonia nga stafilokoku
D) Pneumonia nga mycoplazma pneumonie
E) Pneumoniti nga pneumocisti carini

84. Ateroskleroza eshte:


A) Nje fenomen fiziologjik i lidhur me moshen e pacientit
B) Nje patologji ne proporcion te drejte me rritjen e nivelit te kolesterolit ne gjak
C) Nje ngjarje me etiologji multifaktoriale
D) Ka ne menyre ekskluzive si moment te pare, fenomene te tipit infeksioz ne dem te arterieve
mekaliber te mesem e te madh.
E) Nje patologji qe nuk lidhet me diabetin e cregullimet metabolike

85. Te gjithe agjentet patogene te meposhtem jane shkak i shpeshte per shock septik pervec:
A) Koket gram pozitive (Stafilokok Enterokok)
B) Leptospiroza
C) Enterobacteret
D) Pseudomonas spp
E) Neisseria meningitis

86. Cili nga cregullimet elektrolitike jep shfaqjen e nje zgjatje te QT dhe pranine evales U ne
EKG.
A) Hipokalemia
B) Alkaloza
C) Hipercalcemia
D) Hiperkalemia
E) Hiponatremia

87. Cila nga klasat farmakologjike nuk i jepet pacienti me bllok A-V me temadhe se grada e I
ose me Sindrom te Sinusit te Semure
A) ACE-Inhibitoret
B) Diuretike te Anses
C) Beta- bllokuesit
D) ARB
E) Calci-bllokuesit

88. Cistet granulare provokojne lezione te mukozes:

203
A) Nazale
B) Orofaringeale
C) Ezofageale
D) Auricolar
E) Rectale

89. Arteriti i Hortonit mund te kete komplikacion shfaqjen e:


A) Aneurizem te aortes abdominale
B) Disekacion te harkut te aortes
C) Obstruksion te Arteries Subclavia
D) Amaurozis
E) Te gjitha jane te verteta

90. Me Eriteme kuptojme:


A) Skuqje te lekures qe zhduket nga presioni dixhital
B) Skuqje te lekures qe nuk zhduket nga presioni dixhital
C) Skuqje te lekures me vezikula
D) Ngritjes te nje zone te lekures e rrethuar me edeme
E) Skuqje te lejures me prurit te shprehur

91. Dieta Hipo sodike pershkruhet:


A) Ne te gjithe pacientet me IRK me terapi konservative
B) Vetem ne pacientet me IRK, Hipertension Arterial e Edema
C) Vetem ne pacientet me IRK si pasoje e GNK
D) Vetem ne pacientet me IRK nga Pyelonefriti Chronik
E) Vetem ne pacientet uremike ne Hemodialize

92. Te gjithe pohimet e per limphomen Hodgkiane jane te sakta pervec:


A) Semundja cfaqet ne disa paciente vetem me prurit te gjeneralizuar
B) Eshte karakteristike limphoadenomegalia e vetme ose e shumfishte jo dhimbshme
C) Ethe, djersitje dhe humbje te peshes mbi 10%, te pranishme njekohesisht, keqesojne dukshem
prognozen
D) Trajtimi e prognoza varen nga stadi klinik
E) Limphoma e Hodgkinit eshte zakonisht rezistente ndaj Radioterapise

93. Demtimet qe mund te shfaqen nga nje radio terapi jane:


A) Proporcionalisht me dozen e administruar
B) Proporcionale me llojin e fraksioneve
C) Proporcionale me zgjatjen e radio terapise
D) Proporcionale me siperfaqen e rezatuar te indit
E) Te gjitha pergjigjet e mesiperme

94. Polimialgjia Rheumatizmale:


A) Eshte nje sinovit jo-eroziv i artikulacioneve te shpatullave e te strukturave periartikulare
B) Prek me shume burrat nen 40 vjec
C) Shkaktohet nga infeksionet e perseritura streptokoksike te Tonsilave
D) Shoqeron purpuren Schonlejn-Henoch
E) Shoqerohet me pozitivitetin e anitikorpit JO1

95. Te gjitha anomalite imunologjike lidhe me infeksioni nga HIV me perjashtim te:
A) Ulje e numurit qarkullues te limfociteve T4
B) Raporti i limfociteve T4 e T8 (supresor) > 1
C) Alergjia Kutane ndaj antigeneve te zakonshem per teste Kutane
D) Ulje e funsiont te qelizave T citotoksike
E) Rritje te funksionit te beta2- microglobulines

204
96. Cili nga diuretiket e meposhtem nuk eshte nje Kalikursyes:
A) Amiloridi
B) Spironolactoni
C) Triamtereni
D) Acidi Etacrinik
E) Metoprololi

97. Te gjithe kushtet e jane shkaqe tipike te Insuficiences Kardiake Diastolike pervec:
A) Hipertensioni Arterial me hipertrofi te ventrikulit te majte
B) Takiaritmia
C) Hipertireoza
D) Miokarditi Viral
E) Fibroza e Miokardit

98. Cfare shenje nuk gjendet ne Stenozen Mitrale:


A) Zhurme diastolike me theksim presistolik
B) Puls i shpejte
C) Theksim te tonit te dyte ne maje
D) Theksim te tonit te dyte ne baze
E) Toni i forte i hapjes

99. Cili nga t eshte shkaku me i shpeshte i Embolise Pulmonare


A) Tromboze e venave siperfaqesore te gjymtyreve inferiore
B) Tromboze e atriumit te djathe
C) Tromboze e venave te thella te gjymtyreve te poshtme
D) Endokarditi bakterial
E) Emboli gazore nga frakturat

100. Cila nga barnat e poshte shenuara jane te indikuar ne terapine e krize se gutes?
A) Penicilina
B) Klofibratet
C) Kolestiramina
D) Kortizoni
E) Allopurinol

101. Cili nga barnat e eshte i kundraindikuar ne nje pacient i diagnostikuar rishtaz, rastesisht
me ane te nje EKG-je me fibrilacion atrial?
A) Beta bllokues +ACE inhibitor
B) Digoxina
C) Beta stimulues
D) Acenokumaroli
E) Kalci-antagonistet

102. Jane te gjitha shkaqe te deficitit te Vit 12 pervec:


A) Diete e papershtatshme (Psh vegjetarizmi)
B) Absorbim i pamjaftueshem
C) Ekskretimi rritur
D) Ulcer ezofageale
E) Perdorimi i pamjaftueshem

103. Cila nga semundjet infektive kane transmetim orofekal?


A) Difteria
B) Malaria
C) Varicela

205
D) Sifilizi
E) Tifo

104. Te gjitha pohimet e rreth astmes bronkiale jane korrekte, pervec:


A) Eshte nje semundje inflamatore e mukozes bronkiale
B) Eshte nje semundje kronike me shperthime riakutizuese nga faktore te ndryshem
C) Eshte nje semundje qe karakterizohet nga dispne dhe ndjesia e mbytjes torakale
D) Eshte nje semundje e karakterizohet nga bronkospazma reversibile
E) Eshte nje semundje qe shoqerohet me dekompensim kardiak

105. Ne hiperaldosteronizmin primitiv verehet:


A) Rritje e renines dhe aldosteronit plazmatik
B) Hipokalemi dhe acidoze metabolike
C) Ulje e renines dhe rritje e aldosteronit plazmatik
D) Rritje e ekskretimit urinar te Na dhe Cl
E) Ulje e ekskretimit te K dhe Cl24

106. Cila nga opsionet e te ben te mendosh se nuk kemi te bejme me nje sinkop, por mekrize
epileptike?
A) Zgjatja mbi 10 sekonda
B) Nje plage ne skalpin e kokes
C) Parandjenja e episodit
D) Urinimi i pavullnetshem
E) Pergjumja ne momentin e berjes koshient

107. Nivelet e larta te prolaktines mund:


A) Te bllokojne sekretimin pulsatil te GnRH
B) Te shkaktoje ulje te nivelit te testosteronit
C) Shkakton ulje te libidos
D) Te gjitha jane te sakta
E) Te provokoje amenorre

108. Pacientet me trauma (kontuzion) kardiake mund te trajtohen me te gjitha medikamentet e


pervec se me:
A) Antiaritmike
B) Anksiolitike
C) Antikoagulante
D) Analgjezike
E) Antibiotike

109. Ne pacientet me ketoacidoze diabetike eshte me e rendesishme te normalizohet:


A) Niveli i fosfatit
B) Bikarbonatet
C) Kalciumi
D) Kaliumi
E) Magneziumi

110. Pacientia 73 vjec, me diabet tip II, qe trajtohet me insuline, gjendet nga familjaret pa ndjenja
paraTV dhe dergohet ne urgjence. Aty regjistrohen keto te dhena: pacientja ne gjendje kome, qe
ipergjigjet vetem stimujve te dhimbshem, TA 90/70 mmHg, pulsi i rregullt me frekuence
3425rr/min, frekuenca respiratore 20/min. Lekura e ftohte dhe pak e djeresitur. Sat.i O2, 94%,
glicemia 64mg/dl. Ne EKG vihet re bllok atrio-ventrikular i grades se trete. Cila nderhyrje
terapeutike eshte me urgjente?
A) Intubimi i pacientes dhe ventilimi mekanik
B) Vendosja e nje pace-maker te perkohshem ne pritje te atij perfundimtar

206
C) Infuzioni me solucion NaCl 0.9%, 2l/ore
D) Infuzioni venoz i 50ml glukoze 20%
E) Infuzioni i dopamines 7mcg/kg/min

111. Cili nga elementet e meposhtem perben kunderindikacion relative ne terapine me digitalike?
A) Hipokalemia
B) Hemiblloku anterior i majte
C) Hipertireoza
D) Terapia me kortikosteroide
E) Terapia me spironolakton

112. Pulsi i dobet eshte karakteristike e te gjitha semundjeve pervec:


A) Shokut kardiogjen
B) Hipertensionit arterial sistolo-diastolik
C) Stenozes se aortes
D) Perikarditit
E) Infarktit akut te miokardit

113. Ne klasifikimin NYHA ne dekompensimin kardiak jane te verteta te gjitha theniet e pervec
njeres:
A) Ne shkallen e pare nuk ka kufizim te aktivitetit fizik
B) Ne shkallen IV shenjat e dekompensimit kardiak jane te pranishme edhe ne qetesi
C) Edema e kaviljeve nuk perben kriter klasifikimi
D) Prania e dhimbjeve anginoze rrit shkallen NYHA nga III ne IV
E) Ne shkallen II pacientet jane asimptomatik ne qetesi

114. Metoda me e mire per te diagnostikuar steatorrene eshte:


A) Testi i sakarozes
B) Testi i ksilozes
C) Dozimi i kriperave biliare26
D) Dozimi i yndyrnave ne fece
E) Biopsia intestinale

115. Diabeti tip I karakterizohet nga te gjitha theniet e pervec:


A) Poliuri
B) Polifagi
C) Rritja ne peshe
D) Polidipsia
E) Dhimbje abdominale

116. Akromegalia eshte:


A) Nje osteopati e artikulacioneve
B) Nje organomegali ne lartesi te medha
C) Hipersekrecion kronik i GH, pas pubertetit
D) Nje forme e rende hipomaniakale
E) Policitemia e aviatoreve

117. Nje burre 75 vjec, vdes nga dekompenimi i insuficiences kardiake kongjestive. Ne anamneze
kahistori te hipertensionit arterial per nje kohe te gjate dhe ndonje episodi anginoz ne efort.
Neautopsi u vu re hipertrofi koncentrike e ventrikulit te majte, ne miokard vihen re njolla te
voglame diameter disa mm, me ngjyre te bardhe. Ne arteriet koronare vihen re stenoza mbi 75%
nete 3 deget e medha. Cila nga diagnoza perputhet?
A) Kardiopatia iskemike kronike
B) Prolapsi mitral
C) Kardiomiopatia aritmogjene

207
D) Kardiomiopatia e dilatuar
E) Perfundimi ciktricial i nje infarkti te miokardit

118. Ne cilen nga patologjite e vihet re dyfishium i toneve kardiake?


A) Ne prezence te bllokut te deges se djathte
B) Ne stenozen e aortes
C) Ne stenozen mitrale
D) Ne hipertensionin pulmonar
E) Ne aneurizmen e aortes ascendente

119. Ne cilen nga patologjite e vihet re ne kuadrin radiologjik te kolones vertebrale, imazhi tipik
i shkopit te bambuse?
A) Sindromin Reiter
B) Artritin rheumatoid
C) Spondilitin ankilozant
D) Enteroartriti
E) Sindromi Sjogren

120. Ne cilen nga keto semundje vihen re eritrocite ne formen e tabeles se qitjes?
A) Anemia hemolitike autoimmune
B) Talasemia
C) Leukemia akute
D) Limfoma non Hodgkin
E) Morbus Addison

121. Tek nje burre 35 vjec, alkolist kronik, me diabet mellitus, me hepatosplenomegali,
hiperpigmentim kutan dhe insuficience kardiake, cila nga keto diagnoza eshte me e mundshme?
A) Pankreatiti akut
B) Cirroza biliare primare
C) Morbus Addison
D) Hemokromatoza
E) Neoplazia endokrine multiple tip I (MEN I)

122. Specifiteti i nje testi diagnostik do te thote:


A) Mosdhenia e rezultateve fals positive
B) Mosdhenia e rezultateve fals negative
C) Mosdhenia e rezultateve fals pozitive dhe negative
D) Te japi te njejtin rezultat ne kampione te ndryshem
E) Te qenit specifik per ate lloj dyshimi

123. Ekzaminimi i cili perdoret per konfirmimin e angines se qendrueshme eshte:


A) EKG
B) Koronarografia
C) Kateterizimi i arterive koronare
D) Testi i efortit
E) Ventrikulografia

124. Te gjitha theniet e jane te verteta per karcinomen e ezofagut pervec:


A) Incidence me e madhe eshte tek meshkujt (raporti mashkull/femer - 3/1)
B) Prek me shpesh segmentin inferior te ezofagut
C) Hernia hiatale perben qarte nje faktor risku
D) Disfagia progressive eshte shpesh prezente
E) Risku me i larte per tu prekur eshte tek fumatoret se sa tek jofumatoret

208
125. Nje burre 39 vjec referon nje dhimbje ne fjankun e majte pak ore pas nje trauma ne
hipokondrine majteekzaminimi objektiv rezulton normal. TA 100/65 mmHg, Frekuenca
kardiake 86/min, neanalizat e urines vihet re prania e eritrociteve 6-7/fushe. Cili nga keto
ekzaminime eshte me i pershtatshem?
A) Ekografia renale
B) Urografia
C) Qendrimi ne pritje
D) CT
E) Shintigrafia

126. Pacienti 35 vjec, alkolist paraqitet ne urgjence me dhimbje te forte periumbilikale dhe
epigastrikee cila irradiohet deri ne nivelin e 1/3 se poshtme te toraksit. Cila nga diagnozat
hipotetike eshteme e mundshme:
A) Ulcera gastrike
B) Semundje e kolonit transvers
C) Ulcera duodenale
D) Pankretiti akut
E) Semundje te kolonit descendent

127. Ne anemine e shkaktuar nga mungesa e hekurit cila nga ekzaminimet vlereson gjendjen
edepozites se hekurit ne organizem:
A) MCV
B) Hemoglobinemia
C) Hemoglobina korpuskulare e mesme
D) Ferritinemia serike
E) Transferina serike

128. Ne te moshuarit mund te shkaktoje ulje te degjimit:


A) Infarkti i miokardit
B) Arteropatia obliterante e anesive inferiore
C) Arteriti i Hortonit
D) Koliti ulceroz
E) Hernia hiatale

129. Tek policitemia vera vihet re:


A) Ulje e nivelit te hemoglobines dhe nr. te eritrociteve
B) Rritja e nivelit te hemoglobines dhe nr. te eritrociteve
C) Ulja e nivelit te hemoglobimes dhe zvogelim i mases se eritrociteve
D) Rritja e nivelit te hemoglobines dhe zvogelim i mases se eritrociteve
E) Rritja ose ulja e nivelit te hemoglobines dhe zvogelim i mases se eritrociteve

130. Ne diagnozen etiologjike te hipotiroidizmit eshte e nevojshme te vleresohet:


A) Estradioli
B) Autoantikorpet antitireoglobuline e anti TPO
C) Tireoglobulina
D) fT3
E) rT3

131. Ne me te shumten e rasteve ketoacidoza shfaqet si komplikacion i:


A) Diabetit tip 1
B) Diabetit tip 2
C) HTA
D) Diabetit insipid
E) Hipertiroidizmit

209
132. Nje femer 23 vjevare diagnostikohet me anemi mikrocitare. Cilat nga shkaqet e eshte me e
mundshme?
A) Hemoragjia kronike
B) Beta talasemia major
C) Alfa talasemia
D) Deficit i folateve
E) Deficit i vit B12

133. Pervec PaO2 qe eshte faktor determinant ne saturimin e Hb ne nivel te gjakut arterial, cili
eshtefaktor tjeter determinant?
A) Perqendrimi i O2 ne ajrin e inspiruar (FiO2)
B) pH, glicemia, perqendrimi I 2,3 DPG ne rruazat e kuqe
C) FiO2, ventilimi, difuzioni dhe perfuzioni
D) pH, Pa CO2, temperature, perqendrimi i 2,3 DPG ne rruazat e kuqe
E) FiO2, ventilimi, raporti ventilim- perfusion

134. Te gjitha situatat e mund te shkaktojne hipernatremi, pervec:


A) Hipokortikosurenaloma
B) Diabeti insipid
C) Diuretiket e tepert
D) Ushqimi me sonde
E) Polidipsia e te moshuarit

135. Ne insuficiencen renale akute te tipit funksional pesha specifike e urines eshte:
A) E ulet
B) E ndryshueshme
C) Kushtezohet nga prania e cilindrave
D) E rritur
E) E pamatshme

136. Ne leucemine limfatike kronike gamaglobulinat jane:


A) Te ulura
B) Mungojne
C) Normale
D) Te rritura
E) Shume te rritura

137. Nje grua 52 vjec, alkoliste prej shume vitesh, ankon per dhimbje abdominale rekurente
ketomuajt e fundit. CT tregon pranine e nje mase kistike me diameter 7-8cm ne bishtin e
pankreasit.Cila eshte diagnoza me e mundshme ?
A) Pankreatit akut
B) Pseudokiste pankreatike
C) Adenokarcinoma e pankreasit
D) Karcinoma metastatike
E) Adenoma insulare

138. Nje burre 30 vjec, paraqitet ne urgjence me shenjat e abstinences nga heroina ose opiate te
tjera. Ka te gjitha shenjat e pervec njeres:
A) Diarre
B) Dhimbje muskulare
C) Mioze
D) Hipertermi
E) Rinorrea

139. Cili nga komplikacionet renale mund te shfaqet gjate dekursit te amiloidozes?

210
A) Ptoza renale
B) Insuficienca renale
C) Kalkuloza renale
D) Karcinoma renale
E) Sindromi nefrotik

140. Cili nga infeksionet kongenitale mund te dyshohet tek nje i porsalindur me mikrocefali,
kalcifikime cerebrale, korioretinit?
A) Varicella zoster virus
B) Herpes simplex virus
C) Toksoplazmoza
D) Rubeola
E) Citomegalovirusi

141. Cila nga shenjat dhe simptomat eshte e pranishme ne IKK te rende?
A) Rritja e nivelit plazmatik te Hormonit Antidiuretik
B) Edema e gjeneralizuar
C) Zmadhimi i heparit
D) Etja
E) Te gjitha se bashku

142. Cili nga parametrat laboratorike duhet monitoruar ne menyre strikte ne fund te
terapisefarmakologjike te encefalopatise hepatike?
A) Kalemia
B) Glicemia
C) Uremia
D) Transaminazat
E) Kreatinemia

143. Cila nga shenjat elektrokardiografike eshte karakteristike e fibrilacionit atrial?


A) Q-T e gjate
B) Subdenivelim e segmentit ST
C) Interval PR > 0.20 msc
D) Mungesa e vales P
E) QRS e ngushte

144. Fremitus vokalis eshte i dobesuar ne te gjitha rastet pervec


A) Pneumotoraksit
B) Versamentit pleural
C) Kondesamenti i pulmonit me bronket e vogla
D) Obstruksioni i bronkeve
E) Emfizema

145. Frederiku eshte nje 19 vjecar ushtar. Perpara nje jave ka patur dhimbje fyti dhe
temperature 38-38,5C, per 3-4 dite. Mjeku i batalionit i ka rekomanduar vetem antipiretike. Dje
ka vene re disalimfonodula laterocervikal dhe subcervikal bilateral, pak te dhimbshem ne
palpacion, televizshem. Cila nga patologjite e eshte me e mundshme?
A) Limfoma malinje
B) Infeksion kronik
C) Metastaza te nje neoplazie solide
D) Patologji sistemike infektive
E) Reaksion adenopatik te nje patologjie me lokalizim akut (PSh abscess)

146. Cila nga keto semundje shoqerohet me shpesh me aneurizem te koronareve?


A) Skarlatina

211
B) Semundje reumatizmale
C) Semundja Kawasaki
D) Artriti rheumatoid
E) Endokarditi bacterial

147. Hipokalcemia eshte nje komplikacion i pashmangshem:


A) Pas efortit fizik
B) Hipogonadizem
C) Pas tiroidektomise totale
D) Pas menopauzes jatrogjene
E) Gjate trajtimit me anksiolitike

148. Ne cilin nga rastet e , shtrimi ne spital i nje pacienti me anemi kronike eshte i justifikuar?
A) Pacient me angina pectoris
B) Pacient me alterim te gjendjes mendore
C) Pacient me nivel te Hb; 7 g/dl
D) Pacient ma pancitopeni te sapodiagnostikuar
E) Te gjitha pergjigjet jane te sakta

149. Cila nga enzimat merr pjese ne procesin vaskular te ereksionit?


A) Hidroksilaza
B) NO sintetaza + fosfodiesteraza
C) Peroksidaza
D) Aromataza + hidroksilaza
E) Transferaza

150. Te gjithe jane shkaktare te dekompensimit kardiak pervec:


A) Terapia insulinike
B) Anemia34
C) Marrja e medikamenteve inhibitore te prostaglandines
D) Aritmia me frekuence te larte
E) Pneumonia

151. Vala T e rritur, me ST te subdeniveluar, me zvogelim dhe largim te vales P, bllok A-V,
deformim i QRS, bradikardi, ritem nodal, fibrilacion ventrikular jane te gjitha shenje te:
A) Hipokalemise
B) Hiperkalemise
C) Hipertiroidizmit
D) Hiperkalcemise
E) Hipomagnezemise

152. Tek nje pacient ne gjendje kome indikohet ushqimi:


A) Parenteral periferik
B) Parenteral qendror
C) Enteral me sonde
D) Nga goja gjysem te forta
E) Ushqim i lengshem nga goja

153. Cila nga shkaqet e te isuficiences renale akute do te klasifikohej si postrenale?


A) Insuficienca kardiake
B) Septicemia
C) Kalkulat
D) Rabdomioliza
E) Glomerulonefriti akut

212
154. Skorbuti eshte nje gjendje e:
A) Malnutricioni proteinik-energjitik
B) Malnutricioni proteinik
C) Deficit i Vit C
D) Deficit i Vit D
E) Deficit i fosfateve

155. Cili nga parametrat e eshte me sensitiv ne nefropatine diabetike?


A) Kreatinemia
B) Klirenci i kreatinines
C) Albuminuria
D) Testi i tolerances se glukozes
E) Ekografia renale

156. Cili nga medikamentet e nuk shkakton hipnoze ne anestezine totale?


A) Ketamina
B) Propofoli
C) Midazolami
D) Mivacurium
E) Tiopentali ne trajte kripe

157. Nje 21 vjecar ankon per cefale, polidipsi dhe poliuri, ka hipokalemi dhe vlera te larta te
renines plazmatike. CT abdominale tregon per pranine e nje mase 2cm ne polin superior te
veshkes se djathte. Veshka e majte eshte normal. Diagnoza me e mundshme eshte:
A) Tumor i qelizave juksta-glomerulare
B) Karcinoma e korteksit surenalien
C) Feokromocitoma
D) Aldosteroma
E) Karcinoma me qeliza te qarta

158. Ne nje pacient me episode sinkopale, bllok atrio-ventrikular i plote dhe kriza te shpeshta te
takikardise atriale eshte e nevojshme:
A) Dhenia e amiodaronit
B) Dhenia e beta-bllokuesit
C) Dhenia e verapamilit
D) Dhenia e digitalikut
E) Implantimi i nje pace-maker ventrikular

159. Ne poliurine hipotonike pesha specifike e urines eshte:


A) 1020-1030
B) Mbi 1030
C) Me pak se 1006
D) 101036
E) 1010-1020

160. Cili nga pohimet e eshte i vertete ne nje pacient me hiperkalcemi nga sarkoidoza?
A) Rg e toraksit eshte gjithmone normale
B) Perthithja intestinale e kalciumit eshte e rritur
C) Kalciuria eshte normale
D) Niveli i PTH serik eshte i rritur
E) Shpesh eshte e pranishme hipogamaglobulinemia

161. Te gjitha theniet e per insuficiencen renale akute jane te verteta pervec: (e gabuar)
A) Ne fazen funksionale prerenale ekskretimi i Na dhe pesha specifike e urines jane te ruajtura
B) Ne fazen akute eshte e pranishme hipokalemia

213
C) Ne fazen funksionale prerenale shfaqet nje vazokonstriksion i zones kortikale
D) Nekrotizimi i epitelit tubular eshte me i shpejte ne format toksike dhe me i ngadalte ne atoiskemike
E) Ne urine vihet re prania e cilindrave granuloz dhe tubular

162. Nje nder komplikacionet me te shpeshta te divertikulozes se kolonit eshte:


A) Fistula colo-vezikale
B) Okluzioni intestinal nga perdredhja
C) Fistula enterokutane
D) Fistula anale
E) Koliti iskemik

163. Ne patologjite obstruktive te rrugeve respiratore me FEV dhe CV ndodh:


A) Rritet FEV, ulet CV
B) Ulet FEV, rritet CV
C) Rriten te dyja
D) Te dyja ulen
E) Nuk alterohen

164. Cili nga veset kardiake mund te shkaktoje me shpesh hemoptoe?


A) Stenoza mitrale
B) Duktus Botali apertus
C) Insuficienca aortale
D) Insuficienca trikuspidale
E) Koarktacioni i aortes

165. Perparesite e aktvitetit fizik te kontrolluar vihen re me teper:


A) Ne pacientet me HTA
B) Tek diabetiket
C) Ne obeze
D) Ne te gjitha pergjigjet
E) Ne dislipidemi

166. Ne artritin reumatoid nuk preket:


A) Kavilja
B) Kyci
C) Shpatulla
D) Berryli
E) Artikulacioni sakro-iliak

167. Ushtrimet fizike te zgjatura dhe te perseritura:


A) Rrisin HDL- kolesterolin
B) Ulin kolesterolemine
C) Permiresojne tolerancen glucidike
D) Kane te gjitha efektet e treguara
E) Permiresojne BMI (body mass index)

168. Cili eshte shkaku me i zakonshem i disfunksionit erectil?


A) Depresioni
B) Edukimi, eksperienca, fobite
C) Patologjite vaskulare
D) Patologjite endokrine
E) Patologjite neurologjike

169. Shkaqet e vdekjeve tek sportistet me moshe nen 35 vjec jane:


A) Miokardiopatia hipertrofike

214
B) Displazia aritmogjene e ventrikulit te djathte
C) Anomalite me orgjine koronare
D) Te gjitha pergjigjet
E) Infarkti i miokardit

170. Cili nga kriteret e diferencon insuficiencen kardiake te majte nga ajo e djathte?
A) Enzimat hepatike te larta
B) Edema pulmonare
C) Asciti
D) Edema periferike
E) Distensioni i jugulareve

171. Per te trajtuar disfunksionin erektil ne pacientet me hipogonadizem perdoret fillimisht:


A) Inhibitori i 5-fosfodiesterazes (sildenafil)
B) Inhibitori i 5-fosfodiesterazes pas testosteronit transdermik
C) Testosteron me rruge orale + Inhibitori i 5-fosfodiesterazes
D) Asnje nga pergjigjet
E) Psikoterapi

172. Cili medikament duhet te perdoret i pari tek pacientet me hiperkalemi dhe QRS te zgjeruar
ne EKG:
A) Klorur kalciumi
B) Oksalate
C) Digoksina
D) Furasemidi
E) Bikarbonati i natriumit

173. Perqendrimi i Na+ ne urine ne insuficiencen renale kronike ne fazen stacionare eshte:
A) Me pak se 20mEq/L
B) Me e vogel se sasia e kripes se marre me ushqime
C) E barabarte me sasine e kripes se marre me ushqime
D) Me e madhe se sasia e marre me ushqim
E) Konstante, mbi 300mEq/L39

174. Cila eshte etiologjia me e shpeshte e insuficences surenale akute?


A) Sindromi Sheehan
B) Insuficienca sekondare e surenales nga kortikoterapia
C) Sindromi Shmidt
D) Sindromi Friderichsen – Waterhouse
E) Insuficienca primare e surenales me etiologji tuberkulare

175. Ne intoksikacionin ushqimor nga Clostridiumi perfrigans simptomat shfaqen:


A) Menjehere pas marrjes se ushqimit te kontaminuar
B) 1-6 ore pas marrjes se ushqimit te kontaminuar
C) 6-24 ore pas marrjes se ushqimit te kontaminuar
D) 1-2 dite me pas
E) 93-94 dite

176. Sa duhet te mbahet niveli i LDL-Cholesterolit ne te semuret qe kane kaluar Infarkt


Miokardi?
A) Deri ne 100 mg/dl
B) Deri ne 130 mg/dl
C) Deri ne 160 mg/dl
D) Deri ne 190 mg/dl
E) Deri ne 200mg/dl

215
177. Nje djale i moshes 18 vjec ka temperature, dhimbje faringeale, asteni dhe limfadenopati
latero-cervikale. Ne ekzaminimin objektiv paraqet: faringo-tonsilit, petekie palatine,
limfadenopaticervikale, ne rajonin axillar dhe inguinal, hepatosplenomegali. Ne ekzaminimin
laboratorik evidentohetnje numer normal eritrocitesh dhe nje test pozitiv per hemoaglutininen
esterofile. Cili eshte agjenti shkaktar i kesaj semundje:
A) Virusi Epstein-Barr
B) Neisseria Gonorrhaeae
C) CMV
D) Mikoplazma hominis
E) Streptokoku i grupit A

178. Nje burre 49 vjec shkon te mjeku i tij per shkak te nje cefale mengjezore prej disa muajsh,
qe i pergjigjet pjeserisht terapive rastesore me AIJS. Pesha 105 kg, gjatesia 174 cm, pi 35-40
cigare ne dite40,5-6 kanace birre dhe pije alkolike te forta cdo dite. TA 185/110 mmHg, FC 108
rrahje/min, me ekstrasistola te rralla, ictus cordis i palpueshem ne hapesiren e VI intercostale te
majte ne linjenmediane axilare. Fundus oculi eshte negativ. Cili nga pohimet e rreth TA ne kete
pacienteshte i gabuar:
A) Duhet rikontrolluar TA pas nje periudhe pushimi ne ambulator
B) Nje hasje e vetme e tensionit diastolik 110 mmHg eshte e mjaftueshme per vendosjen e dx
C) Konsumi i alkolit tek ky person ndikon ne menyre te konsiderueshme ne HTA
D) Personi duhet te kthehet pas 1 jave per te rivleresuar TA
E) Nuk duhet filluar terapi antihipertensive ne kete moment

179. Rritja akute ose kronike e presionit arterial si pasoje e okluzionit te pjesshem ose te plote te
njeres ose te dyja arterieve renale ose te degeve te saj quhet:
A) Hipertension esencial
B) Hipertension renoparenkimatoz
C) Hipertension renovaskular
D) Hipertension okluziv
E) Hipertension jatrogjen

180. Te semuret me refluks gastro ezofageal:


A) Kane vetem shqetesime simptomatike
B) Duhet te konsumojne sasi te medha karamele me mente dhe cokollata
C) Kane nje risk te dyfishuar per astma, SPOK, fibroze pulmonare
D) Nuk kane risk te larte per neoplazi
E) Te gjitha pohimet e mesiperme jane te verteta

181. Cila prej metodave terapeutike nuk eshte shume e keshillueshme ne nje te semure obez me
sindromen apne obstruktive nokturne:
A) Renia ne peshe
B) Shina mandibulare
C) Perdorimi i CPAP
D) Kontrolli farmakologjik i vlerave te glicemise
E) Terapi kirurgjikale

182. Ne mungese te nje rruge venoze gjate nje arresti kardiak adrenalina administrohet ne rruge:
A) Intrakardiake
B) Endotrakeale
C) Intramuskulare
D) Intradermale
E) Subkutane

183. Cilat jane kriteret kryesore te intubimit endotrakeal:

216
A) Mbajtja hapur e rrugeve ajrore, parandalimi i inhalacionit, lejimi i ventilimit, lejimi i aspirimit
trakeobronkial
B) Mbajtja hapur e rrugeve ajrore, parandalimi i inhalacionit, mbajtja e ventilimit spontan, nxitja
eekpektoratit
C) Nxitja e ventilimit mekanik, administrimi i ushqimit parenteral, lejimi i aspirimit trakeobronkial,
parandalimi i te vjellave
D) Parandalimi i te vjellave, administrimi i ushqimit parenteral, evitimi i distensionitgastrik, evitimi i
kequshqyerjes
E) Mbajtja hapur e rrugeve ajrore, nxitja e ventilimit dhe ekspektoratit

184-Tek nje person me Diabet mellitus tip 2, mbi peshe, nje diete hipokalorike:
A) Rrit riskun per hipoglicemi
B) Permireson kontrollin glukometabolik
C) Duhet te jete edhe hipoproteinike
D) Rrit nivelin plazmatik te lipideve
E) Asnje nga pergjigjet e mesiperme nuk eshte e sakte

185-Sasia ditore e rekomanduar per proteinat eshte:


A) 40-50% te energjise
B) 30-40% te energjise
C) 0.8 gr/kg peshe
D) 1.5 gr/kg peshe
E) 2 gr/kg peshe

186-Cila prej ve eshte shenja klinike me e shpeshte ne stenozen mitrale:


A) Dhimbja prekordiale
B) Hemoptizi
C) Dispne
D) Iktus
E) Ngjirje e zerit

187-Te gjitha t jane shkaqe te hipokalemise pervec:


A) Terapia me frenues te sistemit renine-angiotenzine
B) Alkaloza metabolike
C) Sindroma Cushing
D) Te vjella te vazhdueshme
E) Terapia me diuretike

188-Nje gjendje hipertensioni portal te rende perfshin te gjitha situatat e pervec:


A) Splenomegali
B) Ascit
C) Poliglobuli dhe trombocitoze
D) Varice ezofageale
E) Qarkullim kolateral

189-Cila prej vlerave te BMI duhet te merret ne konsiderate qe nje grua te quhet mbi peshe:
A) 20-25
B) 25-30
C) 30-35
D) Mbi 27.5
E) Mbi 30

190-Te gjitha pohimet e rreth apnese obstruktive nokturne jane te verteta pervec:
A) Nxitet nga konsumi i alkolit
B) Perfshin periudha te gjata hipoksie dhe hiperkapnie

217
C) Shkaktojne depresion te sistemit simpatik
D) Jane karakteristike te obezitetit te rende
E) Faringu kolabohet lehtesisht

191-Trombocitopenia eshte nje kriter i vlefshem per diagnostikimin e:


A) Artritit rheumatoid
B) Lupus eritematoz sistemik
C) Mikropoliangiti nekrotizant
D) Panarteriti nodoz
E) Vaskuliti nekrotizant

192-Cila prej klasave nuk eshte e indikuar ne ishemine kardiake:


A) B-bllokuesit
B) ACE-inhibitoret
C) Antiagregantet
D) Teofilina
E) Diuretiket

193-Perdorimi joterapeutik i anabolizanteve steroid mund te shkaktoje:


A) Infertilitet
B) Tumore hepatike beninje dhe hepatokarcinoma
C) Psikoza maniako-depressive
D) Iktus
E) Te gjitha te mesipermet

194-Me cilen prej patologjive eshte i lidhur deficiti i Vitamines D:


A) Skorbuti
B) Rakitizmi
C) Anemia pernicioze
D) Anemia hemolitike
E) Kwashiorkor

195-Te gjitha t jane faktore risku per osteoporozen pervec:


A) Menopauza e parakohshme kirurgjikale
B) Duhanpirja
C) Terapia me fenobarbital
D) Jeta sedentare
E) Obeziteti

196-Ne cilen prej patologjive kemi nje renie te peshes specifike urinare:
A) Diabet Mellitus
B) Cirroze hepatike
C) Diabet insipid
D) Hiperaldosteronizmi
E) Hipertensioni arterial

197-Cila prej te dhenave nuk eshte e pranishme ne semundjen e Addison:


A) Rritja e Natremise
B) Rritja e Kalemise
C) Rritja e azotemise
D) Hollimi i urines
E) Rritja e hematokritit

198-Vitamina B-12 perthithet ne:


A) Nivel gastrik

218
B) Nivel duodenal
C) Nivel te ileumit terminal
D) Nivel te kolonit
E) Gjate gjithe tubit tretes

199-Ne cilen prej semundjeve eshte e pranishme hiperkalemia:


A) Semundja e Addisonit
B) Diabet Mellitus
C) Sindroma Gilbert
D) Cirroze hepatike
E) Tireotoksikoza

200-Krioglobulinemia mikse eshte e shoqeruar me pozitivitet te HCV ne:


A) 1-10% te rasteve
B) 11-20% te rasteve
C) 21-30% te rasteve
D) Me shume se ne 80% te rasteve
E) Asnjera nga pergjigjet nuk eshte e sakte

201-Listerioza eshte:
A) Semundje infektive e adultit imunosupresiv i shkaktuar nga nje bakter gram negative
B) Nje semundje ekzentematike virale
C) Nje variant klinik i mononukleozes infektive
D) Semundje infektive tek i porsalinduri qe shkakton sepsis dhe meningit
E) Intoksikacion kronik nga esteret fosforoorganike

202-Ne nje transplant veshke perputhshmeria ABO eshte kusht:


A) I domosdoshem vetem nese ka perputhshmeri te HLA
B) Jo i domosdoshem vetem nese ka perputhshmeri te HLA
C) I pavarur
D) I domosdoshem
E) I domosdoshem vetem nese ka nje transplant te meparshem

203-Cili prej medikamenteve nuk shkakton hipokalemi:


A) Agonistet B-adrenergjik
B) Teofilina
C) Antagonistet e kalciumit
D) Diuretiket
E) Amphotericina

204-Cila prej patologjive shoqerohet me shpesh me HTA esencial.


A) Fibrilacioni atrial
B) Guta
C) Reduktimi i tolerances se glukozes
D) Astma bronkiale
E) Prola PSi i valvules mitrale

205-Ndryshe nga shoku kardiogjen dhe shoku hemoragjik te gjitha situatat e hasen neshokun
septik pervec:
A) Diureza ruhet nga vazodilatacioni renal
B) Lekura eshte shume shpesh e nxehte
C) Lekura eshte shume shpesh e ftohte
D) Eshte shpesh si komplikacion i KID-it
E) Shok pulmonar

219
206-Hipotensioni ortostatik eshte (zgjidh pergjigjen e gabuar):
A) Eshte ulja e TA qe verehet gjate kalimit nga pozicioni shtrire ne ulur
B) E perkufizuar normalisht si nje grumbullim gjaku ne sistemin venoz periferik teekstremiteteve
inferiore dhe te trungut
C) E shkaktuar nga nje reduktim transistor i rikthimit venoz dhe per pasoje te debitit kardiak
D) Rezulton normalisht nga aktivizimi i baroreceptoreve te harkut te aortes dhe sinuseve karotideqe
aktivizojne reflekset autonome duke induktuar nje takikardi provizore me normalizim temetejshem te
vlerave te tensionit arterial
E) Quhet kur diferenca e TA nga shtrire-ulur eshte mbi 20-30 mmHg

207-Cili prej ushtrimeve fizike eshte me i indikuar ne parandalimin dhe rehabilitimin e


semundjeve kardiovaskulare:
A) Anaerobike
B) Aerobike
C) Izokinetike
D) Ushtrime force
E) Ushtrime fuqie

208-Ne rastet e fatkeqesive natyrale duhet ti jepet prioritet:


A) Femijeve
B) Pleqve
C) Grave
D) Njerezve te rendesishem nga pikpamja shoqerore
E) Te gjithe atyre qe me aktivitetin e tyre mund te jene te nevojshem ne shpetimin e njerezve tetjere

209-Pulsi arterial qe palpohet me shpesh gjate procedurave te Basic Life Suport eshte:
A) Radial
B) Popliteal
C) Brachial
D) Temporal
E) Carotid

210-Terapia e dekompensimit kardiak perfshin perdorimin e te gjithe medikamenteve pervec:


A) B-Bllokues
B) ACE-inhibitore
C) Statina
D) Eritropoetine
E) Diuretike

211-Ne SPOK te riakutizuar observohet:


A) Acidoze metabolike
B) Hipoksi dhe hiperkapni
C) Hiperkapni pa hipoksemi
D) Alkaloze respiratore
E) Hiperkalemi grave

212. Kuadri i repolarizimit ne EKG qe verehet tek sportivet:


A) Konsiston ne ngritje te traktit ST
B) Eshte tregues i kardiopatise ishemike
C) Eshte tregues i nje miokardiopatie
D) Konsiston ne nje rritje te nivelit te traktit ST dhe konkavitet ne pjesen e siperme
E) Eshte me i shpeshte ne paciente me hiperkolesterolemi

213. Hiperparatiroidizmi sekondar ne insuficiencen renale kronike vjen si rezultat i:


A) Tumor beninj i paratiroides

220
B) Tumor malinj i paratiroides
C) Hipertiroidizmi
D) Hipotiroidizmi
E) Rritja e fosforemise dhe ulja e kalcemise

214. Te gjitha pohimet e ne lidhje me ACE inhibitoret jane te verteta, pervec:


A) Nuk ndikojne ne gjendjen metabolike te pacientit
B) Ulin proteinurine e lidhur me nefropatine diabetike
C) Mund te pershpejtojne ecurine e insuficiences renale
D) Nuk duhen te perdoren ne paciente me terapi trombolitike
E) Duhet te perdoren me kujdes ne paciente qe marrin diuretike

215. Cili nga medikamentet e ka efekt me te madh bronkodilatator:


A) Kortikosteroidet inhalatore
B) Kortikosteroidet orale
C) Beta 2 mimetiket
D) Antihistaminiket
E) Teofillina

216. Ne sinkopin vazo vagal te gjithe simptomat e jane te verteta pervec:


A) Dhimbja e gjoksit
B) Djersitje
C) Hiperstaltike
D) Bradikardi
E) Vazokonstriksion kutan

217. Megakoloni toksik rendon nje nga gjendjet e :


A) Rektokolitin ulceroz
B) Morbus Crohn
C) Divertikulozen e kolonit
D) Enterokolitin
E) Nje stenoze neoplazike te kolonit

218. Te gjitha pohimet e ne lidhje me insuf. respiratore jane te verteta, pervec:


A) Raporti ventilim\perfuzion alterohet ne SPOK dhe ne emfizeme
B) Hipoksia eshte gjithmone e pranishme
C) Kapaciteti vital ulet ne rastin e insuficiences respiratore te tipit restriktiv
D) Hiperkapnia nuk eshte gjithmone prezente
E) Difuzioni alveolar i oksigjenit eshte me i madh se i dioksidit te karbonit

219. Per diagnozen e hipogonadizmit mashkullor eshte e rendesishme te percaktohet:


A) Perqendrimi plazmatik I testosteronit te lire
B) Perqendrimi plazmatik I testosteronit total
C) LH, testosterone dhe SHBG
D) FSH, PRL dhe estradioli
E) Testi i hCG per testosteronin49

220. Karcinoma e tiroides pergjithesisht karakterizohet:


A) Eutiroidizmi
B) Hipertiroidizmi
C) Hipotiroidizmi
D) Hipotiroidizem me rritje te titrit te antitrupave
E) Ekzoftalmi

221. Te gjitha t jane shkaqe te hiperkalemise grave, pervec:

221
A) Insuficienca renale acute
B) Insuficienca renale kronike
C) Acidoza metabolike
D) Alkaloza metabolike
E) Morbus Addison

222. Nje burre 47 vjec ka temperature, atralgji, renie ne peshe, dhimbje abdominale, dhimbje
pleuritike prej 2muajsh. Prej nje jave ka vene re veshtiresi ne dorsofleksion te gishtit te madh te
djathte. Mjekuka vene re nje rritje te presionit arterial (150\95mmHg). Ekzaminimet
laboratorike tregojne nje rritjete VES, anemi, leukocitoze dhe hematuri. Pacienti nuk perdor
medikamente. Diagnoza me e mundshme eshte:
A) Arterit gjigando-qelizor
B) Granulomatoze alergjike
C) Granulomatoza Wegener
D) Poliarteriti nodoz
E) Vaskuliti nga hipersensibiliteti

223. Kush percakton ne menyre me te thjeshte dhe te besueshme ndryshimet e retensionit hidrik
nenje pacient me edema?
A) Hematokriti
B) Diureza
C) Pesha e feceve
D) Pesha trupore
E) Presioni venoz qendror

224. Ne insuficiencen kronike te zemres se djathte:


A) Rritet shpejtesia e fluksit te gjakut venoz
B) Volume ekstraqelizor mbetet i pandryshuar
C) Rritet kthimi venoz
D) Ulet presioni parcial i oksigjenit
E) Ulet presioni diastolik i ventrikulit te djathte

225. Nje pacient shtrohet per te percaktuar renien e tij ne peshe dhe ne analizen laboratorike
verehetnje hematokrit 30% dhe trombocitet 1.300.000\mm3. Cila nga shkaqet e eshte me
eshpeshte ne trombocitoze?
A) Semundje malinje
B) Infeksion
C) Semundje vaskulare e kolagjenit
D) Semundje mieloproliferative
E) Humbja kronike e gjakut

226. Cilet nga grupet e te medikamenteve antihipertensive duhen perdorur ne fillim ne


njepacient burre, 45vjec, qe nuk ka semundje te tjera shoqeruese?
A) Kalcium antagoniste
B) ACE inhibitoret
C) Diuretiket
D) Beta bllokuesit
E) Nuk ka rendesi lloji i medikamenteve te zgjedhura kur arrihet te mbahet nen kontroll presioni

227. Prokalcitonina:
A) Eshte nje tregues inflamator ne infeksionet bakteriale sistemike
B) Eshte nje neurotransmetues i SNQ
C) Eshte nje hormon i prodhuar nga corpus luteo
D) Eshte nje tregues i inflamacioneve virale
E) Asnjera

222
228. Ne hipoaldosteronizem, hiponatremia shoqerohet me:
A) Hipokalcemi
B) Hipokalemi
C) Hiperkalemi
D) Hiperkalcemi
E) Asnjera

229. Eozinofilet jane te pranishme ne:


A) Infeksione parazitare
B) Infeksione bakteriale
C) Reaksione alergjike
D) ”a”dhe “c”jane te sakta
E) Asnjera nga te mesipermet

230. Cilet medikamente duhen perdorur ne kriza te gutes akute:


A) AIJS
B) Alopurinoli
C) Kortikosteroide
D) Antibakteriale
E) Diuretike

231. Aksese kolle gjate nates, dhimbje te forta torakale dhe ngjirje zeri jane simptomat me te
shpeshta qe shoqerojne:
A) Gastritin atrofik kronik
B) Gastritin akut
C) Ulceren gastrike
D) Infarktin e miokardit
E) Hernie hiatale

232. Osteomalacia karakterizohet nga:


A) Hipokalcemia
B) Mineralizim i pamjaftueshem i matriksit kockor
C) Mineralizim i tepert i matriksit kockor
D) Sasi e pamjaftueshme e materialit kockor te sintetizuar nga qelizat kockore
E) Rritje te aktivitetit te osteoklasteve

233. Cili eshte shkaku me i shpeshte i embolise pulmonare:


A) Fibrilacioni atrial
B) Endokarditi bakterial52
C) Abuzimi me droge
D) Tromboza e venave te thella te ekstremiteteve te poshtme
E) Tromboza e venes cava

234. Nje pacient i ri vjen ne urgjence me nje dhimbje akute ne ijen e djathte qe i perhapet
neregjionin inguinal te djathte, pacienti eshte shume i axhituar, dhe nuk gjen nje pozicion per tu
qetesuar, cila eshte diagnoza me e mundshme:
A) Apendisiti akut
B) Kolecistiti akut
C) Hernia diskale L5-S1
D) Disekacioni i aortes
E) Kalkuloza renale

235. Cila nga pohimet e qe lidhen me diabetin tip 2 nuk eshte e sakte:
A) Nuk trajtohet kurre me insuline

223
B) Ndikon predispozicioni familjar
C) Shpesh lidhet me obezitetin
D) Ben pjese ne sindromin metabolik
E) Shfaqet gati gjithmone ne moshat e medha

236. Nje burre rreth 40vjec vuan nga diabet mellitus, hepatomegali, dekompesim kardiak dhe
hiperpigmentim te lekures. Cila eshte semundja me e mundshme:
A) Pankreatit kronik
B) Hemokromatoze
C) Cirroze biliare primare
D) Morbus Addison
E) Hipotiroidizem

237. Ne rastin e nje procedure dentare ne cilen nga gjendjet e duhet perdorur profilaksi per
endokarditin bacterial:
A) Ne te gjithe pacientet qe perdorin terapi me antikoagulante
B) B. Ne paciente qe kane bere by-pass koronar
C) Ne paciente me prolaps te valvules mitrale
D) Ne paciente me insuficience kardiake kronike
E) Ne paciente me ektazi te bulbit te aortes53

238. Ne insuficiencen kardiake te shkalles NYHA 1 kemi:


A) Shfaqjen e simptomave te pacientet gjate aktivitetit te perditshem
B) Shfaqjen e simptomave tek pacientet ne eforte minimale
C) Mungesen e simptomave te pacientet gjate efortit
D) Shfaqjen e simptomave te pacientet ne eforte te konsiderueshme
E) Shfaqjen e simptomave pavaresisht marrjes se terapise

239. Cili eshte niveli i LDL-kolesterolit qe duhet mbajtur tek pacientet risk te infarktit te
miokardit:
A) <130mg/dL
B) <200mg/dL
C) <100mg/dL
D) <180mg/dL
E) <110mg/dL

240. Nje burre rreth 55vjec, obez dhe hipertensiv, me vlera te pakontrolluara te presionit nga
terapia qeperdor; cilen nga ekzaminimet diagnostikuese do perdorje ne fillim:
A) Ekografi abdominale
B) Polisonografi dinamike
C) Monitorim te presionit per 24 ore
D) Vetem “a” dhe “c”
E) Te gjitha

241. Nje grua rreth 65vjec e shtruar pas nje nderhyrje kardiokirurgjikale per zevendesimin e
valvules mitrale, ne anamnezen e saj ka nje histori per semunde reumatizmale ne moshe te re,
SPOK dhe tiroidektomi te shoqeruar me terapi zevendesuse; pacientja ka nje dhimbje te
shoqeruar me kontraksione muskulare te gjymtyreve te poshtem, cili mund te jete shkaku me i
mundshem:
A) Atak ishemik tranzitor
B) Shfaqja e hiperkalcemise
C) Shfaqja e hipokalcemise
D) Shfaqja e hiperkaliemise
E) Shfaqja e hipokaliemise

224
242. Ne kushtet e nje sindrome endokrine multiple, MEN1, cili nga organet e preket me pak:
A) Paratiroidja
B) Pankreasi
C) Hipofiza
D) Mbiveshkorja
E) Te gjitha
243. Te gjitha gjendjet e karakterizohen nga hiperkalcemia, pervec:
A) Mieloma multiple
B) Rritja e rezines gjate shkembimit jonik
C) Sarkoidoza
D) Hiperparatiroidizmi
E) Intoksikimi me vitamina

244. Pankreatiti akut mund te shkaktohet nga te gjitha t, pervec:


A) Kalkuloza biliare
B) Abuzimi me alkool
C) Nderhyrje kirurgjikale ne rruget biliare ose ne stomak
D) Toksina si klorotiazide
E) Hipertrofia e pilorit

245. Sindroma e Leriche karakterizohet nga:


A) Obstruksion i vena porta
B) Obstruksion i vena cava inferior
C) Obstruksion i bifurkacionit te aortes
D) Obstruksion i vena cava superior
E) Obstruksion i venava suprahepatike

246. Ne intoksikimin me monoksid karboni cila nga shfaqjet kutane eshte me karakteristike:
A) Cianoze e buzeve dhe thonjve
B) Zbehje te lekures dhe djersitje
C) Ikter
D) Lezione kutane eritematoze
E) Lekure si mermer

247. Dhimbja anginoze eshte me shpesh e lokalizuar ne:


A) Epigaster
B) Regjionin nen gjendren mamare te majte
C) Hemitoraksin e djathte
D) Regjionin retrosternal
E) Fyt

248. Simptomatologjia e hematomes ekstradurale shfaqet me shpesh:


A) Menjehere mbas traumes
B) Mbas nje periudhe qartesie te shkurter
C) Mbas 48 oreve
D) Ne faze kronike (ne distance)
E) Papritmas

249. Te gjitha situatat e jane shkaktare te hiperkalemise, pervec njeres:


A) Hemolize e kampionit te marre te gjakut
B) Intoksikimi me digitalike
C) Insuficienca renale akute
D) Rabdomioliza
E) Alkaloza metabolike

225
250. Ne sindromin e shtypjes insuficienca renale akute vjen si pasoje e:
A) Shokut hipovolemik relativ
B) Shokut septik
C) KID-it
D) Mioglobinemise me mioglobinurise
E) Shokut hipovolemik absolut

251. Ne traumat e shkaktuara nga frenimi, cila eshte zona e aortes torakale qe preket me shpesh?
A) Aorta ashendente
B) Harku i aortes
C) Aorta deshendente
D) Rrenja e aortes
E) Deget supra-aortike

252. Cila nga situatat klinike shkakton insuficiencen akute te ventrikulit te djathte?
A) Stenoza e aortes
B) Embolia pulmonare
C) Hipertensioni pulmonar
D) Insuficienca e ventrikulit te majte
E) Stenoza mitrale

253. Incidenca e invaginacionit intestinal eshte me e larte tek:


A) I sapolinduri
B) Midis 2-3 muajve te pare
C) Midis 5-9 muajve
D) Mbas vitit te pare te jetes
E) Midis vitit te pare dhe te dyte te jetes

254. Cila eshte rruga e fillimit terapeutik tek nje pacient 53 vjecar, shtruar per angine qe ben ne
menyre te papritur humbje te koshiences, me puls qe nuk kapet dhe ne monitor shihet fibrilacion
ventrikular:
A) Nitroglicerine dhe heparine
B) Intubim dhe ventilim me ambu
C) Defibrilim 200 J
D) Lidocaine 100 mg bolus iv
E) Adrenaline 1 mg bolus iv

255. Gjate anestezise se pergjithshme, hiperventilimi i zgjatur mund te shkaktoje:


A) Vazodilatacion cerebral
B) Hipokapni
C) Hiperkapni
D) Acidoze respiratore
E) Hipertermi malinje

256. Ne nje pacient te traumatizuar, me fraktura te brinjeve X-XI majtas, duhet te dyshojme
perprekje te:
A) Pankreasit
B) Shpretkes
C) Melcise
D) Veshkes
E) Vezikes

257. Cila nuk mund te jete shenja e menjehershme e gelltitjes se nje acidi:
A) Mediastinit akut

226
B) Perforacion i stomakut
C) Perforacion i ezofagut
D) Hematemeze
E) Stenoze cikatrizale

258. Dermatomykoza furfuraca shkaktohet nga:


A) Pitirosporum orbikular
B) Kandida Albikans
C) Trikofiton mentagrofitis
D) Mikrospori audouini
E) Chlamidia pneumonie

259. Recetat ‘e bardha ’mbi letren drejtuar mjekut jane te vlefshme:


A) Per nje periudhe jo me shume se 3 muaj, te perdorshme jo me shume se 5 here
B) Per nje periudhe 3 mujore, pa limit perdorimi
C) Per nje muaj, i perdorshem jo me shume se 3 here
D) Vetem nje here te vetme brenda nje jave nga data e shkruar
E) Per 6 muaj, por te perdorshme vetem nje here

260. Cfare eshte klasifikimi ATC?


A) Eshte nje sistem klasifikimi per medikamentet qe jane vendosur nga Keshilli Nordik i Mjekesise
ne UppSala, Sverige
B) Liste e medikamenteve per kompanite
C) Literature shkencore
D) Klasifikim per molekulat
E) Asnje nga pergjigjet e mesiperme

261. Recetat speciale perdoren per pershkrimin e:


A) Cdo medikamenti per te cilin eshte i detyruar pershkrimi nga mjeku
B) Medikamente te cliruara nga SSN per te cilat pacienti gezon nje clirim
C) Substanca dhe preparate narkotike dhe Psikotrope qe shenohen ne tabela te vecanta
D) Klasa te vecanta antibiotikesh
E) Medikamente te cliruara nga SSN, vecanerisht te shtrenjta

262. Periudha mesatare e inkubacionit te fruthit eshte:


A) 1-3 dite
B) 5-7 dite
C) 10-14 dite
D) 14-21 dite
E) Mund te jete me shume se 30 dite

263. Pershkrimi i medikamenteve per indikacione te paparashikuara ne skeden teknike eshte e


mundur?
A) Kurre
B) Ndonjehere
C) Gjithmone
D) Eshte e lejuar pasi efikasiteti dhe toleranca jane shkencerisht te dokumentara
E) Eshte e lejuar pasi efikasiteti dhe toleranca jane shkencerisht te dokumentara dhe vetem pasi
eshte siguruar konsensusi i shkruar i pacientit, i cili eshte paraprakisht i informuar

264. Dermatiti nga qendrimi eshte kryesisht i lidhur me:


A) Infeksionet mykotike te lekures
B) Insuficienca venoze
C) Insuficienca arteriale
D) Insuficience kardiake kronike

227
E) Infeksione bakteriale

265. Pershtatja klinike eshte:


A) Masa ne te cilen nje nderhyrje e vecante eshte po aq efikase dhe e indikuar per personin qe e merr
B) Matja e diteqendrit ne spital
C) Koha qe i kushtohet nje performance
D) Te gjitha pergjigjet e mesiperme59
E) Asnje nga pergjigjet e mesiperme

266. Cilat nga t nuk jane karakteristike e suindromit te apnese se gjumit?


A) Zgjime te shpeshta gjate gjumit me ndjesine e “urise per ajer”
B) Gerhitje
C) Apne qendrore
D) Ulje e saturimit arterial te O2
E) Pergjumje gjate dites

267. Gjate zevendesimit te nje mjeku te Mjekesise se pergjithshme ju telefonon nje pacient
68vjecar, me nje histori te njohur per angine, qe ju referon per nje shtim te dhimbjes prekordiale,
qe i ka filluar mbas nje sforcimi fizik te moderuar, qe prej 30 minutash. Çfare do t’i keshilloni ju
pacientit?
A) Te marre vete nitratet, ne pritje te zhdukjes se shenjave
B) I thoni te paraqitet ne studion tuaj per nje ekzaminim objektiv me te kujdesshem
C) E qetesoni, duke kembengulur mbi patologjine baze, dhe i thoni te prese zhdukjen e shenjave
D) I thoni qe te prese viziten tuaj ne banese ne fund te dites
E) Te marre vete nitratet dhe i thoni te telefonoje, per t’u transferuar ne urgjencen me te afert per
ekzaminime te tjera

268. Osteomieliti mund te shkaktohet nga:


A) Vetem nga mikrobe anaerobe
B) Vetem nga mikrobe aerobe
C) Vetem nga mikobaktere te vecante
D) Vetem nga mikrobe dhe funge anaerobe
E) Nga mikrobe aerobe dhe anaerobe, mykobaktere dhe funge

269. Mjeku mund t’i refuzoje nje dokument shendetesor nje qytetari?
A) Po
B) Ndonjehere
C) Jo, nuk mund te refuzoje t’i leshoje direkt nje qytetari dokumente qe flasin per gjendjen e
tijshendetesore dhe duhet te perpiloje dokumentat duke vleresuar dhe garantuar vetem te
dhenaklinike qe i ka konstatuar vete.
D) Po nese eshte MMG
E) Po nese eshte pediater

270. Ne demencen primare (tipi degjerenativ), demtimi i kujteses eshte:


A) Njesoj per memorien e vone dhe te larget
B) Me teper per memorien e vone se ate te larget
C) Me pak per memorien e vone se ate te larget
D) E limituar per ngjarje vecanerisht traumatike
E) Mund te paraqise karakteristika te perbashketa me te gjitha pershkrimet e mesiperme

271. Gjate zevendesimit te nje mjeku te Mjekesise se pergjithshme ju telefonon nje pacient 54
vjecar, i cili iu referon qe eshte rrezuar nga nje shkalle ne shtepi pak ore me pare. Gjate renies
kagoditur koken me nje mobilje dhe referon se ka humbur vetedijen per disa minuta.
Çfarekeshilloni ju per pacientin?

228
A) I kerkoni informacione te tjera pacientit dhe e siguroni per gjendjen e tij klinike, meqe nuk katani
shenja te tjera.
B) I kerkoni te paraqitet ne studion tuaj per nje ekzaminim objektiv me te kujdesshem.
C) I kerkoni te prese ne banesen e tij shfaqjen e mundshme te shenjave te tjera dhe t’juritelefonoje mbas
disa oresh.
D) E keshilloni te shkoje me mundesite e tij tek urgjenca per nje vizite neurologjike ngaspecialisti i
turnit.
E) I keshilloni te marre 118 dhe te cohet me urgjence ne urgjence per ekzaminime te tjera dheper nje
periudhe observimi.

272. Ne trajtimin e kancerit te gjirit nuk jane indikacion hormonet e :


A) Estrogjeni
B) Progesteroni
C) Frenusit e aromatazes
D) Antiestrogeni
E) Te gjitha pergjigjet e mesiperme jane te verteta

273. Cilet nga markuesit molekulare te meposhtem eshte i lidhur me trashegueshmerine per
zhvillimin e kancerit te gjirit dhe ovarit?
A) Tel-2
B) BRCA-1
C) C-myc
D) APC
E) TNF

274. Tumori i kolonit, jep metastaza kryesisht tek:


A) Kockat e skeletit
B) Truri
C) Melci
D) Peritoneum
E) Indet e buta

275. Tumori renal me i shpeshte eshte:


A) Tumori Wilms
B) Karcinoma me qeliza renale
C) Adenoma
D) Leiomioma
E) Leiomiosarkoma

276. Per ke nga neoplazite e nuk ekziston nje procedure “screening” e miratuar?
A) Karcinoma e prostates
B) Karcinoma e gl. mamme
C) Karcinoma e pulmonit
D) Karcinoma e qafes se mitres
E) Karcinoma e rektumit

277. Cili nga faktoret e meposhtem ka rendesi prognostike ne pjesen me te madhe te neoplazive?
A) Histopatologjia
B) Gjendja
C) Masa tumorale
D) Prekja e limfonodujve
E) Te gjitha keto

278. Gjate ekzaminimit objektiv te paketave limfonodujve:


A) Limfonodujt nenmandibular palpohen me me kujdes nese mjeku eshte pas shpatullave tepacientit

229
B) Per te ekzaminuar aksilen e majte eshte me e pershtatshme te perdoresh doren e majte
C) Eshte shume e veshtire te thuash nese limfonodujt e vegjel inguinale qe palpohen janepatologjik
D) Edhe ne paraqitjen e dukshme te adenomegalise limfonodujt paraaortike nuk mund te prekengjate
palpimit te abdomenit62
E) Jane te sakta pohimet a, c, d.

279. Cili nga gjenet e , nese ben mutacion, eshte pergjegjes per sindromin kongenital teLi-
Fraumeni, karakterizuar nga nje rritje e sensibilitetit per zhvillimin e leukemise, limfomes,
sarkomes dhe tumorit te gjirit:
A) ras
B) p53
C) BRCA-1
D) c-myc
E) abl

280. CEA rritet tek personat me:


A) Tumor te kolonit
B) Tumor te melcise
C) Tumor te pankreasit
D) Fumatoret
E) Te gjitha pergjigjet jane te verteta

281. Karcinoma e prostates pergjithesisht metastazon me:


A) Pulmonin
B) Heparin
C) Trurin
D) Kolonin
E) Kocka

282. Tumori malinj me i shpeshte qe kap heparin eshte:


A) Hepatoma
B) Adenokarcinoma
C) Neuroblastoma
D) Karcinoma metastatike
E) Kolangiokarcinoma

283. Cila nga keto eshte neoplazia malinje me e shpeshte e trupit te uterusit?
A) Adenokarcinoma
B) Adenoakantoma
C) Koriokarcinoma
D) Sarkoma
E) Karcinoma me qeliza skuamoze

284. Cila nga keto neoplazi te orofaringut eshte konsideruar e lidhur me virusin Epstein-Barr?
A) Hemangioma nazale
B) Papilloma squamoze vestibulare
C) Angiofibroma e re e rinofaringut
D) Karcinoma rinofaringeale
E) Granuloma malinje e hundes

285. Inoperimi i nje tumori te stomakut eshte i pritshem nese ka:


A) Ascit
B) Metastaza ne limfonodujt supraklavikulare
C) Infiltrim ne omentum dhe perhapje ne melci dhe shpretke
D) Te gjitha jane te verteta

230
E) Asnje nuk eshte e vertete

286. Sistemi TNM:


A) Tregon shtrirjen e semundjes
B) Tregon shkallen histologjike te malinjitetit
C) Tregon gjendjen e pergjithshme te pacientit
D) Tregon efikasitetin e mundshem te trajtimit
E) Tregon aktivitetin proliferativ te neoplazise

287.Tipike eshte shoqerimi i akantozes nigricans me nje paraneoplazi te:


A) Leukoza akute mieloide
B) Mastocitoza sistemike
C) Adenokarcinoma gastrike
D) Karcinoma e qafes te uterusit
E) Sarkoma Ewing

288. Ne cilen nga keto neoplazi mund te clirohet nje sekretim hormonal i papershtatshem i
gonadotropinave qe con ne nje pubertet prekoks tek mashkulli?
A) Karcinoma renale
B) Karcinoma bronkiale
C) Karcinoma e ezofagut
D) Hepatoma
E) Feokromocitoma

289.Cili nga kushtet e eshte i lidhur ne menyre tipike me lizen tumorale?


A) Hipokalemia
B) Hiperuricemia
C) Hipertrigliceridemia
D) Hipokalcemia
E) Hipofosforemia

290. Prognoza e kancerit te gjirit te operuar eshte e lidhur me:


A) Limfonodujt aksilare pozitive
B) Forma histologjike e tumorit
C) Prezenca e receptoreve hormonale
D) Dimensionet e tumorit
E) Te gjitha keto

291. Termi permiresim i pjesshem mund te perdoret ne onkologji ne nje nga situatat e :
A) Permiresim i shenjave klinike
B) Reduktim i permasave tumorale > 50%
C) Zhdukja e tumorit primar edhe pse persistojne metastazat
D) Prezenca e tumorit por normalizimi i markuesve tumorale
E) Te gjitha te siperpermendurat

292. Kush nga pohimet e eshte i sakte lidhur me seminomen e testikulit?


A) Eshte radio-kimio sensibile
B) Eshte radiosensibile por jo kimiosensibile
C) Eshte kimiosensibile por jo radiosensibile
D) Eshte e nevojshme siguria histologjike para kryerjes se inguinotomise
E) Tregon shume shpesh markues tumorale

293. Cila nga sindromat e poshte shenuara mund te jete e lidhur me kancerin bronkopulmonar?
A) Hiperkalcemia
B) Osteopatia hipertrofike e Pierre Marie

231
C) Sindroma Cushing
D) Polineuriti
E) Te gjitha keto

294. Te gjitha pohimet e per trajtimin kimioterapeutik si linje e pare ne karcinomen ovariale te
avancuar jane te gabuara pervec njeres:
A) Eshte shume aktive ne pjesen me te madhe te pacienteve
B) Kemi pergjigje ne me pak se 20% te rasteve
C) Mund te perdorim medikamente te tilla si metotrexate dhe bleomicine
D) Ka te njejtin efikasitet si radioterapia
E) Eshte e indikuar ne paciente me receptore estrogjenike negative

295. Kush eshte trajtimi i zgjedhur ne kancerin pulmonar me qeliza te vogla ne stadin e IV?
A) Kirurgjia
B) Kimioterapia
C) Radioterapia
D) Imunoterapia
E) Te gjitha te verteta

296. Hiperkalcemia ne pacientet me neoplazi:


A) Eshte e rralle ne kancerin e gjirit
B) Eshte crregullim metabolik jashtezakonisht irralle
C) Eshte gjithmone asimptomatike
D) Eshte rrjedhoje e prodhimit paraneoplazik te nje molekule te ngjashme me kalcitoninen
E) Eshte rrjedhoje e clirimit paraneoplazik te nje molekule te ngjashme me parathormoni

297. Cili tumor i traktit intenstinal ka prognozen me te keqe?


A) Karcinoma e ezofagut
B) Karcinoma e kolonit
C) Karcinoma e intestinit66
D) Karcinoidi i intestint
E) Karcinoidi i apendixit

298. Metastazat kockore gjenden kryesisht ne:


A) Karcinomen e ovarit
B) Sarkoma e Ewing
C) Karcinome e gl. mammare
D) Karcinome e prostates
E) Seminoma

299. Te gjitha t jane medikamente imunosupresore me shpesh te perdorura pervec:


A) 5-hidroksitriptamina
B) 6-merkaptourina
C) Metotrexate
D) Azatioprina
E) Ciklofosfamide

300. Cili nga indikatoret klinike te meposhtem perben risk relativ madhor per te zhvilluar
karcinome te gl mamare?
A) Menarke e hershme
B) Anamneze familjare per karcinome te gl mamare
C) Me pak se tre gravidanca ne terme
D) Me shume se tre gravidanca te perfunduara
E) Mosushqyerje me gji

232
301. Cili është faktori kryesor prognostik në karcinomën e gjirit të operuar në mënyrë radikale?
A) Mosha
B) Mutacioni i BRCA 1 (breast cancer)
C) Përdorimi i estroprogesterinikëve
D) Prekja e limfonodjeve
E) Dimensionet e tumorit

302. Simptoma e hershme më e shpeshtë e një tumori të ezofagut është:


A) Disfagia
B) Disfonia
C) Hematemeza masive
D) Rektorragjia
E) Ethe e shtuar

303. Asbesti është përgjegjës për cilin tumor profesional?


A) Kanceri i vezikës
B) Karcinoma e prostatës
C) Hepatokarcinoma primitive
D) Mezotelioma
E) Karcinoma e veshkës

304. Në sistemin e stadifikimit TNM, M-ja tregon:


A) Receptorët hormonalë
B) Metastazat në distancë
C) Tumorin primitiv
D) Limfonodujt
E) Asnjë prej përgjigjeve të mësipërme

305. Kompresioni medular është:


A) Kompresioni i palcës kockore
B) Kompresioni medulës spinale
C) Një komplikacion i trajtimeve kimioterapeutike
D) Një efekt jatrogjen i terapisë hormonale
E) I shpeshtë në leukeminë akute

306. Hetimet e depistimit


A) Vlerësojnë përhapjen e një neoplazie pas një ndërhyrjeje kirurgjikale
B) Kërkojnë praninë e një neoplazie specifike në një popullatë të shëndetshme me risk për atë
neoplazi
C) Kërkojnë metastaza eventuale përpara një ndërhyrjeje kirurgjikale
D) Kërkojnë praninë e një neoplazie specifike në subjekte me simptoma për atë neoplazi
E) Bëjnë pjesë në parandalimin primar të tumoreve68

307. Cili prej këtyre është një tumor lidhor beninj?


A) Cistoadenoma
B) Adenokarcinoma
C) Liposarkoma me qeliza të rrumbullakta
D) Neurofibrosarkoma
E) Fibroma

308. Në onkologji, terapia ndihmëse (adjuvant E) është:


A) Administrohet për të përmirësuar cilësinë e jetës së pacientit onkologjik
B) Përdoret për të reduktuar toksicitetin e terapisë onkologjike
C) Përdoret për të reduktuar nauzenë e shkaktuar nga kimioterapia
D) Administrohet pas trajtimit loko-regjional për të ulur rrezikun e metastazave

233
E) Përdoret për të ulur masën tumorale

309. Cili nga ekzaminimet instrumentale të mëposhtme nuk përdoret për karakterizimin
emetastazave kockore?
A) Shintigrafia kockore
B) TAC
C) Radiografia
D) Ekografia
E) RMN

310. Treguesi i efikasitetit të depistimit mamografik është:


A) Zgjatja e jetëgjatësisë
B) Reduktimi i vdekshmërisë
C) Reduktimi i numrit të tumoreve të avancuar
D) Reduktimi i rasteve simptomatike
E) Të gjitha përgjigjet janë të sakta

311. Cili nga pohimet e mëposhtme që kanë të bëjnë me adenokarcinomën e pankreasit është i
gabuar?
A) Simptoma fillestare më e shpeshtë është një ikter obstruktiv
B) Pjesa më e madhe e tumoreve është e lokalizuar në kokën e pankreasit69
C) Mosha mesatare e pacientëve të prekur është ndërmjet 60 dhe 70 vjet
D) Mbijetesa përtej dy viteve që nga diagnostikimi është e pazakontë
E) Pjesa më e madhe e pacientëve ka një masë tumorale të trajtueshme kirurgjikisht

312. Sarkomat metastazojnë më shumë me rrugë:


A) Limfatike
B) Hematologjike
C) Per continuitatem
D) Likid cerebrospinal
E) Intraperitoneale

313. Karcinoma e qafës së uterusit shoqërohet në më shumë se 95% të rasteve me infeksione nga:
A) Virusi E Pstein-Barr (EBV)
B) Virusi herpetik i tetë (HHV8)
C) Papilomavirus humane (HPV)
D) Nesseria gonorrea
E) Candida albicans

314. Rruga më e përdorshme për administrimin trajtimeve citostatike është:


A) Endovenoze
B) Orale
C) Subkutane
D) Rektale
E) Loko-regionale

315. Cila nga simptomat e mëposhtme është zakonisht e pranishme në kancerin e kolon
ascendens?
A) Inkontinenca
B) Tenezma
C) Diarrea ose pseudodiarrea
D) Malabsorbimi
E) Dhimbja abdominale e tipit peristaltic

234
316. Cila nga këto neoplazi është e karakterizuar nga sekretimi i shtuar i gonadotropinës
korionike humane?
A) Hemangioma
B) Korionkarcinoma
C) Adenokarcinoma e endometrit
D) Leiomioma
E) Cistoadenoma seroze

317. Stadifikimi i tumorit të pulmonit:


A) Nuk mund të përcaktohet nga vlerësimi i markuesve tumoralë
B) Ka nevojë për një laparotomi dhe një proces kompleks
C) Eshtë e indikuar vetëm në pacientët duhanpirës
D) Kërkon shpesh TAC dhe bronkoskopi
E) Nuk përfshin shintigrafinë kockore

318. Një trajtim përfundimtar aktiv:


A) Zgjat mbijetesën, por nuk redukton dimensionet e tumorit
B) Redukton dimensionet e tumorit
C) Ka efekte anësore të kufizuara
D) Ka efekte të ndjeshme anësore
E) Të gjitha përgjigjet e mësipërme

319. Markuesit tumoralë janë:


A) Tregues pak a shumë specifikë të pranisë së një neoplazie
B) Tregues prognostikë në neoplazi të veçanta
C) Faktorë parashikues të përgjigjes ndaj kimioterapisë
D) Tregues të aktivitetit proliferues
E) Tregues të masës tumorale

320. Cila nga pjesët e mëposhtme të traktit gastroenterik është vendi më i zakonshëm i limfomave
malinje?
A) Rektumi
B) Stomaku
C) Duodeni
D) Apendiksi
E) Cekumi71

321. Cili nga faktorët etiologjikë të mëposhtëm është i përfshirë në fillimin e kancerit të colon
rectum?
A) Dietë e pasur me acide yndyrore
B) Tymi i duhanit
C) Dietë e pasur me ushqime të gjalla
D) Ushqyerje hipokalorike
E) Ushqyerje shumë e pasur me fruta dhe perime

322. Anemia e shoqëruar me karcinomën e prostatës në fazë të avancuar i detyrohet:


A) Metastazave kockore të përhapura
B) Mungesa e hekurit
C) Mungesa e folateve
D) Terapia antiandrogjenike
E) Terapia anabolizante

323. Markuesi tumoral më i përdorshëm në neoplazitë e collon rectum është:


A) PSA
B) CA 125

235
C) CA 15-3
D) Kromogranina
E) CEA

324. Terapia e zgjedhur e limfomave jo Hodgkiniane agresive është:


A) Radioterapia
B) Polikimioterapia
C) Monokimioterapia
D) Imunoterapia
E) Asnjëra prej përgjigjeve të mësipërme

325. Një burrë i prekur prej 10 vitesh nga koliti ulçeroz i rektumit, ka një probabilitet të shprehur
nëpërqindje për të zhvilluar kancer të barabartë me:
A) 20%
B) 4%72
C) 40%
D) 10%
E) 0%

326. Toksiciteti dozë-grumbullues më i shpeshtë shoqërues i terapisë me antraciklinë është:


A) Retinik
B) Hepatik
C) Neurologjik
D) Nefrologjik
E) Kardiak

327. Cila nga këto substanca, është faktor rritjeje hematopoetik?


A) Interleukina 2
B) Eritropoetina
C) GM-CSF
D) G-CSF
E) Të gjitha përgjigjet e mësipërme

328. Prevalenca e një neoplazie rritet për shkak se:


A) Rritet incidenca dhe rritet vdekshmëria
B) Ulet incidenca dhe ulet vdekshmëria
C) Rritet incidenca dhe ulet vdekshmëria
D) Ulet incidenca dhe rritet vdekshmëria
E) Asnjëra prej përgjigjeve të mësipërme

329. Kriteri i vlerësimit të efikasitetit të terapisë ndihmëse është:


A) Zgjatja e jetëgjatësisë
B) Arritja e remisionit komplet
C) Reduktimi i masës tumorale
D) Reduktimi i simptomave të lidhura me sëmundjen
E) Asnjë prej përgjigjeve të mësipërme

330. Në një karcinomë të kuadrantit supra ekstern të gjirit dx (cT1NO), cili prej interventeve
tëmëposhtëm nuk i përgjigjet kritereve të radikalitetit?
A) Mastektomia sipas Patey
B) Mastektomia sipas Halstead
C) Lumpektomia me heqje të limfonodulit “roje” dhe radioterapi mbi mamelën dx të mbetur
D) Kuadrantektomia supra eksterne
E) Nodulektomia me 2 cm margo, limfadenektomia aksilare dhe radioterapia mbi mamelën embetur

236
331. Cila nga neoplazitë e mëposhtme të sferës gjenitale femërore ka incidencë më të madhe
nëshkallë botërore?
A) Kanceri i qafës së uterusit
B) Kanceri i endometrit
C) Karcinoma e vulvës
D) Karcinoma e vaginës
E) Tumori i ovarit

332. Cila është neoplazia solide më e shpeshtë tek subjektet meshkuj ndërmjet 20 dhe 35 vjeç?
A) Tumori me qeliza germinale
B) Sarkoma e Ewing
C) Melanoma
D) Karcinoma
E) Glioblastoma

333. Një ndërlikim i zakonshëm i korionepiteliomës është:


A) Okluzioni intestinal
B) Vdekje e papritur
C) Perforacion i uterusit dhe hemorragji e bollshme
D) Tromboflebit
E) Asnjëra prej përgjigjeve të mësipërme

334. Cilat janë mënyrat dhe qëllimet për të cilat kryhet kimioterapia?
A) Ndihmëse
B) Neondihmëse
C) Paliative
D) Shpëtuese
E) Të gjitha bashkë

335. Cila prej këtyre neoplazive mund të japë një sekretim të papërshtatshëm të ADH?
A) Fibrosarkoma retroperitoneale
B) Karcinoma surenaliene
C) Feokromocitoma
D) Karcinoma bronkiale me qeliza në formë “të kokrrës së tërshërës”
E) Hepatoma

336. Efektet anësore më të zakonshme të trajtimeve citostatike janë:


A) Alopecia, nauzea, të vjella, diarrea, leukopenia
B) Sterilitet, dermatit
C) Cistit, neuropati periferike, paralizë e ileumit
D) Otalgjia, rinorhea, konjuktivit
E) Takikardia, hipertensioni, ekstrasistola, kollë, konstipacion

337. Mikrocitoma pulmonare është një neoplazi që mjekohet me:


A) Kimioradioterapi
B) Kirurgji
C) Radioterapi
D) Kimioterapi
E) Radiokirurgji

338. Si përkufizohet mbijetesa e lirë e një sëmundjeje:


A) Intervali kohor ndërmjet datës së terapisë primare dhe datës së recidivës
B) Intervali kohor ndërmjet datës së terapisë primare dhe datës së vdekjes
C) Intervali kohor ndërmjet datës së recidivës dhe datës së vdekjes

237
D) Intervali kohor ndërmjet datës së diagnozës dhe datës së terapisë
E) Asnjëra prej përgjigjeve

339. Cili bar duhet konsideruar si pikë referimi në terapinë ndihmëse në karcinomën e rektumit?
A) 5-fluorouracili
B) Metotreksati
C) Irinotekani75
D) Oksaliplatini
E) Të gjitha përgjigjet e mësipërme

340. Metastazat cerebrale mund të shfaqen më shpesh në rastet e:


A) Karcinomës së vezikës
B) Karcinomës së pankreasit
C) Karcinomës pulmonare
D) Karcinomës së ovarit
E) Karcinoma e kolonit

341. Trajtimi i zgjedhur i mikrocitomës është:


A) Polikimioterapia
B) Hormonoterapia
C) Radioterapia mbi mediastinin
D) Terapia biologjike
E) Kirurgjia

342. Cili është vendi më i zakonshëm i tumorit gjigando-qelizor të kockës (osteoklastoma)?


A) Kolona vertebrale
B) Mandibula
C) Kafka
D) Ekstremiteti distal i femurit
E) Falanget e gjymtyreve te siperme

238

You might also like